GI NCLEX

अब Quizwiz के साथ अपने होमवर्क और परीक्षाओं को एस करें!

In a client with Crohn's disease, which of the following symptoms should not be a direct result from antibiotic therapy? A. Decrease in bleeding B. Decrease in temperature C. Decrease in body weight D. Decrease in the number of stools

C A decrease in body weight may occur during therapy due to inadequate dietary intake, but isn't related to antibiotic therapy. Effective antibiotic therapy will be noted by a decrease in temperature, number of stools, and bleeding.

The client has just had surgery to create an ileostomy. The nurse assesses the client in the immediate post-op period for which of the following most frequent complications of this type of surgery? A. Intestinal obstruction B. Fluid and electrolyte imbalance C. Malabsorption of fat D. Folate deficiency

C A major complication that occurs most frequent following an ileostomy is fluid and electrolyte imbalance. The client requires constant monitoring of intake and output to prevent this from happening. Losses require replacement by intravenous infusion until the client can tolerate a diet orally. Intestinal obstruction is a less frequent complication. Fat malabsorption and folate deficiency are complications that could occur later in the postoperative period.

A male client with a peptic ulcer is scheduled for a vagotomy and the client asks the nurse about the purpose of this procedure. Which response by the nurse best describes the purpose of a vagotomy? A. Halts stress reactions B. Heals the gastric mucosa C. Reduces the stimulus to acid secretions D. Decreases food absorption in the stomach

C A vagotomy, or cutting of the vagus nerve, is done to eliminate parasympathetic stimulation of gastric secretion. Options A, B, and D are incorrect descriptions of a vagotomy.

Which of the following factors is believed to be linked to Crohn's disease? A. Constipation B. Diet C. Hereditary D. Lack of exercise

C Although the definite cause of Crohn's disease is unknown, it's thought to be associated with infectious, immune, or psychological factors. Because it has a higher incidence in siblings, it may have a genetic cause.

When a client has peptic ulcer disease, the nurse would expect a priority intervention to be: A. Assisting in inserting a Miller-Abbott tube B. Assisting in inserting an arterial pressure line C. Inserting a nasogastric tube D. Inserting an I.V

C An NG tube insertion is the most appropriate intervention because it will determine the presence of active GI bleeding. A Miller-Abbott tube (1) is a weighted, mercury-filled ballooned tube used to resolve bowel obstructions. There is no evidence of shock or fluid overload in the client; therefore, an arterial line (2) is not appropriate at this time and an IV (4) is optional.

Medical management of the client with diverticulitis should include which of the following treatments? A. Reduced fluid intake B. Increased fiber in diet C. Administration of antibiotics D. Exercises to increase intra-abdominal pressure

C Antibiotics are used to reduce the inflammation. The client isn't typically isn't allowed anything orally until the acute episode subsides. Parenteral fluids are given until the client feels better; then it's recommended that the client drink eight 8-ounce glasses of water per day and gradually increase fiber in the diet to improve intestinal motility. During the acute phase, activities that increase intra-abdominal pressure should be avoided to decrease pain and the chance of intestinal obstruction.

The most important pathophysiologic factor contributing to the formation of esophageal varices is: A. Decreased prothrombin formation B. Decreased albumin formation by the liver C. Portal hypertension D. Increased central venous pressure

C As the liver cells become fatty and degenerate, they are no longer able to accommodate the large amount of blood necessary for homeostasis. The pressure in the liver increases and causes increased pressure in the venous system. As the portal pressure increases, fluid exudes into the abdominal cavity. This is called ascites.

Which of the following associated disorders may the client with Crohn's disease exhibit? A. Ankylosing spondylitis B. Colon cancer C. Malabsorption D. Lactase deficiency

C Because of the transmural nature of Crohn's disease lesions, malaborption may occur with Crohn's disease. Ankylosing spondylitis and colon cancer are more commonly associated with ulcerative colitis. Lactase deficiency is caused by a congenital defect in which an enzyme isn't present.

Proton pump inhibitors: A. Gastric ulcer formation B. GERD C. Achlorhydria D. Diverticulosis

C Because the proton pump inhibitors stop the final step of acid secretion, they can block up to 90% of acid secretion, leading to achlorhydria (without acid).

The nurse is preparing a discharge teaching plan for the client who had an umbilical hernia repair. Which of the following would the nurse include in the plan? A. Restricting pain medication B. Maintaining bedrest C. Avoiding coughing D. Irrigating the drain

C Bedrest is not required following this surgical procedure. The client should take analgesics as needed and as prescribed to control pain. A drain is not used in this surgical procedure, although the client may be instructed in simple dressing changes. Coughing is avoided to prevent disruption of the tissue integrity, which can occur because of the location of this surgical procedure.

The mechanism of action of diphenoxylate (Lotomil) is: A. An increase in intestinal excretion of water B. An increase in intestinal motility C. A decrease in peristalsis in the intestinal wall D. A decrease in the reabsorption of water in the boweL

C Diphenoxylate acts on the smooth muscle of the intestinal tract to inhibit GI motility and excessive propulsion of the GI tract (peristalsis).

he nurse is providing discharge instructions to a male client following gastrectomy and instructs the client to take which measure to assist in preventing dumping syndrome? A. Ambulate following a meal B. Eat high carbohydrate foods C. Limit the fluid taken with meal D. Sit in a high-Fowler's position during meals

C Dumping syndrome is a term that refers to a constellation of vasomotor symptoms that occurs after eating, especially following a Billroth II procedure. Early manifestations usually occur within 30 minutes of eating and include vertigo, tachycardia, syncope, sweating, pallor, palpitations, and the desire to lie down. The nurse should instruct the client to decrease the amount of fluid taken at meals and to avoid high-carbohydrate foods, including fluids such as fruit nectars; to assume a low-Fowler's position during meals; to lie down for 30 minutes after eating to delay gastric emptying; and to take antispasmodics as prescribed.

The nurse is monitoring a client for the early signs of dumping syndrome. Which symptom indicates this occurrence? A. Abdominal cramping and pain B. Bradycardia and indigestion C. Sweating and pallor D. Double vision and chest pain

C Early manifestations of dumping syndrome occur 5 to 30 minutes after eating. Symptoms include vertigo, tachycardia, syncope, sweating, pallor, palpitations, and the desire to lie down.

Which of the following symptoms is common with a hiatal hernia? A. Left arm pain B. Lower back pain C. Esophageal reflux D. Abdominal cramping

C Esophageal reflux is a common symptom of hiatal hernia. This seems to be associated with chronic exposure of the lower esophageal sphincter to the lower pressure of the thorax, making it less effective.

A client's ulcerative colitis symptoms have been present for longer than 1 week. The nurse recognizes that the client should be assessed carefully for signs of which of the following complications? A. Heart failure B. DVT C. Hypokalemia D. Hypocalcemia

C Excessive diarrhea causes significant depletion of the body's stores of sodium and potassium as well as fluid. The client should be closely monitored for hypokalemia and hyponatremia. Ulcerative colitis does not place the client at risk for heart failure, DVT, or hypocalcemia.

Five days after undergoing surgery, a client develops a small-bowel obstruction. A Miller-Abbott tube is inserted for bowel decompression. Which nursing diagnosis takes priority? A. Imbalanced nutrition: Less than body requirements B. Acute pain C. Deficient fluid volume D. Excess fluid volume

C Fluid shifts to the site of the bowel obstruction, causing a fluid deficit in the intravascular spaces. If the obstruction isn't resolved immediately, the client may experience an imbalanced nutritional status (less than body requirements); however, deficient fluid volume takes priority. The client may also experience pain, but that nursing diagnosis is also of lower priority than deficient fluid volume.

A client who has ulcerative colitis has persistent diarrhea. He is thin and has lost 12 pounds since the exacerbation of his ulcerative colitis. The nurse should anticipate that the physician will order which of the following treatment approaches to help the client meet his nutritional needs? A. Initiate continuous enteral feedings B. Encourage a high protein, high-calorie diet C. Implement total parenteral nutrition D. Provide six small meals a day

C Food will be withheld from the client with severe symptoms of ulcerative colitis to rest the bowel. To maintain the client's nutritional status, the client will be started on TPN. Enteral feedings or dividing the diet into 6 small meals does not allow the bowel to rest. A high-calorie, high-protein diet will worsen the client's symptoms.

The nurse has given instructions to the client with an ileostomy about foods to eat to thicken the stool. The nurse determines that the client needs further instructions if the client stated to eat which of the following foods to make the stools less watery? A. Pasta B. Boiled rice C. Bran D. Low-fat cheese

C Foods that help thicken the stool of the client with an ileostomy include pasta, boiled rice, and low-fat cheese. Bran is high in dietary fiber and thus will increase output of watery stool by increasing propulsion through the bowel. Ileostomy output is liquid. Addition or elimination of various foods can help thicken or loosen this liquid drainage.

Marie, a 51-year-old woman, is diagnosed with cholecystitis. Which diet, when selected by the client, indicates that the nurse's teaching has been successful? A. 4-6 small meals of low-carbohydrate foods daily B. High-fat, high-carbohydrate meals C. Low-fat, high-carbohydrate meals D. High-fat, low protein meals

C For the client with cholecystitis, fat intake should be reduced. The calories from fat should be substituted with carbohydrates. Reducing carbohydrate intake would be contraindicated. Any diet high in fat may lead to another attack of cholecystitis.

A male client who is recovering from surgery has been advanced from a clear liquid diet to a full liquid diet. The client is looking forward to the diet change because he has been "bored" with the clear liquid diet. The nurse would offer which full liquid item to the client? A. Tea B. Gelatin C. Custard D. Popsicle

C Full liquid food items include items such as plain ice cream, sherbet, breakfast drinks, milk, pudding and custard, soups that are strained, and strained vegetable juices. A clear liquid diet consists of foods that are relatively transparent. The food items in options A, B, and D are clear liquids.

Which of the following definitions best describes gastritis? A. Erosion of the gastric mucosa B. Inflammation of a diverticulum C. Inflammation of the gastric mucosa D. Reflux of stomach acid into the esophagus

C Gastritis is an inflammation of the gastric mucosa that may be acute (often resulting from exposure to local irritants) or chronic (associated with autoimmune infections or atrophic disorders of the stomach). Erosion of the mucosa results in ulceration. Inflammation of a diverticulum is called diverticulitis; reflux of stomach acid is known as gastroesophageal disease.

A client with advanced cirrhosis has been diagnosed with hepatic encephalopathy. The nurse expects to assess for: A. Malaise B. Stomatitis C. Hand tremors D. Weight loss

C Hepatic encephalopathy results from the accumulation of neurotoxins in the blood, therefore the nurse wants to assess for signs of neurological involvement. Flapping of the hands (asterixis), changes in mentation, agitation, and confusion are common. These clients typically have ascites and edema so experience weight gain. Malaise and stomatitis are not related to neurological involvement.

Nurse Joy is preparing to administer medication through a nasogastric tube that is connected to suction. To administer the medication, the nurse would: A. Position the client supine to assist in medication absorption B. Aspirate the nasogastric tube after medication administration to maintain patency C. Clamp the nasogastric tube for 30 minutes following administration of the medication D. Change the suction setting to low intermittent suction for 30 minutes after medication administration

C If a client has a nasogastric tube connected to suction, the nurse should wait up to 30 minutes before reconnecting the tube to the suction apparatus to allow adequate time for medication absorption. Aspirating the nasogastric tube will remove the medication just administered. Low intermittent suction also will remove the medication just administered. The client should not be placed in the supine position because of the risk for aspiration.

The nurse is performing a colostomy irrigation on a client. During the irrigation, a client begins to complain of abdominal cramps. Which of the following is the most appropriate nursing action? A. Notify the physician B. Increase the height of the irrigation C. Stop the irrigation temporarily D. Medicate with dilaudid and resume the irrigation

C If cramping occurs during a colostomy irrigation, the irrigation flow is stopped temporarily and the client is allowed to rest. Cramping may occur from an infusion that is too rapid or is causing too much pressure. Increasing the height of the irrigation will cause further discomfort. The physician does not need to be notified. Medicating the client for pain is not the most appropriate action

When administering sucralfate (Carafate) to a patient with a nasogastric tube, it is important to: A. Crush the tablet into a fine powder before mixing with water B. Administer with a bolus tube feeding C. Allow the tablet to dissolve in water before administering D. Administer with an antacid for maximum benefit

C It is important to give sucralfate on an empty stomach so that it may dissolve and form a protective barrier over the gastric mucosa. The tablet form will not dissolve in water when crushed; it must be left whole and allowed to dissolve. Crushing the medication so that it will not dissolve could lead to clogging of the nasogastric tube and decreased effectiveness of the drug.

A client has been taking aluminum hydroxide 30 mL six times per day at home to treat his peptic ulcer. He tells the nurse that he has been unable to have a bowel movement for 3 days. Based on this information, the nurse would determine that which of the following is the most likely cause of the client's constipation? A. The client has not been including enough fiber in his diet B. The client needs to increase his daily exercise C. The client is experiencing a side effect of the aluminum hydroxide. D. The client has developed a gastrointestinal obstruction

C It is most likely that the client is experiencing a side effect of the antacid. Antacids with aluminum salt products, such as aluminum hydroxide, form insoluble salts in the body. These precipitate and accumulate in the intestines, causing constipation. Increasing dietary fiber intake or daily exercise may be a beneficial lifestyle change for the client but is not likely to relieve the constipation caused by the aluminum hydroxide. Constipation, in isolation from other symptoms, is not a sign of bowel obstruction.

Which of the following interventions should be included in the medical management of Crohn's disease? A. Increasing oral intake of fiber B. Administering laxatives C. Using long-term steroid therapy D. Increasing physical activity

C Management of Crohn's disease may include long-term steroid therapy to reduce the inflammation associated with the deeper layers of the bowel wall. Other management focuses on bowel rest (not increasing oral intake) and reducing diarrhea with medications (not giving laxatives). The pain associated with Crohn's disease may require bed rest, not an increase in physical activity.

The nurse is reviewing the physician's orders written for a male client admitted to the hospital with acute pancreatitis. Which physician order should the nurse question if noted on the client's chart? A. NPO status B. Nasogastric tube inserted C. Morphine sulfate for pain D. An anticholinergic medication

C Meperidine (Demerol) rather than morphine sulfate is the medication of choice to treat pain because morphine sulfate can cause spasms in the sphincter of Oddi. Options A, B, and D are appropriate interventions for the client with acute pancreatitis.

After a subtotal gastrectomy, care of the client's nasogastric tube and drainage system should include which of the following nursing interventions? A. Irrigate the tube with 30 ml of sterile water every hour, if needed B. Reposition the tube if it is not draining well C. Monitor the client for N/V, and abdominal distention D. Turn the machine to high suction of the drainage is sluggish on low suction

C Nausea, vomiting, or abdominal distention indicated that gas and secretions are accumulating within the gastric pouch due to impaired peristalsis or edema at the operative site and may indicate that the drainage system is not working properly. Saline solution is used to irrigate nasogastric tubes. Hypotonic solutions such as water increase electrolyte loss. In addition, a physician's order is needed to irrigate the NG tube, because this procedure could disrupt the suture line. After gastric surgery, only the surgeon repositions the NG tube because of the danger of rupturing or dislodging the suture line. The amount of suction varies with the type of tube used and is ordered by the physician. High suction may create too much tension on the gastric suture line.

The patient is having an esophagoenterostomy with anastomosis of a segment of the colon to replace the resected portion. What initial postoperative care should the nurse expect when this patient returns to the nursing unit? A) Turn, deep breathe, cough, and use spirometer every 4 hours. B) Maintain an upright position for at least 2 hours after eating. C) NG will have bloody drainage, and it should not be repositioned. D) Keep in a supine position to prevent movement of the anastomosis.

C) The patient will have bloody drainage from the NG tube for 8 to 12 hours, and it should not be repositioned or reinserted without contacting the surgeon. Turning and deep breathing will be done every 2 hours, and the spirometer will be used more often than every 4 hours. Coughing would put too much pressure in the area and should not be done. Because the patient will have the NG tube, the patient will not be eating yet. The patient should be kept in a semi-Fowler's or Fowler's position, not supine, to prevent reflux and aspiration of secretions.

A patient unable to tolerate oral medications may be prescribed which of the following proton pump inhibitors to be administered intravenously? A. lansoprazole (Prevacid) B. omeprazole (Prilosec) C. pantoprazole (Protonix) D. esomeprazole (Nexium)

C Pantoprazole is the only proton pump inhibitor that is available for intravenous administration. The other medications in this category may only be administered orally.

A client is to take one daily dose of ranitidine (Zantac) at home to treat her peptic ulcer. The nurse knows that the client understands proper drug administration of ranitidine when she says that she will take the drug at which of the following times? A. Before meals B. With meals C. At bedtime D. When pain occurs

C Ranitidine blocks secretion of hydrochloric acid. Clients who take only one daily dose of ranitidine are usually advised to take it at bedtime to inhibit nocturnal secretion of acid. Clients who take the drug twice a day are advised to take it in the morning and at bedtime.

The nurse is caring for a hospitalized female client with a diagnosis of ulcerative colitis. Which finding, if noted on assessment of the client, would the nurse report to the physician? A. Hypotension B. Bloody diarrhea C. Rebound tenderness D. A hemoglobin level of 12 mg/dL

C Rebound tenderness may indicate peritonitis. Bloody diarrhea is expected to occur in ulcerative colitis. Because of the blood loss, the client may be hypotensive and the hemoglobin level may be lower than normal. Signs of peritonitis must be reported to the physician.

A client with ulcerative colitis has an order to begin salicylate medication to reduce inflammation. The nurse instructs the client to take the medication: A. 30 minutes before meals B. On an empty stomach C. After meals D. On arising

C Salicylate compounds act by inhibiting prostaglandin synthesis and reducing inflammation. The nurse teaches the client to take the medication with a full glass of water and to increase fluid intake throughout the day. This medication needs to be taken after meals to reduce GI irritation.

Sucralfate (Carafate) achieves a therapeutic effect by: A. Neutralizing gastric acid B. Enhancing gastric absorption C. Forming a protective barrier around gastric mucosa D. Inhibiting gastric acid secretion

C Sucralfate has a local effect only on the gastric mucosa. It forms a paste-like substance in the stomach, which adheres to the gastric lining, protecting against adverse effects related to gastric acid. It also stimulates healing of any ulcerated areas of the gastric mucosa.

The nurse is performing an abdominal assessment and inspects the skin of the abdomen. The nurse performs which assessment technique next? A. Palpates the abdomen for size B. Palpates the liver at the right rib margin C. Listens to bowel sounds in all four quadrants D. Percusses the right lower abdominal quadrant

C The appropriate sequence for abdominal examination is inspection, auscultation, percussion, and palpation. Auscultation is performed after inspection to ensure that the motility of the bowel and bowel sounds are not altered by percussion or palpation. Therefore, after inspecting the skin on the abdomen, the nurse should listen for bowel sounds.

The client with a duodenal ulcer may exhibit which of the following findings on assessment? A. Hematemesis B. Malnourishment C. Melena D. Pain with eating

C The client with a duodenal ulcer may have bleeding at the ulcer site, which shows up as melena (black tarry poop). The other findings are consistent with a gastric ulcer.

During the assessment of a client's mouth, the nurse notes the absence of saliva. The client is also complaining of pain near the area of the ear. The client has been NPO for several days because of the insertion of a NG tube. Based on these findings, the nurse suspects that the client is developing which of the following mouth conditions? A. Stomatitis B. Oral candidiasis C. Parotitis D. Gingivitis

C The lack of saliva, pain near the area of the ear, and the prolonged NPO status of the client should lead the nurse to suspect the development of parotitis, or inflammation of the parotid gland. Parotitis usually develops in cases of dehydration combined with poor oral hygiene or when clients have been NPO for an extended period. Preventative measures include the use of sugarless hard candy or gum to stimulate saliva production, adequate hydration, and frequent mouth care. Stomatitis (inflammation of the mouth) produces excessive salivation and a sore mouth.

A client with suspected gastric cancer undergoes an endoscopy of the stomach. Which of the following assessments made after the procedure would indicate the development of a potential complication? A. The client complains of a sore throat B. The client displays signs of sedation C. The client experiences a sudden increase in temperature D. The client demonstrates a lack of appetite

C The most likely complication of an endoscopic procedure is perforation. A sudden temperature spike with 1 to 2 hours after the procedure is indicative of a perforation and should be reported immediately to the physician. A sore throat is to be anticipated after an endoscopy. Clients are given sedatives during the procedure, so it is expected that they will display signs of sedation after the procedure is completed. A lack of appetite could be the result of many factors, including the disease process.

Mucosal barrier fortifiers are used in peptic ulcer disease management for which of the following indications? A. To inhibit mucus production B. To neutralize acid production C. To stimulate mucus production D. To stimulate hydrogen ion diffusion back into the mucosa

C The mucosal barrier fortifiers stimulate mucus production and prevent hydrogen ion diffusion back into the mucosa, resulting in accelerated ulcer healing. Antacids neutralize acid production.

Nurse Berlinda is assigned to a 41-year-old client who has a diagnosis of chronic pancreatitis. The nurse reviews the laboratory result, anticipating a laboratory report that indicates a serum amylase level of: A. 45 units/L B. 100 units/L C. 300 units/L D. 500 units/L

C The normal serum amylase level is 25 to 151 units/L. With chronic cases of pancreatitis, the rise in serum amylase levels usually does not exceed three times the normal value. In acute pancreatitis, the value may exceed five times the normal value. Options A and B are within normal limits. Option D is an extremely elevated level seen in acute pancreatitis.

Which of the following medications is most effective for treating the pain associated with irritable bowel disease? A. Acetaminophen B. Opiates C. Steroids D. Stool softeners

C The pain with irritable bowel disease is caused by inflammation, which steroids can reduce. Stool softeners aren't necessary. Acetaminophen has little effect on the pain, and opiate narcotics won't treat its underlying cause

The client being treated for esophageal varices has a Sengstaken-Blakemore tube inserted to control the bleeding. The most important assessment is for the nurse to: A. Check that the hemostat is on the bedside B. Monitor IV fluids for the shift C. Regularly assess respiratory status D. Check that the balloon is deflated on a regular basis

C The respiratory system can become occluded if the balloon slips and moves up the esophagus, putting pressure on the trachea. This would result in respiratory distress and should be assessed frequently. Scissors should be kept at the bedside to cut the tube if distress occurs. This is a safety intervention.

A 30-year old client experiences weight loss, abdominal distention, crampy abdominal pain, and intermittent diarrhea after birth of her 2nd child. Diagnostic tests reveal gluten-induced enteropathy. Which foods must she eliminate from her diet permanently? A. Milk and dairy products B. Protein-containing foods C. Cereal grains (except rice and corn) D. Carbohydrates

C To manage gluten-induced enteropathy, the client must eliminate gluten, which means avoiding all cereal grains except for rice and corn. In initial disease management, clients eat a high calorie, high-protein diet with mineral and vitamin supplements to help normalize nutritional status.

Pierre who is diagnosed with acute pancreatitis is under the care of Nurse Bryan. Which intervention should the nurse include in the care plan for the client? A. Administration of vasopressin and insertion of a balloon tamponade B. Preparation for a paracentesis and administration of diuretics C. Maintenance of nothing-by-mouth status and insertion of nasogastric (NG) tube with low intermittent suction D. Dietary plan of a low-fat diet and increased fluid intake to 2,000 ml/day

C With acute pancreatitis, the client is kept on nothing-by-mouth status to inhibit pancreatic stimulation and secretion of pancreatic enzymes. NG intubation with low intermittent suction is used to relieve nausea and vomiting, decrease painful abdominal distention, and remove hydrochloric acid. Vasopressin would be appropriate for a client diagnosed with bleeding esophageal varices. Paracentesis and diuretics would be appropriate for a client diagnosed with portal hypertension and ascites. A low-fat diet and increased fluid intake would further aggravate the pancreatitis.

During the treatment of the patient with bleeding esophageal varices, it is most important that the nurse a. prepare the patient for immediate portal shunting surgery b. perform guaiac testing on all stools to detect occult blood c. maintain the patient's airway and prevent aspiration of blood d. monitor for the cardiac effects of IV vasopressin and nitroglycerin

C- Bleeding esophageal varices are a medical emergency. During an episode of bleeding, management of the airway and prevention of aspiration of blood are critical factors. Occult blood as well as fresh blood from the GI tract would be expected and is not tested. Vasopressin causes vasoconstriction, decreased HR, and decreased coronary blood flow; nitroglycerin is given with the vasopressin to counter these side effects. Portal shunting surgery is performed for esophageal varices but not during an acute hemorrhage

During the incubation period of viral hepatitis, the nurse would expect the patient to report a. pruritus and malaise b. dark urine and easy fatigability c. anorexia and right upper quadrant discomfort d. constipation or diarrhea with light colored stools

C- Incubation symptoms occur before the onset of jaundice and include a variety of GI symptoms as well as discomfort and heaviness in the upper right quadrant of the abdomen. Pruritus, dark urine, and light colored stools occur with the onset of jaundice in the acute phase.

A nurse is teaching an elderly client about good bowel habits. Which statement by the client indicates to the nurse that additional teaching is required? a) "I need to drink 2 to 3 liters of fluids every day." b) "I should exercise four times per week." c) "I need to use laxatives regularly to prevent constipation." d) "I should eat a fiber-rich diet with raw, leafy vegetables, unpeeled fruit, and whole grain bread."

C) "I need to use laxatives regularly to prevent constipation." The client requires more teaching if he states that he'll use laxatives regularly to prevent constipation. The nurse should teach this client to gradually eliminate the use of laxatives because using laxatives to promote regular bowel movements may have the opposite effect. A high-fiber diet, ample amounts of fluids, and regular exercise promote good bowel health.

A patient is scheduled to receive "Colace 100 mg PO." The patient asks to take the medication in liquid form, and the nurse obtains an order for the interchange. Available is a syrup that contains 150 mg/15 ml. How many milliliters does the nurse administer? A) 3 B) 5 C) 10 D) 12

C) 10 mL The concentration of the syrup is 10 mg/ml. Therefore, a 100-mg dose necessitates 10 ml.

A nurse is receiving report from the emergency room regarding a new client being admitted to the medical-surgical unit with a diagnosis of peptic ulcer disease. The nurse expects the age of the client will be between a) 20 and 30 years b) 15 and 25 years c) 40 and 60 years d) 60 and 80 years

C) 40 to 60 years Peptic ulcer disease occurs with the greatest frequency in people 40 to 60 years old. It is relatively uncommon in women of childbearing age, but it has been observed in children and even in infants.

The patient tells the nurse she had a history of abdominal pain, so she had a surgery to make an opening into the common bile duct to remove stones. The nurse knows that this surgery is called a A) colectomy B) cholecystectomy C) choledocholithotomy D) choledochojejunostomy

C) A choledocholithotomy is an opening into the common bile duct for the removal of stones. A colectomy is the removal of the colon. The cholecystectomy is the removal of the gallbladder. The choledochojejunostomy is an opening between the common bile duct and the jejunum.

A 24-year-old athlete is admitted to the trauma unit following a motor-vehicle collision. The client is comatose and has developed ascites as a result of the accident. You are explaining the client's condition to his parents. In your education, what do you indicate is the primary function of the small intestine? a) Digest proteins b) Digest fats c) Absorb nutrients d) Absorb water

C) Absorb nutrients The primary function of the small intestine is to absorb nutrients from the chyme.

A 54-year-old patient admitted with diabetes mellitus, malnutrition, osteomyelitis, and alcohol abuse has a serum amylase level of 280 U/L and a serum lipase level of 310 U/L. To which of the following diagnoses does the nurse attribute these findings? A) Malnutrition B) Osteomyelitis C) Alcohol abuse D) Diabetes mellitus

C) Alcohol Abuse The patient with alcohol abuse could develop pancreatitis as a complication, which would increase the serum amylase (normal 30-122 U/L) and serum lipase (normal 31-186 U/L) levels as shown.

A patient with cholelithiasis needs to have the gallbladder removed. Which patient assessment is a contraindication for a cholecystectomy? A) Low-grade fever of 100° F and dehydration B) Abscess in the right upper quadrant of the abdomen C) Activated partial thromboplastin time (aPTT) of 54 seconds D) Multiple obstructions in the cystic and common bile duct

C) An aPTT of 54 seconds is above normal and indicates insufficient clotting ability. If the patient had surgery, significant bleeding complications postoperatively are very likely. Fluids can be given to eliminate the dehydration; the abscess can be assessed, and the obstructions in the cystic and common bile duct would be relieved with the cholecystectomy.

A longitudinal tear or ulceration in the lining of the anal canal is termed a (an) a) anorectal abscess. b) anal fistula. c) anal fissure. d) hemorrhoid.

C) Anal fissure Fissures are usually caused by the trauma of passing a large, firm stool or from persistent tightening of the anal canal secondary to stress or anxiety (leading to constipation). An anorectal abscess is an infection in the pararectal spaces. An anal fistula is a tiny, tubular, fibrous tract that extends into the anal canal from an opening located beside the anus. A hemorrhoid is a dilated portion of vein in the anal canal.

A longitudinal tear or ulceration in the lining of the anal canal is termed a (an) a) hemorrhoid. b) anorectal abscess. c) anal fissure. d) anal fistula.

C) Anal fissure Fissures are usually caused by the trauma of passing a large, firm stool or from persistent tightening of the anal canal secondary to stress or anxiety (leading to constipation). An anorectal abscess is an infection in the pararectal spaces. An anal fistula is a tiny, tubular, fibrous tract that extends into the anal canal from an opening located beside the anus. A hemorrhoid is a dilated portion of vein in the anal canal.

The nurse is preparing to measure the client's abdominal girth as part of the physical examination. At which location would the nurse most likely measure? a) At the lower border of the liver b) In the right upper quadrant c) At the umbilicus d) Just below the last rib

C) At the umbilicus Measurement of abdominal girth is done at the widest point, which is usually the umbilicus. The right upper quadrant, lower border of the liver, or just below the last rib would be inappropriate sites for abdominal girth measurement.

A client is prescribed tetracycline to treat peptic ulcer disease. Which of the following instructions would the nurse give the client? a) "Take the medication with milk." b) "Do not drive when taking this medication." c) "Be sure to wear sunscreen while taking this medicine." d) "Expect a metallic taste when taking this medicine, which is normal."

C) Be sure to wear sunscreen while taking this medicine Tetracycline may cause a photosensitivity reaction in clients. The nurse should caution the client to use sunscreen when taking this drug. Dairy products can reduce the effectiveness of tetracycline, so the nurse should not advise him or her to take the medication with milk. A metallic taste accompanies administration of metronidazole (Flagyl). Administration of tetracycline does not necessitate driving restrictions.

The nurse is to obtain a stool specimen from a client who reported that he is taking iron supplements. The nurse would expect the stool to be which color? a) Dark brown b) Red c) Black d) Green

C) Black Ingestion of iron can cause the stool to turn black. Meat protein causes stool to appear dark brown. Ingestion of large amounts of spinach may turn stool green while ingestion of carrots and beets may cause stool to turn red.

Which of the following terms is used to refer to intestinal rumbling? a) Diverticulitis b) Tenesmus c) Borborygmus d) Azotorrhea

C) Borborygmus Borborygmus is the intestinal rumbling that accompanies diarrhea. Tenesmus is the term used to refer to ineffectual straining at stool. Azotorrhea is the term used to refer to excess of nitrogenous matter in the feces or urine. Diverticulitis refers to inflammation of a diverticulum from obstruction (by fecal matter) resulting in abscess formation.

A nurse is caring for a client who had gastric bypass surgery 2 days ago. Which assessment finding requires immediate intervention? a) The client states he has been passing gas. b) The client states he is nauseated. c) The client's right lower leg is red and swollen. d) The client complains of pain at the surgical site.

C) Client's right lower leg is red and swollen A red, swollen extremity is a possible sign of a thromboembolism, a common complication after gastric surgery because of the fact that the clients are obese and tend to ambulate less than other surgical clients. The nurse should inform the physician of the finding. Pain at the surgical site should be investigated, but the red, swollen leg is a higher priority. It isn't unusual for a client to be nauseated after gastric bypass surgery. The nurse should follow up with the finding, but only after she has notified the physician about the possible thromboembolism. Passing gas is normal and a sign that the client's intestinal system is beginning to mobilize.

A client is scheduled for an esophagogastroduodenoscopy (EGD) to detect lesions in the gastrointestinal tract. The nurse would observe for which of the following while assessing the client during the procedure? a) Signs of perforation b) Gag reflex c) Client's tolerance for pain and discomfort d) Client's ability to retain the barium

C) Client's tolerance for pain and discomfort The nurse has to assess the client's tolerance for pain and discomfort during the procedure. The nurse should assess the signs of perforation and the gag reflex after the procedure of EGD and not during the procedure. Assessing the client's level for retaining barium is important for a diagnostic test that involves the use of barium. EGD does not involve the use of barium.

A client is scheduled for several diagnostic tests to evaluate her gastrointestinal function. After teaching the client about these tests, the nurse determines that the client has understood the teaching when she identifies which test as not requiring the use of a contrast medium? a) Computer tomography b) Small bowel series c) Colonoscopy d) Upper GI series

C) Colonoscopy A colonoscopy is a direct visual examination of the entire large intestine. It does not involve the use of a contrast agent. Contrast medium may be used with a small bowel series, computed tomography, and upper GI series.

A client with a gastrojejunostomy is beginning to take solid food. Which finding would lead the nurse to suspect that the client is experiencing dumping syndrome? a) Slowed heart beat b) Hyperglycemia c) Diarrhea d) Dry skin

C) Diarrhea Clients with a gastrojejunostomy are at risk for developing the dumping syndrome when they begin to take solid food. This syndrome produces weakness, dizziness, sweating, palpitations, abdominal cramps, and diarrhea, which result from the rapid emptying (dumping) of large amounts of hypertonic chyme (a liquid mass of partly digested food) into the jejunum. This concentrated solution in the gut draws fluid from the circulating blood into the intestine, causing hypovolemia. The drop in blood pressure can produce syncope. As the syndrome progresses, the sudden appearance of carbohydrates in the jejunum stimulates the pancreas to secrete excessive amounts of insulin, which in turn causes hypoglycemia.

A client is scheduled for magnetic resonance imaging (MRI). During the client teaching, the nurse will discuss which of the following? a) "The examination will take only 15 minutes." b) "You must be NPO for the day before the examination." c) "Do you experience any claustrophobia?" d) "You must remove all jewelry but can wear your wedding ring."

C) Do you experience any claustrophobia? MRI is a noninvasive technique that uses magnetic fields and radio waves to produce images of the area being studied. Clients must be NPO for 6 to 8 hours before the study and remove all jewelry and other metals. The examination takes 60 to 90 minutes and can induce feelings of claustrophobia, because the scanner is close fitting.

When assessing a patient's abdomen, what would be most appropriate for the nurse to do? A) Palpate the abdomen before auscultation. B) Percuss the abdomen before auscultation. C) Auscultate the abdomen before palpation. D) Perform deep palpation before light palpation.

C) During examination of the abdomen, auscultation is done before percussion and palpation because these latter procedures may alter the bowel sounds.

A female patient has a sliding hiatal hernia. What nursing interventions will prevent the symptoms of heartburn and dyspepsia that she is experiencing? A) Keep the patient NPO. B) Put the bed in the Trendelenberg position. C) Have the patient eat 4 to 6 smaller meals each day. D) Give various antacids to determine which one works for the patient.

C) Eating smaller meals during the day will decrease the gastric pressure and the symptoms of hiatal hernia. Keeping the patient NPO or in a Trendelenberg position are not safe or realistic for a long period of time for any patient. Varying antacids will only be done with the care provider's prescription, so this is not a nursing intervention.

The health care team is assessing a male patient for acute pancreatitis after he presented to the emergency department with severe abdominal pain. Which laboratory value is the best diagnostic indicator of acute pancreatitis? A) Gastric pH B) Blood glucose C) Serum amylase D) Serum potassium

C) Elevated serum amylase levels indicate early pancreatic dysfunction and are used to diagnose acute pancreatitis. Serum lipase levels stay elevated longer than serum amylase in acute pancreatitis. Blood glucose, gastric pH, and potassium levels are not direct indicators of acute pancreatic dysfunction.

A client is admitted to the healthcare facility suspected of having acute pancreatitis and undergoes laboratory testing. Which of the following would the nurse expect to find? a) Decreased white blood cell count b) Decreased liver enzyme levels c) Elevated urine amylase levels d) Increased serum calcium levels

C) Elevated urine amylase levels Elevated serum and urine amylase, lipase, and liver enzyme levels accompany significant pancreatitis. If the common bile duct is obstructed, the bilirubin level is above normal. Blood glucose levels and white blood cell counts can be elevated. Serum electrolyte levels (calcium, potassium, and magnesium) are low.

The nurse determines that a patient has experienced the beneficial effects of medication therapy with famotidine (Pepcid) when which of the following symptoms is relieved? A) Nausea B) Belching C) Epigastric pain D) Difficulty swallowing

C) Epigastric Pain Famotidine is an H2-receptor antagonist that inhibits parietal cell output of HCl acid and minimizes damage to gastric mucosa related to hyperacidity, thus relieving epigastric pain.

The nurse determines that a pnt has experienced the beneficial effects of medication therapy with famotidine (Pepcid) when which of the following symptoms is relieved? A) Nausea B) Belching C) Epigastric pain D) Difficulty swallowing

C) Epigastric pain Famotidine is an H2-receptor antagonist that inhibits parietal cell output of HCl acid and minimizes damage to gastric mucosa related to hyperacidity, thus relieving epigastric pain.

Which of the following would be the least important assessment in a patient diagnosed with ascites? a) Measurement of abdominal girth b) Palpation of abdomen for a fluid shift c) Foul-smelling breath d) Weight

C) Foul smelling breath Foul-smelling breath would not be considered an important assessment for this patient. Measurement of abdominal girth, weight, and palpation of the abdomen for a fluid shift are all important assessment parameters for the patient diagnosed with ascites.

A nurse is preparing a client for surgery. During preoperative teaching, the client asks where is bile stored. The nurse knows that bile is stored in the: a) Cystic duct b) Duodenum c) Gallbladder d) Common bile duct

C) Gallbladder The gallbladder functions as a storage depot for bile.

Which of the following surgical procedures for obesity utilizes a prosthetic device to restrict oral intake? a) Vertical-banded gastroplasty b) Roux-en-Y gastric bypass c) Gastric banding d) Biliopancreatic diversion with duodenal switch

C) Gastric banding In gastric banding, a prosthetic device is used to restrict oral intake by creating a small pouch of 10 to 15 milliliters that empties through the narrow outlet into the remainder of the stomach. Roux-en-Y gastric bypass uses a horizontal row of staples across the fundus of the stomach to create a pouch with a capacity of 20 to 30 mL. Vertical-banded gastroplasty involves placement of a vertical row of staples along the lesser curvature of the stomach, creating a new, small gastric pouch. Biliopancreatic diversion with duodenal switch combines gastric restriction with intestinal malabsorption.

The nurse is caring for a man who has experienced a spinal cord injury. Throughout his recovery, the client expects to gain control of his bowels. The nurse's best response to this client would be which of the following? a) "Over time, the nerve fibers will regrow new tracts, and you can have bowel movements again." b) "Wearing an undergarment will become more comfortable over time." c) "Having a bowel movement is a spinal reflex requiring intact nerve fibers. Yours are not intact." d) "It is not going to happen. Your nerve cells are too damaged."

C) Having a bowel movement is a spinal reflex requiring intact nerve fibers. Yours are not intact The act of defecation is a spinal reflex involving the parasympathetic nerve fibers. Normally, the external anal sphincter is maintained in a state of tonic contraction. With a spinal cord injury, the client no longer has this nervous system control and is often incontinent.

A nurse is preparing a presentation for a local community group about hepatitis. Which of the following would the nurse include? a) Hepatitis B is transmitted primarily by the oral-fecal route. b) Hepatitis A is frequently spread by sexual contact. c) Hepatitis C increases a person's risk for liver cancer. d) Infection with hepatitis G is similar to hepatitis A.

C) Hep C increases a person's risk for liver cancer Infection with hepatitis C increases the risk of a person developing hepatic (liver) cancer. Hepatitis A is transmitted primarily by the oral-fecal route; hepatitis B is frequently spread by sexual contact and infected blood. Hepatitis E is similar to hepatitis A whereas hepatitis G is similar to hepatitis C.

"The nurse is caring for a patient following an appendectomy. The patient takes a deep breath, coughs, and then winces in pain. Which of the following statements, if made by the nurse to the patient, is BEST? "A.) "Take three deep breaths, hold your incision, and then cough." B.) "That was good. Do that again and soon it won't hurt as much." C.) "It won't hurt as much if you hold your incision when you cough." D.) "Take another deep breath, hold it, and then cough deeply."

"(1) correct-most effective way of deep breathing and coughing, dilates airway and expands lung surface area (2) should splint incision before coughing to reduce discomfort and increase efficiency (3) partial answer, should take three deep breaths before coughing (4) implies coughing routine is adequate, incision needs to be splinted"

Which of the following statements are accurate as they relate to medications used to manage GERD? "A)Magnesium-containing antacids can cause diarrhea. B) Aluminum-containing antacids can cause constipation. C) Cimetidine (Tagamet HB) causes osteomalacia and hypophosphatemia. D)Misoprostol's (Cytotec) major side effect is G.I. bleeding

"A)Magnesium-containing antacids can cause diarrhea. B) Aluminum-containing antacids can cause constipation. Rationale: Magnesium-containing antacids can cause diarrhea, and should be used with caution in older persons with renal dysfunction. Aluminum-containing antacids can cause constipation, osteomalacia, and hypophosphatemia. Cimetidine has the greatest chance for adverse reactions, including erectile dysfunction, Gynecomastia, and confusion. Misoprostol's major side effects are diarrhea and abdominal pain

A client is admitted with right lower quadrant pain, anorexia, nausea, low-grade fever, and elevated white blood cell count. Which complication is most likely the cause? 1. A. fecalith 2. Bowel Kinking 3. Internal blowel occlusion 4. Abdominal wall swelling

"Answer 1 Rational: The client is experiencing appendicitis. A. fecalith is a fecal calculus, or stone, that occludes the lumen of the appendix and is the most common cause of appendicitis. Bowel wall swelling, kinking of the appendix, and external occlusion not internal occlusion, of the bowel by adhesions can also be cause of appendicitis."

"A client with appendicitis is experiencing excruciating abdominal pain. An abdominal X-ray film reveals intraperitoneal air. The nurse should prepare the client for: a) colonoscopy. b) surgery. c) nasogastric (NG) tube insertion. d) barium enema."

"B) Surgery The client should be prepared for surgery because his signs and symptoms indicate bowel perforation. Appendicitis is the most common cause of bowel perforation in the United States. Because perforation can lead to peritonitis and sepsis, surgery wouldn't be delayed to perform other interventions, such as colonoscopy, NG tube insertion, or a barium enema. These procedures aren't necessary at this point."

"During the assessment of a patient with acute abdominal pain, the nurse should: a. Perform deep palpation before ascultation b. Obtain blood pressure and pulse rate to determine hypervolemic changes c. Ascultate bowel sounds because hyperactive bowel sounds suggest paralytic ileus d. Measure body temperature because an elevated temperature may indicate an inflammatory or infectious process"

"Correct answer: d Rationale: For the patient complaining of acute abdominal pain, the nurse should take vital signs immediately. Increased pulse and decreasing blood pressure (BP) are indicative of hypovolemia. An elevated temperature suggests an inflammatory or infectious process. Intake and output measurements provide essential information about the adequacy of vascular volume. Inspect the abdomen first and then auscultate bowel sounds. Palpation is performed next and should be gentle."

An adult client is to have a sigmoidoscopy in the morning.What should the nurse plan to do? 1. Give him an enema 1 hour before the examination. 2. Keep him NPO for 8 hours before the examination. 3. Order a low fat, low residue diet for breakfast. 4. Administer enemas until clear this evening

(1) An enema 1 hour before the exam will clear the sigmoid colon. A client having an upper GI series will be NPO. Low fat diet is indicated prior to a gallbladder series. Low residue diet is part of the preparation for barium enema. Enemas until clear are sometime sordered prior to a barium enema

A barium enema is ordered for an adult male client. The nurse is teaching him what to expect regarding the procedure. Which statement should be included in the teaching? 1. Fecal matter must be cleansed from the bowel for good visualization. 2. There will be no food restrictions before the test .3. He will not have to change positions during the procedure. 4. He will be asked to drink barium during the procedure

(1) The bowel must be free of fecal material for good visualization of the bowel. He will be on a clear liquid or low residue diet for the day preceding the exam. The client is put in several positions during the test. Barium is given by enema. It is given by mouth in an upper GI series

An adult male client is admitted with a diagnosis of probable duodenal ulcer. Which of the following laboratory tests would it be most essential for the nurse to assess immediately? 1. Hemoglobin and Hematocrit 2. SGPT and SGOT 3. Na and K 4. BUN and creatinine

(1)Hgb and Hct would indicate if there had been any bleeding from the ulcer. SGPT and SGOT elevations indicate liver damage. Na and K indicate electrolyte imbalances. BUN and creatinine elevationswould indicate renal disease.

The nurse is preparing a client with Crohn's disease for discharge. Which statement he makes indicates he needs further teaching? 1. "Stress can make it worse." 2. "Since I have Crohn's disease I don't have to worry about colon cancer." 3. "I realize I shall always have to monitor my diet." 4. "I understand there is a high incidence of familial occurrence with this disease

(2) Persons with Crohn's disease are at high risk for the development of colon cancer. The other answers are all correct

The client asks how he contracted hepatitis A. He reports all of the following. Which one is most likely related to hepatitis A? 1. He ate home canned tomatoes. 2. He ate oysters his roommate brought home from a fishing trip. 3. He stepped on a nail 2 weeks ago. 4. He donated blood 2 weeks before he got sick

(2) Shellfish that grow in contaminated waters may have the virus. Home canned tomatoes might cause food poisoning. Stepping on a nail might cause tetanus.Donating blood will not cause hepatitis. Receiving blood might cause hepatitis B or C

An adult client is to have a gastroduodenoscopy in the morning.The nurse's instructions should include the information that he will be 1. given a general anesthetic during the procedure. 2. given a local anesthetic to ease the discomfort during the procedure. 3. asked to assist by coughing during the procedure. 4. asked to assist by performing a Valsalva maneuver during the procedure

(2)Gastroduodenoscopy is visualization of the esophagus, stomach and duodenal through a flexible tube inserted orally. The exam is uncomfortable because the muscles of the GI tract have spasms as the tube is passed. This causes difficulty swallowing. The client is usually given a local anesthetic to the posterior pharynx to reduce the discomfort during the passage of the tube.He may also be given conscious sedation. He will not given a general anesthetic because he must be able to assist by swallowing. Coughing and the performance of a Valsalva maneuver would impede the passage of the tube.

Which nursing intervention is essential immediately following a gastroduodenoscopy? 1. Force fluids. 2. Position him supine. 3. Instruct him not to eat or drink. 4. Encourage coughing and deep breathing

(3) It is essential to keep him NPO until the cough and gag reflexes have returned. He should be in a semi-Fowler's position to reduce edema formation

"Which assessment data support the client's diagnosis of gastric ulcer? 1.Presence of blood in the client's stool for the past month. 2.Complaints of a burning sensation that moves like a wave. 3.Sharp pain in the upper abdomen after eating a heavy meal. 4 .Comparison of complaints of pain with ingestion of food and sleep

(4) In a client diagnosed with a gastric ulcer,pain usually occurs 30-60 minutes after eating, but not at night. In contrast, a client with a duodenal ulcer has pain during thenight that is often relieved by eating food.Pain occurs 1-3 hours after meals

"The nurse is caring for an adult client diagnosed with gastroesophageal reflux disease(GERD). Which condition is the most common comorbid disease associated with GERD? 1.Adult-onset asthma. 2.Pancreatitis. 3.Peptic ulcer disease. 4.Increased gastric emptying

*1. Of adult-onset asthma cases, 80%-90% are caused by gastroesophageal reflux disease(GERD)* 2. Pancreatitis is not related to GERD 3. Peptic ulcer disease is related to H. pylori bacterial infections and can lead to increased levelsof gastric acid, but it is not related to reflux. 4.GERD is not related to increased gastricemptying. Increased gastric emptying would bea benefit to a client with decreased functioningof the lower esophageal sphincter

Hepatic encephalopathy develops when the blood level of which substance increases? 1. Ammonia 2. Amylase 3. Calcium 4. Potassium

1 Ammonia levels increase d/t improper shunting of blood, causing ammonia to enter systemic circulation, which carries it to the brain.

You're caring for Betty with liver cirrhosis. Which of the following assessment findings leads you to suspect hepatic encephalopathy in her? 1. Asterixis 2. Chvostek's sign 3. Trousseau's sign 4. Hepatojugular reflex

1 Asterixis is an early neurologic sign of hepatic encephalopathy elicited by asking the patient to hold her arms stretched out. Asterixis is present if the hands rapidly extend and flex.

You are developing a care plan on Sally, a 67 y.o. patient with hepatic encephalopathy. Which of the following do you include? 1. Administering a lactulose enema as ordered. 2. Encouraging a protein-rich diet. 3. Administering sedatives, as necessary. 4. Encouraging ambulation at least four times a day.

1 You may administer the laxative lactulose to reduce ammonia levels in the colon.

The nurse caring for a client diagnosed with GERD writes the client problem of "behavior modification." which intervention should be included for this problem? 1. the client should elevate the head of the bed on blocks or use a foam wedge to use gravity to help keep the gastric acid in the stomach and prevent reflux into the esophagus. behavior modification is changing one's behavior. 2. encourage the client to decrease the amount of smoking. 3. instruct the client to take over the counter medication for relief of pain. 4. discuss the need to attend alcoholics anonymous to quit drinking

1 (correct): the client should elevate the hdad of the bed on blocks or use afoam wedge to use gravity to help keep the grastric acid in the stomach and prevent reflux into the esophagus. behavior modification is changing one's behavior.

The nurse is performing an admission assessment on a client diagnosed with gastroesophageal reflux disease (GERD). Which signs and symptoms would indicate GERD? 1. Pyrosis, water brash, and flatulence 2. Weight loss, dysarthria, and diarrhea 3. Decreased abdominal fat, proteinuria, and constipation 4. Mid-epigastric pain, positive H. pylori test, and melena

1 (pyrosis, water brash, and flatulence)1. Pyrosis is heartburn, water brash is the feeling of saliva secretion as a result of reflux, and flatulence is gas—all symptoms of GERD 2. Gastroesophageal reflux disease does not cause weight loss 3. There is no change in abdominal fat, no proteinuria (the result of a filtration problem inthe kidney), and no alteration in bowel elimination for the client diagnosed with GERD 4. Mid-epigastric pain, a positive H. pylori test, and melena are associated with gastric ulcer disease

You're caring for Jane, a 57 y.o. patient with liver cirrhosis who developed ascites and requires paracentesis. Before her paracentesis, you instruct her to: 1. Empty her bladder. 2. Lie supine in bed. 3. Remain NPO for 4 hours. 4. Clean her bowels with an enema.

1 A full bladder can interfere with paracentesis and be punctured inadvertently

The client with hiatal hernia chronically experiences heartburn following meals. The nurse plans to teach the client to avoid which action because it is contraindicated with a hiatal hernia? 1. Lying recumbent following meals. 2. Taking in small, frequent bland meals. 3. Raising the head of the bed on 6-inch block. 4. Taking H2-receptor antagonist medication

1 Hiatal hernia is caused by a protrusion of a portion of the stomach above the diaphragm where the esophagus is normally positioned. he client usually experiences pain from reflux caused by ingestion of irritating foods, lying flat following meals or at night, and eating large or fatty meals. Option 2-4, and actually elevating the thorax after a meal, provide relief

The client with a hiatal hernia chronically experiences heartburn following meals. The nurse planc to teach the client to avoid which action because it is contraindicated with hiatal hernia? 1. Lying recumbent following meals 2. Taking in small, frequent, bland meals 3. Raising the head of the bed on 6-inch blocks 4. Taking H2-receptor antagonist medication

1 Laying recumbant following meals or at night will cause reflux and pain. Relief is usually achieved with the intake of small, bland meals, use of H2 receptor antagonists and antacids, and elevation of the thorax after meals and during sleep

Symptoms indicating progression into hepatic coma include: 1. flapping tremor 2. nystagmus 3. fruity odor breath 4. fetid breath

1 and 4

In preparation for an abdomino-perineal resection the client is placed on a low residue diet. Which of the following food lists is appropriate for him to eat on a low residue diet? 1. Ground lean beef, soft boiled eggs, tea. 2. Lettuce, spinach, corn. 3. Prunes, grapes, apples. 4. Bran cereal, whole wheat toast, coffee

1)All of these foods are low in residue. Fruits,vegetables and whole grains are high in residue.

A patient diagnosed with viral hepatitis is in the pre-icteric phase. When assessing the patient, which of these findings should the healthcare provider anticipate? Select all that apply. Select all that apply. 1. Nausea 2. Pruritis 3. Tarry stools 4. Anorexia 5. Dark urine

1, 4

A nurse is assessing a client who has been admitted with a diagnosis of an obstruction in the small intestine. The nurse should assess the client for? Select all that apply. 1. Projectile vomiting. 2. Significant abdominal distention. 3. Copious diarrhea. 4. Rapid onset of dehydration. 5. Increased bowel sounds.

1, 4, 5. Signs and symptoms of intestinal obstructions in the small intestine may include projectile vomiting and rapidly developing dehydration and electrolyte imbalances. The client will also have increased bowel sounds, usually high-pitched and tinkling. The client would not normally have diarrhea and would have minimal abdominal distention. Pain is intermittent, being relieved by vomiting. Intestinal obstructions in the large intestine usually evolve slowly, produce persistent pain, and vomiting is less common. Clients with a large-intestine obstruction may develop constipation and significant abdominal distention.

The client with hiatal hernia chronically experiences heartburn following meals. The nurses plans to teach the client to avoid which action because it is contraindicated with a hiatal hernia? 1. Lying recumbent following meals 2. Taking in small, frequent, bland meals 3. Raising the head of bed on 6-inch blocks 4. Taking H2-receptor antagonist medication

1, Hiatal hernia is caused by a protrusion of a portion of the stomach above the diaphragm where the esophagus usually is positioned. The client usually experiences pain from reflux caused by ingestion of irritating foods, lying flat following meals or at night, and eating large or fatty meals. Relief is obtained with the intake of small, frequent meals, use of H2-receptor antagonists and antacids, and elevation of the thorax following meals and during sleep

A patient has a vagotomy with antrectomy to treat a duodenal ulcer. Postoperatively, the patient develops dumping syndrome. Which of the following statements, if made by the patient, should indicate to the nurse that further dietary teaching is needed? 1. I should eat bread with each meal 2. I should eat smaller meals more frequently. 3. I should lie down after eating. 4. I should avoid drinking fluids with my meals

1, Patient should decrease intake of carbohydrates

A client is admitted with a bowel obstruction. The client has nausea, vomiting, and crampy abdominal pain. The physician has written orders for the client to be up ad lib, to have narcotics for pain, to have a nasogastric tube inserted if needed, and for I.V. Ringer's Lactate and hyperalimentation fluids. The nurse should do the following in order of priority from first to last: 1. Assist with ambulation to promote peristalsis 2. Administer Ringer's Lactate 3. Insert a nasogastric tube. 4. Start and infusion of hyperalimentation fluids

1,2,3,4 The nurse should first help the client ambulate to try to induce peristalsis; this may be effective and require the least amount of invasive procedures. I.V. fluid therapy can be done to correct fluid and electrolyte imbalances (sodium and potassium), and normal saline or Ringer's Lactate to correct interstitial fluid deficit. Nasogastric (NG) decompression of G.I. tract to reduce gastric secretions and nasointestinal tubes may also be used. Hyperalimentation can be used to correct protein deficiency from chronic obstruction, paralytic ileus, or infection.

The physician orders intestinal decompression with a Cantor tube for a client with an intestinal obstruction. In order to determine effectiveness of intestinal decompression the nurse should evaluate the client to determine if: 1. Fluid and gas have been removed from the intestine. 2. The client has had a bowel movement. 3. The client's urinary output is adequate. 4. The client can sit up without pain.

1. Intestinal decompression is accomplished with a Cantor, Harris, or Miller-Abbott tube. These 6- to 10-foot tubes are passed into the small intestine to the obstruction. They remove accumulated fluid and gas, relieving the pressure. The client will not have an adequate bowel movement until the obstruction is removed. The pressure from the distended intestine should not obstruct urinary output. While the client may be able to more easily sit up, and the pain caused by the intestinal pressure will be less, these are not the primary indicators for successful intestinal decompression.

One hour before a client is to undergo abdominal surgery, the physician orders atropine, 0.3 mg I.M. The client asks the nurse why this drug must be administered. How should the nurse respond?

1. "Atropine decreases salivation and gastric secretions."

A client admitted for treatment of a gastric ulcer is being prepared for discharge on antacid therapy. Discharge teaching should include which instruction?

1. "Continue to take antacids, even if your symptoms subside."

A client with extreme weakness, pallor, weak peripheral pulses, and disorientation is admitted to the emergency department. His wife reports that he has been "spitting up blood." A Mallory-Weiss tear is suspected, and the nurse begins taking a client history from the client's wife. The question by the nurse that demonstrates her understanding of Mallory-Weiss tearing is:

1. "Tell me about your husband's alcohol usage."

A nurse is assigned to care for four clients. Which client should a nurse assess first?

1. A postoperative client who just returned from surgery and is vomiting

A client is evaluated for severe pain in the right upper abdominal quadrant, which is accompanied by nausea and vomiting. The physician diagnoses acute cholecystitis and cholelithiasis. For this client, which nursing diagnosis takes top priority?

1. Acute pain related to biliary spasms

When preparing a client for a hemorrhoidectomy, the nurse should take which action?

1. Administer an enema as ordered.

While obtaining a client's medication history, the nurse learns that the client takes ranitidine (Zantac), as prescribed, to treat a peptic ulcer. The nurse continues gathering medication history data to assess for potential drug interactions. The nurse should instruct the client to avoid taking a drug from which class with ranitidine?

1. Antacids

To verify the placement of a gastric feeding tube, the nurse should perform at least two tests. One test requires instilling air into the tube with a syringe and listening with a stethoscope for air passing into the stomach. What is another test method?

1. Aspiration of gastric contents and testing for a pH less than 6

A client, age 82, is admitted to an acute care facility for treatment of an acute flare-up of a chronic GI condition. In addition to assessing the client for complications of the current illness, the nurse monitors for age-related changes in the GI tract. Which age-related change increases the risk of anemia?

1. Atrophy of the gastric mucosa

When assessing the client with the diagnosis of peptic ulcer disease, which physical examination should the nurse implement first? 1. Auscultate the client's bowel sounds in all four quadrants. 2.Palpate the abdominal area for tenderness. 3.Percuss the abdominal borders to identify organs. 4.Assess the tender area progressing to nontender

1. Auscultation should be used prior to palpa-tion or percussion when assessing the abdomen. If the nurse manipulates the abdomen, the bowel sounds can be altered and give false information

The physician orders morphine for a client who complains of postoperative abdominal pain. For maximum pain relief, when should the nurse anticipate administering morphine?

1. Before the pain becomes severe

A client with inflammatory bowel disease undergoes an ileostomy. On the first day after surgery, the nurse notes that the client's stoma appears dusky. How should the nurse interpret this finding?

1. Blood supply to the stoma has been interrupted.

Which nursing intervention should the nurse perform for a client receiving enteral feedings through a gastrostomy tube?

1. Change the tube feeding solutions and tubing at least every 24 hours.

Which infections require contact precautions?

1. Clostridium difficile, 3. Methicillin-resistant staphylococcus aureus

What is the primary nursing diagnosis for a client with a bowel obstruction?

1. Deficient fluid volume

A client with mild diarrhea, fever, and abdominal discomfort is being evaluated for inflammatory bowel disease (IBD). Which statement about IBD is true?

1. Diarrhea is the most common sign of IBD.

Which diagnostic test would be used first to evaluate a client with acute upper GI bleeding?

1. Endoscopy

A client is preparing to undergo abdominal paracentesis. Which nursing interventions should be performed before the procedure?

1. Explain the procedure to the client., 2. Make sure informed consent was obtained., 3. Instruct the client to void.

The nurse is teaching a client how to irrigate his stoma. Which action indicates that the client needs more teaching?

1. Hanging the irrigation bag 24" to 36" (60 to 90 cm) above the stoma

While a client is being prepared for discharge, the nasogastric (NG) feeding tube becomes clogged. To remedy this problem and teach the client's family how to handle it at home, what should the nurse do?

1. Irrigate the tube with cola

A client is receiving a cleansing enema. During the procedure, the client reports abdominal cramping. What should the nurse do?

1. Lower the fluid bag so that the instillation slows.

A nurse is assigned the care of six clients and has the aid of a nursing assistant. Which task is appropriate for the nurse to delegate to the nursing assistant?

1. Measuring and recording nasogastric tube output

A client undergoes total gastrectomy. Several hours after surgery, the nurse notes that the client's nasogastric (NG) tube has stopped draining. How should the nurse respond?

1. Notify the physician.

A nurse is caring for a client with an ileostomy. What is the most common complication of this procedure?

1. Peristomal skin irritation

When caring for a client who has had constipation for 4 days, what should be the nurse's primary client care concern?

1. Promoting defecation

A client is admitted to the health care facility with abdominal pain, a low-grade fever, abdominal distention, and weight loss. The physician diagnoses acute pancreatitis. What is the primary goal of nursing care for this client?

1. Relieving abdominal pain

Why are antacids administered regularly, rather than as needed, to treat peptic ulcer disease?

1. To keep gastric pH at 3.0 to 3.5

A client comes to the emergency department with suspected cholecystitis. Which data collection findings are characteristic of this diagnosis?

1. Transient epigastric pain radiating to the back and right shoulder, 2. Burning in the chest after eating fried foods, 3. Flatulence, 4. Nausea

When collecting data on a client during a routine checkup, the nurse reviews the history and notes that the client had aphthous stomatitis at the time of the last visit. Aphthous stomatitis is best described as:

1. a canker sore of the oral soft tissues.

A client had a nephrectomy 2 days ago and is now complaining of abdominal pressure and nausea. The first nursing action should be to:

1. auscultate bowel sounds.

The nurse is caring for a client with a colostomy. The client tells the nurse that he makes small pin holes in the drainage bag to help relieve gas. The nurse should teach him that this action:

1. destroys the odor-proof seal.

A client who can't tolerate oral feedings begins receiving intermittent enteral feedings. When monitoring for evidence of intolerance to these feedings, the nurse must stay alert for:

1. diaphoresis, vomiting, and diarrhea.

A client with severe inflammatory bowel disease is receiving total parenteral nutrition (TPN). When monitoring TPN, the nurse must take care to maintain the prescribed flow rate because giving TPN too rapidly may cause:

1. hyperglycemia.

While preparing a client for cholecystectomy, the nurse explains that incentive spirometry will be used after surgery primarily to:

1. increase respiratory effectiveness.

A client is recovering from an ileostomy that was performed to treat inflammatory bowel disease. During discharge teaching, the nurse should stress the importance of:

1. increasing fluid intake to prevent dehydration.

A client with amebiasis, an intestinal infection, is prescribed metronidazole (Flagyl). When teaching the client about adverse reactions to this drug, the nurse should mention:

1. metallic taste.

A 32-year-old male client with appendicitis is experiencing severe abdominal pain. An abdominal X-ray film reveals intraperitoneal air. The nurse should prepare the client for:

1. surgery.

A client with cholelithiasis has a gallstone lodged in the common bile duct. When assessing this client, the nurse expects to note:

1. yellow sclerae.

What is the primary concern of the nurse administering narcotics and sedatives to a client with cirrhosis? 1. The kidneys cannot excrete the drug. 2. The liver cannot metabolize drugs effectively. 3. Hepatic enzymes potentiate these drugs to high levels. 4. Narcotics and sedatives are contraindicated in clients with cirrhosis.

2

Develop a teaching care plan for Angie who is about to undergo a liver biopsy. Which of the following points do you include? 1. "You'll need to lie on your stomach during the test." 2. "You'll need to lie on your right side after the test." 3. "During the biopsy you'll be asked to exhale deeply and hold it." 4. "The biopsy is performed under general anesthesia."

2 After a liver biopsy, the patient is placed on the right side to compress the liver and to reduce the risk of bleeding or bile leakage.

Ralph has a history of alcohol abuse and has acute pancreatitis. Which lab value is most likely to be elevated? 1. Calcium 2. Glucose 3. Magnesium 4. Potassium

2 Glucose level increases and diabetes mellitus may result d/t the pancreatic damage to the islets of langerhans.

Stephanie, a 28 y.o. accident victim, requires TPN. The rationale for TPN is to provide: 1. Necessary fluids and electrolytes to the body. 2. Complete nutrition by the I.V. route. 3. Tube feedings for nutritional supplementation. 4. Dietary supplementation with liquid protein given between meals.

2 TPN is given I.V. to provide all the nutrients your patient needs. TPN isn't a tube feeding nor is it a liquid dietary supplement.

You're patient is complaining of abdominal pain during assessment. What is your priority? 1. Auscultate to determine changes in bowel sounds. 2. Observe the contour of the abdomen. 3. Palpate the abdomen for a mass. 4. Percuss the abdomen to determine if fluid is present.

2 The first step in assessing the abdomen is to observe its shape and contour, then auscultate, palpate, and then percuss.

The nurse determines that a patient has experienced the beneficial effects of medication therapy with famotidine (Pepcid) when which of the following symptoms is relieved? 1. Ice tea 2. Dry toast 3. warm broth 4. plain hamburger

2 Dry toast (Dry toast or crackers may alleviate the feeling of nausea and prevent further vomiting. Extremely hot or cold liquids and fatty foods are generally not well tolerated

A client is admitted to the hospital with ulcerative colitis.Admitting orders include a low residue diet. Which food would be contraindicated for this client? . Roast beef. 2. Fresh peas .3. Mashed potatoes .4. Baked chicken

2) Fresh peas are high in residue. The other foods are low in residue

The client with a duodenal ulcer is ready for discharge. Which statement made by the client indicates a need for more teaching about his diet? 1. "It's a good thing I gave up drinking alcohol last year." 2. "I will have to drink lots of milk and cream every day." 3. "I will stay away from cola drinks after I am discharged." 4. "Eating three nutritious meals and snacks every day is okay.

2) Milk and cream are now known to cause rebound acidity and are not prescribed for ulcer clients.The other choices all indicate good knowledge. He should not drink alcohol or cola. Three meals and snacks will help to keep the stomach from staying empty for long periods

The client is diagnosed with an acute exacerbation of ulcerative colitis. Which inter- vention should the nurse implement? 1. Provide a low-residue diet. 2.Monitor intravenous fluids. 3.Assess vital signs daily. 4.Administer antacids orally

2, "1. The client's bowel should be placed on rest andno foods or fluids should be introduced intothe bowel. 2. (Correct) The client requires fluids to help prevent dehydration from diarrhea and to replacethe fluid lost through normal body func-tioning. 3.The vital signs must be taken more often thandaily in a client who is having an acute exacer-bation of ulcerative colitis. 4.The client will receive anti-inflammatory andantidiarrheal medications, not antacids, whichare used for gastroenteritis

Which of the following statements about nasoenteric tubes is correct? 1. The tube cannot be attached to suction. 2. The tube contains a soft rubber bag filled with mercury. 3. The tube is taped securely to the client's cheek after insertion. 4. The tube can have its placement determined only by auscultation

2. A nasoenteric tube has a small balloon at its tip that is weighted with mercury. The weight of the mercury helps advance the tube by gravity through the intestine. Nasoenteric tubes are attached to suction. A nasoenteric tube is not taped in position until it has reached the obstruction. Because the tube has a radiopaque strip, its progress through the intestinal tract can be followed by fluoroscopy.

After insertion of a nasoenteric tube, the nurse should place the client in which position? 1. Supine. 2. Right side-lying. 3. Semi-Fowler's. 4. Upright in a bedside chair.

2. The client is placed in a right side-lying position to facilitate movement of the mercury-weighted tube through the pyloric sphincter. After the tube is in the intestine, the client is turned from side to side or encouraged to ambulate to facilitate tube movement through the intestinal loops. Placing the client in the supine or semi-Fowler's position, or having the client sitting out of bed in a chair will not facilitate tube progression

The client with an intestinal obstruction continues to have acute pain even though the nasoenteric tube is patent and draining. Which action by the nurse would be most appropriate? 1. Reassure the client that the nasoenteric tube is functioning. 2. Assess the client for a rigid abdomen. 3. Administer an opioid as ordered. 4. Reposition the client on the left side.

2. The client's pain may be indicative of peritonitis, and the nurse should assess for signs and symptoms, such as a rigid abdomen, elevated temperature, and increasing pain. Reassuring the client is important, but accurate assessment of the client is essential. The full assessment should occur before pain relief measures are employed. Repositioning the client to the left side will not resolve the pain.

To prevent gastroesophageal reflux in a client with hiatal hernia, the nurse should provide which discharge instruction?

2. "Avoid coffee and alcoholic beverages."

The nurse teaches the client about an anti-ulcer diet. Which of the following statements by the client indicates to the nurse that dietary teaching was successful? 1. "I must eat bland foods to help my stomach heal." 2. "I can eat most foods, as long as they don't bother my stomach." 3. "I cannot eat fruits and vegetables because they cause too much gas." 4. "I should eat a low-fiber diet to delay gastric emptying -

2. "I can eat most foods, as long as they don't bother my stomach."

The nurse is teaching an elderly client about good bowel habits. Which statement by the client would indicate to the nurse that additional teaching is required?

2. "I need to use laxatives regularly to prevent constipation."

A client who is about to undergo gastric bypass surgery calls the nurse into the room. The client says she's concerned that friends will learn about her upcoming surgery. She pleads with the nurse to keep her surgery a secret. Which response by the nurse is best?

2. "I'm not at liberty to discuss your case with anyone except those directly involved in your care unless you authorize me to do so."

After admission for acute appendicitis, a client undergoes an appendectomy. He complains of moderate postsurgical pain for which the physician prescribes pentazocine (Talwin), 50 mg by mouth every 4 hours. How soon after administration of this drug can the nurse expect the client to feel relief?

2. 15 to 30 minutes

The nurse must administer an enema to an adult client. The appropriate distance for inserting an enema into an average-sized adult is:

2. 3" to 4".

When a client resumes oral feedings after having gastric resection, the nurse watches for early manifestations of dumping syndrome. The vasomotor disturbances associated with this syndrome usually occur how soon after eating?

2. 5 to 30 minutes

The nurse is monitoring a client receiving paregoric to treat diarrhea for drug interactions. Which drugs can produce additive constipation when given with an opium preparation?

2. Anticholinergic drugs

A client undergoes a barium swallow fluoroscopy that confirms gastroesophageal reflux disease (GERD). Based on this diagnosis, the client should be instructed to take which action?

2. Avoid caffeine and carbonated beverages., 4. Stop smoking., 5.Take antacids 1 hour and 3 hours after meals.

"The nurse is caring for the client diagnosed with chronic gastritis. Which symptom(s) would support this diagnosis? 1. Rapid onset of mid-sternal discomfort. 2. Epigastric pain relieved by eating food 3. Dyspepsia and hematemesis. 4. Nausea and projectile vomiting

2. Chronic pain in the epigastric area that is relieved by ingesting food is a sign of chronic gastritis (CORRECT). "Rationale by answer: 1. Acute gastritis is characterized by sudden epigastric pain or discomfort, not mid-sternal chest pain. 2. Chronic pain in the epigastric area that is relieved by ingesting food is a sign of chronic gastritis (CORRECT). 3. Dyspepsia (heartburn) and hematemesis (vomiting blood) are frequent symptoms of acute gastritis. 4. Projective vomiting is not a sign of chronic gastritis

A 72-year-old client seeks help for chronic constipation. This is a common problem for elderly clients due to several factors related to aging. Which of the following is one such factor?

2. Decreased abdominal strength

The nurse is developing a plan of care for a client with hepatitis A. What is the main route of transmission of this hepatitis virus?

2. Feces

One day after undergoing a traditional cholecystectomy, a client is scheduled to stand at the bedside and walk. What should a nurse teach the client to do before standing and walking for the first time after surgery?

2. Flex her legs when moving to a sitting position.

"Which statement made by the client indicates to the nurse the client may be experiencing GERD? 1. "My chest hurts when I walk up the stairs in my home." 2. "I take antacid tablets with me wherever I go." 3. My spouse tells me I snore very loudly at night." 4. I drink six (6) to seven (7) soft drinks every day

2. Frequent use of antacids indicates an acid reflux problem.

An 86-year-old client with a history of atrial fibrillation takes 5 mg of warfarin (Coumadin) daily. Warfarin therapy makes the client at risk for which complications?

2. Hemorrhage, 3. Hepatitis, 5. Hematuria

A 58-year-old client with osteoarthritis is admitted to the hospital with peptic ulcer disease. Which findings are commonly associated with peptic ulcer disease?

2. History of nonsteroidal anti-inflammatory drug (NSAID) use, 3. Epigastric pain that's relieved by antacids, 5. Nausea and weight loss

The nurse is assessing a client who is receiving total parenteral nutrition (TPN). Which finding suggests that the client has developed hyperglycemia?

2. Increased urine output

The nurse is assessing a client who complains of abdominal pain, nausea, and diarrhea. When examining the client's abdomen, which sequence should the nurse use?

2. Inspection, auscultation, percussion, and palpation

A client takes 30 ml of magnesium hydroxide and aluminum hydroxide with simethicone (Maalox TC) by mouth 1 hour and 3 hours after each meal and at bedtime for treatment of a duodenal ulcer. Why does the client take this antacid so frequently?

2. It has a short duration of action.

A client seeks medical attention after developing acute abdominal pain. Which action by the nurse would help ensure accurate auscultation of the client's bowel sounds?

2. Making sure the client's bladder is empty before auscultating

The male client tells the nurse he has been experiencing "heartburn" at night that awakens him. Which assessment question should the nurse ask? " 1. "How much weight have you gained recently?" 2. "What have you done to alleviate the heartburn?" 3. "Do you consume many milk and dairy products?" 4. "Have you been around anyone with a stomach virus?

2. Most clients with GERD have been self- medicating with over-the-counter medications prior to seeking advice from a health-care provider. It is important to know what the client has been using to treat the problem. 1. Clients with heartburn are frequently diagnosed as having GERD. GERD can occasionally cause weight loss, but not weight gain. 2. Most clients with GERD have been self- medicating with over-the-counter medications prior to seeking advice from a health-care provider. It is important to know what the client has been using to treat the problem. 3. Milk and dairy products contain lactose, which are important if considering lactose intolerance, but are not important for "heartburn." 4. Heartburn is not a symptom of a viral illness

You're performing an abdominal assessment on Brent who is 52 y.o. In which order do you proceed? 1. Observation, percussion, palpation, auscultation 2. Observation, auscultation, percussion, palpation 3. Percussion, palpation, auscultation, observation 4. Palpation, percussion, observation, auscultation

2. Observation, auscultation, percussion, palpation

A client with severe abdominal pain is being evaluated for appendicitis. What is the most common cause of appendicitis?

2. Obstruction of the appendix

When preparing a client, age 50, for surgery to treat appendicitis, the nurse assists in formulating a nursing diagnosis of Risk for infection related to inflammation, perforation, and surgery. What is the rationale for choosing this nursing diagnosis?

2. Obstruction of the appendix reduces arterial flow, leading to ischemia, inflammation, and rupture of the appendix

Which condition is most likely to have a nursing diagnosis of Deficient fluid volume?

2. Pancreatitis

When caring for a client with acute pancreatitis, the nurse should use which comfort measure?

2. Positioning the client on the side with the knees flexed

A client is scheduled for an endoscopy. On admission, the nurse asks the client if he has an advance directive, and the client states, "No." What should the nurse do next?

2. Provide the client with information about an advance directive.

As part of a routine screening for colorectal cancer, a client must undergo fecal occult blood testing. Which foods should the nurse instruct the client to avoid 48 to 72 hours before the test and throughout the collection period?

2. Red meat,. 3. Turnips, 4. Horseradish

A client is scheduled for bowel resection with anastomosis involving the large intestine. Because of the surgical site, the nurse assists in formulating the nursing diagnosis of Risk for infection. To complete the nursing diagnosis statement, which "related-to" phrase should be added?

2. Related to the presence of bacteria at the surgical site

After undergoing a liver biopsy, the client should be placed in which position?

2. Right lateral decubitus position

As a result of a viral infection, a client develops gastroenteritis. The physician prescribes kaolin and pectin mixture (Kaopectate), 60 ml by mouth after each loose bowel movement, up to eight doses daily. The client asks the nurse how soon the medication will take effect. How should the nurse respond?

2. Within 30 minutes

A client has just been diagnosed with hepatitis A. On assessment, the nurse expects to note:

2. anorexia, nausea, and vomiting.

The nurse is caring for a client with an endotracheal tube who receives enteral feedings through a feeding tube. Before each tube feeding, the nurse checks for tube placement in the stomach as well as residual volume. The purpose of the nurse's actions is to avoid:

2. aspiration.

Following a liver transplant a client develops ascites. The nurse should teach the client to:

2. brace the abdomen with a pillow during coughing.

For a client who must undergo colon surgery, the physician orders preoperative cleansing enemas and neomycin sulfate (Mycifradin). The rationale for neomycin use in this client is to:

2. decrease the intestinal bacteria count.

When evaluating a client for complications of acute pancreatitis, the nurse would observe for:

2. decreased urine output.

For a client with cirrhosis, deterioration of hepatic function is best indicated by:

2. difficulty in arousal.

A client with gastroenteritis is admitted to an acute care facility with severe dehydration and electrolyte imbalances. Diagnostic tests reveal the Norwalk virus as the cause of gastroenteritis. Based on this information, the nurse knows that:

2. enteric precautions must be continued.

The nurse is performing an assessment on a client who has developed a paralytic ileus. The client's bowel sounds will be:

2. hypoactive.

A client is admitted with suspected cirrhosis. During assessment, the nurse is most likely to detect:

2. muscle wasting.

A client with advanced cirrhosis has a prothrombin time (PT) of 15 seconds, compared with a control time of 11 seconds. The nurse expects to administer:

2. phytonadione (Mephyton).

The nurse is teaching a client about malabsorption syndrome and its treatment. The client asks which part of the GI tract absorbs food. The nurse tells the client that products of digestion are absorbed mainly in the:

2. small intestine.

A client with peptic ulcer disease is prescribed aluminum-magnesium complex (Riopan). When teaching about this antacid preparation, the nurse should instruct the client to take it with:

2. water.

"The nurse is teaching the patient a client with a peptic ulcer discharge instructions. The client asks the nurse which type of analgesic he may take. Which of the following responses by the nurse would be most accurate? "1. Aspirin 2. Acetaminophen 3. Naproxen 4. Ibuprofen

2.Acetaminophen is recommended for pain relief because it does no promote irritation of the mucosa. Aspirin, and nonsteroidal anti- inflammatory drugs suchs as naproxen and ibuprofen, may cause irritation of the mucosa and subsequent bleeding

What response should a nurse offer to a client who asks why he's having a vagotomy to treat his ulcer? 1. To repair a hole in the stomach 2. to reduce the ability of the stomach to produce acid 3. to prevent the stomach from sliding into the chest 4. to remove a potentially malignant lesion in the stomach

2: A vagotomy is perfomred to elimniate the acid-secreting stimulus to gastric cells. a perforation would be repaired with a gastric resection. Repair of hiatal hernia (fundoplication) prevents the stomach from sliding through the diaphragm. Removal of a potentially malignant tumor wouldn't reduce the entire acid-producing mechanism

The nurse is assessing the client diagnosed with chronic gastritis. Which symptom(s) support this diagnosis? 1. Rapid onset of midsternal discomfort 2. Epigastric pain relieved by eating food 3. Dyspepsia and hematemesis 4. Nausea and projectile vomiting

2: Chronic pain in the epigastric area relieved by ingesting food is a sign of chronic gastritis

Brenda, a 36 y.o. patient is on your floor with acute pancreatitis. Treatment for her includes: 1. Continuous peritoneal lavage. 2. Regular diet with increased fat. 3. Nutritional support with TPN. 4. Insertion of a T tube to drain the pancreas.

3 With acute pancreatitis, you need to rest the GI tract by TPN as nutritional support.

Britney, a 20 y.o. student is admitted with acute pancreatitis. Which laboratory findings do you expect to be abnormal for this patient? 1. Serum creatinine and BUN 2. Alanine aminotransferase (ALT) and aspartate aminotransferase (AST) 3. Serum amylase and lipase 4. Cardiac enzymes

3 Pancreatitis involves activation of pancreatic enzymes, such as amylase and lipase. These levels are elevated in a patient with acute pancreatitis.

"Which of the following types of gastritis is associated with Helicobacter pylori and duodenal ulcers? 1. Erosive (hemorrhagic) gastritis 2. Fundic gland gastritis (type A) 3. Antral gland gastritis (type B) 4.Aspiring-induced gastric ulcer

3 - Erosive (hemorrhagic) gastritis can be caused by ingestion of substances that irritate the gastric mucosa. Fundic gland gastritis (type A) is associated with diffuse severe mucosal atrophy and the presence of pernicious anemia. Antral gland gastritis (type B) is the most common form of gastritis, and is associated with Helicobacter pylori and duodenal ulcers

Which of the following types of gastritis ic associated with Helicobacter pylori and duodenal ulcers? 1. Erosive (hemorrhagic) gastritis 2. Fundic gland gastritis (type A) 3. Antral gland gastritis (type B) 4. Aspiring-induced gastric ulcer

3 Antral gland gastritis ( type B). Rationale: Antral gland gastritis is the most common form of gastritis and is associated with Helicobacter pylori and duodenal ulcers

A male client is diagnosed with acute gastritis secondary to alcoholism and cirrhosis. When obtaining the client's history, the nurse gives priority to the client's statement that: 1) His pain increases after meals. 2) He experiences nausea frequently. 3) His stools have a black appearance. 4) He recently joined Alcoholics Anonymous

3) His stools have a black appearance.

Because a client has a nasogastric tube attached to intermittent drainage the nurse should be particularly alert for the development of which complication? 1. Hypocalcemia. 2. Hypermagnesemia. 3. Hypokalemia. 4. Hypoglycemia

3) Potassium is present in GI fluids and is lost during suctioning.

An adult is admitted with a duodenal ulcer. On the second day after admission, the client develops severe, persistent pain radiating to the shoulder. What action should the nurse take first? 1. Notify the physician. 2. Place client in a high-Fowler's position to decrease pressure on the gastric area and shoulder. 3. Examine the client for board-like rigidity of the abdomen. 4. Administer ordered prn pain medication

3) The nurse should first do a quick assessment todetermine if the cause of the pain is more apt to beperforation of the ulcer or something else such as cardiacpain. If the ulcer has perforated the client's abdomen willbe tender and rigid - board like

he nurse is caring for a client who has had a colostomy.Which of the following client behaviors is indicative of a willingness to be involved in self-care following a colostomy? 1. Discussing the cost of his hospitalization. 2. Asking what time the surgeon will be in. 3. Asking questions about the equipment being used. 4. Complaining about the noise in the adjacent room

3) When the client asks questions about theequipment being used, he indicates a readiness to learn.None of the other responses indicate a willingness tolearn about his colostom

Atropine 0.5 mg is ordered for a client having an acute attack of cholecystitis. What is the primary purpose of this drug for this client? To 1. decrease skeletal muscle spasms. 2. increase gastrointestinal peristalsis 3. decrease smooth muscle contractions 4. decrease anxiety

3)Atropine is an anticholinergic drug , which will decrease contractions of the gallbladder

The condition of a patient who has cirrhosis of the liver has deteriorated. Which diagnostic study would help determine if the patient has developed liver cancer? A) Serum α-fetoprotein level B) Ventilation/perfusion scan C) Hepatic structure ultrasound D) Abdominal girth measurement

C) Hepatic structure ultrasound, CT, and MRI are used to screen and diagnose liver cancer. Serum α-fetoprotein level may be elevated with liver cancer or other liver problems. Ventilation/perfusion scans do not diagnose liver cancer. Abdominal girth measurement would not differentiate between cirrhosis and liver cancer

Gastroesophageal reflux disease (GERD) weakens the lower esophageal sphincter, predisposing older persons to risk for impaired swallowing. In managing the symptoms associated with GERD, the nurse should assign the highest priority to which of the following interventions? 1. Decrease daily intake of vegetables and water, and ambulate frequently 2. Drink coffee diluted with milk at each meal, and remain in an upright position for 30 minutes. 3. Eat small, frequent meals, and remain in an upright position for at least 30 minutes after eating 4. Avoid over-the-counter drugs that have antacids in them

3, Eating small and frequent meals requires less release of hydrochloric acid. Remaining in an upright position for 30 minutes after meals prevents reflux into the esophagus which is often exacerbated when lying down, expecially after a large meal which makes the patient tired

which is the most common upper GI problem? " 1. peptic ulcer disease 2. Crohns 3. Gerd 4. ulcerative colitis

3, Gerd is the only upper GI problem

A client with a peptic ulcer is about to begin a therapeutic regimen that includes a bland diet, antacids, and ranitidine (Zantac). Before the client is discharged, the nurse should provide which instruction?

3. "Avoid aspirin and products that contain aspirin."

The nurse is providing dietary instructions to a client with a history of pancreatitis. Which instruction is correct?

3. "Maintain a high-carbohydrate, low-fat diet."

A client with acute diarrhea is prescribed paregoric, 5 ml by mouth up to four times daily, until the diarrhea subsides. The client asks the nurse how soon the medication will start to work after the first dose is taken. How should the nurse respond?

3. "Within 1 hour"

A client who has been treated for diverticulitis is being discharged on oral propantheline bromide (Pro-Banthine). The nurse should instruct the client to take the drug at which times?

3. 30 minutes before meals and at bedtime

After checking the client's chart for possible contraindications, the nurse is administering meperidine (Demerol), 50 mg I.M., to a client with pain after an appendectomy. Which type of drug would contraindicate the use of meperidine?

3. A monoamine oxidase (MAO) inhibitor

"The nurse is planning the care of a client diagnosed with lower esophageal sphincter dysfunction. Which dietary modifications should be included in the plan of care? 1. Allow any of the client's favorite foods as long as the amount is limited. 2. Have the client perform eructation exercises several times a day. 3. Eat four (4) to six (6) small meals a day and limit fluids during mealtimes. 4. Encourage the client to consume a glass of red wine with one (1) meal a day

3. Clients should eat small, frequent meals and limit fluids with the meals to prevent reflux into the esophagus from a distended stomach 1. The client is instructed to avoid spicy and acidic foods and any food producing symptoms. 2. Eructation means belching, which is a symptom of GERD. 4. Clients are encouraged to forgo all alcoholic beverages because alcohol relaxes the lower esophageal sphincter and increases the risk of reflux

A client is admitted with increased ascites related to cirrhosis. Which nursing diagnosis should receive top priority?

3. Ineffective breathing pattern

A nurse approaches a client with an 0800 dose of his scheduled pancreatin. The client states, "I'm not going to take that medicine. It makes me nauseated." What should the nurse do first?

3. Instruct the client about the benefit of taking the medication.

A client with pancreatitis has been receiving total parenteral nutrition (TPN) for the past week. Which nursing intervention helps determine if TPN is providing adequate nutrition?

3. Monitoring the client's weight every day

A client with Crohn's disease is admitted to a semiprivate room late in the afternoon. The next day, the client reports that he was not able to sleep during the night because the hallway lights bothered him. He asks that he be moved to a bed next to a window. What should the nurse do?

3. Move him to the next available window-side bed.

A client with abdominal pain secondary to a malignant mass in the colon is receiving fentanyl (Duragesic) by transdermal patch. His current patch expires in 48 hours and he reports a pain level of 8 on a 1-to-10 scale. What should a nurse do?

3. Notify the client's physician.

A 53-year-old client undergoes colonoscopy for colorectal cancer screening. A polyp was removed during the procedure. Which nursing interventions are necessary when caring for the client immediately after colonoscopy?

3. Observe the client closely for signs and symptoms of bowel perforation., 4. Monitor vital signs frequently until they're stable., 5. Inform the client that there may be blood in his stool and that he should report excessive blood immediately.

The nurse should expect to administer which vaccine to the client after a splenectomy?

3. Pneumovax 23

The nurse is caring for a client with cirrhosis. Which data collection findings indicate that the client has deficient vitamin K absorption caused by this hepatic disease?

3. Purpura and petechiae

Which of the following is a warning sign of colon cancer?

3. Rectal bleeding

A client is admitted to the emergency department with complaints of double vision, difficulty swallowing, dry mouth, and muscle weakness. A nurse also observes that the client has drooping eyelids and slurred speech. He states that he recently ate home-canned green beans. The nurse suspects exposure to botulism. What type of infection control precaution is necessary?

3. Standard precautions

A client comes to the emergency department complaining of acute GI distress. When obtaining the client's history, the nurse inquires about his family history. Which disorder has a familial basis?

3. Ulcerative colitis

An elderly client with Alzheimer's disease begins supplemental tube feedings through a gastrostomy tube to provide adequate calorie intake. The nurse should be concerned most with the potential for:

3. aspiration.

A client is admitted to the health care facility with a diagnosis of a bleeding gastric ulcer. The nurse expects this client's stools to be:

3. black and tarry.

The physician orders a stool culture to help diagnose a client with prolonged diarrhea. The nurse who obtains the stool specimen should:

3. collect the specimen in a sterile container.

A client with cholecystitis is receiving propantheline bromide (Pro-Banthine). The client is given this medication because it:

3. inhibits contraction of the bile duct and gallbladder.

The nurse is caring for a client who underwent a subtotal gastrectomy 24 hours ago. The client has a nasogastric (NG) tube. The nurse should:

3. irrigate the NG tube gently with normal saline solution.

When planning care for a client with a small-bowel obstruction, the nurse should consider the primary goal to be:

3. maintaining fluid balance.

A 68-year-old male is being admitted to the hospital with abdominal pain, anemia, and bloody stools. He complains of feeling weak and dizzy. He has rectal pressure and needs to urinate and move his bowels. The nurse should help him:

3. onto the bedpan.

A client who received an inhalation anesthetic during GI surgery experiences severe shivering postoperatively. In addition to providing extra blankets, the nurse should:

3. provide oxygen as prescribed.

A client with viral hepatitis A is being treated in an acute care facility. Because the client requires enteric precautions, the nurse should:

3. wear gloves when caring for the client and wash her hands after touching the client.

An adult who has cholecystitis reports clay colored stools and moderate jaundice. Which is the best explanation for the presence of clay colored stools and jaundice? 1. There is an obstruction in the pancreatic duct. 2. There are gallstones in the gallbladder. 3. Bile is no longer produced by the gallbladder. 4. There is an obstruction in the common bile duct.

4) Clay colored stools means bile is not getting through to the duodenum. The bile duct is obstructed so bile backs up into the bloodstream causing jaundice

The client with appendicitis asks the nurse for a laxative to help relieve her constipation. The nurse explains to her that laxatives are not given to persons with possible appendicitis. What is the primary reason for this? 1. Laxatives will decrease the spread of infection. 2. Laxatives are not given prior to any type of surgery. 3. The patient does not have true constipation. She only has pressure. 4. Laxatives could cause rupture of the appendix.

4) Laxatives cause increased peristalsis, which may cause the appendix to rupture. #2 is not a true statement. Laxatives may well be given prior to gynecological, rectal and colon surgery. #3 is true but is not the primary reason why laxatives are not given

A nurse provides feeding instructions to a mother of an infant diagnosed with gastroesophageal reflux disease. To assist in reducing the episodes of emesis, the nurse tells the mother to 1) Provide less frequent, larger feedings. 2) Burp the infant less frequently during feedings. 3) Thin the feedings by adding water to the formula. 4) Thicken the feedings by adding rice cereal to the formula

4) Thicken the feedings by adding rice cereal to the formula. Rationale: GERD is backflow of gastric contents into the esophagus as a result of relaxation or incompetence of the lower esophageal or cardiac sphincter. Small, more frequent feedings with frequent burping often are prescibed in the treatment of GER. Feedings thickened with rice cereal may reduce episodes of emesis. If thickened formula is used, cross-cutting of the nipple may be required

A client has an order for irrigation of a nasogastric tube. What should the nurse do before irrigating the nasogastric tube? 1. Inject a small amount of air while listening with a stethoscope over the stomach for a "swoosh." 2. Instill 5 cc of normal saline and observe for development of coughing and dyspnea. 3. Place the end of the nasogastric tube in a glass of water and observe for bubbles. 4. Aspirate and check the pH

4) To determine if the tube is in the stomach, the nurse should aspirate and check the pH. It should be less than 5. Never instill saline. If the tube were in the-bronchi instead of the stomach, saline would cause respiratory distress. Placing the end of the tube in a glass

Following a cholecystectomy, drainage form the T tube for the first 24 hours postoperative was 350 cc. Proper nursing action in response to this should be to 1. notify the physician . 2. raise the level of the drainage bag to decrease rate of flow. 3. increase the IV flow rate to compensate for the loss. 4. continue to observe and measure drainage

4)350 cc in 24 hours after surgery is a normal amount of bile drainage

An adult male is admitted to the hospital complaining of burning epigastric pain. He reports to the nurse that he has gained 14 pounds over the last two months. Which nursing response is best? 1. "Why were you eating more?" 2. "Has the weight gain been intentional?" 3. "Does your weight usually fluctuate this much?" 4. "How did your eating habits change?"

4)Weight gain may occur due to increased consumption of food as the client tries to feed a duodenal ulcer. "Why" questions are threatening to clients. #3asks for a yes or no answer. This will not give as much information as asking about the eating habits

The client with gastroesophageal reflux disease (GERD) complains of a chronic cough. The nurse understands that in a client with GERD this symptom may be indicative of which of the following conditions? " 1. Development of laryngeal cancer. 2. Irritation of the esophagus. 3. Esophageal scar tissue formation. 4. Aspiration of gastric contents

4, "Clients with GERD can develop pulmonary symptoms, such as coughing, wheezing, and dyspnea, that are caused by the aspiration of gastric contents. GERD does not predispose the client to the development of laryngeal cancer. Irritation of the esophagus and esophageal scar tissue formation can develop as a result of GERD. However, GERD is more likely to cause painful and difficult swallowing

The nurse is monitoring a client with a diagnosis of peptic ulcer. Which assessment finding would most likely indicate perforation of the ulcer? 1. Bradycardia 2. Numbness in the legs 3. Nausea and vomiting 4. A rigid, board-like abdomen

4, Perforation of an ulcer is a surgical emergency and is characterized by sudden, sharp, intolerable severe pain beginning in the midepigastric area and spreading over the abdomen, which become rigid and board-like. Nausea and vomiting may occur. Tachycardia may occur as hypovolemic shock develops. Numbness in the legs is not an associated finding

The nurse is monitoring a client with a diagnosis of peptic ulcer. Which assessment finding would most likely indicate perforation of the ucler? 1. Bradycardia 2. Numbness in the legs 3. Nausea and vomiting 4. A rigid, board-like abdomen

4, Rationale: Perforation of an ulcer is a surgical emergency and is characterized by sudden, sharp, intolerable severe pain beginning in the midepigastric area and spreading over the abdomen, which becomes rigid and board-like. Nausea and vomiting may occur. Tachycardia may occur as hypovolemic shock develops. Numbness in the legs is not an associated finding

The nurse is preparing a client diagnosed with GERD for discharge following an esophagogastroduodenoscopy. Which statement indicates the client understands the discharge instructions?7 1. I should not eat for 24 hours following this procedure. 2 I can lie down whenever iI want after a meal. It own't make a difference. 3. The stomach contents won't bother my esophagus but will make me nauseous. 4. I should avoid drinking orange jice and eating tomatoes until my esophagus heals.

4, oragne and tomato juices are acidic, and the client diagnosed with GERD shouldavoid acidic foods until the esophagus hashad a chance to heal - A client hospitalized with a gastric ulcer is scheduled for discharge.

the nurse is monitoring a client with a diagnosis of peptic ulcer. which assessment finding would most likely indicate perforation of the ulcer? 1. bradycardia 2. numbness in legs 3. N&V 4. a rigid board-like abdomen

4, perforation of ulcer is a surgical emergency and is characterized by sudden, sharp, intolderable severe pain beginning in the midepigastric area and spreading over the abdomen, which becomes rigid and board-like. nausea and vomiting may also occur. tachycardia may occur as hypovolemic shock develops. numbness of the legs is not an associated finding

Which of the following is a protrusion of the intestine through a weakened area in the abdominal wall? a) Tumor b) Adhesion c) Hernia d) Volvulus

C) Hernia A hernia is a protrusion of intestine through a weakened area in the abdominal muscle or wall. A tumor that extends into the intestinal lumen, or a tumor outside the intestine causes pressure on the wall of the intestine. Volvulus occurs when the bowel twists and turns on itself. An adhesion occurs when loops of intestine become adherent to areas that heal slowly or scar after abdominal surgery.

Before abdominal surgery for an intestinal obstruction, the nurse monitors the client's urine output and finds that the total output for the past 2 hours was 35 mL. The nurse then assesses the client's total intake and output over the last 24 hours and notes that he had 2,000 mL of I.V. fluid for intake, 500 mL of drainage from the nasogastric tube, and 700 mL of urine for a total output of 1,200 mL. This would indicate which of the following? 1. Decreased renal function. 2. Inadequate pain relief. 3. Extension of the obstruction. 4. Inadequate fluid replacement.

4. Considering that there is usually 1 L of insensible fluid loss, this client's output exceeds his intake (intake, 2,000 mL; output, 2,200 mL), indicating deficient fluid volume. The kidneys are concentrating urine in response to low circulating volume, as evidenced by a urine output of less than 30 mL/ hour. This indicates that increased fluid replacement is needed. Decreasing urine output can be a sign of decreased renal function, but the data provided suggest that the client is dehydrated. Pain does not affect urine output. There are no data to suggest that the obstruction has worsened.

A client with recent onset of epigastric discomfort is scheduled for an upper GI series (barium swallow). When teaching the client how to prepare for the test, which instruction should the nurse provide?

4. "Avoid eating or drinking anything for 6 to 8 hours before the test."

During a client-teaching session, which instruction should the nurse give to a client receiving kaolin and pectin (Kaopectate) for treatment of diarrhea?

4. "Drink 8 to 13 8-oz glasses (2 to 3 L) of fluid daily."

A client is diagnosed with a hiatal hernia. Which statement indicates effective client teaching about hiatal hernia and its treatment?

4. "I'll eat frequent, small, bland meals that are high in fiber."

A client who has just been diagnosed with hepatitis A asks, "How could I have gotten this disease?" What is the nurse's best response?

4. "You may have eaten contaminated restaurant food."

A nursing assistant is assisting a nurse with feeding clients. Which client should the nurse assign to the nursing assistant?

4. A client with bilateral blindness

The nurse caring for a client with small-bowel obstruction would plan to implement which nursing intervention first?

4. Administering I.V. fluids

A nurse is working with a nursing assistant, who is given the task of calculating three clients' intake and output at the end of the shift. When the nurse reviews the nursing assistant's work, she discovers inaccuracies in the nursing assistant's results. What should the nurse do?

4. Ask the nursing assistant to show her how she determined the results.

Which food should be included in a client's diet during the first 6 to 8 weeks after ileostomy surgery?

4. Banana

One year ago, a client was diagnosed with cirrhosis of the liver caused by alcohol abuse. Since then, he has been noncompliant with the prescribed protein-restricted diet. After a friend finds him semiconscious at home, the client is admitted to the hospital. When initial laboratory test results show an elevated ammonia level, he's diagnosed with hepatic encephalopathy. The physician prescribes lactulose (Cephulac), 200 g diluted in 700 ml of tap water, given as a retention enema every 4 hours. For which other condition is lactulose prescribed?

4. Constipation

The doctor has ordered Tagamet for a client admitted with gastroesophageal reflux disease (GERD). After looking up the drug in the Physician's Desk Reference, you understand it is being used to: 1. Neutralize stomach acid. 2. Treat a hiatal hernia. 3. Aid in the digestion of food. 4. Decrease stomach acid production

4. Decrease stomach acid production

Which medication should the nurse expect to administer to a client with constipation?

4. Docusate sodium (Colace)

The surgeon is discussing surgery with a client diagnosed with colon cancer. The client is visibly shaken over the possibility of a colostomy. Based on the client's response, the surgeon should collaborate with which health team member?

4. Enterostomal nurse

A nurse is irrigating an open wound of the abdomen. In which direction should she arrange for the irrigation solution to flow through the wound?

4. From the top inside of the wound, through the wound, and then out

Which assessment data support the client's diagnosis of gastric ulcer?" 1. Presence of blood in the client's stool for the past month. 2.Complaints of a burning sensation that moves like a wave. 3.Sharp pain in the upper abdomen after eating a heavy meal. 4.Comparison of complaints of pain with ingestion of food and sleep

4. In a client diagosed with a gastric ulcer, pain usually occurs 30-60 minutes after eating, but not at night. In contrast, a client with duodenal ulcer has pain durin ghte night that is often relieved by eating food. Pain occurs 1-3 hours after meals

When caring for a client with hepatitis B, the nurse should monitor closely for the development of which finding associated with a decrease in hepatic function?

4. Irritability and drowsiness

While palpating a client's right upper quadrant, the nurse would expect to find which of the following structures?

4. Liver

A client with left hemiparesis is having difficulty handling eating utensils. A nurse asks the physician to request a consult with which discipline?

4. Occupational therapy

The nurse is monitoring the client with a diagnosis of peptic ulcer. Which assessment finding would most likely indicate perforation of the ulcer? 1. Bradycardia 2. Numbness in Legs 3. Nausea and Vomiting 4. A rigid, board-like abdomen

4. Perforation of an ulcer is a surgical emergency and is a sudden, sharp, intolerable severe pain beginning in the midepigastric area and spreading over the abdomen, which becomes rigid and board-like. Nausea and vomiting may occur. Tachycardia may occur as hypovolemic shock develops. Numbness in the leg is not an associated finding

A client is admitted to the health care facility with nausea, vomiting, and abdominal cramps and distention. Which test result is most significant?

4. Serum potassium level of 3 mEq/L

A physician asks a nurse to witness an informed consent of a client scheduled for gastric bypass surgery. What should the nurse do?

4. Sign the consent only if she sees the client sign it.

After taking an antacid, the client asks the nurse where antacids act in the body. How should the nurse respond?

4. Stomach

Which outcome indicates effective client teaching to prevent constipation?

4. The client reports engaging in a regular exercise regimen.

A client has a newly created colostomy. After participating in counseling with the nurse and receiving support from the spouse, the client decides to change the colostomy pouch unaided. Which behavior suggests that the client is beginning to accept the change in body image?

4. The client touches the altered body part.

A client is undergoing an extensive diagnostic workup for a suspected GI problem. The nurse discovers that the client has a family history of ulcer disease. Which blood type also is a risk factor for duodenal ulcers?

4. Type O

The nurse is teaching a group of middle-aged men about peptic ulcers. When discussing risk factors for peptic ulcers, the nurse should mention:

4. alcohol abuse and smoking.

A client is in the late stage of cirrhosis. When planning the client's diet, the nurse should focus on providing increased amounts of:

4. carbohydrate.

A client with hepatitis C develops liver failure and GI hemorrhage. The blood products that would most likely bring about hemostasis in the client are:

4. cryoprecipitate and fresh frozen plasma.

The nurse is caring for a client who underwent a subtotal gastrectomy. To manage dumping syndrome, the nurse should advise the client to:

4. drink liquids only between meals.

The physician prescribes lactulose (Cephulac), 30 ml by mouth three times daily, when a client with cirrhosis develops an increased serum ammonia level. To evaluate the effectiveness of lactulose, the nurse should monitor:

4. level of consciousness (LOC).

Alterations in hepatic blood flow resulting from a drug interaction also can affect:

4. metabolism and excretion.

A client is diagnosed with shigellosis. The nurse teaches the client and family how the disease is transmitted and treated and discusses the need for enteric precautions. The nurse should explain that enteric precautions must be maintained:

4. until three fecal cultures are negative for Shigella.

The nurse is monitoring a female client with a diagnosis of peptic ulcer. 1.Bradycardia 2.Numbness in the legs 3.Nausea and vomiting 4.A rigid, board-like abdomen

4.Perforation of an ulcer is a surgical emergency and is characterized by sudden, sharp, intolerable severe pain beginning in the midepigastric area and spreading over the abdomen, which becomes rigid and board-like

The nurse is planning to teach a client with GERD about substances that will increase the LES pressure.Which item shoud the nurse include on this list. 1. Coffee 2. Chocolate 3. Fatty Foods 4. Nonfat MIlk

4Nonfat MIlk Foods that will increase LES pressure will decrease reflux and lessen the symptoms of GERD. The food that will increase LES pressure is nonfat milk. The other substances listed decrease LES pressure, thus increasing reflux symptoms. Aggravating substances include the others listed and alcohol

A 22 yr old calls the clinic complaining of N&V and RLQ abdominal pain. The nurse advises the patient to: a. have the symptoms evaluated by a MD right away b. use a heating pad c. drink at least 2 qts of juice d. take a laxative to empty the bowel before exam at clinic

A

A Blakemore-Sengstaken tube is inserted to prevent bleedingfrom esophageal varices. The nurse responsibility in this instancewould be to: A. alternate inflate and deflate the esophageal balloon B. make certain that the desired degree of pressure is constantlymaintained C. deflate both balloons periodically D. encourage Kim to swallow frequently while tube is I place

A

A client is admitted to the hospital complaining of nervousness,heat intolerance and muscle weakness. Her pulse rate is 118 and shehas exopthalmos. An essential part of her assessment will be: A. palpation of the thyroid gland B. evaluation of fluid and electrolyte balance C. evaluation of deep tendon reflexes D. use of the Glasgow Coma Scale

A

A client is admitted to the medical-surgical floor with a diagnosis of acute pancreatitis. His blood pressure is 136/76 mm Hg, pulse 96 beats/minute, respirations 22 breaths/minute, temperature 99°F (38.3°C), and he has been experiencing severe vomiting for 24 hours. His past medical history reveals hyperlipidemia and alcohol abuse. The physician prescribes a nasogastric (NG) tube for the client. Which of the following is the primary purpose for insertion of the NG tube? A. Empty the stomach of fluids and gas to relieve vomiting. B. Prevent spasms at the sphincter of Oddi. C. Prevent air from forming in the small and large intestines. D. Remove bile from the gallbladder.

A

A client is being evaluated for the possibility of Grave'sdisease. The nurse teaches that the best laboratory test for evaluatingwhether a client has hypothyroidism or hyperthyroidism is the serumlevel of: A. thyroxine (T4) C. TSH B. triiodothyroinine (T3) D. epinephrine

A

A client is taking Levothyroxine (synthroid) for hypothyroidism. The nurse teaches the client to: A. monitor the pulse regularly B. restrict sodium in the diet C. take the drug with meals D. measure urinary output

A

A client with cirrhosis of the liver develops ascites. Which of the following orders would the nurse expect? A. Restrict fluid to 1000 mL per day. B. Ambulate 100 ft. three times per day. C. High-sodium diet. D. Maalox 30 ml P.O. BID.

A

A client with liver and renal failure has severe ascites. On initial shift rounds, his primary nurse finds his indwelling urinary catheter collection bag too full to store more urine. The nurse empties more than 2,000 ml from the collection bag. One hour later, she finds the collection bag full again. The nurse notifies the physician, who suspects that a bladder rupture is allowing the drainage of peritoneal fluid. The physician orders a urinalysis to be obtained immediately. The presence of which substance is considered abnormal? a) Albumin b) Chloride c) Urobilinogen d) Creatinine

A

A client with severe and chronic liver disease is showing manifestations related to inadequate vitamin intake and metabolism. He reports difficulty driving at night because he cannot see well. Which of the following vitamins is most likely deficient for this client? a) Vitamin K b) Vitamin A c) Riboflavin d) Thiamine

A

A knowledge of factors associated with colorectal cancer guides the nurse when obtaining a nursing history to ask specifically about: a. usual diet b. history of smoking c. history of alcohol d. environmental exposure to carcinogens

A

A nursing assessment for initial signs of hypoglycemia willinclude: A. Pallor, blurred vision, weakness, behavioral changes B. frequent urination, flushed face, pleural friction rub C. abdominal pain, diminished deep tendon reflexes, double vision D. weakness, lassitude, irregular pulse, dilated pupils

A

A patient with NG tube develops nausea and increased upper abominal bowel sounds. Appropriate action is to: a. check the patency of the NG tube b. place client in recumbant position c. assess vital signs d. ecourage deep breathing

A

A patient's vomitus is dark brown and has a coffee-ground appearance. the nurse recognizes that this emsis is charactristic of: a. stomach bleeding b. an intestinal obstruction c. bile reflux d. active bleeding of lower esophagus

A

Diagnostic testing is planned fr a patient with suspected peptic ulcer. Most reliable test is: a. endoscopy b. gastric analysis c. barium swallow d. serologic test for H pylori

A

During an acute attack of diverticulitis, the patient is: a. monitored for signs of peritonitis b. treated with daily med enemas c. prepared for surgery to resect the involved colon d. provided with heathing pad to apply to LLQ

A

Management of patient with upper GI bleeding is effective the lab results reveal: a. decreasing BUN b. normal hematocrit c. urine output of 20 ml hr d. specific gravity of 1.03

A

Nurse determines teaching need when patient with dumping syndrome says a. "i should eat bread with every meal" b. "i should avoid drinking fluids with meals" c. i should eat small meals about 6x day" d. "i need t olie down for 30-60 min after meals

A

Nursing management of the patient with chronic gastritis includes teaching the patient to: a. maintain a bland diet with six small meals a day b. take antacids before meals c. use NSAIDS instead of aspirin for pain relief d. eliminate alcohol and caffeine from diet

A

Patient admitted to ER has profuse bright-red hematemesis. During intial care of the patient, the nurse's first priority is to: a. perform a nursing assessment of patient's status b. establish 2 IV sites c. obtain a thorough health history d. perform a gastric lavage with cool tap water in prep for endoscopic exam

A

Patient with inflammatory bwel disease has a nursing diagnosis of imbalanced nutrition: less than body requirements r/t decreased nutritional intake and decreased intestinal absorption. Data to support this is: a. pallor and hair loss b. frequent diarrhea stools c. anorectal excoriation and pain d. hypotension and urine output below 30 ml /hr

A

Patient with ulcerative colitis has a total colectomy with formation of a terminal ileum stoma. an important nursing interention for this patient postop is to: a. measure the ileosotmy output to determine the status of patient's fluid balance b. change ileostomy q 3-4 hrs c. emphasize that ostomy is temporary d. teach about high fiber diet required to maintain normal ostomy drainage

A

Postop patient has nursing diagnosis of pain r/t to immobility, meds, and decreased motility as evidneced by abdominal pain and distention and inability to pass flatus. An apropriate nursing intervention for the patient is to: a. ambulate patient more frequently b. assess abdomen for bowel sounds c. place patient in high fowlers d. withhold narcotics because they decrease bowel motility

A

Regardless of precipitating factor, the injury to mucosal cells in PUD is caused by: a. acid back-diffusion into the mucosa b. ammonia formation in the mucus wall c. breakdown of gastric mucosal barrier d. release of histamine for cells

A

The nurse is caring for a client who requires a nasogastric (NG) tube for feeding. What should the nurse do immediately after inserting an NG tube for enteral feedings? A. Aspirate for gastric secretions with a syringe. B. Begin feeding slowly to prevent cramping. C. Get an X-ray of the tip of the tube within 24 hours. D. Clamp off the tube until the feedings begin.

A

A patient with a history of peptic ulcer disease is admitted to the emergency department with complaints of severe abdominal pain and a rigid, board-like abdomen. The health care team suspects a perforated ulcer. Which action should you anticipate? A. Providing IV fluids and monitoring nasogastric tube drainage B. Administering calcium gluconate and testing the patient's gastric pH level C. Performing a fecal occult blood test and administering IV calcium gluconate D. Starting parenteral nutrition and placing the patient in a high Fowler's position

A Rationale A perforated peptic ulcer requires IV replacement of fluid losses and continued gastric aspiration by a nasogastric tube. Nothing is given by mouth, and gastric pH testing is not a priority. Calcium gluconate is not a medication directly relevant to the patient's suspected diagnosis, and parenteral nutrition is not a priority in the short term. Reference: 995

After administration of a dose of promethazine (Phenergan) to a patient with nausea and vomiting, which symptom may be experienced as a common temporary adverse effect of the medication? A. Drowsiness B. Reduced hearing C. Sensation of falling D. Photosensitivity

A Rationale Although given to this patient as an antiemetic, promethazine also has sedative and amnesic properties. For this reason, the patient is likely to experience drowsiness as an adverse effect of the medication. Reference: 965

A patient had been vomiting and is now tolerating small amounts of clear liquids. What would be an appropriate food or fluid choice? A. Dry toast B. Milk C. Iced tea D. Creamed corn

A Rationale Clear liquids are liquids that can be seen through and do not include milk products. Because the patient is tolerating clear liquids, he or she can be advanced to a soft, bland diet of bananas, rice, applesauce, and toast (BRAT). Patients do better avoiding extremely cold fluids because they can cause cramping. Creamed corn contains milk products, and the hull can be irritating and is not part of a bland diet. Reference: 965

The patient is using a corticosteroid inhaler and has white patches in the mouth. What action should you teach? A. Rinse out the mouth after using the inhaler. B. Brush the tongue to remove bacteria. C. Rinse with an alcohol-based mouthwash to kill germs. D. Chew gum to promote secretion of saliva.

A Rationale Corticosteroid inhalers put the patient at risk for oral infections, particularly candidiasis. Rinsing the mouth will help minimize this risk. An important element of reducing oral infections is identification of the cause. The other measures are not directly related to candidiasis. Reference: 968

The patient had a gastroduodenostomy (Billroth I operation) for a peptic ulcer that did not respond to more conservative treatment. The patient now reports having palpitations, abdominal cramping, and runny diarrhea 30 minutes after eating. What is the best explanation of what is happening? A. Dumping syndrome from a meal of hyperosmolar composition B. Gastric irritation from loss of the intrinsic factor C. Postprandial hyperglycemia from uncontrolled gastric emptying D. Bile reflux gastritis from dumping of bile into the stomach

A Rationale Dumping syndrome results because the stomach cannot control the amount of gastric chyme entering the small intestine. A hypertonic amount enters the intestine and draws fluid into the lumen, causing distention of the bowel and rapid transit. This is a long-term potential problem and not related to immediate postoperative irritation. The more likely complication is postprandial hypoglycemia. Bile reflux gastritis is prolonged contact of bile that can cause gastritis and recurrence of PUD. Reference: 996

The patient is diagnosed with Escherichia coli infection. What is the most likely cause? A. Undercooked hamburger B. Milk left at room temperature C. A pet turtle D. Cleaning the cat litter box

A Rationale E. coli is primarily found in undercooked meats, particularly poultry and hamburger. It can also be found in contaminated vegetables, fruits, and nuts. It can occur in raw or unpasteurized milk. Turtles carry Salmonella. Trichomonas can be transmitted through cat litter or contaminated bathroom facilities. Reference: 1001

During administration of a dose of metoclopramide (Reglan), a patient complains of nausea. You should teach the patient to report which potential adverse effect? A. Tremors B. Constipation C. Double vision D. Numbness in the fingers and toes

A Rationale Extrapyramidal side effects, including tremors and dyskinesias, may be a response to metoclopramide (Reglan) administration. Reference: 965

What is an important cause of chronic gastritis? A. Helicobacter pylori B. Ascorbic acid C. Streptococci D. Herpes zoster

A Rationale H. pylori is an important cause of chronic gastritis. It is more likely in underdeveloped countries and in people with low socioeconomic status. Ascorbic acid is vitamin C. Streptococci and herpes zoster infections are not related to gastritis. Herpes zoster is the causative organism for shingles, chickenpox, and varicella. Reference: 985

Which symptom is commonly seen with achalasia? A. Halitosis B. Frequent eructation C. Flatus D. Canker sores

A Rationale Halitosis (foul-smelling breath) is a symptom of achalasia. It is caused by accumulation of "old" food. Inability to eructate (belch) is a common symptom. Flatus and canker sores are not specifically related to this condition. Reference: 980

Which is the most likely cause of foodborne illness (food poisoning)? A. Prepackaged cookie dough left at room temperature B. Contaminated shellfish C. Frozen ice cream D. Unpasteurized milk

A Rationale Most foodborne illness is caused by contaminated raw food or uncooked food left at room temperature. The bacteria can multiply rapidly. One example is prepackaged cookie dough. Contaminated shellfish is a source in cases of hepatitis A. The other options are not common causes in the United States. Reference: 1001

The older adult with a history of frequent heartburn reports episodes of wheezing and coughing. What should you investigate related to the cause of these new symptoms? A. Drinking caffeine-containing beverages B. Starting a new exercise program to lose weight C. Using over-the-counter antihistamines D. Consuming iodine-containing food

A Rationale Older adults with hiatal hernia and GERD are sometimes asymptomatic or have only minor symptoms. The first indication may be a respiratory complication due to the aerosolization or aspiration of gastric contents. These complications are more likely with age because the LES weakens or with the start of new medications (e.g., nitrates, calcium channel blockers, antidepressants) that decrease LES pressure. Caffeine affects the LES, and modification of habits can help. The other options are not specifically related to LES pressure and GERD or hiatal hernia. Reference: 976

The intoxicated patient in the emergency department has a decreased level of consciousness and is reporting feeling nauseated. What is most important for you to do? A. Place the patient in a side-lying position. B. Obtain an emesis basin for the patient. C. Encourage the patient to take deep breaths. D. Obtain an order for metoclopramide (Reglan).

A Rationale Patients with a decreased level of consciousness are at risk for aspiration with vomiting. They should be placed in a semi-Fowler or side-lying position. Preventing aspiration is a priority over the other options. Nausea is traditionally treated with fluids or an antiemetic. Reglan is more commonly used for delayed gastric emptying.

The patient has oral mucositis after radiation therapy for pharyngeal cancer. Which manifestation should you expect to find when assessing the patient? A. Oral erythema and taste changes B. Gingivitis and halitosis C. Plaque and cheilitis D. Caries and leukoplakia

A Rationale Redness and taste changes occur with oral mucositis. Gingivitis, bad breath, plaque, lip inflammation, cavities, and white oral lesions are not manifestations of mucositis. Reference: 968

The drug prochlorperazine (Compazine) should not be given for postoperative nausea and vomiting if the patient has which condition? A. Glaucoma B. Cancer C. History of diabetes D. Constipation

A Rationale The drug has anticholinergic actions and is contraindicated for the patient with glaucoma, prostatic hyperplasia, pyloric or bladder neck obstruction, or biliary obstruction. There is no reason not to use the drug with a patient who has cancer, although often other antiemetics are used. Administration of prochlorperazine is not contraindicated for a patient who has diabetes. Constipation is a side effect of the drug, but it is not a contraindication; bowel ileus or obstruction would be a contraindication. Reference: 965

Twenty-four hours after a surgical repair of a hiatal hernia, you notice bright red bloody drainage in the nasogastric tube. What should you do first? A. Notify the primary health care provider. B. Irrigate the nasogastric tube with ice water. C. Document the findings. D. Place the patient in a side-lying position.

A Rationale This is a sign of active gastrointestinal bleeding and needs further follow-up. Bloody drainage is expected for 8 to 12 hours. Because there is the potential for surgical complications or active ulcer bleeding, the health care provider must be consulted rather than only attempting to stop the bleeding with ice water irrigations. Action is required rather than documentation. Repositioning the patient will not resolve the problem. Reference: 976

What is the best test to diagnose stomach cancer? A. Upper endoscopic examination B. Barium enema C. Endoscopic retrograde cholangiopancreatogram (ERCP) D. Magnetic resonance imaging (MRI)

A Rationale Upper endoscopic examination is the best diagnostic test for stomach cancer. It allows direct visualization and biopsy. Barium enema is for lower gastrointestinal problems and does not allow direct visualization. ERCP is an endoscopic procedure used to examine the liver, gallbladder, and pancreas. MRI is not used to diagnose stomach cancer. Reference: 99

While a female client is being prepared for discharge, the nasogastric (NG) feeding tube becomes clogged. To remedy this problem and teach the client's family how to deal with it at home, what should the nurse do? a. Irrigate the tube with cola. b. Advance the tube into the intestine. c. Apply intermittent suction to the tube. d. Withdraw the obstruction with a 30-ml syringe.

A The nurse should irrigate the tube with cola because its effervescence and acidity are suited to the purpose, it's inexpensive, and it's readily available in most homes. Advancing the NG tube is inappropriate because the tube is designed to stay in the stomach and isn't long enough to reach the intestines. Applying intermittent suction or using a syringe for aspiration is unlikely to dislodge the material clogging the tube but may create excess pressure. Intermittent suction may even collapse the tube.

The client being seen in a physician's office has just been scheduled for a barium swallow the next day. The nurse writes down which of the following instructions for the client to follow before the test? A. Fast for 8 hours before the test B. Eat a regular supper and breakfast C. Continue to take all oral medications as scheduled D. Monitor own bowel movement pattern for constipation

A A barium swallow is an x-ray study that uses a substance called barium for contrast to highlight abnormalities in the GI tract. The client should fast for 8 to 12 hours before the test, depending on the physician instructions. Most oral medications also are withheld before the test. After the procedure the nurse must monitor for constipation, which can occur as a result of the presence of barium in the GI tract.

A female client being seen in a physician's office has just been scheduled for a barium swallow the next day. The nurse writes down which instruction for the client to follow before the test? A. Fast for 8 hours before the test B. Eat a regular supper and breakfast C. Continue to take all oral medications as scheduled D. Monitor own bowel movement pattern for constipation

A A barium swallow is an x-ray study that uses a substance called barium for contrast to highlight abnormalities in the gastrointestinal tract. The client should fast for 8 to 12 hours before the test, depending on physician instructions. Most oral medications also are withheld before the test. After the procedure, the nurse must monitor for constipation, which can occur as a result of the presence of barium in the gastrointestinal tract.

Which of the following complications is thought to be the most common cause of appendicitis? A. A fecalith B. Bowel kinking C. Internal bowel occlusion D. Abdominal bowel swelling

A A fecalith is a fecal calculus, or stone, that occludes the lumen of the appendix and is the most common cause of appendicitis. Bowel wall swelling, kinking of the appendix, and external occlusion, not internal occlusion, of the bowel by adhesions can also be causes of appendicitis.

Which of the following diets is most commonly associated with colon cancer? A. Low-fiber, high fat B. Low-fat, high-fiber C. Low-protein, high-carbohydrate D. Low carbohydrate, high protein

A A low-fiber, high-fat diet reduced motility and increases the chance of constipation. The metabolic end products of this type of diet are carcinogenic. A low-fat, high-fiber diet is recommended to prevent colon cancer.

After a subtotal gastrectomy, the nurse should anticipate that nasogastric tube drainage will be what color for about 12 to 24 hours after surgery? A. Dark brown B. Bile green C. Bright red D. Cloudy white

A About 12 to 24 hours after a subtotal gastrectomy, gastric drainage is normally brown, which indicates digested blood. Bile green or cloudy white drainage is not expected during the first 12 to 24 hours after a subtotal gastrectomy. Drainage during the first 6 to 12 hours contains some bright red blood, but large amounts of blood or excessively bloody drainage should be reported to the physician promptly.

The nurse would teach patients that antacids are effective in treatment of hyperacidity because they: A. Neutralize gastric acid B. Decrease stomach motility C. Decrease gastric pH D. Decrease duodenal pH

A Antacids work by neutralizing gastric acid, which would cause an increase in pH. They do not affect gastric motility.

A client has just had surgery for colon cancer. Which of the following disorders might the client develop? A. Peritonitis B. Diverticulosis C. Partial bowel obstruction D. Complete bowel obstruction

A Bowel spillage could occur during surgery, resulting in peritonitis. Complete or partial bowel obstruction may occur before bowel resection. Diverticulosis doesn't result from surgery or colon cancer.

The nurse provides medication instructions to a client with peptic ulcer disease. Which statement, if made by the client, indicates the best understanding of the medication therapy? A. "The cimetidine (Tagamet) will cause me to produce less stomach acid." B. "Sucralfate (Carafate) will change the fluid in my stomach." C. "Antacids will coat my stomach." D. "Omeprazole (Prilosec) will coat the ulcer and help it heal."

A Cimetidine (Tagamet), a histamine H2 receptor antagonist, will decrease the secretion of gastric acid. Sucralfate (Carafate) promotes healing by coating the ulcer. Antacids neutralize acid in the stomach. Omeprazole (Prilosec) inhibits gastric acid secretion.

The nurse is reviewing the record of a female client with Crohn's disease. Which stool characteristics should the nurse expect to note documented in the client's record? A. Diarrhea B. Chronic constipation C. Constipation alternating with diarrhea D. Stools constantly oozing from the rectum

A Crohn's disease is characterized by nonbloody diarrhea of usually not more than four to five stools daily. Over time, the diarrhea episodes increase in frequency, duration, and severity. Options B, C, and D are not characteristics of Crohn's disease.

Which of the following symptoms indicated diverticulosis? A. No symptoms exist B. Change in bowel habits C. Anorexia with low-grade fever D. Episodic, dull, or steady midabdominal pain

A Diverticulosis is an asymptomatic condition. The other choices are signs and symptoms of diverticulitis.

The nurse is monitoring a female client for the early signs and symptoms of dumping syndrome. Which of the following indicate this occurrence? A. Sweating and pallor B. Bradycardia and indigestion C. Double vision and chest pain D. Abdominal cramping and pain

A Early manifestations of dumping syndrome occur 5 to 30 minutes after eating. Symptoms include vertigo, tachycardia, syncope, sweating, pallor, palpitations, and the desire to lie down.

Which of the following dietary measures would be useful in preventing esophageal reflux? A. Eating small, frequent meals B. Increasing fluid intake C. Avoiding air swallowing with meals D. Adding a bedtime snack to the dietary plan

A Esophageal reflux worsens when the stomach is overdistended with food. Therefore, an important measure is to eat small, frequent meals. Fluid intake should be decreased during meals to reduce abdominal distention. Avoiding air swallowing does not prevent esophageal reflux. Food intake in the evening should be strictly limited to reduce the incidence of nighttime reflux, so bedtime snacks are not recommended.

Which of the following areas is the most common site of fistulas in client's with Crohn's disease? A. Anorectal B. Ileum C. Rectovaginal D. Transverse colon

A Fistulas occur in all these areas, but the anorectal area is most common because of the relative thinness of the intestinal wall in this area.

Dr. Smith has determined that the client with hepatitis has contracted the infection from contaminated food. The nurse understands that this client is most likely experiencing what type of hepatitis? A. Hepatitis A B. Hepatitis B C. Hepatitis C D. Hepatitis D

A Hepatitis A is transmitted by the fecal-oral route via contaminated food or infected food handlers. Hepatitis B, C, and D are transmitted most commonly via infected blood or body fluids.

Histamine2-receptor antagonists: A. Compete with histamine for binding sites on the parietal cells B. Irreversibly bind to H+/K+ATPase C. Cause a decrease in stomach pH D. Decrease signs and symptoms of allergies related to histamine release

A Histamine receptor blocking agents decrease gastric acid by competing with histamine for binding sites on the parietal cells.

The nurse is caring for a client following a Billroth II procedure. On review of the post-operative orders, which of the following, if prescribed, would the nurse question and verify? A. Irrigating the nasogastric tube B. Coughing a deep breathing exercises C. Leg exercises D. Early ambulation

A In a Billroth II procedure the proximal remnant of the stomach is anastomased to the proximal jejunum. Patency of the NG tube is critical for preventing the retention of gastric secretions. The nurse should never irrigate or reposition the gastric tube after gastric surgery, unless specifically ordered by the physician. In this situation, the nurse would clarify the order.

Which assessment finding indicates that lactulose is effective in decreasing the ammonia level in the client with hepatic encephalopathy? A. Passage of two or three soft stools daily B. Evidence of watery diarrhea C. Daily deterioration in the client's handwriting D. Appearance of frothy, foul-smelling stools

A Lactulose reduces serum ammonia levels by inducing catharsis, subsequently decreasing colonic pH and inhibiting fecal flora from producing ammonia from urea. Ammonia is removed with the stool. Two or three soft stools daily indicate effectiveness of the drug. Watery diarrhea indicates overdose. Daily deterioration in the client's handwriting indicates an increase in the ammonia level and worsening of hepatic encephalopathy. Frothy, foul-smelling stools indicate steatorrhea, caused by impaired fat digestion.

Which of the following types of diets is implicated in the development of diverticulosis? A. Low-fiber diet B. High-fiber diet C. High-protein diet D. Low-carbohydrate diet

A Low-fiber diets have been implicated in the development of diverticula because these diets decrease the bulk in the stool and predispose the person to the development of constipation. A high-fiber diet is recommended to help prevent diverticulosis. A high-protein or low-carbohydrate diet has no effect on the development of diverticulosis.

The client has orders for a nasogastric (NG) tube insertion. During the procedure, instructions that will assist in the insertion would be: A. Instruct the client to tilt his head back for insertion in the nostril, then flex his neck for the final insertion B. After insertion into the nostril, instruct the client to extend his neck C. Introduce the tube with the client's head tilted back, then instruct him to keep his head upright for final insertion D. Instruct the client to hold his chin down, then back for insertion of the tube

A NG insertion technique is to have the client first tilt his head back for insertion into the nostril, then to flex his neck forward and swallow. Extension of the neck (2) will impede NG tube insertion.

Risk factors for the development of hiatal hernias are those that lead to increased abdominal pressure. Which of the following complications can cause increased abdominal pressure? A. Obesity B. Volvulus C. Constipation D. Intestinal obstruction

A Obesity may cause increased abdominal pressure that pushes the lower portion of the stomach into the thorax.

Which of the following aspects is the priority focus of nursing management for a client with peritonitis? A. Fluid and electrolyte balance B. Gastric irrigation C. Pain management D. Psychosocial issues

A Peritonitis can advance to shock and circulatory failure, so fluid and electrolyte balance is the priority focus of nursing management. Gastric irrigation may be needed periodically to ensure patency of the nasogastric tube. Although pain management is important for comfort and psychosocial care will address concerns such as anxiety, focusing on fluid and electrolyte imbalance will maintain hemodynamic stability.

Radiation therapy is used to treat colon cancer before surgery for which of the following reasons? A. Reducing the size of the tumor B. Eliminating the malignant cells C. Curing the cancer D. Helping the bowel heal after surgery

A Radiation therapy is used to treat colon cancer before surgery to reduce the size of the tumor, making it easier to be resected. Radiation therapy isn't curative, can't eliminate the malignant cells (though it helps define tumor margins), can could slow postoperative healing.

Which of the following measures should the nurse focus on for the client with esophageal varices? A. Recognizing hemorrhage B. Controlling blood pressure C. Encouraging nutritional intake D. Teaching the client about varices

A Recognizing the rupture of esophageal varices, or hemorrhage, is the focus of nursing care because the client could succumb to this quickly. Controlling blood pressure is also important because it helps reduce the risk of variceal rupture. It is also important to teach the client what varices are and what foods he should avoid such as spicy foods.

Side effects of loperamide (Imodium) include all of the following except? A. Diarrhea B. Epigastric pain C. Dry mouth D. Anorexia

A Side effects associated with loperamide include CNS fatigue and dizziness, epigastric pain, abdominal cramps, nausea, dry mouth, vomiting, and anorexia. Diarrhea is an indication, not a side effect.

Nurse Juvy is caring for a client with cirrhosis of the liver. To minimize the effects of the disorder, the nurse teaches the client about foods that are high in thiamine. The nurse determines that the client has the best understanding of the dietary measures to follow if the client states an intention to increase the intake of: A. Pork B. Milk C. Chicken D. Broccoli

A The client with cirrhosis needs to consume foods high in thiamine. Thiamine is present in a variety of foods of plant and animal origin. Pork products are especially rich in this vitamin. Other good food sources include nuts, whole grain cereals, and legumes. Milk contains vitamins A, D, and B2. Poultry contains niacin. Broccoli contains vitamins C, E, and K and folic acid

Fistulas are most common with which of the following bowel disorders? A. Crohn's disease B. Diverticulitis C. Diverticulosis D. Ulcerative colitis

A The lesions of Crohn's disease are transmural; that is, they involve all thickness of the bowel. These lesions may perforate the bowel wall, forming fistulas with adjacent structures. Fistulas don't develop in diverticulitis or diverticulosis. The ulcers that occur in the submucosal and mucosal layers of the intestine in ulcerative colitis usually don't progress to fistula formation as in Crohn's disease.

The nurse is doing an admission assessment on a client with a history of duodenal ulcer. To determine whether the problem is currently active, the nurse would assess the client for which of the following most frequent symptom(s) of duodenal ulcer? A. Pain that is relieved by food intake B. Pain that radiated down the right arm C. N/V D. Weight loss

A The most frequent symptom of duodenal ulcer is pain that is relieved by food intake. These clients generally describe the pain as burning, heavy, sharp, or "hungry" pain that often localizes in the midepigastric area. The client with duodenal ulcer usually does not experience weight loss or N/V. These symptoms are usually more typical in the client with a gastric ulcer.

The client with Crohn's disease has a nursing diagnosis of acute pain. The nurse would teach the client to avoid which of the following in managing this problem? A. Lying supine with the legs straight B. Massaging the abdomen C. Using antispasmodic medication D. Using relaxation techniques

A The pain associated with Crohn's disease is alleviated by the use of analgesics and antispasmodics and also is reduced by having the client practice relaxation techniques, applying local cold or heat to the abdomen, massaging the abdomen, and lying with the legs flexed. Lying with the legs extended is not useful because it increases the muscle tension in the abdomen, which could aggravate the inflamed intestinal tissues as the abdominal muscles are stretched.

The client has been admitted with a diagnosis of acute pancreatitis. The nurse would assess this client for pain that is: A. Severe and unrelenting, located in the epigastric area and radiating to the back. B. Severe and unrelenting, located in the left lower quadrant and radiating to the groin. C. Burning and aching, located in the epigastric area and radiating to the umbilicus. D. Burning and aching, located in the left lower quadrant and radiating to the hip.

A The pain associated with acute pancreatitis is often severe and unrelenting, is located in the epigastric region, and radiates to the back.

A 40-year-old male client has been hospitalized with peptic ulcer disease. He is being treated with a histamine receptor antagonist (cimetidine), antacids, and diet. The nurse doing discharge planning will teach him that the action of cimetidine is to: A. Reduce gastric acid output B. Protect the ulcer surface C. Inhibit the production of hydrochloric acid (HCl) D. Inhibit vagus nerve stimulation

A These drugs inhibit action of histamine on the H2 receptors of parietal cells, thus reducing gastric acid output.

The nurse is teaching the client how to perform a colostomy irrigation. To enhance the effectiveness of the irrigation and fecal returns, what measure should the nurse instruct the client to do? A. Increase fluid intake B. Reduce the amount of irrigation solution C. Perform the irrigation in the evening D. Place heat on the abdomen

A To enhance effectiveness of the irrigation and fecal returns, the client is instructed to increase fluid intake and prevent constipation.

The nurse is teaching a female client how to perform a colostomy irrigation. To enhance the effectiveness of the irrigation and fecal returns, what measure should the nurse instruct the client to do? A. Increase fluid intake B. Place heat on the abdomen C. Perform the irrigation in the evening D. Reduce the amount of irrigation solution

A To enhance effectiveness of the irrigation and fecal returns, the client is instructed to increase fluid intake and to take other measures to prevent constipation. Options B, C and D will not enhance the effectiveness of this procedure.

When teaching a community group about measures to prevent colon cancer, which instruction should the nurse include? A. "Limit fat intake to 20% to 25% of your total daily calories." B. "Include 15 to 20 grams of fiber into your daily diet." C. "Get an annual rectal examination after age 35." D. "Undergo sigmoidoscopy annually after age 50."

A To help prevent colon cancer, fats should account for no more than 20% to 25% of total daily calories and the diet should include 25 to 30 grams of fiber per day. A digital rectal examination isn't recommended as a stand-alone test for colorectal cancer. For colorectal cancer screening, the American Cancer society advises clients over age 50 to have a flexible sigmoidoscopy every 5 years, yearly fecal occult blood tests, yearly fecal occult blood tests PLUS a flexible sigmoidoscopy every 5 years, a double-contrast barium enema every 5 years, or a colonoscopy every 10 years.

A client has a percutaneous endoscopic gastrostomy tube inserted for tube feedings. Before starting a continuous feeding, the nurse should place the client in which position? A. Semi-Fowlers B. Supine C. Reverse Trendelenburg D. High Fowler's

A To prevent aspiration of stomach contents, the nurse should place the client in semi-Fowler's position. High Fowler's position isn't necessary and may not be tolerated as well as semi-Fowler's.

Nurse Oliver checks for residual before administering a bolus tube feeding to a client with a nasogastric tube and obtains a residual amount of 150 mL. What is appropriate action for the nurse to take? A. Hold the feeding B. Reinstill the amount and continue with administering the feeding C. Elevate the client's head at least 45 degrees and administer the feeding D. Discard the residual amount and proceed with administering the feeding

A Unless specifically indicated, residual amounts more than 100 mL require holding the feeding. Therefore options B, C, and D are incorrect. Additionally, the feeding is not discarded unless its contents are abnormal in color or characteristics

A patient with chronic alcohol abuse is admitted with liver failure. You closely monitor the patient's blood pressure because of which change that is associated with the liver failure? A. Hypoalbuminemia B. Increased capillary permeability C. Abnormal peripheral vasodilation D. Excess rennin release from the kidneys

A Blood pressure decreases as the body is unable to maintain normal oncotic pressure with liver failure, so patients with liver failure require close blood pressure monitoring. Increased capillary permeability, abnormal peripheral vasodilation, and excess rennin released from the kidney's aren't direct ramifications of liver failure.

A client diagnosed with acute pancreatitis is being transferred to another facility. The nurse caring for the client completes the transfer summary, which includes information about the client's drinking history and other assessment findings. Which assessment findings confirm his diagnosis? a) Recent weight loss and temperature elevation b) Presence of easy bruising and bradycardia c) Adventitious breath sounds and hypertension d) Presence of blood in the client's stool and recent hypertension

A client diagnosed with acute pancreatitis is being transferred to another facility. The nurse caring for the client completes the transfer summary, which includes information about the client's drinking history and other assessment findings. Which assessment findings confirm his diagnosis? a) Recent weight loss and temperature elevation b) Presence of easy bruising and bradycardia c) Adventitious breath sounds and hypertension d) Presence of blood in the client's stool and recent hypertension Recent weight loss and temperature elevation Assessment findings associated with pancreatitis include recent weight loss and temperature elevation. Inflammation of the pancreas causes a response that elevates temperature and leads to abdominal pain that typically occurs with eating. Nausea and vomiting may occur as a result of pancreatic tissue damage that's caused by the activation of pancreatic enzymes. The client may experience weight loss because of the lost desire to eat. Blood in stools and recent hypertension aren't associated with pancreatitis; fatty diarrhea and hypotension are usually present. Presence of easy bruising and bradycardia aren't found with pancreatitis; the client typically experiences tachycardia, not bradycardia. Adventitious breath sounds and hypertension aren't associated with pancreatitis.

1. Assist the client into the left-lateral Sims' position., 2. Lubricate the distal end of the rectal catheter., 6. Be sure to keep the solution container below 18" above bed level.

A client with constipation is prescribed an irrigating enema. Which steps should the nurse take when administering an enema?

Which of the following nursing interventions should the nurse perform for a female client receiving enteral feedings through a gastrostomy tube? a. Change the tube feeding solutions and tubing at least every 24 hours. b. Maintain the head of the bed at a 15-degree elevation continuously. c. Check the gastrostomy tube for position every 2 days. d. Maintain the client on bed rest during the feedings.

A male client with pancreatitis complains of pain. The nurse expects the physician to prescribe meperidine (Demerol) instead of morphine to relieve pain because: a. meperidine provides a better, more prolonged analgesic effect. b. morphine may cause spasms of Oddi's sphincter. c. meperidine is less addictive than morphine. d. morphine may cause hepatic dysfunction.

A patient with a history of peptic ulcer disease has presented to the emergency department with complaints of severe abdominal pain and a rigid, boardlike abdomen, prompting the health care team to suspect a perforated ulcer. Which of the following actions should the nurse anticipate? A. Providing IV fluids and inserting a nasogastric tube B. Administering oral bicarbonate and testing the patient's gastric pH level. C. Performing a fecal occult blood test and administering IV calcium gluconate. D. Starting parenteral nutrition and placing the patient in high-Fowler's position

A perforated peptic ulcer requires IV replacement of fluid losses and continued gastric aspiration by NG tube. Nothing is given by mouth and gastric pH testing is not a priority. Calcium gluconate is not a medication directly relevant to the patient's suspected diagnosis and parenteral nutrition is not a priority in the short term

A 72-year-old patient was admitted with epigastric pain due to a gastric ulcer. Which patient assessment warrants an urgent change in the nursing plan of care? A) Chest pain relieved with eating or drinking water B) Back pain 3 or 4 hours after eating a meal C) Burning epigastric pain 90 minutes after breakfast D) Rigid abdomen and vomiting following indigestion

A rigid abdomen with vomiting in a patient who has a gastric ulcer indicates a perforation of the ulcer, especially if the manifestations of perforation appear suddenly. Midepigastric pain is relieved by eating, drinking water, or antacids with duodenal ulcers, not gastric ulcers. Back pain 3-4 hours after a meal is more likely to occur with a duodenal ulcer. Burning epigastric pain 1-2 hours after a meal is an expected manifestation of a gastric ulcer related to increased gastric secretions and does not cause an urgent change in the nursing plan of care.

The nurse is checking the residual content for a client who is receiving intermittent feedings. Which residual content, if obtained, would lead the nurse to delay the feeding? a) 120 mL b) 60 mL c) 30 mL d) 90 mL

A) 120 mL Feedings typically are delayed if the residual content measures more than 100 mL for intermittent feedings or 10% to 20% of the hourly amount of a continuous feeding. Thus a residual content of 120 mL would require the nurse to delay the feeding.

When preparing a patient for a capsule endoscopy study, what should the nurse do? A) Ensure the patient understands the required bowel preparation. B) Have the patient return to the procedure room for removal of the capsule. C) Teach the patient to maintain a clear liquid diet throughout the procedure. D) Explain to the patient that conscious sedation will be used during placement of the capsule.

A) A capsule endoscopy study involves the patient performing a bowel prep to cleanse the bowel before swallowing the capsule. The patient will be on a clear liquid diet for 1 to 2 days before the procedure and will remain NPO for 4 to 6 hours after swallowing the capsule. The capsule is disposable and will pass naturally with the bowel movement, although the monitoring device will need to be removed.

After assessing a client with peritonitis, the nurse most likely would document the client's bowel sounds as: a) Absent. b) High-pitched. c) Mild. d) Hyperactive.

A) Absent Since lack of bowel motility typically accompanies peritonitis, bowel sounds are absent. Therefore, the nurse will not observe mild, high-pitched, or hyperactive bowel sounds.

A client with liver and renal failure has severe ascites. On initial shift rounds, his primary nurse finds his indwelling urinary catheter collection bag too full to store more urine. The nurse empties more than 2,000 ml from the collection bag. One hour later, she finds the collection bag full again. The nurse notifies the physician, who suspects that a bladder rupture is allowing the drainage of peritoneal fluid. The physician orders a urinalysis to be obtained immediately. The presence of which substance is considered abnormal? a) Albumin b) Chloride c) Creatinine d) Urobilinogen

A) Albumin Albumin is an abnormal finding in a routine urine specimen. Ascites present in liver failure contain albumin; therefore, if the bladder ruptured, ascites containing albumin would drain from the indwelling urinary catheter because the catheter is no longer contained in the bladder. Creatinine, urobilinogen, and chloride are normally found in urine.

A client with liver and renal failure has severe ascites. On initial shift rounds, his primary nurse finds his indwelling urinary catheter collection bag too full to store more urine. The nurse empties more than 2,000 ml from the collection bag. One hour later, she finds the collection bag full again. The nurse notifies the physician, who suspects that a bladder rupture is allowing the drainage of peritoneal fluid. The physician orders a urinalysis to be obtained immediately. The presence of which substance is considered abnormal? a) Albumin b) Chloride c) Urobilinogen d) Creatinine

A) Albumin Albumin is an abnormal finding in a routine urine specimen. Ascites present in liver failure contain albumin; therefore, if the bladder ruptured, ascites containing albumin would drain from the indwelling urinary catheter because the catheter is no longer contained in the bladder. Creatinine, urobilinogen, and chloride are normally found in urine.

When caring for a client with an acute exacerbation of a peptic ulcer, the nurse finds the client doubled up in bed with severe pain to his right shoulder. The intial appropriate action by the nurse is to a) Assess the client's abdomen and vital signs. b) Irrigate the client's NG tube. c) Place the client in the high-Fowler's position. d) Notify the health care provider.

A) Assess the client's abdomen and vital signs Signs and symptoms of perforation includes sudden, severe upper abdominal pain (persisting and increasing in intensity); pain may be referred to the shoulders, especially the right shoulder, because of irritation of the phrenic nerve in the diaphragm. The nurse should assess the vital signs and abdomen prior to notifying the physician. Irrigation of the NG tube should not be performed because the additional fluid may be spilled into the peritoneal cavity, and the client should be placed in a position of comfort, usually on the side with the head slightly elevated.

The nurse is to obtain a stool specimen from a client who reported that he is taking iron supplements. The nurse would expect the stool to be which color? a) Black b) Red c) Dark brown d) Green

A) Black Ingestion of iron can cause the stool to turn black. Meat protein causes stool to appear dark brown. Ingestion of large amounts of spinach may turn stool green while ingestion of carrots and beets may cause stool to turn red.

A client is admitted to the health care facility with a diagnosis of a bleeding gastric ulcer. The nurse expects this client's stools to be: a) black and tarry. b) coffee-ground-like. c) bright red. d) clay-colored.

A) Black and tarry Black, tarry stools are a sign of bleeding high in the GI tract, as from a gastric ulcer, and result from the action of digestive enzymes on the blood. Vomitus associated with upper GI tract bleeding commonly is described as coffee-ground-like. Clay-colored stools are associated with biliary obstruction. Bright red stools indicate lower GI tract bleeding.

Which of the following is an accurate statement regarding cancer of the esophagus? a) Chronic irritation of the esophagus is a known risk factor. b) It is three times more common in women in the U.S. than men. c) It is seen more frequently in Caucasian Americans than in African Americans. d) It usually occurs in the fourth decade of life.

A) Chronic irritation of the esophagus is a known risk factor In the United States, cancer of the esophagus has been associated with the ingestion of alcohol and the use of tobacco. In the United States, carcinoma of the esophagus occurs more than three times more often in men as in women. It is seen more frequently in African Americans than in Caucasian Americans. It usually occurs in the fifth decade of life

A client is admitted for suspected GI disease. Assessment data reveal muscle wasting, a decrease in chest and axillary hair, and increased bleeding tendency. The nurse suspects the client has: a) cirrhosis. b) cholelithiasis. c) appendicitis. d) peptic ulcer disease.

A) Cirrhosis Muscle wasting, a decrease in chest and axillary hair, and increased bleeding tendencies are all symptoms of cirrhosis. The client may also have mild fever, edema, abdominal pain, and an enlarged liver. Clients with peptic ulcer disease complain of a dull, gnawing epigastric pain that's relieved by eating. Appendicitis is characterized by a periumbilical pain that moves to the right lower quadrant and rebound tenderness. Cholelithiasis is characterized by severe abdominal pain that presents several hours after a large meal.

A nurse is reviewing the history and physical of a client admitted for a hemorrhoidectomy. Which predisposing condition does the nurse expect to see? a) Constipation b) Hypoglycemia c) Lactic acidosis d) Hyperkalemia

A) Constipation Orthostatic hypertension and other conditions associated with persistently high intra-abdominal pressure (such as pregnancy) can lead to hemorrhoids. The passing of hard stools, not diarrhea, can aggravate hemorrhoids. Diverticulosis has no relationship to hemorrhoids. Rectal bleeding is a symptom of hemorrhoids, not a predisposing condition.

Which of the following is the primary symptom of achalasia? a) Difficulty swallowing b) Pulmonary symptoms c) Chest pain d) Heartburn

A) Difficulty swallowing The primary symptom of achalasia is difficulty in swallowing both liquids and solids. The patient may also report chest pain and heartburn that may or may not be associated with eating. Secondary pulmonary complications may result from aspiration of gastric contents.

The client cannot tolerate oral feedings due to an intestinal obstruction and is NPO. A central line has been inserted, and the client is being started on parenteral nutrition (PN). The nurse performs the following actions while the client receives PN (select all that apply): a) Document intake and output. b) Use clean technique for all catheter dressing changes. c) Weigh the client every day. d) Cover insertion site with a transparent dressing that is changed daily. e) Check blood glucose level every 6 hours.

A) Document intake and output; C) Weigh the client every day; E) Check blood glucose level every 6 hours When a client is receiving PN through a central line, the nurse weighs the client daily, checks blood glucose level every 6 hours, and documents intake and output. These actions are to ensure the client is receiving optimal nutrition. The nurse also performs activities to prevent infection, such as covering the insertion site with a transparent dressing that is changed weekly and/or prn and using sterile technique during catheter site dressing changes.

After administering a dose of promethazine (Phenergan) to a patient with nausea and vomiting, the nurse explains that which of the following may be experienced as a common temporary adverse effect of the medication? A) Drowsiness B) Reduced hearing C) Sensation of falling D) Photosensitivity

A) Drowsiness Although being given to this patient as an antiemetic, promethazine also has sedative and amnesic properties. For this reason, the patient is likely to experience drowsiness as an adverse effect of the medication.

A nurse is preparing a client with Crohn's disease for a barium enema. What should the nurse do the day before the test? a) Encourage plenty of fluids. b) Order a high-fiber diet. c) Serve dairy products. d) Serve the client his usual diet.

A) Encourage plenty of fluids The nurse should encourage plenty of fluids because adequate fluid intake is necessary to avoid dehydration that may be caused by the bowel preparation and to prevent fecal impaction after the procedure. The client may be placed on a low-residue diet 1 to 2 days before the procedure to reduce the contents in the GI tract. Fiber intake is limited in a low-residue diet. Because dairy products leave a residue, they aren't allowed the evening before the test. Clear liquids only are allowed the evening before the test.

When reviewing the history of a client with pancreatic cancer, the nurse would identify which of the following as a possible risk factor? a) Ingestion of caffeinated coffee b) Ingestion of a low-fat diet c) History of pancreatitis d) One-time exposure to petrochemicals

C) History of pancreatitis Pancreatitis is associated with the development of pancreatic cancer. Other factors that correlate with pancreatic cancer include diabetes mellitus, a high-fat diet, and chronic exposure to carcinogenic substances (i.e., petrochemicals). Although data are inconclusive, a relationship may exist between cigarette smoking and high coffee consumption (especially decaffeinated coffee) and the development of pancreatic carcinoma.

A patient had a stomach resection for stomach cancer. The nurse should teach the patient about the loss of the hormone that stimulates gastric acid secretion and motility and maintains lower esophageal sphincter tone. Which hormone will be decreased with a gastric resection? A) Gastrin B) Secretin C) Cholecystokinin D) Gastric inhibitory peptide

A) Gastrin is the hormone activated in the stomach (and duodenal mucosa) by stomach distention that stimulates gastric acid secretion and motility and maintains lower esophageal sphincter tone. Secretin inhibits gastric motility and acid secretion and stimulates pancreatic bicarbonate secretion. Cholecystokinin allows increased flow of bile into the duodenum and release of pancreatic digestive enzymes. Gastric inhibitory peptide inhibits gastric acid secretion and motility.

Which diagnostic test is used first to evaluate a client with upper GI bleeding? a) Hemoglobin levels and hematocrit (HCT) b) Endoscopy c) Arteriography d) Upper GI series

A) Hemoglobin levels and hematocrit Hemoglobin and HCT are typically performed first in clients with upper GI bleeding to evaluate the extent of blood loss. Endoscopy is then performed to directly visualize the upper GI tract and locate the source of bleeding. An upper GI series, or barium study, usually isn't the diagnostic method of choice, especially in a client with acute active bleeding who's vomiting and unstable. An upper GI series is also less accurate than endoscopy. Although an upper GI series might confirm the presence of a lesion, it wouldn't necessarily reveal whether the lesion is bleeding. Arteriography is an invasive study associated with life-threatening complications and wouldn't be used for an initial evaluation.

Diet therapy for patients diagnosed with IBS include which of the following? a) High-fiber diet b) Fluids with meals c) Caffeinated products d) Spicy foods

A) High fiber diet A high-fiber diet is prescribed to help control diarrhea and constipation. Foods that are possible irritants such as caffeine, spicy foods, lactose, beans, fried foods, corn, wheat, alcohol should be avoided. Fluids should not be taken with meals because this results in abdominal distention.

Patients diagnosed with esophageal varices are at risk for hemorrhagic shock. Which of the following is a sign of potential hypovolemia? a) Hypotension b) Bradycardia c) Polyuria d) Warm moist skin

A) Hypotension Signs of potential hypovolemia include cool, clammy skin, tachycardia, decreased blood pressure, and decreased urine output.

Patients diagnosed with esophageal varices are at risk for hemorrhagic shock. Which of the following is a sign of potential hypovolemia? a) Hypotension b) Bradycardia c) Polyuria d) Warm moist skin

A) Hypotension Signs of potential hypovolemia include cool, clammy skin, tachycardia, decreased blood pressure, and decreased urine output.

Patients diagnosed with esophageal varices are at risk for hemorrhagic shock. Which of the following is a sign of potential hypovolemia? a) Hypotension b) Bradycardia c) Warm moist skin d) Polyuria

A) Hypotension Signs of potential hypovolemia include cool, clammy skin, tachycardia, decreased blood pressure, and decreased urine output.

A physician plans to send a client home with supplies to complete a hemoccult test on all stools for 3 days. During the client education, the nurse informs the client to avoid which of the following medications while collecting stool for the test? a) ibuprofen (Advil) b) ciprofloxacin (Cipro XR) c) docusate sodium (Colace) d) acetaminophen (Tylenol)

A) Ibprofen (Advil) Fecal occult blood testing (FOBT) is one of the most commonly performed stool tests. FOBT can be done at the bedside, in the physician's office, or at home. The client is taught to avoid aspirin, red meats, nonsteroidal antiinflammatory agents, and horseradish for 72 hours prior to the examination. Advil is an anti-inflammatory drug and should be avoided with FOBT.

The patient with cirrhosis is being taught self-care. Which statement indicates the patient needs more teaching? A) "If I notice a fast heart rate or irregular beats, this is normal for cirrhosis." B) "I need to take good care of my belly and ankle skin where it is swollen." C) "A scrotal support may be more comfortable when I have scrotal edema." D) "I can use pillows to support my head to help me breathe when I am in bed."

A) If the patient with cirrhosis experiences a fast or irregular heart rate, it may be indicative of hypokalemia and should be reported to the health care provider, as this is not normal for cirrhosis. Edematous tissue is subject to breakdown and needs meticulous skin care. Pillows and a semi-Fowler's or Fowler's position will increase respiratory efficiency. A scrotal support may improve comfort if there is scrotal edema

Two days following a colectomy for an abdominal mass, a patient reports gas pains and abdominal distention. The nurse plans care for the patient based on the knowledge that the symptoms occur as a result of which of the following? A) Impaired peristalsis B) Irritation of the bowel C) Nasogastric suctioning D) Anastomosis site inflammation

A) Impaired peristalsis Until peristalsis returns to normal following anesthesia, the patient may experience slowed gastrointestinal motility leading to gas pains and abdominal distention.

The wound, ostomy, and continence (WOC) nurse selects the site where the ostomy will be placed. What should be included in the consideration for the site? A) The patient must be able to see the site. B) Outside the rectus muscle area is the best site. C) It is easier to seal the drainage bag to a protruding area. D) The ostomy will need irrigation, so area should not be tender.

A) In selection of the ostomy site, the WOC nurse will want a site visible to the patient so the patient can take care of it, within the rectus muscle to avoid hernias, and on a flat surface to more easily create a good seal with the drainage bag.

A nursing instructor tells the class that review of oral hygiene is an important component during assessment of the gastrointestinal system. One of the students questions this statement. Which of the following explanations from the nurse educator is most appropriate? a) "Injury to oral mucosa or tooth decay can lead to difficulty in chewing food." b) "Mouth sores are caused by bacteria that can thin the villi of the small intestine." c) "Decaying teeth secrete toxins that decrease the absorption of nutrients." d) "Bad breath will encourage ingestion of fatty foods to mask odor."

A) Injury to the oral mucosa or tooth decay can lead to difficulty in chewing food Poor oral hygiene can result in injury to the oral mucosa, lip, or palate; tooth decay; or loss of teeth. Such problems may lead to disruption in the digestive system. The ability to chew food or even swallow may be hindered.

After 20 seconds of auscultating for bowel sounds on a client recovering from abdominal surgery, the nurse hears nothing. Which of the following should the nurse do based on the assessment findings? a) Listen longer for the sounds. b) Call the physician to report absent bowel sounds. c) Document that the client is constipated. d) Return in 1 hour and listen again to confirm findings.

A) Listen longer for sounds Auscultation is used to determine the character, location, and frequency of bowel sounds. The frequency and character of sounds are usually heard as clicks and gurgles that occur irregularly and range from 5 to 35 per minutes. Normal sounds are heard about every 5 to 20 seconds, whereas hypoactive sounds can be one or two sounds in 2 minutes. Postoperatively, it is common for sounds to be reduced; therefore, the nurse needs to listen at least 3 to 5 minutes to verify absent or no bowel sounds.

A physician orders spironolactone (Aldactone), 50 mg by mouth four times daily, for a client with fluid retention caused by cirrhosis. Which finding indicates that the drug is producing a therapeutic effect? a) Loss of 2.2 lb (1 kg) in 24 hours b) Serum potassium level of 3.5 mEq/L c) Blood pH of 7.25 d) Serum sodium level of 135 mEq/L

A) Loss of 2.2 lb (1 kg) in 24 hours Daily weight measurement is the most accurate indicator of fluid status; a loss of 2.2 lb (1 kg) indicates loss of 1 L of fluid. Because spironolactone is a diuretic, weight loss is the best indicator of its effectiveness. This client's serum potassium and sodium levels are normal. A blood pH of 7.25 indicates acidosis, an adverse reaction to spironolactone.

The nurse is caring for a 68-year-old patient admitted with abdominal pain, nausea, and vomiting. The patient has an abdominal mass and a bowel obstruction is suspected. The nurse auscultating the abdomen listens for which of the following types of bowel sounds that is consistent with the patient's clinical picture? A) Low pitched and rumbling above the area of obstruction B) High pitched and hypoactive below the area of obstruction C) Low pitched and hyperactive below the area of obstruction D) High pitched and hyperactive above the area of obstruction

A) Low pitched and rumbling above the area of obstruction Early in intestinal obstruction, the patient's bowel sounds are hyperactive and high pitched, sometimes referred to as "tinkling" above the level of the obstruction. This occurs because peristaltic action increases to "push past" the area of obstruction. As the obstruction becomes complete, bowel sounds decrease and finally become absent.

A physician has ordered a liver biopsy for a client whose condition is deteriorating. Which of the following places the client at high risk due to her altered liver function during the biopsy? a) Low platelet count b) Low hemoglobin c) Decreased prothrombin time d) Low sodium level

A) Low platelet count Certain blood tests provide information about liver function. Prolonged prothrombin time (PT) and low platelet count place the client at high risk for hemorrhage. The client may receive intravenous (IV) administration of vitamin K or infusions of platelets before liver biopsy to reduce the risk of bleeding.

The nurse is conducting discharge teaching for a patient with metastatic lung cancer who was admitted with a bowel impaction. Which of the following instructions would be most helpful to prevent further episodes of constipation? A) Maintain a high intake of fluid and fiber in the diet. B) Reduce intake of medications causing constipation. C) Eat several small meals per day to maintain bowel motility. D) Sit upright during meals to increase bowel motility by gravity.

A) Maintain a high intake of fluid and fiber int he diet Increased fluid intake and a high-fiber diet reduce the incidence of constipation caused by immobility, medications, and other factors. Fluid and fiber provide bulk that in turn increases peristalsis and bowel motility.

A patient with type 2 diabetes and cirrhosis asks the nurse if it would be okay to take silymarin (milk thistle) to help minimize liver damage. The nurse responds based on knowledge that A) Milk thistle may affect liver enzymes and thus alter drug metabolism. B) Milk thistle is generally safe in recommended doses for up to 10 years. C) There is unclear scientific evidence for the use of milk thistle in treating cirrhosis. D) Milk thistle may elevate the serum glucose levels and is thus contraindicated in diabetes.

A) Milk thistle may affect liver enzymes and thus alter drug metabolism There is good scientific evidence for the use of milk thistle as an antioxidant to protect the liver cells from toxic damage in the treatment of cirrhosis. It is noted to be safe for up to 6 years, not 10 years, and it may lower, not elevate, blood glucose levels. It does affect liver enzymes and thus could alter drug metabolism. Therefore patients will need to be monitored for drug interactions.

A client recovering from gastric bypass surgery accidentally removes the nasogastric tube. It is best for the nurse to a) Notify the surgeon about the tube's removal. b) Reinsert the nasogastric tube to the stomach. c) Document the discontinuation of the nasogastric tube. d) Place the nasogastric tube to the level of the esophagus.

A) Notify the surgeon about the tube's removal If the nasogastric tube is removed accidently in a client who has undergone esophageal or gastric surgery, it is usually replaced by the physician. Care is taken to avoid trauma to the suture line. The nurse will not insert the tube to the esophagus or to the stomach in this situation. The nurse needs to do more than just document its removal. The nurse needs to notify the physician who will make a determination of leaving out or inserting a new nasogastric tube.

A stroke patient who primarily uses a wheelchair for mobility has diarrhea with fecal incontinence. What should the nurse assess first? A) Fecal impaction B) Perineal hygiene C) Dietary fiber intake D) Antidiarrheal agent use

A) Patients with limited mobility are at risk for fecal impactions due to constipation that may lead to liquid stool leaking around the hardened impacted feces, so assessing for fecal impaction is the priority. Perineal hygiene can be assessed at the same time. Assessing the dietary fiber and fluid intake and antidiarrheal agent use will be assessed and considered next.

Which of the following is caused by improper catheter placement and inadvertent puncture of the pleura? a) Pneumothorax b) Sepsis c) Fluid overload d) Air embolism

A) Pneumothorax A pneumothorax is caused by improper catheter placement and inadvertent puncture of the pleura. Air embolism can occur from a missing cap on a port. Sepsis can be caused by the separation of dressings. Fluid overload is caused by fluids infusing too rapidly.

A nurse is interviewing a client about his past medical history. Which preexisting condition may lead the nurse to suspect that a client has colorectal cancer? a) Polyps b) Weight gain c) Hemorrhoids d) Duodenal ulcers

A) Polyps Colorectal polyps are common with colon cancer. Duodenal ulcers and hemorrhoids aren't preexisting conditions of colorectal cancer. Weight loss — not gain — is an indication of colorectal cancer.

The most common cause of esophageal varices includes which of the following? a) Portal hypertension b) Asterixis c) Jaundice d) Ascites

A) Portal hypertension Esophageal varices are almost always caused by portal hypertension, which results from obstruction of the portal circulation within the damaged liver. Jaundice occurs when the bilirubin concentration in the blood is abnormally elevated. Ascites results from circulatory changes within the diseased liver. Asterixis is an involuntary flapping movement of the hands associated with metabolic liver dysfunction.

The nurse observes dry mucous membranes in a client who is receiving tube feedings after an oral surgery. The client also complains of unpleasant tastes and odors. Which of the following measures should be included in the client's plan of care? a) Provide frequent mouth care. b) Keep the feeding formula refrigerated. c) Ensure adequate hydration with additional water. d) Flush the tube with water before adding the feedings.

A) Provide frequent mouth care Frequent mouth care helps to relieve the discomfort from dryness and unpleasant odors and tastes. It can be done with the help of ice chips and analgesic throat lozenges, gargles, or sprays. Adequate hydration is essential. If urine output is less than less than 500 mL/day, formula and additional water can be given as ordered. Keeping the feeding formula refrigerated and unopened until it is ready for use and flushing the tube with water before adding feedings are measures to protect the client from infections.

A patient with a history of peptic ulcer disease has presented to the emergency department with complaints of severe abdominal pain and a rigid, boardlike abdomen, prompting the health care team to suspect a perforated ulcer. Which of the following actions should the nurse anticipate? A) Providing IV fluids and inserting a nasogastric tube B) Administering oral bicarbonate and testing the patient's gastric pH level C) Performing a fecal occult blood test and administering IV calcium gluconate D) Starting parenteral nutrition and placing the patient in a high-Fowler's position

A) Providing IV fluids and inserting a NG tube A perforated peptic ulcer requires IV replacement of fluid losses and continued gastric aspiration by NG tube. Nothing is given by mouth and gastric pH testing is not a priority. Calcium gluconate is not a medication directly relevant to the patient's suspected diagnosis and parenteral nutrition is not a priority in the short term.

A pt with a history of peptic ulcer disease has presented to the emergency department with complaints of severe abdominal pain and a rigid, boardlike abdomen, prompting the health care team to suspect a perforated ulcer. Which of the following actions should the nurse anticipate? A) Providing IV fluids and inserting a nasogastric tube B) Administering oral bicarbonate and testing the patient's gastric pH level C) Performing a fecal occult blood test and administering IV calcium gluconate D) Starting parenteral nutrition and placing the patient in a high-Fowler's position

A) Providing IV fluids and inserting a nasogastric tube A perforated peptic ulcer requires IV replacement of fluid losses and continued gastric aspiration by NG tube. Nothing is given by mouth and gastric pH testing is not a priority. Calcium gluconate is not a medication directly relevant to the patient's suspected diagnosis and parenteral nutrition is not a priority in the short term.

A nurse is assisting with preoperative care for a client who requires an appendectomy. The nurse is aware that the surgery will involve which abdominal quadrant? A) RLQ B) RUQ C) LLQ D) LUQ

A) RLQ The appendix is in the right lower quadrant.

A patient scheduled to undergo an abdominal ultrasonography is advised to do which of the following? a) Restrict eating of solid food for 6 to 8 hours before the test. b) Do not consume anything sweet for 24 hours before the test c) Do not undertake any strenuous exercise for 24 hours before the test d) Avoid exposure to sunlight for at least 6 to 8 hours before the test

A) Restrict eating of solid food for 6 to 8 hours before the test For a patient who is scheduled to undergo an abdominal ultrasonography, the patient should restrict herself from solid food for 6 to 8 hours to avoid having images of her test obscured with gas and intestinal contents. Ultrasonography records the reflection of sound waves. Strenuous exercises, the consumption of sweets, and exposure to sunlight do not affect the results of the test in any way.

The nurse is caring for a patient in the emergency department with complaints of acute abdominal pain, nausea, and vomiting. When the nurse palpates the patient's left lower abdominal quadrant, the patient complains of pain in the right lower quadrant. The nurse will document this as which of the following diagnostic signs of appendicitis? A) Rovsing sign B) Referred pain C) Chvostek's sign D) Rebound tenderness

A) Rovsing sign In patients with suspected appendicitis, Rovsing sign may be elicited by palpation of the left lower quadrant, causing pain to be felt in the right lower quadrant.

Which of the following medications used for the treatment of obesity prevents the reuptake of serotonin and norepinephrine? a) Sibutramine hydrochloride (Meridia) b) Orlistat (Xenical) c) Bupropion hydrochloride (Wellbutrin) d) Fluoxetine hydrochloride (Prozac)

A) Sibutramine hydrochloride (Meridia) Sibutramine hydrochloride (Meridia) inhibits the reuptake of serotonin and norepinephrine. Meridia decreases appetite. Orlistat (Xenical) prevents the absorption of triglycerides. Side effects of Xenical may include increased bowel movements, gas with oily discharge, decreased food absorption, decreased bile flow, and decreased absorption of some vitamins. Bupropion hydrochloride (Wellbutrin) is an antidepressant medication. Fluoxetine hydrochloride (Prozac) has not been approved by the FDA for use in the treatment of obesity.

A client with acute liver failure exhibits confusion, a declining level of consciousness, and slowed respirations. The nurse finds him very difficult to arouse. The diagnostic information which best explains the client's behavior is: a) subnormal serum glucose and elevated serum ammonia levels. b) subnormal clotting factors and platelet count. c) elevated liver enzymes and low serum protein level. d) elevated blood urea nitrogen and creatinine levels and hyperglycemia.

A) Subnormal serum glucose and elevated serum ammonia levels In acute liver failure, serum ammonia levels increase because the liver can't adequately detoxify the ammonia produced in the GI tract. In addition, serum glucose levels decline because the liver isn't capable of releasing stored glucose. Elevated serum ammonia and subnormal serum glucose levels depress the level of a client's consciousness. Elevated liver enzymes, low serum protein level, subnormal clotting factors and platelet count, elevated blood urea nitrogen and creatine levels, and hyperglycemia aren't as directly related to the client's level of consciousness.

Blood shed in sufficient quantities into the upper GI tract, produces which color of stool? a) Tarry-black b) Milky white c) Green d) Bright red

A) Tarry-black Blood shed in sufficient quantities into the upper GI tract produces a tarry-black stool. Blood entering the lower portion of the GI tract or passing rapidly through it will appear bright or dark red. A milky white stool is indicative "of" a patient who received barium. A green stool is indicative of a patient who has eaten spinach.

The nurse is preparing to insert a nasogastric (NG) tube into a 68-year-old female patient who is nauseated and vomiting. She has an abdominal mass and suspected small intestinal obstruction. The patient asks the nurse why this procedure is necessary. What response by the nurse is most appropriate? A) "The tube will help to drain the stomach contents and prevent further vomiting." B) "The tube will push past the area that is blocked and thus help to stop the vomiting." C) "The tube is just a standard procedure before many types of surgery to the abdomen." D) "The tube will let us measure your stomach contents so that we can plan what type of IV fluid replacement would be best."

A) The NG tube is used to decompress the stomach by draining stomach contents and thereby prevent further vomiting. The NG tube will not push past the blocked area. Potential surgery is not currently indicated. The location of the obstruction will determine the type of fluid to use, not measure the amount of stomach contents.

A client with dysphagia is being prepared for discharge. Which outcome indicates that the client is ready for discharge? a) The client is free from esophagitis and achalasia. b) The client doesn't exhibit rectal tenesmus. c) The client has normal gastric structures. d) The client reports diminished duodenal inflammation.

A) The client is free from esophagitis and achalasia Dysphagia may be the reason why a client with esophagitis or achalasia seeks treatment. Therefore, when the client is free of esophagitis or achalasia, he is ready for discharge. Dysphagia isn't associated with rectal tenesmus, duodenal inflammation, or abnormal gastric structures.

A client with dysphagia is being prepared for discharge. Which outcome indicates that the client is ready for discharge? a) The client is free from esophagitis and achalasia. b) The client reports diminished duodenal inflammation. c) The client has normal gastric structures. d) The client doesn't exhibit rectal tenesmus.

A) The client is free from esophagitis and achalasia. Dysphagia may be the reason why a client with esophagitis or achalasia seeks treatment. Therefore, when the client is free of esophagitis or achalasia, he is ready for discharge. Dysphagia isn't associated with rectal tenesmus, duodenal inflammation, or abnormal gastric structures.

A nurse enters the room of a client who has returned to the unit after having a radical neck dissection. Which assessment finding requires immediate intervention? a) The client lying in a lateral position, with the head of bed flat b) Foley catheter bag containing 500 ml of amber urine c) Serosanguineous drainage on the dressing d) A piggyback infusion of levofloxacin (Levaquin)

A) The client lying in a lateral position, with the head of bed flat A client who has had neck surgery is at risk for neck swelling. To prevent respiratory complications, the head of the bed should be at least at a 30-degree angle. This position gives the lungs room to expand and decreases swelling by promoting venous and lymphatic drainage. This position also minimizes the risk of aspiration. Serosanguineous drainage on the dressing, a Foley bag containing amber urine, and levofloxacin infusing aren't causes for concern.

Which outcome indicates effective client teaching to prevent constipation? a) The client reports engaging in a regular exercise regimen. b) The client limits water intake to three glasses per day. c) The client verbalizes consumption of low-fiber foods. d) The client maintains a sedentary lifestyle.

A) The client reports engaging in a regular exercise regimen. The client having a regular exercise program indicates effective teaching. A regular exercise regimen promotes peristalsis and contributes to regular bowel elimination patterns. A low-fiber diet, a sedentary lifestyle, and limited water intake would predispose the client to constipation.

When evaluating the patient's understanding about the care of the ileostomy, what statement by the patient indicates the patient needs more teaching? A) "I will be able to regulate when I have stools." B) "I will be able to wear the pouch until it leaks." C) "Dried fruit and popcorn must be chewed very well." D) "The drainage from my stoma can damage my skin."

A) The ileostomy is in the ileum and drains liquid stool frequently, unlike the colostomy which has more formed stool the further distal the ostomy is in the colon. The ileostomy pouch is usually worn 4-7 days or until it leaks. It must be changed immediately if it leaks because the drainage is very irritating to the skin. To avoid obstruction, popcorn, dried fruit, coconut, mushrooms, olives, stringy vegetables, food with skin, and meats with casings must be chewed extremely well before swallowing because of the narrow diameter of the ileostomy lumen.

The nurse is preparing to insert a nasogastric tube into a 68-year-old patient with an abdominal mass and suspected bowel obstruction. The patient asks the nurse why this procedure is necessary. Which of the following responses is most appropriate? A) "The tube will help to drain the stomach contents and prevent further vomiting." B) "The tube will push past the area that is blocked, and thus help to stop the vomiting." C) "The tube is just a standard procedure before many types of surgery to the abdomen." D) "The tube will let us measure your stomach contents, so that we can plan what type of IV fluid replacement would be best."

A) The tube will help drain the stomach contents and prevent further vomiting The nasogastric tube is used to decompress the stomach by draining stomach contents, and thereby prevent further vomiting.

A patient with type 2 diabetes and cirrhosis asks the nurse if it would be okay to take silymarin (milk thistle) to help minimize liver damage. The nurse responds based on knowledge that: A. Milk thistle may affect liver enzymes and thus alter drug metabolism. B. Milk thistle is generally safe in recommended doses for up to 10 years. C. is unclear scientific evidence for the use of milk thistle in treating cirrhosis. D. Milk thistle may elevate the serum glucose levels and is thus contraindicated in diabetes.

A) There is good scientific evidence that there is no real benefit from using milk thistle to protect the liver cells from toxic damage in the treatment of cirrhosis. Milk thistle does affect liver enzymes and thus could alter drug metabolism. Therefore patients will need to be monitored for drug interactions. It is noted to be safe for up to 6 years, not 10 years, and it may lower, not elevate, blood glucose levels.

When administered a dose of metoclopramide (Reglan), a patient complains of nausea. The nurse would teach the patient to report which of the following potential adverse effects? A) Tremors B) Constipation C) Double vision D) Numbness in the fingers and dose

A) Tremors (Extrapyramidal side effects, including tremors and dyskinesias, may occur as a result of metoclopramide (Reglan) administration.)

Two days following a colectomy for an abdominal mass, a patient reports gas pains and abdominal distention. The nurse plans care for the patient based on the knowledge that the symptoms are occurring as a result of A) impaired peristalsis. B) irritation of the bowel. C) nasogastric suctioning. D) inflammation of the incision site.

A) Until peristalsis returns to normal following anesthesia, the patient may experience slowed gastrointestinal motility leading to gas pains and abdominal distention. Irritation of the bowel, nasogastric suctioning, and inflammation of the surgical site do not cause gas pains or abdominal distention.

A client with GERD develops espophagitis. Which diagnostic test would the nurse expect the physician to order to confirm the diagnosis? a) Upper endoscopy with biopsy b) Stool testing for occult blood c) 24-hour esophageal pH monitoring d) Barium swallow

A) Upper endoscopy with biopsy Upper endoscopy with biopsy confirms esophagitis. Barium-swallow would reveal inflammation or stricture formation from chronic esophagitis. Tests of stool may show positive findings of blood. Ambulatory 24-hour esophageal pH monitoring allows for observation of the frequency of reflux episodes and their associated symptoms.

The nurse inserts a nasoduodenal tube for feeding of the client. To check best for placement, the nurse a) Verifies location with an abdominal x-ray b) Aspirates contents and checks the color of the aspirate c) Auscultates when injecting air d) Adds 8 to 10 inches of the tube after inserting to the xiphoid process

A) Verifies location with an abdominal x-ray Initially, an x-ray should be used to confirm placement of the nasoduodenal tube. It is the most accurate method to verify tube placement. Adding 8 to 10 inches to the length of the tube after measuring from nose to earlobe to xiphoid process is not supported, because it does not indicate that the tube will be in the correct position. Intestinal aspirate is usually clear and yellow to bile-colored. Gastric aspirate is usually cloudy and green, tan, off-white, or brown. Food particles may be present. The traditional method of injecting air through the tube while auscultating the epigastric area with a stethoscope to detect air insufflation is also an unreliable indicator.

Initially, which diagnostic should be completed following placement of a NG tube? a) X-ray b) Measurement of tube length c) pH measurement of aspirate d) Visual assessment of aspirate

A) X-ray Instead of auscultation, a combination of three methods is recommended: measurement of tube length, visual assessment of aspirate, and pH measurement of aspirate.

A 90-year-old healthy man is suffering from dysphagia. The nurse explains what age-related change of the GI tract is the most likely cause of his difficulty? A) Xerostomia B) Esophageal cancer C) Decreased taste buds D) Thinner abdominal wall

A) Xerostomia, decreased volume of saliva, leads to dry oral mucosa and dysphagia. Esophageal cancer is not an age-related change. Decreased taste buds and a thinner abdominal wall do not contribute to difficulty swallowing.

"A patient with a history of peptic ulcer disease has presented to the ED with complaints of severe abdominal pain and a rigid, boardlike abdome, prompting the health care team to suspect a perforated ulcer. Which of the following actions should the nurse anticiptate? source: http://quizlet.com/20002414/nclex-review-lower-gi-problems-ibd-crohns-disease-ulcerative-colitis-flash-cards/ or Lewis chapt. 42 Upper GI NCLEX" " A: Providing IV fluids and inserting a nasogastric tube B:Administering oral bicarbonate and testing patient's gastric pH level C:Performing a fecal occult blood test and administering IV calcium gluconate D: Starting parenteral nutrition and placing the patient in high-fowler's position

A, "A: providing IV fluids and inserting a nasogastric tube rationale: A perforated peptic ulcer requires IV replacement of fluid losses and continued gastric aspiration by NG tube. Nothing is given by mouth and gastric pH testing is not a priority. Calcium gluconate is not a medication directly relevent to the patient's suspected diagnosis and parenteral nutrition is not a priority in the short term

"A patient with a history of peptic ulcer disease has presented to the emergency department with complaints of severe abdominal pain and a rigid, boardlike abdomen, prompting the health care team to suspect a perforated ulcer. Which of the following actions should the nurse anticipate? A. Providing IV fluids and inserting a nasogastric tube B. Administering oral bicarbonate and testing the patient's gastric pH level C. Performing a fecal occult blood test and administering IV calcium gluconate D. Starting parenteral nutrition and placing the patient in a high-Fowler's position

A, A perforated peptic ulcer requires IV replacement of fluid losses and continued gastric aspiration by NG tube. Nothing is given by mouth and gastric pH testing is not a priority. Calcium gluconate is not a medication directly relevant to the patient's suspected diagnosis and parenteral nutrition is not a priority in the short term

A male patient suspected of having peptic ulcer disease (PUD) has received 3 weeks of drug therapy with a proton pump inhibitor but reports no improvement. What are some factors that may cause his lack of improvement (select all that apply)? A. He may not be compliant with taking medication B. He may need additional medication C. He is not drinking enough milk D. Symptoms may have a cardiac origin E. Symptoms may be caused by stomach cancer

A, B, D, E Rationale PUD is frequently treated empirically based on symptoms. The symptoms suggestive of PUD may also be caused by heart disease or stomach cancer. It is most important to rule out stomach cancer in a patient who has had no relief after 3 weeks of treatment for PUD. A benign ulcer should start to heal if the patient has been compliant with taking the medication. If he has been compliant, he may need additional medication to treat PUD. Reference: 1000

The client attends two sessions with the dietitian to learn about diet modifications to minimize gastroesophageal reflux. The teaching would be considered successful if the client says that she will decrease her intake of which of the following foods? A) Fats, B)Carbohydrates, C) High-calcium foods, D) High-Sodium foods

A, Fats are associated with decreased esophageal sphincter tone, which increases reflux. Obesity contributes to the development of hiatal hernia, and a low-fat diet might also aid in weight loss. Carbohydrates and foods high in sodium or calcium do not affect gastroesophageal reflux

"The nurse is reviewing the record of a client with Crohn's disease. Which stool characteristic should the nurse expect to note documented in the client's record? " a. Diarrhea b. Chronic constipation c. Constipation alternating with diarrhea d. Stool constantly oozing from the rectum

A,"(A) Crohns disease is characterized by nonbloody diarrhea of usually not more than 4 or 5 stools daily. overtime the stools increase frequency duration and severity

When caring for a patient with a biliary obstruction, the nurse will anticipate administering which of the following vitamin supplements (select all that apply)? A) Vitamin A B) Vitamin D C) Vitamin E D) Vitamin K E) Vitamin B

A,B,C,D Biliary obstruction prevents bile from entering the small intestine and thus prevents the absorption of fat-soluble vitamins. Vitamins A, D, E, and K are all fat soluble and thus would need to be supplemented in a patient with biliary obstruction.

When caring for a patient with a biliary obstruction, the nurse will anticipate administering which of the following vitamin supplements (select all that apply)? A. Vitamin A B. Vitamin D C. Vitamin E D. Vitamin K E. Vitamin B

A,B,C,D) Biliary obstruction prevents bile from entering the small intestine and thus prevents the absorption of fat-soluble vitamins. Vitamins A, D, E, and K are all fat-soluble and thus would need to be supplemented in a patient with biliary obstruction.

When caring for a patient with liver disease, the nurse recognizes the need to prevent bleeding resulting from altered clotting factors and rupture of varices. Which of the following nursing interventions would be appropriate to achieve this outcome (select all that apply)? A) Use smallest gauge possible when giving injections or drawing blood. B) Teach patient to avoid straining at stool, vigorous blowing of nose, and coughing. C) Advise patient to use soft-bristle toothbrush and avoid ingestion of irritating food. D) Apply gentle pressure for the shortest possible time period after performing venipuncture. E) Instruct patient to avoid aspirin and NSAIDs to prevent hemorrhage when varices are present.

A,B,C,E Using the smallest gauge for injections will minimize the risk of bleeding into the tissues. Avoiding straining, nose blowing, and coughing will reduce the risk of hemorrhage at these sites. The use of a soft-bristle toothbrush and avoidance of irritating food will reduce injury to highly vascular mucous membranes. The nurse should apply gentle but prolonged pressure to venipuncture sites to minimize the risk of bleeding.

When caring for a patient with liver disease, the nurse recognizes the need to prevent bleeding resulting from altered clotting factors and rupture of varices. Which of the following nursing interventions would be appropriate to achieve this outcome (select all that apply)? A. Use smallest gauge possible when giving injections or drawing blood. B. Teach patient to avoid straining at stool, vigorous blowing of nose, and coughing. C. Advise patient to use soft-bristle toothbrush and avoid ingestion of irritating food. D. Apply gentle pressure for the shortest possible time period after performing venipuncture. E. Instruct patient to avoid aspirin and NSAIDs to prevent hemorrhage when varices are present.

A,B,C,E) Using the smallest gauge needle for injections will minimize the risk of bleeding into the tissues. Avoiding straining, nose blowing, and coughing will reduce the risk of hemorrhage at these sites. The use of a soft-bristle toothbrush and avoidance of irritating food will reduce injury to highly vascular mucous membranes. The nurse should apply gentle but prolonged pressure to venipuncture sites to minimize the risk of bleeding. Aspirin and NSAIDs should not be used in patients with liver disease because they interfere with platelet aggregation, thus increasing the risk for bleeding.

The patient with cirrhosis has an increased abdominal girth from ascites. The nurse should know that this fluid gathers in the abdomen for which reasons (select all that apply)? A) There is decreased colloid oncotic pressure from the liver's inability to synthesize albumin. B) Hyperaldosteronism related to damaged hepatocytes increases sodium and fluid retention. C) Portal hypertension pushes proteins from the blood vessels, causing leaking into the peritoneal cavity. D) Osmoreceptors in the hypothalamus stimulate thirst, which causes the stimulation to take in fluids orally. E) Overactivity of the enlarged spleen results in increased removal of blood cells from the circulation, which decreases the vascular pressure.

A,B.C) The ascites related to cirrhosis are caused by decreased colloid oncotic pressure from the lack of albumin from liver inability to synthesize it and the portal hypertension that shifts the protein from the blood vessels to the peritoneal cavity, and hyperaldosteronism which increases sodium and fluid retention. The intake of fluids orally and the removal of blood cells by the spleen do not directly contribute to ascites.

Mr. Hasakusa is in end-stage liver failure. Which interventions should the nurse implement when addressing hepatic encephalopathy? (Select all that apply.) A. Assessing the client's neurologic status every 2 hours B. Monitoring the client's hemoglobin and hematocrit levels C. Evaluating the client's serum ammonia level D. Monitoring the client's handwriting daily E. Preparing to insert an esophageal tamponade tube F. Making sure the client's fingernails are short

A,C,D Hepatic encephalopathy results from an increased ammonia level due to the liver's inability to covert ammonia to urea, which leads to neurologic dysfunction and possible brain damage. The nurse should monitor the client's neurologic status, serum ammonia level, and handwriting. Monitoring the client's hemoglobin and hematocrit levels and insertion of an esophageal tamponade tube address esophageal bleeding. Keeping fingernails short address jaundice.

The nurse is doing pre-op teaching with the client who is about to undergo creation of a Kock pouch. The nurse interprets that the client has the best understanding of the nature of the surgery if the client makes which of the following statements? A. "I will need to drain the pouch regularly with a catheter." B. "I will need to wear a drainage bag for the rest of my life." C. "The drainage from this type of ostomy will be formed." D. "I will be able to pass stool from my rectum eventually."

A. A Kock pouch is a continent ileostomy. As the ileostomy begins to function, the client drains it every 3 to 4 hours and then decreases the draining to about 3 times a day or as needed when full. The client does not need to wear a drainage bag but should wear an absorbent dressing to absorb mucus drainage from the stoma. Ileostomy drainage is liquid. The client would be able to pass stool only from the rectum if an ileal-anal pouch or anastamosis were created. This type of operation is a two-stage procedure.

The client being seen in a physician's office has just been scheduled for a barium swallow the next day. The nurse writes down which of the following instructions for the client to follow before the test? A. Fast for 8 hours before the test B. Eat a regular supper and breakfast C. Continue to take all oral medications as scheduled. D. Monitor own bowel movement pattern for constipation

A. A barium swallow is an x-ray study that uses a substance called barium for contrast to highlight abnormalities in the GI tract. The client should fast for 8 to 12 hours before the test, depending on the physician instructions. Most oral medications also are withheld before the test. After the procedure the nurse must monitor for constipation, which can occur as a result of the presence of barium in the GI tract.

A client with which of the following conditions may be likely to develop rectal cancer? A. Adenomatous polyps B. Diverticulitis C. Hemorrhoids D. Peptic ulcer disease

A. A client with adenomatous polyps has a higher risk for developing rectal cancer than others do. Clients with diverticulitis are more likely to develop colon cancer. Hemorrhoids don't increase the chance of any type of cancer. Clients with peptic ulcer disease have a higher incidence of gastric cancer.

Which of the following complications is thought to be the most common cause of appendicitis? A. A fecalith B. Bowel kinking C. Internal bowel occlusion D. Abdominal bowel swelling

A. A fecalith is a fecal calculus, or stone, that occludes the lumen of the appendix and is the most common cause of appendicitis. Bowel wall swelling, kinking of the appendix, and external occlusion, not internal occlusion, of the bowel by adhesions can also be causes of appendicitis.

Which of the following diets is most commonly associated with colon cancer? A. Low-fiber, high fat B. Low-fat, high-fiber C. Low-protein, high-carbohydrate D. Low carbohydrate, high protein

A. A low-fiber, high-fat diet reduced motility and increases the chance of constipation. The metabolic end products of this type of diet are carcinogenic. A low-fat, high-fiber diet is recommended to prevent colon cancer.

A client has just had surgery for colon cancer. Which of the following disorders might the client develop? A. Peritonitis B. Diverticulosis C. Partial bowel obstruction D. Complete bowel obstruction

A. Bowel spillage could occur during surgery, resulting in peritonitis. Complete or partial bowel obstruction may occur before bowel resection. Diverticulosis doesn't result from surgery or colon cancer.

Which of the following symptoms indicated diverticulosis? A. No symptoms exist B. Change in bowel habits C. Anorexia with low-grade fever D. Episodic, dull, or steady midabdominal pain

A. Diverticulosis is an asymptomatic condition. The other choices are signs and symptoms of diverticulitis.

Which of the following areas is the most common site of fistulas in client's with Crohn's disease? A. Anorectal B. Ileum C. Rectovaginal D. Transverse colon

A. Fistulas occur in all these areas, but the anorectal area is most common because of the relative thinness of the intestinal wall in this area.

Which of the following types of diets is implicated in the development of diverticulosis? A. Low-fiber diet B. High-fiber diet C. High-protein diet D. Low-carbohydrate diet

A. Low-fiber diets have been implicated in the development of diverticula because these diets decrease the bulk in the stool and predispose the person to the development of constipation. A high-fiber diet is recommended to help prevent diverticulosis. A high-protein or low-carbohydrate diet has no effect on the development of diverticulosis.

Which of the following aspects is the priority focus of nursing management for a client with peritonitis? A. Fluid and electrolyte balance B. Gastric irrigation C. Pain management D. Psychosocial issues

A. Peritonitis can advance to shock and circulatory failure, so fluid and electrolyte balance is the priority focus of nursing management. Gastric irrigation may be needed periodically to ensure patency of the nasogastric tube. Although pain management is important for comfort and psychosocial care will address concerns such as anxiety, focusing on fluid and electrolyte imbalance will maintain hemodynamic stability.

Radiation therapy is used to treat colon cancer before surgery for which of the following reasons? A. Reducing the size of the tumor B. Eliminating the malignant cells C. Curing the cancer D. Helping the bowel heal after surgery

A. Radiation therapy is used to treat colon cancer before surgery to reduce the size of the tumor, making it easier to be resected. Radiation therapy isn't curative, can't eliminate the malignant cells (though it helps define tumor margins), can could slow postoperative healing.

The client with a new colostomy is concerned about the odor from the stool in the ostomy drainage bag. The nurse teaches the client to include which of the following foods in the diet to reduce odor? A. Yogurt B. Broccoli C. Cucumbers D. Eggs

A. The client should be taught to include deodorizing foods in the diet, such a beet greens, parsley, buttermilk, and yogurt. Spinach also reduces odor but is a gas forming food as well. Broccoli, cucumbers, and eggs are gas forming foods.

Fistulas are most common with which of the following bowel disorders? A. Crohn's disease B. Diverticulitis C. Diverticulosis D. Ulcerative colitis

A. The lesions of Crohn's disease are transmural; that is, they involve all thickness of the bowel. These lesions may perforate the bowel wall, forming fistulas with adjacent structures. Fistulas don't develop in diverticulitis or diverticulosis. The ulcers that occur in the submucosal and mucosal layers of the intestine in ulcerative colitis usually don't progress to fistula formation as in Crohn's disease.

The nurse is doing an admission assessment on a client with a history of duodenal ulcer. To determine whether the problem is currently active, the nurse would assess the client for which of the following most frequent symptom(s) of duodenal ulcer? A. Pain that is relieved by food intake B. Pain that radiated down the right arm C. N/V D. Weight loss

A. The most frequent symptom of duodenal ulcer is pain that is relieved by food intake. These clients generally describe the pain as burning, heavy, sharp, or "hungry" pain that often localizes in the midepigastric area. The client with duodenal ulcer usually does not experience weight loss or N/V. These symptoms are usually more typical in the client with a gastric ulcer.

The client with Crohn's disease has a nursing diagnosis of acute pain. The nurse would teach the client to avoid which of the following in managing this problem? A. Lying supine with the legs straight B. Massaging the abdomen C. Using antispasmodic medication D. Using relaxation techniques

A. The pain associated with Crohn's disease is alleviated by the use of analgesics and antispasmodics and also is reduced by having the client practice relaxation techniques, applying local cold or heat to the abdomen, massaging the abdomen, and lying with the legs flexed. Lying with the legs extended is not useful because it increases the muscle tension in the abdomen, which could aggravate the inflamed intestinal tissues as the abdominal muscles are stretched.

The client has been admitted with a diagnosis of acute pancreatitis. The nurse would assess this client for pain that is: A. Severe and unrelenting, located in the epigastric area and radiating to the back. B. Severe and unrelenting, located in the left lower quadrant and radiating to the groin. C. Burning and aching, located in the epigastric area and radiating to the umbilicus. D. Burning and aching, located in the left lower quadrant and radiating to the hip.

A. The pain associated with acute pancreatitis is often severe and unrelenting, is located in the epigastric region, and radiates to the back.

The nurse instructs the ileostomy client to do which of the following as a part of essential care of the stoma? A. Cleanse the peristomal skin meticulously B. Take in high-fiber foods such as nuts C. Massage the area below the stoma D. Limit fluid intake to prevent diarrhea.

A. The peristomal skin must receive meticulous cleansing because the ileostomy drainage has more enzymes and is more caustic to the skin than colostomy drainage. Foods such as nuts and those with seeds will pass through the ileostomy. The client should be taught that these foods will remain undigested. The area below the ileostomy may be massaged if needed if the ileostomy becomes blocked by high fiber foods. Fluid intake should be maintained to at least six to eight glasses of water per day to prevent dehydration.

The nurse is teaching the client how to perform a colostomy irrigation. To enhance the effectiveness of the irrigation and fecal returns, what measure should the nurse instruct the client to do? A. Increase fluid intake B. Reduce the amount of irrigation solution C. Perform the irrigation in the evening D. Place heat on the abdomen

A. To enhance effectiveness of the irrigation and fecal returns, the client is instructed to increase fluid intake and prevent constipation.

When teaching a community group about measures to prevent colon cancer, which instruction should the nurse include? A. "Limit fat intake to 20% to 25% of your total daily calories." B. "Include 15 to 20 grams of fiber into your daily diet." C. "Get an annual rectal examination after age 35." D. "Undergo sigmoidoscopy annually after age 50."

A. To help prevent colon cancer, fats should account for no more than 20% to 25% of total daily calories and the diet should include 25 to 30 grams of fiber per day. A digital rectal examination isn't recommended as a stand-alone test for colorectal cancer. For colorectal cancer screening, the American Cancer society advises clients over age 50 to have a flexible sigmoidoscopy every 5 years, yearly fecal occult blood tests, yearly fecal occult blood tests PLUS a flexible sigmoidoscopy every 5 years, a double-contrast barium enema every 5 years, or a colonoscopy every 10 years.

A client has a percutaneous endoscopic gastrostomy tube inserted for tube feedings. Before starting a continuous feeding, the nurse should place the client in which position? A. Semi-Fowlers B. Supine C. Reverse Trendelenburg D. High Fowler's

A. To prevent aspiration of stomach contents, the nurse should place the client in semi-Fowler's position. High Fowler's position isn't necessary and may not be tolerated as well as semi-Fowler's.

Which of the following nursing interventions should the nurse perform for a female client receiving enteral feedings through a gastrostomy tube? " a. Change the tube feeding solutions and tubing at least every 24 hours b. Maintain the head of the bed at a 15-degree elevation continuously. c. Check the gastrostomy tube for position every 2 days. d. Maintain the client on bed rest during the feedings

A. Tube feeding solutions and tubing should be changed every 24 hours, or more frequently if the feeding requires it. Doing so prevents contamination and bacterial growth. The head of the bed should be elevated 30 to 45 degrees continuously to prevent aspiration. Checking for gastrostomy tube placement is performed before initiating the feedings and every 4 hours during continuous feedings. Clients may ambulate during feedings

A client with gastroesophageal reflux disease complains about having difficulty sleeping at night, what should the nurse instruct the client to do? A. sleep on several pillows B. eliminate carbohydrates from the diet C. suggest a glass of milk before retiring D. take antacids such as sodium bicarbonate

A. sleeping on pillows raises the upper torso and minimizes reflux of the gastic contents

A client is diagnosed with a hiatal hernia. Which statement indicates effective client teaching about hiatal hernia and its treatment? a) "I'll lie down immediately after a meal." b) "I'll eat three large meals every day without any food restrictions." c) "I'll eat frequent, small, bland meals that are high in fiber." d) "I'll gradually increase the amount of heavy lifting I do."

C) I'll eat frequent, small, bland meals that are high in fiber In hiatal hernia, the upper portion of the stomach protrudes into the chest when intra-abdominal pressure increases. To minimize intra-abdominal pressure and decrease gastric reflux, the client should eat frequent, small, bland meals that can pass easily through the esophagus. Meals should be high in fiber to prevent constipation and minimize straining on defecation (which may increase intra-abdominal pressure from the Valsalva maneuver). Eating three large meals daily would increase intra-abdominal pressure, possibly worsening the hiatal hernia. The client should avoid spicy foods, alcohol, and tobacco because they increase gastric acidity and promote gastric reflux. To minimize intra-abdominal pressure, the client shouldn't recline after meals, lift heavy objects, or bend.

A 54-year-old patient admitted with diabetes mellitus, malnutrition, osteomyelitis, and alcohol abuse has a serum amylase level of 280 U/L and a serum lipase level of 310 U/L. To which of the following diagnoses does the nurse attribute these findings? A. Malnutrition B. Osteomyelitis C. Alcohol abuse D. Diabetes mellitus

C) The patient with alcohol abuse could develop pancreatitis as a complication, which would increase the serum amylase (normal 30-122 U/L) and serum lipase (normal 31-186 U/L) levels as shown.

A patient who is scheduled for surgery with general anesthesia in 1 hour is observed with a moist, but empty water glass in his hand. Which assessment finding may indicate that the patient drank a glass of water? A) Flat abdomen without movement upon inspection B) Tenderness at left upper quadrant upon palpation C) Easily heard, loud gurgling in the right upper quadrant D) High-pitched, hollow sounds in the left upper quadrant

C) If the patient drank water on an empty stomach, gurgling can be assessed without a stethoscope or assessed with auscultation. High-pitched, hollow sounds are tympanic and indicate an empty cavity. A flat abdomen and tenderness do not indicate that the patient drank a glass of water.

Crohn's disease is a condition of malabsorption caused by which of the following pathophysiological processes? a) Gastric resection b) Infectious disease c) Inflammation of all layers of intestinal mucosa d) Disaccharidase deficiency

C) Inflammation of all layers of intestinal mucosa Crohn's disease is also known as regional enteritis and can occur anywhere along the GI tract, but most commonly at the distal ileum and in the colon. Infectious disease causes problems such as small bowel bacterial overgrowth leading to malabsorption. Disaccharidase deficiency leads to lactose intolerance. Postoperative malabsorption occurs after gastric or intestinal resection.

The client is receiving a 25% dextrose solution of parenteral nutrition. The infusion machine is beeping, and the nurse determines the intravenous (IV) bag is empty. The nurse finds there is no available bag to administer. It is most important for the nurse to a) Request a new bag from the pharmacy department. b) Flush the line with 10 mL of sterile saline. c) Infuse a solution containing 10% dextrose and water. d) Catch up with the next bag when it arrives.

C) Infuse a solution containing 10% dextrose and water If the parenteral nutrition solution runs out, a solution of 10% dextrose and water is infused to prevent hypoglycemia. The nurse would then order the next parenteral nutrition bag from the pharmacy. Flushing a peripherally inserted catheter is usually prescribed every 8 hours or per hospital established protocols. It is not the most important activity at this moment. The infusion rate should not be increased to compensate for fluids that were not infused, because hyperglycemia and hyperosmolar diuresis could occur.

Following bowel resection, a patient has a nasogastric (NG) tube to suction, but complains of nausea and abdominal distention. The nurse irrigates the tube as necessary as ordered, but the irrigating fluid does not return. What should be the priority action by the nurse? A) Notify the physician. B) Auscultate for bowel sounds. C) Reposition the tube and check for placement. D) Remove the tube and replace it with a new one.

C) The tube may be resting against the stomach wall. The first action by the nurse (since this is intestinal surgery and not gastric surgery) is to reposition the tube and check it again for placement. The physician does not need to be notified unless the tube function cannot be restored by the nurse. The patient does not have bowel sounds, which is why the NG tube is in place. The NG tube would not be removed and replaced unless it was no longer in the stomach or the obstruction of the tube could not be relieved.

When assisting with the plan of care for a client receiving tube feedings, which of the following would the nurse include to reduce the client's risk for aspiration? a) Administering 15 to 30 mL of water every 4 hours. b) Aspirating for residual contents every 4 to 8 hours. c) Keeping the client in a semi-Fowler's position at all times. d) Giving the feedings at room temperature.

C) Keeping the client in a semi Fowler's position at all times With continuous tube feedings, the nurse needs to keep the client in a semi-Fowler's position at all times to reduce regurgitation and the risk for aspiration. Aspirating for residual contents helps to ensure adequate nutrition and prevent overfeeding. Administering 15 to 30 mL of water every 4 hours helps to maintain tube patency. Giving the feedings at room temperature reduces the risk for diarrhea.

While palpating a client's right upper quadrant (RUQ), the nurse would expect to find which structure? a) Spleen b) Appendix c) Liver d) Sigmoid colon

C) Liver The RUQ contains the liver, gallbladder, duodenum, head of the pancreas, hepatic flexure of the colon, portions of the ascending and transverse colon, and a portion of the right kidney. The sigmoid colon is located in the left lower quadrant; the appendix, in the right lower quadrant; and the spleen, in the left upper quadrant.

Following administration of a dose of metoclopramide (Reglan) to the patient, the nurse determines that the medication has been effective when what is noted? A) Decreased blood pressure B) Absence of muscle tremors C) Relief of nausea and vomiting D) No further episodes of diarrhea

C) Metoclopramide is classified as a prokinetic and antiemetic medication. If it is effective, the patient's nausea and vomiting should resolve. Metoclopramide does not affect blood pressure, muscle tremors, or diarrhea.

When bowel sounds are heard about every 15 seconds, the nurse would record that the bowel sounds are a) absent. b) hypoactive. c) normal. d) sluggish.

C) Normal Normal bowel sounds are heard every 5 to 20 seconds. Hypoactive bowel sound is the description given to auscultation of one to two bowel sounds in 2 minutes. Sluggish is not a term a nurse would use to accurately describe bowel sounds. The nurse records that bowel sounds are absent when no sound is heard in 3 to 5 minutes.

The nurse asks a client to point to where she feels pain. The client asks why this is important. The nurse's best response would be which of the following? a) "This determines the pain medication to be ordered." b) "If the doctor massages over the exact painful area, the pain will disappear." c) "Often the area of pain is referred from another area." d) "The area may determine the severity of the pain."

C) Often the area of pain is referred from another area Pain can be a major symptom of disease. The location and distribution of pain can be referred from a different area. If a client points to an area of pain and has other symptoms associated with a certain disease, this is valuable information for treatment.

Which of the following represents the medication classification of a proton (gastric acid) pump inhibitor? a) Famotidine (Pepcid) b) Metronidazole (Flagyl) c) Omeprazole (Prilosec) d) Sucralfate (Carafate)

C) Omeprazole Omeprazole decreases gastric acid by slowing the hydrogen-potassium adenosine triphosphatase pump on the surface of the parietal cells. Sucralfate is a cytoprotective drug. Famotidine is a histamine-2 receptor antagonist. Metronidazole is an antibiotic, specifically an amebicide.

The patient receiving chemotherapy rings the call bell and reports an onset of nausea. The nurse should prepare a prn dose of which of the following medications? A) Morphine sulfate B) Zolpidem (Ambien) C) Ondansetron (Zofran) D) Dexamethasone (Decadron)

C) Ondansetron (Zofran) Ondansetron is a 5-HT3 receptor antagonist antiemetic that is especially effective in reducing cancer chemotherapy-induced nausea and vomiting.

The nurse recognizes that the patient diagnosed with a duodenal ulcer will likely experience a) weight loss. b) vomiting. c) pain 2 to 3 hours after a meal. d) hemorrhage.

C) Pain 2 to 3 hours after a meal The patient with a gastric ulcer often awakens between 1 to 2 with pain, and ingestion of food brings relief. Vomiting is uncommon in the patient with duodenal ulcer. Hemorrhage is less likely in the patient with duodenal ulcer than the patient with gastric ulcer. The patient with a duodenal ulcer may experience weight gain.

The patient with chronic gastritis is being put on a combination of medications to eradicate H. pylori. Which drugs does the nurse know will probably be used for this patient? A) Antibiotic(s), antacid, and corticosteroid B) Antibiotic(s), aspirin, and antiulcer/protectant C) Antibiotic(s), proton pump inhibitor, and bismuth D) Antibiotic(s) and nonsteroidal antiinflammatory drugs (NSAIDs)

C) To eradicate H. pylori, a combination of antibiotics, a proton pump inhibitor, and possibly bismuth (for quadruple therapy) will be used. Corticosteroids, aspirin, and NSAIDs are drugs that can cause gastritis and do not affect H. pylori.

When caring for a client with acute pancreatitis, the nurse should use which comfort measure? a) Encouraging frequent visits from family and friends b) Administering frequent oral feedings c) Positioning the client on the side with the knees flexed d) Administering an analgesic once per shift, as ordered, to prevent drug addiction

C) Postitioning the client on the side with the knees flexed The nurse should place the client with acute pancreatitis in a side-lying position with knees flexed; this position promotes comfort by decreasing pressure on the abdominal muscles. The nurse should administer an analgesic, as needed and ordered, before pain becomes severe, rather than once each shift. Because the client needs a quiet, restful environment during the acute disease stage, the nurse should discourage frequent visits from family and friends. Frequent oral feedings are contraindicated during the acute stage to allow the pancreas to rest.

A client is being treated for prolonged diarrhea. Which of the following foods should the nurse encourage the client to consume? a) Protein-rich foods b) High-fiber foods c) Potassium-rich foods d) High-fat foods

C) Potassium rich foods The nurse should encourage the client with diarrhea to consume potassium-rich foods. Excessive diarrhea causes severe loss of potassium. The nurse should also instruct the client to avoid high-fiber or fatty foods because these foods stimulate gastrointestinal motility. The intake of protein foods may or may not be appropriate depending on the client's status.

A client presents with complaints of blood in her stools. Upon inspection, the nurse notes streaks of bright red blood visible on the outer surface of formed stool. Which of the following will the nurse further investigate with this client? a) Ingestion of cherry soda b) Ingestion of cocoa c) Presence or history of hemorrhoids d) Recent barium studies

C) Presence or history of hemorrhoids Stool is normally light to dark brown. Blood in the stool can present in various ways and must be investigated. Lower rectal or anal bleeding is suspected if there is streaking of blood on the surface of the stool or blood is noted on toilet tissue.

Which of the following indicates an overdose of lactulose? a) Hypoactive bowel sounds b) Constipation c) Watery diarrhea d) Fecal impaction

C) Watery diarrhea The patient receiving lactulose is monitored closely for the development of watery diarrheal stool, which indicates a medication overdose.

The nurse observes dry mucous membranes in a client who is receiving tube feedings after an oral surgery. The client also complains of unpleasant tastes and odors. Which of the following measures should be included in the client's plan of care? a) Ensure adequate hydration with additional water. b) Keep the feeding formula refrigerated. c) Provide frequent mouth care. d) Flush the tube with water before adding the feedings.

C) Provide frequent mouth care Frequent mouth care helps to relieve the discomfort from dryness and unpleasant odors and tastes. It can be done with the help of ice chips and analgesic throat lozenges, gargles, or sprays. Adequate hydration is essential. If urine output is less than less than 500 mL/day, formula and additional water can be given as ordered. Keeping the feeding formula refrigerated and unopened until it is ready for use and flushing the tube with water before adding feedings are measures to protect the client from infections.

The nurse is caring for a client with cirrhosis. Which assessment findings indicate that the client has deficient vitamin K absorption caused by this hepatic disease? a) Ascites and orthopnea b) Gynecomastia and testicular atrophy c) Purpura and petechiae d) Dyspnea and fatigue

C) Purpura and petechiae A hepatic disorder, such as cirrhosis, may disrupt the liver's normal use of vitamin K to produce prothrombin (a clotting factor). Consequently, the nurse should monitor the client for signs of bleeding, including purpura and petechiae. Dyspnea and fatigue suggest anemia. Ascites and orthopnea are unrelated to vitamin K absorption. Gynecomastia and testicular atrophy result from decreased estrogen metabolism by the diseased liver.

While caring for a comatose patient who is receiving continuous enteral nutrition through a soft nasogastric tube, the nurse notes the presence of new crackles in the patient's lungs. In which order will the nurse take the following actions? Put a comma and space between each answer choice (a, b, c, d, etc.) ____________________ a. Turn off the tube feeding. b. Obtain the patient's oxygen saturation. c. Check the tube feeding residual volume. d. Notify the patient's health care provider.

ANS: A, B, C, D The assessment data indicate that aspiration may have occurred, and the nurse's first action should be to turn off the tube feeding to avoid further aspiration. The next action should be to check the oxygen saturation because this may indicate the need for immediate respiratory suctioning or oxygen administration. The residual volume should be obtained because it provides data about possible causes of aspiration. Finally, the health care provider should be notified and informed of all the assessment data the nurse has just obtained.

When teaching a patient about testing to diagnose metabolic syndrome, which topic would the nurse include? a. Blood glucose test b. Cardiac enzyme tests c. Postural blood pressures d. Resting electrocardiogram

ANS: A A fasting blood glucose test >100 mg/dL is one of the diagnostic criteria for metabolic syndrome. The other tests are not used to diagnose metabolic syndrome although they may be used to check for cardiovascular complications of the disorder

The health care provider prescribes the following therapies for a patient who has been admitted with dehydration and hypotension after 3 days of nausea and vomiting. Which order will the nurse implement first? a. Infuse normal saline at 250 mL/hr. b. Administer IV ondansetron (Zofran). c. Provide oral care with moistened swabs. d. Insert a 16-gauge nasogastric (NG) tube.

ANS: A Because the patient has severe dehydration, rehydration with IV fluids is the priority. The other orders should be accomplished as quickly as possible after the IV fluids are initiated.

When assessing a patient who is a vegan, which finding may indicate the need for cobalamin supplementation? a. Paresthesias b. Ecchymoses c. Dry, scaly skin d. Gingival swelling

ANS: A Cobalamin (vitamin B12) cannot be obtained from foods of plant origin, so the patient will be most at risk for signs of cobalamin deficiency, such as anemia and peripheral neuropathy. The other symptoms listed are associated with other nutritional deficiencies but would not be associated with a vegan diet.

Cobalamin injections have been prescribed for a patient with chronic atrophic gastritis. The nurse determines that teaching regarding the injections has been effective when the patient states, a. "The cobalamin injections will prevent me from becoming anemic." b. "These injections will increase the hydrochloric acid in my stomach." c. "These injections will decrease my risk for developing stomach cancer." d. "The cobalamin injections need to be taken until my inflamed stomach heals."

ANS: A Cobalamin supplementation prevents the development of pernicious anemia. The incidence of stomach cancer is higher in patients with chronic gastritis, but cobalamin does not reduce the risk for stomach cancer. Chronic gastritis may cause achlorhydria, but cobalamin does not correct this. The loss of intrinsic factor secretion with chronic gastritis is permanent, and the patient will need lifelong supplementation with cobalamin.

A patient is hospitalized with vomiting of "coffee-ground" emesis. The nurse will anticipate preparing the patient for a. endoscopy. b. angiography. c. gastric analysis testing. d. barium contrast studies.

ANS: A Endoscopy is the primary tool for visualization and diagnosis of upper gastrointestinal (GI) bleeding. Angiography is used only when endoscopy cannot be done because it is more invasive and has more possible complications. Gastric analysis testing may help with determining the cause of gastric irritation, but it is not used for acute GI bleeding. Barium studies are helpful in determining the presence of gastric lesions, but not whether the lesions are actively bleeding.

The nurse is coaching a community group for individuals who are overweight. Which participant behavior is an example of the best exercise plan for weight loss? a. Walking for 40 minutes 6 or 7 days/week b. Lifting weights with friends 3 times/week c. Playing soccer for an hour on the weekend d. Running for 10 to 15 minutes 3 times/week

ANS: A Exercise should be done daily for 30 minutes to an hour. Exercising in highly aerobic activities for short bursts or only once a week is not helpful and may be dangerous in an individual who has not been exercising. Running may be appropriate, but a patient should start with an exercise that is less stressful and can be done for a longer period. Weight lifting is not as helpful as aerobic exercise in weight loss

Twelve hours after undergoing a gastroduodenostomy (Billroth I), a patient complains of increasing abdominal pain. The patient has absent bowel sounds and 200 mL of bright red nasogastric (NG) drainage in the last hour. The most appropriate action by the nurse at this time is to a. notify the surgeon. b. irrigate the NG tube. c. administer the prescribed morphine. d. continue to monitor the NG drainage.

ANS: A Increased pain and 200 mL of bright red NG drainage 12 hours after surgery indicate possible postoperative hemorrhage, and immediate actions such as blood transfusion and/or return to surgery are needed. Because the NG is draining, there is no indication that irrigation is needed. The patient may need morphine, but this is not the highest priority action. Continuing to monitor the NG drainage is not an adequate response.

The nurse will be teaching self-management to patients after gastric bypass surgery. Which information will the nurse plan to include? a. Drink fluids between meals but not with meals. b. Choose high-fat foods for at least 30% of intake. c. Developing flabby skin can be prevented by exercise. d. Choose foods high in fiber to promote bowel function.

ANS: A Intake of fluids with meals tends to cause dumping syndrome and diarrhea. Food choices should be low in fat and fiber. Exercise does not prevent the development of flabby skin

Which of these nursing actions included in the plan of care for a patient who is receiving intermittent tube feedings through a percutaneous endoscopic gastrostomy (PEG) tube may be delegated to an LPN/LVN? a. Providing skin care to the area around the tube site b. Assessing the patient's nutritional status at least weekly c. Determining the need for the addition of water to the feedings d. Teaching the patient and family how to administer tube feedings

ANS: A LPN/LVN education and scope of practice include actions such as dressing changes and wound care. Patient teaching and complex assessments (such as patient nutrition and hydration status) require RN-level education and scope of practice.

A patient who has just been started on continuous tube feedings of a full-strength commercial formula at 100 mL/hr using a closed system method has six diarrhea stools the first day. Which action should the nurse plan to take? a. Slow the infusion rate of the tube feeding. b. Check gastric residual volumes more frequently. c. Change the enteral feeding system and formula every 8 hours. d. Discontinue administration of water through the feeding tube.

ANS: A Loose stools indicate poor absorption of nutrients and indicate a need to slow the feeding rate or decrease the concentration of the feeding. Water should be given when patients receive enteral feedings to prevent dehydration. When a closed enteral feeding system is used, the tubing and formula are changed every 24 hours. High residual volumes do not contribute to diarrhea.

A patient who has had several episodes of bloody diarrhea is admitted to the emergency department. Which action should the nurse anticipate taking? a. Obtain a stool specimen for culture. b. Administer antidiarrheal medications. c. Teach about adverse effects of nonsteroidal anti-inflammatory drugs (NSAIDs). d. Provide education about antibiotic therapy.

ANS: A Patients with bloody diarrhea should have a stool culture for E. coli O157:H7. NSAIDs may cause occult blood in the stools, but not diarrhea. Antidiarrheal medications usually are avoided for possible infectious diarrhea to avoid prolonging the infection. Antibiotic therapy in the treatment of infectious diarrhea is controversial because it may precipitate kidney complications.

The health care provider orders intravenous (IV) ranitidine (Zantac) for a patient with gastrointestinal (GI) bleeding caused by peptic ulcer disease. When teaching the patient about the effect of the medication, which information will the nurse include? a. "Ranitidine decreases secretion of gastric acid." b. "Ranitidine neutralizes the acid in the stomach." c. "Ranitidine constricts the blood vessels in the stomach and decreases bleeding." d. "Ranitidine covers the ulcer with a protective material that promotes healing."

ANS: A Ranitidine is a histamine-2 (H2) receptor blocker, which decreases the secretion of gastric acid. The response beginning, "Ranitidine constricts the blood vessels" describes the effect of vasopressin. The response beginning "Ranitidine neutralizes the acid" describes the effect of antacids. And the response beginning "Ranitidine covers the ulcer" describes the action of sucralfate (Carafate).

When counseling a patient with a family history of stomach cancer about ways to decrease risk for developing stomach cancer, the nurse will teach the patient to avoid a. smoked foods such as bacon and ham. b. foods that cause abdominal distention. c. chronic use of H2 blocking medications. d. emotionally or physically stressful situations.

ANS: A Smoked foods such as bacon, ham, and smoked sausage increase the risk for stomach cancer. Use of H2 blockers, stressful situations, and abdominal distention are not associated with an increased incidence of stomach cancer.

Which information will be best for the nurse to include when teaching a patient with peptic ulcer disease (PUD) about dietary management of the disease? a. "Avoid foods that cause pain after you eat them." b. "High-protein foods are least likely to cause pain." c. "You will need to remain on a bland diet indefinitely." d. "You should avoid eating many raw fruits and vegetables."

ANS: A The best information is that each individual should choose foods that are not associated with postprandial discomfort. Raw fruits and vegetables may irritate the gastric mucosa, but chewing well seems to decrease this and some patients may tolerate these well. High-protein foods help to neutralize acid, but they also stimulate hydrochloric (HCl) acid secretion and may increase discomfort for some patients. Bland diets may be recommended during an acute exacerbation of PUD, but there is little scientific evidence to support their use.

A 40-year-old obese woman reports that she wants to lose weight. Which question should the nurse ask first? a. "What factors led to your obesity?" b. "Which types of food do you like best?" c. "How long have you been overweight?" d. "What kind of activities do you enjoy?"

ANS: A The nurse should obtain information about the patient's perceptions of the reasons for the obesity to develop a plan individualized to the patient. The other information also will be obtained from the patient, but the patient is more likely to make changes when the patient's beliefs are considered in planning

A patient recovering from a gastrojejunostomy (Billroth II) for treatment of a duodenal ulcer develops dizziness, weakness, and palpitations about 20 minutes after eating. To avoid recurrence of these symptoms, the nurse teaches the patient to a. lie down for about 30 minutes after eating. b. choose foods that are high in carbohydrates. c. increase the amount of fluid intake with meals. d. drink sugared fluids or eat candy after each meal.

ANS: A The patient is experiencing symptoms of dumping syndrome, which may be reduced by lying down after eating. Increasing fluid intake and choosing high carbohydrate foods will increase the risk for dumping syndrome. Having a sweet drink or hard candy will correct the hypoglycemia that is associated with dumping syndrome but will not prevent dumping syndrome.

A patient with a bleeding duodenal ulcer has a nasogastric (NG) tube in place, and the health care provider orders 30 mL of aluminum hydroxide/magnesium hydroxide (Maalox) to be instilled through the tube every hour. To evaluate the effectiveness of this treatment, the nurse a. periodically aspirates and tests gastric pH. b. monitors arterial blood gas values on a daily basis. c. checks each stool for the presence of occult blood. d. measures the amount of residual stomach contents hourly.

ANS: A The purpose for antacids is to increase gastric pH. Checking gastric pH is the most direct way of evaluating the effectiveness of the medication. Arterial blood gases may change slightly, but this does not directly reflect the effect of antacids on gastric pH. Because the patient has upper gastrointestinal (GI) bleeding, occult blood in the stools will appear even after the acute bleeding has stopped. The amount of residual stomach contents is not a reflection of resolution of bleeding or of gastric pH.

A patient who is receiving continuous enteral nutrition through a small-bore silicone feeding tube has a computed tomography (CT) scan ordered and will have to be placed in a flat position for the scan. Which action by the nurse is best? a. Shut the feeding off 30 to 60 minutes before the scan. b. Ask the health care provider to reschedule the CT scan. c. Connect the feeding tube to continuous suction during the scan. d. Send the patient to CT scan with oral suction in case of aspiration.

ANS: A The tube feeding should be shut off 30 to 60 minutes before any procedure requiring the patient to lie flat. Because the CT scan is ordered for diagnosis of patient problems, rescheduling is not usually an option. Prevention, rather than treatment, of aspiration is needed. Small-bore feeding tubes are soft and collapse easily with aspiration or suction, making nasogastric suction of gastric contents unreliable.

Which topic would be of importance for the nurse to include when teaching a patient about testing for possible metabolic syndrome? a. Blood glucose test b. Cardiac enzyme tests c. Postural blood pressures d. Resting electrocardiogram

ANS: A A fasting blood glucose test >100 mg/dL is one of the diagnostic criteria for metabolic syndrome. The other tests are not used to diagnose metabolic syndrome, although they may be used to check for cardiovascular complications of the disorder.

Which patient behavior indicates that an overweight patient has understood the nurse's teaching about the best exercise plan for weight loss? a. Walking for 40 minutes 6 or 7 days/week b. Lifting weights with friends 3 times/week c. Playing soccer for an hour on the weekend d. Running for 10 to 15 minutes 3 times/week

ANS: A Exercise should be done daily for 30 minutes to an hour. Exercising in highly aerobic activities for short bursts or only once a week is not helpful and may be dangerous in an individual who has not been exercising. Running may be appropriate, but a patient should start with an exercise that is less stressful and can be done for a longer period. Weight lifting is not as helpful as aerobic exercise in weight loss

Which information will the nurse plan to include in discharge teaching for a patient after gastric bypass surgery? a. Avoid drinking fluids with meals. b. Choose high-fat foods for at least 30% of intake. c. Choose foods that are high in fiber to promote bowel function. d. Development of flabby skin can be prevented by daily exercise.

ANS: A Intake of fluids with meals tends to cause dumping syndrome and diarrhea. Food choices should be low in fat and fiber. Exercise does not prevent the development of flabby skin.

After the nurse has completed teaching a patient about the recommended amounts of foods from animal and plant sources, which of these menu selections indicates that the initial instructions about diet have been understood? a. 3 oz of pork roast, a cup of corn, and a sliced tomato b. A chicken breast and a cup of tossed salad with nonfat dressing c. A 6 oz can of tuna mixed with nonfat mayonnaise and chopped celery d. 3 oz of roast beef, 2 oz of low-fat cheese, and a half-cup of carrot sticks

ANS: A This selection is most consistent with the recommendation of the American Institute for Cancer Research that one third of the diet should be from animal sources and two thirds from plant source foods. The other choices all have higher ratios of animal origin foods to plant source foods than would be recommended.

During a busy day, the nurse admits all of the following patients to the medical-surgical unit. Which patients are most important to refer to the dietitian for a complete nutritional assessment (select all that apply)? a. A 24-year-old who has a history of weight gains and losses b. A 53-year-old who complains of intermittent nausea for the past 2 days c. A 66-year-old who is admitted for débridement of an infected surgical wound d. A 45-year-old admitted with chest pain and possible myocardial infarction (MI) e. A 32-year-old with rheumatoid arthritis who takes prednisone (Deltasone) daily

ANS: A, C, E Weight fluctuations, use of corticosteroids, and draining or infected wounds all suggest that the patient may be at risk for malnutrition. Patients with chest pain or MI are not usually poorly nourished. Although vomiting that lasts 5 days places a patient at risk, nausea that has persisted for 2 days does not always indicate poor nutritional status or risk for health problems caused by poor nutrition.

Which information about a patient who has just been admitted to the hospital with nausea and vomiting will require the most rapid intervention by the nurse? a. The patient has taken only sips of water. b. The patient is lethargic and difficult to arouse. c. The patient's chart indicates a recent resection of the small intestine. d. The patient has been vomiting several times a day for the last 4 days.

ANS: B A lethargic patient is at risk for aspiration, and the nurse will need to position the patient to decrease aspiration risk. The other information also is important to collect, but it does not require as quick action as the risk for aspiration.

Parenteral nutrition (PN) containing amino acids and dextrose was ordered and hung 24 hours ago for a malnourished patient. The nurse observes that about 50 mL remain in the PN container. Which action is best for the nurse to take? a. Ask the health care provider to clarify the written PN order. b. Add a new container of PN using the current tubing and filter. c. Hang a new container of PN and change the IV tubing and filter. d. Infuse the remaining 50 mL and then hang a new container of PN.

ANS: B All PN solutions are changed at 24 hours. PN solutions containing dextrose and amino acids require a change in tubing and filter every 72 hours rather than daily. Infusion of the additional 50 mL will increase patient risk for infection. Changing the IV tubing and filter more frequently than required will unnecessarily increase costs. The nurse (not the health care provider) is responsible for knowing the indicated times for tubing and filter changes.

Which finding for a patient who has been taking orlistat (Xenical) is most important to report to the health care provider? a. The patient frequently has liquid stools. b. The patient is pale and has many bruises. c. The patient complains of bloating after meals. d. The patient is experiencing a weight loss plateau.

ANS: B Because orlistat blocks the absorption of fat-soluble vitamins, the patient may not be receiving an adequate amount of vitamin K, resulting in a decrease in clotting factors. Abdominal bloating and liquid stools are common side effects of orlistat and indicate that the nurse should remind the patient that fat in the diet may increase these side effects. Weight loss plateaus are normal during weight reduction

Which nursing action is appropriate when coaching obese adults enrolled in a behavior modification program? a. Having the adults write down the caloric intake of each meal b. Asking the adults about situations that tend to increase appetite c. Suggesting that the adults plan rewards, such as sugarless candy, for achieving their goals d. Encouraging the adults to eat small amounts frequently rather than having scheduled meals

ANS: B Behavior modification programs focus on how and when the person eats and de-emphasize aspects such as calorie counting. Nonfood rewards are recommended for achievement of weight-loss goals. Patients are often taught to restrict eating to designated meals when using behavior modification

A patient who has been NPO during treatment for nausea and vomiting caused by gastric irritation is to start oral intake. Which of these should the nurse offer to the patient? a. A glass of orange juice b. A dish of lemon gelatin c. A cup of coffee with cream d. A bowl of hot chicken broth

ANS: B Clear liquids are usually the first foods started after a patient has been nauseated. Acidic foods such as orange juice, very hot foods, and coffee are poorly tolerated when patients have been nauseated.

Which information will the nurse prioritize in planning preoperative teaching for a patient undergoing a Roux-en-Y gastric bypass? a. Educating the patient about the nasogastric (NG) tube b. Instructing the patient on coughing and breathing techniques c. Discussing necessary postoperative modifications in lifestyle d. Demonstrating passive range-of-motion exercises for the legs

ANS: B Coughing and deep breathing can prevent major postoperative complications such as carbon monoxide retention and hypoxemia. Information about passive range of motion, the NG tube, and postoperative modifications in lifestyle will also be discussed, but avoidance of respiratory complications is the priority goal after surgery

Which of these assessment findings in a patient with a hiatal hernia who returned from a laparoscopic Nissen fundoplication 4 hours ago is most important for the nurse to address immediately? a. The patient is experiencing intermittent waves of nausea. b. The patient has absent breath sounds throughout the left lung. c. The patient has decreased bowel sounds in all four quadrants. d. The patient complains of 6/10 (0 to 10 scale) abdominal pain.

ANS: B Decreased breath sounds on one side may indicate a pneumothorax, which requires rapid diagnosis and treatment. The abdominal pain and nausea also should be addressed but they are not as high priority as the patient's respiratory status. The patient's decreased bowel sounds are expected after surgery and require ongoing monitoring but no other action.

Which action should the nurse take first in order to improve calorie and protein intake for a patient who eats only about 50% of each meal because of "feeling too tired to eat much." a. Teach the patient about the importance of good nutrition. b. Serve multiple small feedings of high-calorie, high-protein foods. c. Obtain an order for enteral feedings of liquid nutritional supplements. d. Consult with the health care provider about providing parenteral nutrition (PN).

ANS: B Eating small amounts of food frequently throughout the day is less fatiguing and will improve the patient's ability to take in more nutrients. Teaching the patient may be appropriate, but will not address the patient's inability to eat more because of fatigue. Tube feedings or PN may be needed if the patient is unable to take in enough nutrients orally, but increasing the oral intake should be attempted first.

Which assessment action will help the nurse determine if an obese patient has metabolic syndrome? a. Take the patient's apical pulse. b. Check the patient's blood pressure. c. Ask the patient about dietary intake. d. Dipstick the patient's urine for protein.

ANS: B Elevated blood pressure is one of the characteristics of metabolic syndrome. The other information also may be obtained by the nurse, but it will not assist with the diagnosis of metabolic syndrome

The family member of a patient who has suffered massive abdominal trauma in an automobile accident asks the nurse why the patient is receiving famotidine (Pepcid). The nurse will explain that the medication will a. prevent aspiration of gastric contents. b. inhibit the development of stress ulcers. c. lower the chance for H. pylori infection. d. decrease the risk for nausea and vomiting.

ANS: B Famotidine is administered to prevent the development of physiologic stress ulcers, which are associated with a major physiologic insult such as massive trauma. Famotidine does not decrease nausea or vomiting, prevent aspiration, or prevent H. pylori infection.

A patient with peptic ulcer disease associated with the presence of Helicobacter pylori is treated with triple drug therapy. The nurse will plan to teach the patient about a. sucralfate (Carafate), nystatin (Mycostatin), and bismuth (Pepto-Bismol). b. amoxicillin (Amoxil), clarithromycin (Biaxin), and omeprazole (Prilosec). c. famotidine (Pepcid), magnesium hydroxide (Mylanta), and pantoprazole (Protonix). d. metoclopramide (Reglan), bethanechol (Urecholine), and promethazine (Phenergan).

ANS: B The drugs used in triple drug therapy include a proton pump inhibitor such as omeprazole and the antibiotics amoxicillin and clarithromycin. The other combinations listed are not included in the protocol for H. pylori infection.

When preparing to teach an 82-year-old Hispanic patient who lives with an adult daughter about ways to improve nutrition, which action should the nurse take first? a. Ask the daughter about the patient's food preferences. b. Determine who shops for groceries and prepares the meals. c. Question the patient about how many meals per day are eaten. d. Assure the patient that culturally appropriate foods will be included.

ANS: B The family member who shops for groceries and cooks will be in control of the patient's diet, so the nurse will need to ensure that this family member is involved in any teaching or discussion about the patient's nutritional needs. The other information also will be assessed and used but will not be useful in meeting the patient's nutritional needs unless nutritionally appropriate foods are purchased and prepared.

A 66-year-old patient has a body mass index (BMI) of 31 kg/m2, a normal C-reactive protein level, and low transferrin and albumin levels. The nurse will plan patient teaching to increase the patient's intake of foods that are high in a. iron. b. protein. c. calories. d. carbohydrate.

ANS: B The patient's C-reactive protein and transferrin levels indicate low protein stores. The BMI is in the obese range, so increasing caloric intake is not indicated. The data do not indicate a need for increased carbohydrate or iron intake.

A 22-year-old who is hospitalized with anorexia nervosa is 5 ft 5 in (163 cm) tall and weighs 90 pounds (41 kg). Laboratory tests reveal hypokalemia and iron-deficiency anemia. Which nursing diagnosis has the highest priority for the patient? a. Risk for activity intolerance related to anemia and weakness b. Risk for electrolyte imbalance related to poor eating patterns c. Ineffective health maintenance related to obsession with body image d. Imbalanced nutrition: less than body requirements related to refusal to eat

ANS: B The patient's hypokalemia may lead to life-threatening cardiac dysrhythmias. The other diagnoses also are appropriate for this patient but are not associated with immediate risk for fatal complications.

A patient with a peptic ulcer who has a nasogastric (NG) tube develops sudden, severe upper abdominal pain, diaphoresis, and a very firm abdomen. Which action should the nurse take next? a. Irrigate the NG tube. b. Obtain the vital signs. c. Listen for bowel sounds. d. Give the ordered antacid.

ANS: B The patient's symptoms suggest acute perforation, and the nurse should assess for signs of hypovolemic shock. Irrigation of the NG tube, administration of antacids, or both would be contraindicated because any material in the stomach will increase the spillage into the peritoneal cavity. The nurse should assess the bowel sounds, but this is not the first action that should be taken.

How many grams of protein will the nurse recommend to meet the minimum daily requirement for a patient who weighs 145 pounds (66 kg)? a. 36 b. 53 c. 75 d. 98

ANS: B The recommended daily protein intake is 0.8 to 1 g/kg of body weight, which for this patient is 66 kg × 0.8 g = 52.8 or 53 g/day.

When using a soft, silicone nasogastric tube for enteral feedings, the nurse will need to a. avoid giving medications through the feeding tube. b. flush the tubing after checking for residual volumes. c. administer continuous feedings using an infusion pump. d. replace the tube every 3 to 5 days to avoid mucosal damage.

ANS: B The soft silicone feeding tubes are small in diameter and can easily become clogged unless they are flushed after the nurse checks the residual volume. Either intermittent or continuous feedings can be given. The tubes are less likely to cause mucosal damage than the stiffer polyvinyl chloride tubes used for nasogastric suction and do not need to be replaced at certain intervals. Medications can be given through these tubes, but flushing after medication administration is important to avoid clogging

Which adult will the nurse plan to teach about risks associated with obesity? a. Man who has a BMI of 18 kg/m2 b. Man with a 42 in waist and 44 in hips c. Woman who has a body mass index (BMI) of 24 kg/m2 d. Woman with a waist circumference of 34 inches (86 cm)

ANS: B The waist-to-hip ratio for this patient is 0.95, which exceeds the recommended level of <0.80. A patient with a BMI of 18 kg/m2 is considered underweight. A BMI of 24 kg/m2 is normal. Health risks associated with obesity increase in women with a waist circumference larger than 35 in (89 cm) and men with a waist circumference larger than 40 in (102 cm).

The nurse is caring for a client with cirrhosis. Which assessment findings indicate that the client has deficient vitamin K absorption caused by this hepatic disease? a) Dyspnea and fatigue b) Ascites and orthopnea c) Purpura and petechiae d) Gynecomastia and testicular atrophy

C) Purpura and petechiae A hepatic disorder, such as cirrhosis, may disrupt the liver's normal use of vitamin K to produce prothrombin (a clotting factor). Consequently, the nurse should monitor the client for signs of bleeding, including purpura and petechiae. Dyspnea and fatigue suggest anemia. Ascites and orthopnea are unrelated to vitamin K absorption. Gynecomastia and testicular atrophy result from decreased estrogen metabolism by the diseased liver.

After the nurse teaches a patient about the recommended amounts of foods from animal and plant sources, which menu selections indicate that the initial instructions about diet have been understood? a. 3 oz of lean beef, 2 oz of low-fat cheese, and a tomato slice b. 3 oz of roasted pork, a cup of corn, and a cup of carrot sticks c. Cup of tossed salad and nonfat dressing topped with a chicken breast d. Half cup of tuna mixed with nonfat mayonnaise and a half cup of celery

ANS: B This selection is most consistent with the recommendation of the American Institute for Cancer Research that one third of the diet should be from animal sources and two thirds from plant source foods. The other choices all have higher ratios of animal origin foods to plant source foods than would be recommended

After bariatric surgery, a patient who is being discharged tells the nurse, "I prefer to be independent. I am not interested in any support groups." Which response by the nurse is best? a. "I hope you change your mind so that I can suggest a group for you." b. "Tell me what types of resources you think you might use after this surgery." c. "Support groups have been found to lead to more successful weight loss after surgery." d. "Because there are many lifestyle changes after surgery, we recommend support groups."

ANS: B This statement allows the nurse to assess the individual patient's potential needs and preferences. The other statements offer the patient more information about the benefits of support groups, but fail to acknowledge the patient's preferences

The nurse obtains these assessment data for a patient who has been taking orlistat (Xenical) for several months as part of a weight loss program. Which finding is most important to report to the health care provider? a. The patient frequently has liquid stools. b. The patient is pale and has many bruises. c. The patient is experiencing a plateau in weight loss. d. The patient complains of abdominal bloating after meals.

ANS: B Because orlistat blocks the absorption of fat-soluble vitamins, the patient may not be receiving an adequate amount of vitamin K, resulting in a decrease in clotting factors. Abdominal bloating and liquid stools are common side effects of orlistat and indicate that the nurse should remind the patient that fat in the diet may increase these side effects. Weight loss plateaus are normal during weight reduction

When working with an obese patient who is enrolled in a behavior modification program, which nursing action is appropriate? a. Having the patient write down the caloric intake of each meal b. Asking the patient about situations that tend to increase appetite c. Encouraging the patient to eat small amounts throughout the day rather than having scheduled meals d. Suggesting that the patient have a reward, such as a piece of sugarless candy, after achieving a weight-loss goal

ANS: B Behavior modification programs focus on how and when the person eats and de-emphasize aspects such as calorie counting. Nonfood rewards are recommended for achievement of weight-loss goals. Patients are often taught to restrict eating to designated meals when using behavior modification.

Which assessment will the nurse do to help determine if an obese patient seen in the clinic has metabolic syndrome? a. Take the patient's apical pulse. b. Check the patient's blood pressure. c. Ask the patient about dietary intake. d. Dipstick the patient's urine for protein.

ANS: B Elevated blood pressure is one of the characteristics of metabolic syndrome. The other information also may be obtained by the nurse, but it will not assist with the diagnosis of metabolic syndrome.

A patient has been on a 1000-calorie diet with a daily exercise routine and a prescription for sibutramine (Meridia) for 10 weeks. Which information obtained by the nurse is important to report to the health care provider? a. The patient has not lost any weight for the last 2 weeks. b. The patient tells the nurse about occasional palpitations. c. The patient complains about having chronic constipation. d. The patient reports walking only 3 days during the last week.

ANS: B The patient may be experiencing an increase in heart rate caused by the sibutramine (Meridia) that should be evaluated further by the health care provider. Plateaus during weight loss programs are common. Chronic constipation may be a side effect of the sibutramine, and the nurse should instruct the patient in measures such as eating more high fiber foods and increasing fluid intake. The nurse should reinforce the need to exercise more frequently, but no additional intervention by the health care provider is necessary regarding the patient's activity level.

When admitting a patient with a stroke who is unconscious and unresponsive to stimuli, the nurse learns from the patient's family that the patient has a history of gastroesophageal reflux disease (GERD). The nurse will plan to do frequent assessments of the patient's a. apical pulse. b. bowel sounds. c. breath sounds. d. abdominal girth.

ANS: C Because GERD may cause aspiration, the unconscious patient is at risk for developing aspiration pneumonia. Bowel sounds, abdominal girth, and apical pulse will not be affected by the patient's stroke or GERD and do not require more frequent monitoring than the routine.

A patient with deep partial-thickness burns experiences severe pain associated with nausea during dressing changes. Which action will be most useful in decreasing the patient's nausea? a. The patient NPO for 2 hours before and after dressing changes. b. Avoid performing dressing changes close to the patient's mealtimes. c. Administer the prescribed morphine sulfate before dressing changes. d. Give the ordered prochlorperazine (Compazine) before dressing changes.

ANS: C Because the patient's nausea is associated with severe pain, it is likely that it is precipitated by stress and pain. The best treatment will be to provide adequate pain medication before dressing changes. The nurse should avoid doing painful procedures close to mealtimes, but nausea/vomiting that occur at other times also should be addressed. Keeping the patient NPO does not address the reason for the nausea and vomiting and will have an adverse effect on the patient's nutrition. Administration of antiemetics is not the best choice for a patient with nausea caused by pain.

The nurse implements discharge teaching for a patient following a gastroduodenostomy for treatment of a peptic ulcer. Which patient statement indicates that the teaching has been effective? a. "Persistent heartburn is expected after surgery." b. "I will try to drink liquids along with my meals." c. "Vitamin supplements may be needed to prevent problems with anemia." d. "I will need to choose foods that are low in fat and high in carbohydrate."

ANS: C Cobalamin deficiency may occur after partial gastrectomy, and the patient may need to receive cobalamin via injections or nasal spray. Foods that have moderate fat and low carbohydrate should be chosen to prevent dumping syndrome. Ingestion of liquids with meals is avoided to prevent dumping syndrome. Although peptic ulcer disease may recur, persistent heartburn is not expected after surgery and the patient should call the health care provider if this occurs.

To evaluate an obese patient for adverse effects of lorcaserin (Belviq), which action will the nurse take? a. Take the apical pulse rate. b. Check sclera for jaundice. c. Ask about bowel movements. d. Assess for agitation or restlessness.

ANS: C Constipation is a common side effect of lorcaserin. The other assessments would be appropriate for other weight-loss medications

A patient with a body mass index (BMI) of 17 kg/m2 and a low albumin level is being admitted by the nurse. Which assessment finding will the nurse expect to find? a. Restlessness b. Hypertension c. Pitting edema d. Food allergies

ANS: C Edema occurs when serum albumin levels and plasma oncotic pressure decrease. The blood pressure and level of consciousness are not directly affected by malnutrition. Food allergies are not an indicator of nutritional status.

The nurse will plan to teach the patient with newly diagnosed achalasia that a. a liquid or blenderized diet will be necessary. b. drinking fluids with meals should be avoided. c. endoscopic procedures may be used for treatment. d. lying down and resting after meals is recommended.

ANS: C Endoscopic and laparoscopic procedures are the most effective therapy for improving symptoms caused by achalasia. Patients are advised to drink fluid with meals. Keeping the head elevated after eating will improve esophageal emptying. A semisoft diet is recommended to improve esophageal emptying.

All of the following nursing actions are included in the plan of care for a patient who is malnourished. Which action is appropriate for the nurse to delegate to nursing assistive personnel (NAP)? a. Assist the patient to choose high nutrition items from the menu. b. Monitor the patient for skin breakdown over the bony prominences. c. Offer the patient the prescribed nutritional supplement between meals. d. Assess the patient's strength while ambulating the patient in the room.

ANS: C Feeding the patient and assisting with oral intake are included in NAP education and scope of practice. Assessing the patient and assisting the patient in choosing high nutrition foods require LPN/LVN- or RN-level education and scope of practice.

After 6 hours of parenteral nutrition (PN) infusion, the nurse checks a patient's capillary blood glucose level and finds it to be 120 mg/dL. The most appropriate action by the nurse is to a. obtain a venous blood glucose specimen. b. slow the infusion rate of the PN infusion. c. recheck the capillary blood glucose in 4 hours. d. notify the health care provider of the glucose level.

ANS: C Mild hyperglycemia is expected during the first few days after PN is started and requires ongoing monitoring. Because the glucose elevation is small and expected, notification of the health care provider is not necessary. There is no need to obtain a venous specimen for comparison. Slowing the rate of the infusion is beyond the nurse's scope of practice and will decrease the patient's nutritional intake.

Which statement by the nurse is most likely to help a morbidly obese 22-year-old man in losing weight on a 1000-calorie diet? a. "It will be necessary to change lifestyle habits permanently to maintain weight loss." b. "You will decrease your risk for future health problems such as diabetes by losing weight now." c. "You are likely to notice changes in how you feel with just a few weeks of diet and exercise." d. "Most of the weight that you lose during the first weeks of dieting is water weight rather than fat."

ANS: C Motivation is a key factor in successful weight loss and a short-term outcome provides a higher motivation. A 22-year-old patient is unlikely to be motivated by future health problems. Telling a patient that the initial weight loss is water will be discouraging, although this may be correct. Changing lifestyle habits is necessary, but this process occurs over time and discussing this is not likely to motivate the patient

A few months after bariatric surgery, a 56-year-old man tells the nurse, "My skin is hanging in folds. I think I need cosmetic surgery." Which response by the nurse is most appropriate? a. "The important thing is that you are improving your health." b. "The skinfolds will disappear once most of the weight is lost." c. "Cosmetic surgery is a possibility once your weight has stabilized." d. "Perhaps you would like to talk to a counselor about your body image."

ANS: C Reconstructive surgery may be used to eliminate excess skinfolds after at least a year has passed since the surgery. Skinfolds may not disappear over time, especially in older patients. The response, "The important thing is that your weight loss is improving your health," ignores the patient's concerns about appearance and implies that the nurse knows what is important. Whereas it may be helpful for the patient to talk to a counselor, it is more likely to be helpful to know that cosmetic surgery is available

The health care provider prescribes antacids and sucralfate (Carafate) for treatment of a patient's peptic ulcer. The nurse will teach the patient to take a. antacids 30 minutes before the sucralfate. b. sucralfate at bedtime and antacids before meals. c. antacids after eating and sucralfate 30 minutes before eating. d. sucralfate and antacids together 30 minutes before each meal.

ANS: C Sucralfate is most effective when the pH is low and should not be given with or soon after antacid. Antacids are most effective when taken after eating. Administration of sucralfate 30 minutes before eating and antacids just after eating will ensure that both drugs can be most effective. The other regimens will decrease the effectiveness of the medications.

A patient is receiving tube feedings through a percutaneous endoscopic gastrostomy (PEG). Which action will the nurse include in the plan of care? a. Keep the patient positioned on the left side. b. Obtain a daily x-ray to verify tube placement. c. Check the gastric residual volume every 4 to 6 hours. d. Avoid giving bolus tube feedings through the PEG tube.

ANS: C The gastric residual volume is assessed every 4 to 6 hours to decrease the risk for aspiration. The patient does not need to be positioned on the left side. An x-ray is obtained immediately after placement of the PEG tube to check position, but daily x-rays are not needed. Bolus feedings can be administered through a PEG tube.

After successfully losing 1 lb weekly for several months, a patient at the clinic has not lost any weight for the last month. The nurse should first a. review the diet and exercise guidelines with the patient. b. instruct the patient to weigh and record weights weekly. c. ask the patient whether there have been any changes in exercise or diet patterns. d. discuss the possibility that the patient has reached a temporary weight loss plateau.

ANS: C The initial nursing action should be assessment of any reason for the change in weight loss. The other actions may be needed, but further assessment is required before any interventions are planned or implemented

After receiving change-of-shift report, which patient should the nurse assess first? a. A patient who was admitted yesterday with gastrointestinal (GI) bleeding and has melena b. A patient who is crying after receiving a diagnosis of esophageal cancer c. A patient with esophageal varices who has a blood pressure of 96/54 mm Hg d. A patient with nausea who has a dose of metoclopramide (Reglan) scheduled

ANS: C The patient's history and blood pressure indicate possible hemodynamic instability caused by GI bleeding. The data about the other patients do not indicate acutely life-threatening complications.

A patient with acute gastrointestinal (GI) bleeding is receiving normal saline IV at a rate of 500 mL/hr. Which assessment finding obtained by the nurse is most important to communicate immediately to the health care provider? a. The patient's blood pressure (BP) has increased to 142/94 mm Hg. b. The nasogastric (NG) suction is returning coffee-ground material. c. The patient's lungs have crackles audible to the midline. d. The bowel sounds are very hyperactive in all four quadrants.

ANS: C The patient's lung sounds indicate that pulmonary edema may be developing as a result of the rapid infusion of IV fluid and that the fluid infusion rate should be slowed. The return of coffee-ground material in an NG tube is expected for a patient with upper GI bleeding. The BP is slightly elevated but would not be an indication to contact the health care provider immediately. Hyperactive bowel sounds are common when a patient has GI bleeding.

What information will the nurse include for an overweight 35-year-old woman who is starting a weight-loss plan? a. Weigh yourself at the same time every morning and evening. b. Stick to a 600- to 800-calorie diet for the most rapid weight loss. c. Low carbohydrate diets lead to rapid weight loss but are difficult to maintain. d. Weighing all foods on a scale is necessary to choose appropriate portion sizes.

ANS: C The restrictive nature of fad diets makes the weight loss achieved by the patient more difficult to maintain. Portion size can be estimated in other ways besides weighing. Severely calorie-restricted diets are not necessary for patients in the overweight category of obesity and need to be closely supervised. Patients should weigh weekly rather than daily

The nurse notes that the peripheral parenteral nutrition (PN) bag has only 20 mL left and a new PN bag has not yet arrived from the pharmacy. Which intervention is the priority? a. Monitor the patient's capillary blood glucose until a new PN bag is hung b. Flush the peripheral line with saline and wait until the new PN bag is available c. Infuse 5% dextrose in water until the new PN bag is delivered from the pharmacy d. Decrease the rate of the current PN infusion to 10 mL/hr until the new bag arrives

ANS: C To prevent hypoglycemia, the nurse should infuse a 5% dextrose solution until the next PN bag can be started. Decreasing the rate of the ordered PN infusion is beyond the nurse's scope of practice. Flushing the line and then waiting for the next bag may lead to hypoglycemia. Monitoring the capillary blood glucose is appropriate but is not the priority.

A 61-year-old man is being admitted for bariatric surgery. Which nursing action can the nurse delegate to unlicensed assistive personnel (UAP)? a. Demonstrate use of the incentive spirometer. b. Plan methods for bathing and turning the patient. c. Assist with IV insertion by holding adipose tissue out of the way. d. Develop strategies to provide privacy and decrease embarrassment.

ANS: C UAP can assist with IV placement by assisting with patient positioning or holding skinfolds aside. Planning for care and patient teaching require registered nurse (RN)-level education and scope of practice

The nurse is caring for a 54-year-old female patient on the first postoperative day after a Roux-en-Y gastric bypass procedure. Which assessment finding should be reported immediately to the surgeon? a. Bilateral crackles audible at both lung bases b. Redness, irritation, and skin breakdown in skinfolds c. Emesis of bile-colored fluid past the nasogastric (NG) tube d. Use of patient-controlled analgesia (PCA) several times an hour for pain

ANS: C Vomiting with an NG tube in place indicates that the NG tube needs to be repositioned by the surgeon to avoid putting stress on the gastric sutures. The nurse should implement actions to decrease skin irritation and have the patient cough and deep breathe, but these do not indicate a need for rapid notification of the surgeon. Frequent PCA use after bariatric surgery is expected

In planning preoperative teaching for a patient undergoing a Roux-en-Y gastric bypass as treatment for morbid obesity, the nurse places the highest priority on a. demonstrating passive range-of-motion exercises to the legs. b. discussing the necessary postoperative modifications in lifestyle. c. teaching the patient proper coughing and deep breathing techniques. d. educating the patient about the postoperative presence of a nasogastric (NG) tube.

ANS: C Coughing and deep breathing can prevent major postoperative complications such as carbon monoxide retention and hypoxemia. Information about passive range of motion, the NG tube, and postoperative modifications in lifestyle also will be discussed, but avoidance of respiratory complications is the priority goal after surgery.

When the health care provider in the outpatient clinic is considering prescribing sibutramine (Meridia) for a patient, which patient information is most important for the nurse to discuss with the provider? a. The patient has a permanent pacemaker. b. The patient's goal is to lose 90 lb (41 kg). c. The patient's blood pressure is usually 135-145/85-95. d. The patient used fenfluramine (Pondimin) in the past for weight loss.

ANS: C Side effects of sibutramine (Meridia) include hypertension. A permanent pacemaker and a history of fenfluramine use are not contraindications for sibutramine. Sibutramine is prescribed for patients who have large weight loss goals.

What specific information will the nurse include in patient teaching for an overweight patient who is starting a weight loss plan? a. Weigh yourself at the same time every morning. b. Start dieting with a 600- to 800-calorie diet for rapid weight loss. c. Low carbohydrate diets lead to rapid weight loss but are difficult to maintain. d. Weighing all foods on a scale is necessary to choose appropriate portion sizes.

ANS: C The restrictive nature of fad diets makes the weight loss achieved by the patient more difficult to maintain. Portion size can be estimated in other ways besides weighing. Severely calorie-restricted diets are not necessary for patients in the overweight category of obesity and need to be closely supervised. Patients should weigh weekly rather than daily.

Which assessment finding in a patient who had a total gastrectomy 12 hours previously is most important to report to the health care provider? a. Absent bowel sounds b. Scant nasogastric (NG) tube drainage c. Complaints of incisional pain d. Temperature 102.1° F (38.9° C)

ANS: D An elevation in temperature may indicate leakage at the anastomosis, which may require return to surgery or keeping the patient NPO. The other findings are expected in the immediate postoperative period for patients who have this surgery.

A patient who recently has been experiencing frequent heartburn is seen in the clinic. The nurse will anticipate teaching the patient about a. barium swallow. b. radionuclide tests. c. endoscopy procedures. d. proton pump inhibitors.

ANS: D Because diagnostic testing for heartburn that is probably caused by gastroesophageal reflux disease (GERD) is expensive and uncomfortable, proton pump inhibitors are frequently used for a short period as the first step in the diagnosis of GERD. The other tests may be used but are not usually the first step in diagnosis.

A patient with protein calorie malnutrition who has had abdominal surgery is receiving parenteral nutrition (PN). Which assessment information obtained by the nurse is the best indicator that the patient is receiving adequate nutrition? a. Blood glucose is 110 mg/dL. b. Serum albumin level is 3.5 mg/dL. c. Fluid intake and output are balanced. d. Surgical incision is healing normally.

ANS: D Because poor wound healing is a possible complication of malnutrition for this patient, normal healing of the incision is an indicator of the effectiveness of the PN in providing adequate nutrition. Blood glucose is monitored to prevent the complications of hyperglycemia and hypoglycemia, but it does not indicate that the patient's nutrition is adequate. The intake and output will be monitored but do not indicate that the PN is effective. The albumin level is in the low-normal range but does not reflect adequate caloric intake, which is also important for the patient.

All of the following orders are received for a patient who has vomited 1500 mL of bright red blood. Which order will the nurse implement first? a. Insert a nasogastric (NG) tube and connect to suction. b. Administer intravenous (IV) famotidine (Pepcid) 40 mg. c. Draw blood for typing and crossmatching. d. Infuse 1000 mL of lactated Ringer's solution.

ANS: D Because the patient has vomited a large amount of blood, correction of hypovolemia and prevention of hypovolemic shock are the priorities. The other actions also are important to implement quickly but are not the highest priorities.

Which information will the nurse include when teaching a patient with newly diagnosed gastroesophageal reflux disease (GERD)? a. "Peppermint tea may be helpful in reducing your symptoms." b. "You should avoid eating between meals to reduce acid secretion." c. "Vigorous physical activities may increase the incidence of reflux." d. "It will be helpful to keep the head of your bed elevated on blocks."

ANS: D Elevating the head of the bed will reduce the incidence of reflux while the patient is sleeping. Peppermint will lower LES pressure and increase the chance for reflux. Small, frequent meals are recommended to avoid abdominal distention. There is no need to make changes in physical activities because of GERD.

Which choice from the hospital menu indicates that the patient has understood the nurse's teaching about choosing high calorie, high protein foods? a. Baked fish with applesauce b. Beef noodle soup and canned corn c. Fresh vegetables with yogurt topping d. Fried chicken with potatoes and gravy

ANS: D Foods that are high in calories include fried foods and those covered with sauces. High protein foods include meat and dairy products. The other choices are lower in calories and protein.

The nurse is assessing a patient with gastroesophageal reflux disease (GERD) who is experiencing increasing discomfort. Which patient statement indicates that additional patient education about GERD is needed? a. "I take antacids between meals and at bedtime each night." b. "I sleep with the head of the bed elevated on 4-inch blocks." c. "I quit smoking several years ago, but I still chew a lot of gum." d. "I eat small meals throughout the day and have a bedtime snack."

ANS: D GERD is exacerbated by eating late at night, and the nurse should plan to teach the patient to avoid eating at bedtime. The other patient actions are appropriate to control symptoms of GERD.

After the nurse teaches a patient with gastroesophageal reflux disease (GERD) about recommended dietary modifications, which diet choice for a snack 2 hours before bedtime indicates that the teaching has been effective? a. Chocolate pudding b. Glass of low-fat milk c. Peanut butter sandwich d. Cherry gelatin and fruit

ANS: D Gelatin and fruit are low fat and will not decrease lower esophageal sphincter (LES) pressure. Foods like chocolate are avoided because they lower LES pressure. Milk products increase gastric acid secretion. High-fat foods such as peanut butter decrease both gastric emptying and LES pressure.

A patient who requires daily use of a nonsteroidal anti-inflammatory drug (NSAID) for management of severe rheumatoid arthritis has recently developed melena. The nurse will anticipate teaching the patient about a. substitution of acetaminophen (Tylenol) for the NSAID. b. use of enteric-coated NSAIDs to reduce gastric irritation. c. reasons for using corticosteroids to treat the rheumatoid arthritis. d. the benefits of misoprostol (Cytotec) in protecting the gastrointestinal (GI) mucosa.

ANS: D Misoprostol, a prostaglandin analog, reduces acid secretion and incidence of upper GI bleeding associated with NSAID use. Enteric coating of NSAIDs does not reduce the risk for GI bleeding. Corticosteroids increase the risk for ulcer development and will not be substituted for NSAIDs for this patient. Acetaminophen will not be effective in treating the patient's rheumatoid arthritis.

After vertical banded gastroplasty, a 42-year-old male patient returns to the surgical nursing unit with a nasogastric tube to low, intermittent suction and a patient-controlled analgesia (PCA) machine for pain control. Which nursing action should be included in the postoperative plan of care? a. Offer sips of fruit juices at frequent intervals. b. Irrigate the nasogastric (NG) tube frequently. c. Remind the patient that PCA use may slow the return of bowel function. d. Support the surgical incision during patient coughing and turning in bed.

ANS: D The incision should be protected from strain to decrease the risk for wound dehiscence. The patient should be encouraged to use the PCA because pain control will improve the cough effort and patient mobility. NG irrigation may damage the suture line or overfill the stomach pouch. Sugar-free clear liquids are offered during the immediate postoperative time to decrease the risk for dumping syndrome

The nurse is performing an admission assessment on a 20-year-old college student who is being admitted for electrolyte disorders of unknown etiology. Which assessment finding is most important to report to the health care provider? a. The patient's knuckles are macerated. b. The patient uses laxatives on a daily basis. c. The patient has a history of weight fluctuations. d. The patient's serum potassium level is 2.9 mEq/L.

ANS: D The low serum potassium level may cause life-threatening cardiac dysrhythmias and potassium supplementation is needed rapidly. The other information also will be reported because it suggests that bulimia may be the etiology of the patient's electrolyte disturbances, but it does not suggest imminent life-threatening complications.

A patient who is vomiting bright red blood is admitted to the emergency department. Which assessment should the nurse perform first? a. Checking the level of consciousness b. Measuring the quantity of any emesis c. Auscultating the chest for breath sounds d. Taking the blood pressure (BP) and pulse

ANS: D The nurse is concerned about blood loss and possible hypovolemic shock in a patient with acute gastrointestinal (GI) bleeding; BP and pulse are the best indicators of these complications. The other information also is important to obtain, but BP and pulse rate are the best indicators for hypoperfusion.

A patient with a recent 20-pound unintended weight loss is diagnosed with stomach cancer. Which nursing action will be included in the plan of care? a. Refer the patient for hospice services. b. Infuse IV fluids through a central line. c. Teach the patient about antiemetic therapy. d. Offer supplemental feedings between meals.

ANS: D The patient data indicate a poor nutritional state and improvement in nutrition will be helpful in improving response to therapies such as surgery, chemotherapy, or radiation. Nausea and vomiting are not common clinical manifestations of stomach cancer. There is no indication that the patient requires hospice or IV fluid infusions.

The nurse receives change-of-shift report about the following four patients. Which patient will the nurse assess first? a. A patient who has malnutrition associated with 4+ generalized pitting edema b. A patient whose parenteral nutrition has 10 mL of solution left in the infusion bag c. A patient whose gastrostomy tube is plugged after crushed medications were given through the tube d. A patient who is receiving continuous enteral feedings and has new-onset crackles throughout the lungs

ANS: D The patient data suggest aspiration has occurred and rapid assessment and intervention are needed. The other patients also should be assessed as quickly as possible, but the data about them do not suggest any immediately life-threatening complications.

A patient who has a wound infection after major surgery has only been taking in about 50% to 75% of the ordered meals and states, "Nothing on the menu really appeals to me." Which action by the nurse will be most effective in improving the patient's oral intake? a. Make a referral to the dietician. b. Order at least six small meals daily. c. Teach the patient about high-calorie, high-protein foods. d. Have family members bring in favorite foods from home.

ANS: D The patient's statement that the hospital foods are unappealing indicates that favorite home-cooked foods might improve intake. The other interventions also may help improve the patient's intake, but the most effective action will be to offer the patient more appealing foods.

A patient with recurring heartburn receives a new prescription for esomeprazole (Nexium). In teaching the patient about this medication, the nurse explains that this drug a. neutralizes stomach acid and provides relief of symptoms in a few minutes. b. reduces the reflux of gastric acid by increasing the rate of gastric emptying. c. coats and protects the lining of the stomach and esophagus from gastric acid. d. treats gastroesophageal reflux disease by decreasing stomach acid production.

ANS: D The proton pump inhibitors decrease the rate of gastric acid secretion. Promotility drugs such as metoclopramide (Reglan) increase the rate of gastric emptying. Cryoprotective medications such as sucralfate (Carafate) protect the stomach. Antacids neutralize stomach acid and work rapidly.

A patient who is nauseated and vomiting up blood-streaked fluid is admitted to the hospital with acute gastritis. To determine possible risk factors for gastritis, the nurse will ask the patient about a. the amount of fat in the diet. b. history of recent weight gain or loss. c. any family history of gastric problems. d. use of nonsteroidal anti-inflammatory drugs (NSAIDs).

ANS: D Use of an NSAID is associated with damage to the gastric mucosa, which can result in acute gastritis. Family history, recent weight gain or loss, and fatty foods are not risk factors for acute gastritis.

The nurse is developing a weight loss plan for a 21-year-old patient who is morbidly obese. Which statement by the nurse is most likely to help the patient in losing weight on the planned 1000-calorie diet? a. "It will be necessary to change lifestyle habits permanently to maintain weight loss." b. "You will decrease your risk for future health problems such as diabetes by losing weight now." c. "Most of the weight that you lose during the first weeks of dieting is water weight rather than fat." d. "You are likely to start to notice changes in how you feel with just a few weeks of diet and exercise."

ANS: D Motivation is a key factor in successful weight loss and a short-term outcome provides a higher motivation. A 21-year-old patient is unlikely to be motivated by future health problems. Telling a patient that the initial weight loss is water will be discouraging, although this may be correct. Changing lifestyle habits is necessary, but this process occurs over time and discussing this is not likely to motivate the patient.

A few months after bariatric surgery, a 62-year-old patient tells the nurse, "My skin is hanging in folds. I think I need cosmetic surgery." Which response by the nurse is most appropriate? a. "Perhaps you would like to talk to a counselor about your body image." b. "The important thing is that your weight loss is improving your health." c. "The skinfolds will gradually disappear once most of the weight is lost." d. "Cosmetic surgery is certainly a possibility once your weight has stabilized."

ANS: D Reconstructive surgery may be used to eliminate excess skinfolds after at least a year has passed since the surgery. Skinfolds may not disappear over time, especially in older patients. The response, "The important thing is that your weight loss is improving your health" ignores the patient's concerns about appearance and implies that the nurse knows what is important. Whereas it may be helpful for the patient to talk to a counselor, it is more likely to be helpful to know that cosmetic surgery is available.

A patient returns to the surgical nursing unit following a vertical banded gastroplasty with a nasogastric tube to low, intermittent suction and a patient-controlled analgesia (PCA) machine for pain control. Which nursing action should be included in the postoperative plan of care? a. Irrigate the nasogastric (NG) tube frequently with normal saline. b. Offer sips of sweetened liquids at frequent intervals. c. Remind the patient that PCA use may slow the return of bowel function. d. Support the surgical incision during patient coughing and turning in bed.

ANS: D The incision should be protected from strain to decrease the risk for wound dehiscence. The patient should be encouraged to use the PCA since pain control will improve cough effort and patient mobility. NG irrigation may damage the suture line or overfill the stomach pouch. Sugar-free clear liquids are offered during the immediate postoperative time to decrease the risk for dumping syndrome.

When developing a weight reduction plan for an obese patient who wants to lose weight, which question should the nurse ask first? a. "Which food types do you like best?" b. "How long have you been overweight?" c. "What kind of physical activities do you enjoy?" d. "What factors do you think led to your obesity?"

ANS: D The nurse should obtain information about the patient's perceptions of the reasons for the obesity to develop a plan individualized to the patient. The other information also will be obtained from the patient, but the patient is more likely to make changes when the patient's beliefs are considered in planning.

Which of these patients in the clinic will the nurse plan to teach about risks associated with obesity? a. Patient who has a BMI of 18 kg/m2 b. Patient with a waist circumference 34 inches (86 cm) c. Patient who has a body mass index (BMI) of 24 kg/m2 d. Patient whose waist measures 30 in (75 cm) and hips measure 34 in (85 cm)

ANS: D The waist-to-hip ratio for this patient is 0.88, which exceeds the recommended level of <0.80. A BMI of 24 kg/m2 is normal. Health risks associated with obesity increase in women with a waist circumference larger than 35 in (89 cm) and men with a waist circumference larger than 40 in (102 cm). A patient with a BMI of 18 kg/m2 is considered underweight.

On the first postoperative day the nurse is caring for a patient who has had a Roux-en-Y gastric bypass procedure. Which assessment finding should be reported immediately to the surgeon? a. Use of patient-controlled analgesia (PCA) several times an hour for pain b. Irritation and skin breakdown in skinfolds c. Bilateral crackles audible at both lung bases d. Emesis of bile-colored fluid past the nasogastric (NG) tube

ANS: D Vomiting with an NG tube in place indicates that the NG tube needs to be repositioned by the surgeon to avoid putting stress on the gastric sutures. The nurse should implement actions to decrease skin irritation and have the patient cough and deep breathe, but these do not indicate a need for rapid notification of the surgeon. Frequent PCA use after bariatric surgery is expected.

A client with cirrhosis is ordered to have a daily measurement of his abdominal girth. Identify the anatomical landmark where the tape measure should be placed when obtaining this measurement.

Abdominal girth should be measured at the umbilicus to obtain the most accurate measurement.

A participant in a health fair has asked the nurse about the role of drugs in liver disease. What health promotion teaching has the most potential to prevent drug-induced hepatitis Finish all prescribed courses of antibiotics, regardless of symptom resolution. Adhere to dosing recommendations of OTC analgesics Ensure that expired medications are disposed of safely. Ensure that pharmacists regularly review drug regimens for potential interactions.

Adhere to dosing recommendations of OTC analgesics. Although any medication can affect liver function, use of acetaminophen (found in many over-the-counter medications used to treat fever and pain) has been identified as the leading cause of acute liver failure. Finishing prescribed antibiotics and avoiding expired medications are unrelated to this disease. Drug interactions are rarely the cause of drug-induced hepatitis.

A group of students are studying for an examination on the gastrointestinal (GI) system and are reviewing the structures of the esophagus and stomach. The students demonstrate understanding of the material when they identify which of the following as the opening between the stomach and duodenum? a) Hypoharyngeal sphincter b) Cardiac sphincter c) Pyloric sphincter d) Ileocecal valve

C) Pyloric Sphincter The pyloric sphincter is the opening between the stomach and duodenum. The cardiac sphincter is the opening between the esophagus and the stomach. The hypopharyngeal sphincter or upper esophageal sphincter prevents food or fluids from re-entering the pharynx. The ileocecal valve is located at the distal end of the small intestine and regulates flow of intestinal contents into the large intestine.

Following administration of a dose of metoclopramide (Reglan) to the patient, the nurse determines that the medication has been effective when which of the following is noted? A) Decreased blood pressure B) Absence of muscle tremors C) Relief of nausea and vomiting D) No further episodes of diarrhea

C) Relief of nausea and vomiting Metoclopramide is classified as a prokinetic and antiemetic medication. If it is effective, the patient's nausea and vomiting should resolve.

The patient is having a gastroduodenostomy (Billroth I operation) for stomach cancer. What long-term complication is occurring when the patient reports generalized weakness, sweating, palpitations, and dizziness 15 to 30 minutes after eating? A) Malnutrition B) Bile reflux gastritis C) Dumping syndrome D) Postprandial hypoglycemia

After a Billroth I operation, dumping syndrome may occur 15 to 30 minutes after eating because of the hypertonic fluid going to the intestine and additional fluid being drawn into the bowel. Malnutrition may occur but does not cause these symptoms. Bile reflux gastritis cannot happen when the stomach has been removed. Postprandial hypoglycemia occurs with similar symptoms, but 2 hours after eating.

The nurse is preparing the discharge teaching plan for a client who has recently been diagnosed with his first episode of acute pancreatitis. Which of the following is it essential to include? Glucose testing. Low carbohydrate diet. Insulin administration. Alcohol restriction.

Alcohol restriction

A client with acute liver failure is admitted with ascites and pruritus. Which of the following nursing diagnoses would be appropriate for these symptoms? High-Risk for Infection Activity Intolerance Ineffective Breathing Pattern Altered Comfort Fluid Volume Deficit Altered Thought Processes

Altered Comfort Ineffective Breathing Pattern

When providing discharge teaching for the patient after a laparoscopic cholecystectomy, what information should the nurse include? A) A lower-fat diet may be better tolerated for several weeks. B) Do not return to work or normal activities for 3 weeks. C) Bile-colored drainage will probably drain from the incision. D) Keep the bandages on and the puncture site dry until it heals.

Although the usual diet can be resumed, a low-fat diet is usually better tolerated for several weeks following surgery. Normal activities can be gradually resumed as the patient tolerates. Bile-colored drainage or pus, redness, swelling, severe pain, and fever may all indicate infection. The bandage may be removed the day after surgery, and the patient can shower.

Following administration of a dose of metoclopramide (Reglan) to the patient, the nurse determines that the medication has been effective when which of the following is noted? A) Decreased blood pressure B) Absence of muscle tremors C) Relief of nausea and vomiting D) No further episodes of diarrhea

C) Relief of nausea and vomiting. (Metoclopramide is classified as a prokinetic and antiemetic medication. If it is effective, the patient's nausea and vomiting should resolve).

"The results of a patient's recent endoscopy indicate the presence of peptic ulcer disease (PUD). Which of the following teaching points should the nurse provide to the patient in light of his new diagnosis? " A) "You'll need to drink at least two to three glasses of milk daily." B) "It would likely be beneficial for you to eliminate drinking alcohol." C) "Many people find that a minced or pureed diet eases their symptoms of PUD." D) "Your medications should allow you to maintain your present diet while minimizing symptoms

Although there is no specific recommended dietary modification for PUD, most patients find it necessary to make some sort of dietary modifications to minimize symptoms. Milk may exacerbate PUD and alcohol is best avoided because it can delay healing

The nurse is caring for a client with acute pancreatitis. Which of the following statements is true regarding the care of this patient? Acute pancreatitis is always accompanied by a right pleural effusion. An elevated serum lipase and amylase levels are diagnostic signs. Acute pancreatitis frequently requires surgical drainage of a pseudocyst. Insulin administration is only required when the patient is on TPN (total parenteral nutrition).

An elevated serum lipase and amylase levels are diagnostic signs.

"A nurse is caring for a client admitted to the hospital with a suspected diagnosis of acute appendicitis. Which of the following laboratory results would the nurse expect to note if the client does have appendicitis? 1. Leukopenia with a shift to the right 2. Leukocytosis with a shift to the right 3.Leukocytosis with a shift to the left 4. Leukopenia with a shift to the left"

Answer 2 - no rationale

"A client with acute appendicitis develops a fever, tachycardia, and hypotension. Based on these assessment findings, the nurse should further assess the client for which of the following complications?" " 1. Deficient fluid volume. 2. Intestinal obstruction. 3. Bowel ischemia. 4. Peritonitis.

Answer 4 Complications of acute appendicitis are perforation, peritonitis, and abscess development. Signs of the development of peritonitis include abdominal pain and distention, tachycardia, tachypnea, nausea, vomiting, and fever. Because peritonitis can cause hypovolemic shock, hypotension can develop. Deficient fluid volume would not cause a fever. Intestinal obstruction would cause abdominal distention, diminished or absent bowel sounds, and abdominal paIn. Bowel ischemia has signs and symptoms similar to those found with intestinal obstruction.

The nurse is monitoring a client admitted to the hospital with a dx of appendicitis who is scheduled for surgery in 2 hours. The client begins to complain of increased abdominal pain and begins to vomit. On assessment, the nurse notes that the abdomen is distended and bowel sounds are diminished. Which is appropriate nursing intervention? "A. Notify the physician B. Administer the prescribe pain medication C. Call and ask the operating room team to perform the surgery as soon as possible D. Reposition the client and apply a heating pad on warm setting to the clients abdomen"

Answer A The health-care provider should be noti-fied when the nurse has the needed infor-mation.`

"The nurse is caring for a patient in the emergency department with complaints of acute abdominal pain, nausea, and vomiting. When the nurse palpates the patient's left lower abdominal quadrant, the patient complains of pain in the right lower quadrant. The nurse will document this as which of the following diagnostic signs of appendicitis? "a. Rovsing sign b. referred pain c. Chvostek's sign d. rebound tenderness correct answer: A"

Answer A In patients with suspected appendicitis, Rovsing sign may be elicited by palpation of the left lower quadrant, causing pain to be felt in the right lower quadrant.

The teaching plan for the patient being discharged following an acute episode of upper GI bleeding will concern information concerning the importance of (select all that apply) a. only taking aspirin with milk or bread products b. avoiding taking aspirin and drugs containing aspirin c. taking only drugs prescribed by the health care provider d. taking all drugs 1 hour before mealtime to prevent further bleeding e. reading all OTC drug labels to avoid those containing stearic acid and calcium

Answer A, C Aspirin contributes to thinning the blood and is linked to causing things like peritonitis further increasing the risk for bleeding. Taking only health care prescribed drugs can greatly reduce the risk from accidentally using OTC meds that might contribute to bleeding

A male client with extreme weakness, pallor, weak peripheral pulses, and disorientation is admitted to the emergency department. His wife reports that he has been "spitting up blood." A Mallory-Weiss tear is suspected, and the nurse begins taking a client history from the client's wife. The question by the nurse that demonstrates her understanding of Mallory-Weiss tearing is: a. "Tell me about your husband's alcohol usage." b. "Is your husband being treated for tuberculosis?" c. "Has your husband recently fallen or injured his chest?" d. "Describe spices and condiments your husband uses on food."

Answer A. A Mallory-Weiss tear is associated with massive bleeding after a tear occurs in the mucous membrane at the junction of the esophagus and stomach. There is a strong relationship between ethanol usage, resultant vomiting, and a Mallory-Weiss tear. The bleeding is coming from the stomach, not from the lungs as would be true in some cases of tuberculosis. A Mallory-Weiss tear doesn't occur from chest injuries or falls and isn't associated with eating spicy foods.

A female client being seen in a physician's office has just been scheduled for a barium swallow the next day. The nurse writes down which instruction for the client to follow before the test? a. Fast for 8 hours before the test b. Eat a regular supper and breakfast c. Continue to take all oral medications as scheduled d. Monitor own bowel movement pattern for constipation

Answer A. A barium swallow is an x-ray study that uses a substance called barium for contrast to highlight abnormalities in the gastrointestinal tract. The client should fast for 8 to 12 hours before the test, depending on physician instructions. Most oral medications also are withheld before the test. After the procedure, the nurse must monitor for constipation, which can occur as a result of the presence of barium in the gastrointestinal tract.

A male client undergoes total gastrectomy. Several hours after surgery, the nurse notes that the client's nasogastric (NG) tube has stopped draining. How should the nurse respond? a. Notify the physician b. Reposition the tube c. Irrigate the tube d. Increase the suction leveL

Answer A. An NG tube that fails to drain during the postoperative period should be reported to the physician immediately. It may be clogged, which could increase pressure on the suture site because fluid isn't draining adequately. Repositioning or irrigating an NG tube in a client who has undergone gastric surgery can disrupt the anastomosis. Increasing the level of suction may cause trauma to GI mucosa or the suture line.

A male client is recovering from an ileostomy that was performed to treat inflammatory bowel disease. During discharge teaching, the nurse should stress the importance of: a. increasing fluid intake to prevent dehydration. b. wearing an appliance pouch only at bedtime. c. consuming a low-protein, high-fiber diet. d. taking only enteric-coated medications.

Answer A. Because stool forms in the large intestine, an ileostomy typically drains liquid waste. To avoid fluid loss through ileostomy drainage, the nurse should instruct the client to increase fluid intake. The nurse should teach the client to wear a collection appliance at all times because ileostomy drainage is incontinent, to avoid high-fiber foods because they may irritate the intestines, and to avoid enteric-coated medications because the body can't absorb them after an ileostomy

The nurse is reviewing the record of a female client with Crohn's disease. Which stool characteristics should the nurse expect to note documented in the client's record? a. Diarrhea b. Chronic constipation c. Constipation alternating with diarrhea d. Stools constantly oozing form the rectum

Answer A. Crohn's disease is characterized by nonbloody diarrhea of usually not more than four to five stools daily. Over time, the diarrhea episodes increase in frequency, duration, and severity. Options B, C, and D are not characteristics of Crohn's disease.

The nurse is monitoring a female client for the early signs and symptoms of dumping syndrome. Which of the following indicate this occurrence? a. Sweating and pallor b. Bradycardia and indigestion c. Double vision and chest pain d. Abdominal cramping and pain

Answer A. Early manifestations of dumping syndrome occur 5 to 30 minutes after eating. Symptoms include vertigo, tachycardia, syncope, sweating, pallor, palpitations, and the desire to lie down.

Which diagnostic test would be used first to evaluate a client with upper GI bleeding? a. Endoscopy b. Upper GI series c. Hemoglobin (Hb) levels and hematocrit (HCT) d. Arteriography

Answer A. Endoscopy permits direct evaluation of the upper GI tract and can detect 90% of bleeding lesions. An upper GI series, or barium study, usually isn't the diagnostic method of choice, especially in a client with acute active bleeding who's vomiting and unstable. An upper GI series is also less accurate than endoscopy. Although an upper GI series might confirm the presence of a lesion, it wouldn't necessarily reveal whether the lesion is bleeding. Hb levels and HCT, which indicate loss of blood volume, aren't always reliable indicators of GI bleeding because a decrease in these values may not be seen for several hours. Arteriography is an invasive study associated with life-threatening complications and wouldn't be used for an initial evaluation

Dr. Smith has determined that the client with hepatitis has contracted the infection form contaminated food. The nurse understands that this client is most likely experiencing what type of hepatitis? a. Hepatitis A b. Hepatitis B c. Hepatitis C d. Hepatitis D

Answer A. Hepatitis A is transmitted by the fecal-oral route via contaminated food or infected food handlers. Hepatitis B, C, and D are transmitted most commonly via infected blood or body fluids.

A female client who has just been diagnosed with hepatitis A asks, "How could I have gotten this disease?" What is the nurse's best response? a. "You may have eaten contaminated restaurant food." b. "You could have gotten it by using I.V. drugs." c. "You must have received an infected blood transfusion." d. "You probably got it by engaging in unprotected sex.

Answer A. Hepatitis A virus typically is transmitted by the oral-fecal route — commonly by consuming food contaminated by infected food handlers. The virus isn't transmitted by the I.V. route, blood transfusions, or unprotected sex. Hepatitis B can be transmitted by I.V. drug use or blood transfusion. Hepatitis C can be transmitted by unprotected sex.

Nurse Ryan is assessing for correct placement of a nosogartric tube. The nurse aspirates the stomach contents and check the contents for pH. The nurse verifies correct tube placement if which pH value is noted? a. 3.5 b. 7.0 c. 7.35 d. 7.5

Answer A. If the nasogastric tube is in the stomach, the pH of the contents will be acidic. Gastric aspirates have acidic pH values and should be 3.5 or lower. Option B indicates a slightly acidic pH. Option C indicates a neutral pH. Option D indicates an alkaline pH.

Nurse Juvy is caring for a client with cirrhosis of the liver. To minimize the effects of the disorder, the nurse teaches the client about foods that are high in thiamine. The nurse determines that the client has the best understanding of the dietary measures to follow if the client states an intension to increase the intake of: a. Pork b. Milk c. Chicken d. Broccoli

Answer A. The client with cirrhosis needs to consume foods high in thiamine. Thiamine is present in a variety of foods of plant and animal origin. Pork products are especially rich in this vitamin. Other good food sources include nuts, whole grain cereals, and legumes. Milk contains vitamins A, D, and B2. Poultry contains niacin. Broccoli contains vitamins C, E, and K and folic acid

While a female client is being prepared for discharge, the nasogastric (NG) feeding tube becomes clogged. To remedy this problem and teach the client's family how to deal with it at home, what should the nurse do? a. Irrigate the tube with cola. b. Advance the tube into the intestine. c. Apply intermittent suction to the tube. d. Withdraw the obstruction with a 30-ml syringe.

Answer A. The nurse should irrigate the tube with cola because its effervescence and acidity are suited to the purpose, it's inexpensive, and it's readily available in most homes. Advancing the NG tube is inappropriate because the tube is designed to stay in the stomach and isn't long enough to reach the intestines. Applying intermittent suction or using a syringe for aspiration is unlikely to dislodge the material clogging the tube but may create excess pressure. Intermittent suction may even collapse the tube.

The nurse is teaching a female client how to perform a colostomy irrigation. To enhance the effectiveness of the irrigation and fecal returns, what measure should the nurse instruct the client to do? a. Increase fluid intake b. Place heat on the abdomen c. Perform the irrigation in the evening d. Reduce the amount of irrigation solution

Answer A. To enhance effectiveness of the irrigation and fecal returns, the client is instructed to increase fluid intake and to take other measures to prevent constipation. Options B, C and D will not enhance the effectiveness of this procedure.

Nurse Oliver checks for residual before administering a bolus tube feeding to a client with a nasogastric tube and obtains a residual amount of 150 mL. What is appropriate action for the nurse to take? a. Hold the feeding b. Reinstill the amount and continue with administering the feeding c. Elevate the client's head at least 45 degrees and administer the feeding d. Discard the residual amount and proceed with administering the feeding

Answer A. Unless specifically indicated, residual amounts more than 100 mL require holding the feeding. Therefore options B, C, and D are incorrect. Additionally, the feeding is not discarded unless its contents are abnormal in color or characteristics.

A male client with cholelithiasis has a gallstone lodged in the common bile duct. When assessing this client, the nurse expects to note: a. yellow sclerae. b. light amber urine. c. circumoral pallor. d. black, tarry stools

Answer A. Yellow sclerae may be the first sign of jaundice, which occurs when the common bile duct is obstructed. Urine normally is light amber. Circumoral pallor and black, tarry stools don't occur in common bile duct obstruction; they are signs of hypoxia and GI bleeding, respectively.

"When preparing a male client, age 51, for surgery to treat appendicitis, the nurse formulates a nursing diagnosis of Risk for infection related to inflammation, perforation, and surgery. What is the rationale for choosing this nursing diagnosis? "a. Obstruction of the appendix may increase venous drainage and cause the appendix to rupture. b. Obstruction of the appendix reduces arterial flow, leading to ischemia, inflammation, and rupture of the appendix. c. The appendix may develop gangrene and rupture, especially in a middle-aged client. d. Infection of the appendix diminishes necrotic arterial blood flow and increases venous drainage."

Answer B. A client with appendicitis is at risk for infection related to inflammation, perforation, and surgery because obstruction of the appendix causes mucus fluid to build up, increasing pressure in the appendix and compressing venous outflow drainage. The pressure continues to rise with venous obstruction; arterial blood flow then decreases, leading to ischemia from lack of perfusion. Inflammation and bacterial growth follow, and swelling continues to raise pressure within the appendix, resulting in gangrene and rupture. Geriatric, not middle-aged, clients are especially susceptible to appendix rupture.

"When preparing a male client, age 51, for surgery to treat appendicitis, the nurse formulates a nursing diagnosis of Risk for infection related to inflammation, perforation, and surgery. What is the rationale for choosing this nursing diagnosis? a. Obstruction of the appendix may increase venous drainage and cause the appendix to rupture. b. Obstruction of the appendix reduces arterial flow, leading to ischemia, inflammation, and rupture of the appendix. c. The appendix may develop gangrene and rupture, especially in a middle-aged client. d. Infection of the appendix diminishes necrotic arterial blood flow and increases venous drainage."

Answer B. A client with appendicitis is at risk for infection related to inflammation, perforation, and surgery because obstruction of the appendix causes mucus fluid to build up, increasing pressure in the appendix and compressing venous outflow drainage. The pressure continues to rise with venous obstruction; arterial blood flow then decreases, leading to ischemia from lack of perfusion. Inflammation and bacterial growth follow, and swelling continues to raise pressure within the appendix, resulting in gangrene and rupture. Geriatric, not middle-aged, clients are especially susceptible to appendix rupture.

A client is admitted to the health care facility with abdominal pain, a low-grade fever, abdominal distention, and weight loss. The physician diagnoses acute pancreatitis. What is the primary goal of nursing care for this client? a) Maintaining adequate nutritional status b) Preventing fluid volume overload c) Relieving abdominal pain d) Teaching about the disease and its treatment

C) Relieving abdominal pain The predominant clinical feature of acute pancreatitis is abdominal pain, which usually reaches peak intensity several hours after onset of the illness. Therefore, relieving abdominal pain is the nurse's primary goal. Because acute pancreatitis causes nausea and vomiting, the nurse should try to prevent fluid volume deficit, not overload. The nurse can't help the client achieve adequate nutrition or understand the disease and its treatment until the client is comfortable and no longer in pain.

When preparing a male client, age 51, for surgery to treat appendicitis, the nurse formulates a nursing diagnosis of Risk for infection related to inflammation, perforation, and surgery. What is the rationale for choosing this nursing diagnosis? a. Obstruction of the appendix may increase venous drainage and cause the appendix to rupture. b. Obstruction of the appendix reduces arterial flow, leading to ischemia, inflammation, and rupture of the appendix. c. The appendix may develop gangrene and rupture, especially in a middle-aged client. d. Infection of the appendix diminishes necrotic arterial blood flow and increases venous drainage.

Answer B. A client with appendicitis is at risk for infection related to inflammation, perforation, and surgery because obstruction of the appendix causes mucus fluid to build up, increasing pressure in the appendix and compressing venous outflow drainage. The pressure continues to rise with venous obstruction; arterial blood flow then decreases, leading to ischemia from lack of perfusion. Inflammation and bacterial growth follow, and swelling continues to raise pressure within the appendix, resulting in gangrene and rupture. Geriatric, not middle-aged, clients are especially susceptible to appendix rupture.

Which of the following is the best method for determining nasogastric tube placement? a) Placement of external end of tube under water b) Testing of pH of gastric aspirate c) X-ray d) Observation of gastric aspirate

C) X-ray Radiologic identification of tube placement in the stomach is the most reliable method. Gastric fluid may be grassy green, brown, clear, or odorless while an aspirate from the lungs may be off-white or tan. Hence, checking aspirate is not the best method of determining nasogastric tube placement in the stomach. Gastric pH values are typically lower or more acidic than that of the intestinal or respiratory tract, but not always. Placement of external end of tube under water and watching for air bubbles is not a reliable method for determining nasogastric tube placement in the stomach.

When preparing a male client, age 51, for surgery to treat appendicitis, the nurse formulates a nursing diagnosis of Risk for infection related to inflammation, perforation, and surgery. What is the rationale for choosing this nursing diagnosis?"a. Obstruction of the appendix may increase venous drainage and cause the appendix to rupture. b. Obstruction of the appendix reduces arterial flow, leading to ischemia, inflammation, and rupture of the appendix. c. The appendix may develop gangrene and rupture, especially in a middle-aged client. d. Infection of the appendix diminishes necrotic arterial blood flow and increases venous drainage."

Answer B. A client with appendicitis is at risk for infection related to inflammation, perforation, and surgery because obstruction of the appendix causes mucus fluid to build up, increasing pressure in the appendix and compressing venous outflow drainage. The pressure continues to rise with venous obstruction; arterial blood flow then decreases, leading to ischemia from lack of perfusion. Inflammation and bacterial growth follow, and swelling continues to raise pressure within the appendix, resulting in gangrene and rupture. Geriatric, not middle-aged, clients are especially susceptible to appendix rupture.

When evaluating a male client for complications of acute pancreatitis, the nurse would observe for: a. increased intracranial pressure. b. decreased urine output. c. bradycardia. d. hypertension

Answer B. Acute pancreatitis can cause decreased urine output, which results from the renal failure that sometimes accompanies this condition. Intracranial pressure neither increases nor decreases in a client with pancreatitis. Tachycardia, not bradycardia, usually is associated with pulmonary or hypovolemic complications of pancreatitis. Hypotension can be caused by a hypovolemic complication, but hypertension usually isn't related to acute pancreatitis.

The nurse is caring for a male client postoperatively following creation of a colostomy. Which nursing diagnosis should the nurse include in the plan of care? a. Sexual dysfunction b. Body image, disturbed c. Fear related to poor prognosis d. Nutrition: more than body requirements, imbalanced

Answer B. Body image, disturbed relates to loss of bowel control, the presence of a stoma, the release of fecal material onto the abdomen, the passage of flatus, odor, and the need for an appliance (external pouch). No data in the question support options A and C. Nutrition: less than body requirements, imbalanced is the more likely nursing diagnosis.

The nurse is caring for a male client with a diagnosis of chronic gastritis. The nurse monitors the client knowing that this client is at risk for which vitamin deficiency? a. Vitamin A b. Vitamin B12 c. Vitamin C d. Vitamin E

Answer B. Chronic gastritis causes deterioration and atrophy of the lining of the stomach, leading to the loss of the function of the parietal cells. The source of the intrinsic factor is lost, which results in the inability to absorb vitamin B12. This leads to the development of pernicious anemia. The client is not at risk for vitamin A, C, or E deficiency.

The nurse is preparing a discharge teaching plan for the male client who had umbilical hernia repair. What should the nurse include in the plan? a. Irrigating the drain b. Avoiding coughing c. Maintaining bed rest d. Restricting pain medication

Answer B. Coughing is avoided following umbilical hernia repair to prevent disruption of tissue integrity, which can occur because of the location of this surgical procedure. Bed rest is not required following this surgical procedure. The client should take analgesics as needed and as prescribed to control pain. A drain is not used in this surgical procedure, although the client may be instructed in simple dressing changes.

A female client with dysphagia is being prepared for discharge. Which outcome indicates that the client is ready for discharge? a. The client doesn't exhibit rectal tenesmus. b. The client is free from esophagitis and achalasia. c. The client reports diminished duodenal inflammation. d. The client has normal gastric structures.

Answer B. Dysphagia may be the reason why a client with esophagitis or achalasia seeks treatment. Dysphagia isn't associated with rectal tenesmus, duodenal inflammation, or abnormal gastric structures.

What laboratory finding is the primary diagnostic indicator for pancreatitis? a. Elevated blood urea nitrogen (BUN) b. Elevated serum lipase c. Elevated aspartate aminotransferase (AST) d. Increased lactate dehydrogenase (LD)

Answer B. Elevation of serum lipase is the most reliable indicator of pancreatitis because this enzyme is produced solely by the pancreas. A client's BUN is typically elevated in relation to renal dysfunction; the AST, in relation to liver dysfunction; and LD, in relation to damaged cardiac muscle.

The nurse is instructing the male client who has an inguinal hernia repair how to reduce postoperative swelling following the procedure. What should the nurse tell the client? a. Limit oral fluid b. Elevate the scrotum c. Apply heat to the abdomen d. Remain in a low-fiber diet

Answer B. Following inguinal hernia repair, the client should be instructed to elevate the scrotum and apply ice packs while in bed to decrease pain and swelling. The nurse also should instruct the client to apply a scrotal support when out of bed. Heat will increase swelling. Limiting oral fluids and a low-fiber diet can cause constipation.

A male client with pancreatitis complains of pain. The nurse expects the physician to prescribe meperidine (Demerol) instead of morphine to relieve pain because: a. meperidine provides a better, more prolonged analgesic effect. b. morphine may cause spasms of Oddi's sphincter. c. meperidine is less addictive than morphine. d. morphine may cause hepatic dysfunction.

Answer B. For a client with pancreatitis, the physician will probably avoid prescribing morphine because this drug may trigger spasms of the sphincter of Oddi (a sphincter at the end of the pancreatic duct), causing irritation of the pancreas. Meperidine has a somewhat shorter duration of action than morphine. The two drugs are equally addictive. Morphine isn't associated with hepatic dysfunction.

A male client with a recent history of rectal bleeding is being prepared for a colonoscopy. How should the nurse position the client for this test initially? a. Lying on the right side with legs straight b. Lying on the left side with knees bent c. Prone with the torso elevated d. Bent over with hands touching the floor

Answer B. For a colonoscopy, the nurse initially should position the client on the left side with knees bent. Placing the client on the right side with legs straight, prone with the torso elevated, or bent over with hands touching the floor wouldn't allow proper visualization of the large intestine

A male client has just been diagnosed with hepatitis A. On assessment, the nurse expects to note: a. severe abdominal pain radiating to the shoulder. b. anorexia, nausea, and vomiting. c. eructation and constipation. d. abdominal ascites.

Answer B. Hallmark signs and symptoms of hepatitis A include anorexia, nausea, vomiting, fatigue, and weakness. Abdominal pain may occur but doesn't radiate to the shoulder. Eructation and constipation are common in gallbladder disease, not hepatitis A. Abdominal ascites is a sign of advanced hepatic disease, not an early sign of hepatitis A.

Which condition is most likely to have a nursing diagnosis of fluid volume deficit? a. Appendicitis b. Pancreatitis c. Cholecystitis d. Gastric ulcer

Answer B. Hypovolemic shock from fluid shifts is a major factor in acute pancreatitis. The other conditions are less likely to exhibit fluid volume deficit.

The nurse is performing a colostomy irrigation on a male client. During the irrigation, the client begins to complain of abdominal cramps. What is the appropriate nursing action? a. Notify the physician b. Stop the irrigation temporarily c. Increase the height of the irrigation d. Medicate for pain and resume the irrigation

Answer B. If cramping occurs during a colostomy irrigation, the irrigation flow is stopped temporarily and the client is allowed to rest. Cramping may occur from an infusion that is too rapid or is causing too much pressure. The physician does not need to be notified. Increasing the height of the irrigation will cause further discomfort. Medicating the client for pain is not the appropriate action in this situation.

A client is suspected of having hepatitis. Which diagnostic test result will assist in confirming this diagnosis? a. Elevated hemoglobin level b. Elevated serum bilirubin level c. Elevated blood urea nitrogen level d. Decreased erythrocycle sedimentation rate

Answer B. Laboratory indicators of hepatitis include elevated liver enzyme levels, elevated serum bilirubin levels, elevated erythrocyte sedimentation rates, and leukopenia. An elevated blood urea nitrogen level may indicate renal dysfunction. A hemoglobin level is unrelated to this diagnosis.

A male client is recovering from a small-bowel resection. To relieve pain, the physician prescribes meperidine (Demerol), 75 mg I.M. every 4 hours. How soon after administration should meperidine's onset of action occur? a. 5 to 10 minutes b. 15 to 30 minutes c. 30 to 60 minutes d. 2 to 4 hours

Answer B. Meperidine's onset of action is 15 to 30 minutes. It peaks between 30 and 60 minutes and has a duration of action of 2 to 4 hours.

The nurse is monitoring a female client receiving paregoric to treat diarrhea for drug interactions. Which drugs can produce additive constipation when given with an opium preparation? a. Antiarrhythmic drugs b. Anticholinergic drugs c. Anticoagulant drugs d. Antihypertensive drugs

Answer B. Paregoric has an additive effect of constipation when used with anticholinergic drugs. Antiarrhythmics, anticoagulants, and antihypertensives aren't known to interact with paregoric

male client has undergone a colon resection. While turning him, wound dehiscence with evisceration occurs. The nurse's first response is to: a. call the physician. b. place saline-soaked sterile dressings on the wound. c. take a blood pressure and pulse. d. pull the dehiscence closed.

Answer B. The nurse should first place saline-soaked sterile dressings on the open wound to prevent tissue drying and possible infection. Then the nurse should call the physician and take the client's vital signs. The dehiscence needs to be surgically closed, so the nurse should never try to close it.

To prevent gastroesophageal reflux in a male client with hiatal hernia, the nurse should provide which discharge instruction? a. "Lie down after meals to promote digestion." b. "Avoid coffee and alcoholic beverages." c. "Take antacids with meals." d. "Limit fluid intake with meals.

Answer B. To prevent reflux of stomach acid into the esophagus, the nurse should advise the client to avoid foods and beverages that increase stomach acid, such as coffee and alcohol. The nurse also should teach the client to avoid lying down after meals, which can aggravate reflux, and to take antacids after eating. The client need not limit fluid intake with meals as long as the fluids aren't gastric irritants.

The doctor ordered for a complete blood count. After the test, Nurse Ray received the result from the laboratory. Which laboratory values will confirm the diagnosis of appendicitis? a. RBC 5.5 x 106/mm3 b. Hct 44 % c. WBC 13, 000/mm3 d. Hgb 15 g/dL"

Answer C "Rationale: Increase in WBC counts is suggestive of appendicitis because of bacterial invasion and inflammation. Normal WBC count is 5, 000 - 10, 000/mm3. Other options are normal values."

The nurse is caring for a male client with cirrhosis. Which assessment findings indicate that the client has deficient vitamin K absorption caused by this hepatic disease? a. Dyspnea and fatigue b. Ascites and orthopnea c. Purpura and petechiae d. Gynecomastia and testicular atrophy

Answer C. A hepatic disorder, such as cirrhosis, may disrupt the liver's normal use of vitamin K to produce prothrombin (a clotting factor). Consequently, the nurse should monitor the client for signs of bleeding, including purpura and petechiae. Dyspnea and fatigue suggest anemia. Ascites and orthopnea are unrelated to vitamin K absorption. Gynecomastia and testicular atrophy result from decreased estrogen metabolism by the diseased liver.

A male client with a peptic ulcer is scheduled for a vagotomy and the client asks the nurse about the purpose of this procedure. Which response by the nurse best describes the purpose of a vagotomy? a. Halts stress reactions b. Heals the gastric mucosa c. Reduces the stimulus to acid secretions d. Decreases food absorption in the stomach

Answer C. A vagotomy, or cutting of the vagus nerve, is done to eliminate parasympathetic stimulation of gastric secretion. Options A, B, and D are incorrect descriptions of a vagotomy.

The nurse is providing discharge instructions to a male client following gastrectomy and instructs the client to take which measure to assist in preventing dumping syndrome? a. Ambulate following a meal b. Eat high carbohydrate foods c. Limit the fluid taken with meal d. Sit in a high-Fowler's position during meals

Answer C. Dumping syndrome is a term that refers to a constellation of vasomotor symptoms that occurs after eating, especially following a Billroth II procedure. Early manifestations usually occur within 30 minutes of eating and include vertigo, tachycardia, syncope, sweating, pallor, palpitations, and the desire to lie down. The nurse should instruct the client to decrease the amount of fluid taken at meals and to avoid high-carbohydrate foods, including fluids such as fruit nectars; to assume a low-Fowler's position during meals; to lie down for 30 minutes after eating to delay gastric emptying; and to take antispasmodics as prescribed.

A male client who is recovering from surgery has been advanced from a clear liquid diet to a full liquid diet. The client is looking forward to the diet change because he has been "bored" with the clear liquid diet. The nurse would offer which full liquid item to the client? a. Tea b. Gelatin c. Custard d. Popsicle

Answer C. Full liquid food items include items such as plain ice cream, sherbet, breakfast drinks, milk, pudding and custard, soups that are strained, and strained vegetable juices. A clear liquid diet consists of foods that are relatively transparent. The food items in options A, B, and D are clear liquids.

Which of the following factors can cause hepatitis A? a. Contact with infected blood b. Blood transfusions with infected blood c. Eating contaminated shellfish d. Sexual contact with an infected person

Answer C. Hepatitis A can be caused by consuming contaminated water, milk, or food — especially shellfish from contaminated water. Hepatitis B is caused by blood and sexual contact with an infected person. Hepatitis C is usually caused by contact with infected blood, including receiving blood transfusions

an 18 yr old is admitted with an acute onset of right lower quadrant pain. Appendicitis is suspected. For which clinical indicator should the nurse assess the client to determine if the pain is secondary to appendicitis A) urinary retention B) gastric hyperacidity C) rebound tenderness D) increased lower bowel motility

C) rebound tenderness is a classic subjective sign of appendicitis

Nurse Joy is preparing to administer medication through a nasogastric tube that is connected to suction. To administer the medication, the nurse would: a. Position the client supine to assist in medication absorption b. Aspirate the nasogastric tube after medication administration to maintain patency c. Clamp the nasogastric tube for 30 minutes following administration of the medication d. Change the suction setting to low intermittent suction for 30 minutes after medication administration

Answer C. If a client has a nasogastric tube connected to suction, the nurse should wait up to 30 minutes before reconnecting the tube to the suction apparatus to allow adequate time for medication absorption. Aspirating the nasogastric tube will remove the medication just administered. Low intermittent suction also will remove the medication just administered. The client should not be placed in the supine position because of the risk for aspiration.

"The nurse determines that a patient has experienced the beneficial effects of medication therapy with famotidine (Pepcid) when which of the following symptoms is relieved? " A) Nausea B) Belching C) Epigastric pain D) Difficulty swallowing

C, "Famotidine is an H2-receptor antagonist that inhibits parietal cell output of HCl acid and minimizes damage to gastric mucosa related to hyperacidity, thus relieving epigastric pain

The nurse is caring for a female client following a Billroth II procedure. Which postoperative order should the nurse question and verify? a. Leg exercises b. Early ambulation c. Irrigating the nasogastric tube d. Coughing and deep-breathing exercises

Answer C. In a Billroth II procedure, the proximal remnant of the stomach is anastomosed to the proximal jejunum. Patency of the nasogastric tube is critical for preventing the retention of gastric secretions. The nurse should never irrigate or reposition the gastric tube after gastric surgery, unless specifically ordered by the physician. In this situation, the nurse should clarify the order. Options A, B, and D are appropriate postoperative interventions.

5. Which of the following drugs is a histamine blocker and reduces levels of gastric acid?" A. Omeprazole (Prilosec) B. Metoclopramide (Reglan) C. Cimetidine (Tagamet) D. Magnesium Hydroxide (Maalox)

C, Cimetidine bind to H2 in the tissue and decreases the production of gastric acid

Which clinical manifestations of inflammatory bowel disease are common to both patients with ulcerative colitis (UC) and Crohn's disease (select all that apply)? A) Restricted to rectum B) Strictures are common. C) Bloody, diarrhea stools D) Cramping abdominal pain E) Lesions penetrate intestine.

C, D) Clinical manifestations of UC and Crohn's disease include bloody diarrhea, cramping abdominal pain, and nutritional disorders. Intestinal lesions associated with UC are usually restricted to the rectum before moving into the colon. Lesions that penetrate the intestine or cause strictures are characteristic of Crohn's disease.

The nurse is reviewing the medication record of a female client with acute gastritis. Which medication, if noted on the client's record, would the nurse question? a. Digoxin (Lanoxin) b. Furosemide (Lasix) c. Indomethacin (Indocin) d. Propranolol hydrochloride (Inderal)

Answer C. Indomethacin (Indocin) is a nonsteroidal anti-inflammatory drug and can cause ulceration of the esophagus, stomach, or small intestine. Indomethacin is contraindicated in a client with gastrointestinal disorders. Furosemide (Lasix) is a loop diuretic. Digoxin is a cardiac medication. Propranolol (Inderal) is a β-adrenergic blocker. Furosemide, digoxin, and propranolol are not contraindicated in clients with gastric disorders.

The nurse is reviewing the physician's orders written for a male client admitted to the hospital with acute pancreatitis. Which physician order should the nurse question if noted on the client's chart? a. NPO status b. Nasogastric tube inserted c. Morphine sulfate for pain d. An anticholinergic medication

Answer C. Meperidine (Demerol) rather than morphine sulfate is the medication of choice to treat pain because morphine sulfate can cause spasms in the sphincter of Oddi. Options A, B, and D are appropriate interventions for the client with acute pancreatitis.

The nurse is caring for a hospitalized female client with a diagnosis of ulcerative colitis. Which finding, if noted on assessment of the client, would the nurse report to the physician? a. Hypotension b. Bloody diarrhea c. Rebound tenderness d. A hemoglobin level of 12 mg/dL

Answer C. Rebound tenderness may indicate peritonitis. Bloody diarrhea is expected to occur in ulcerative colitis. Because of the blood loss, the client may be hypotensive and the hemoglobin level may be lower than normal. Signs of peritonitis must be reported to the physician.

The nurse is caring for a female client with active upper GI bleeding. What is the appropriate diet for this client during the first 24 hours after admission? a. Regular diet b. Skim milk c. Nothing by mouth d. Clear liquids

Answer C. Shock and bleeding must be controlled before oral intake, so the client should receive nothing by mouth. A regular diet is incorrect. When the bleeding is controlled, the diet is gradually increased, starting with ice chips and then clear liquids. Skim milk shouldn't be given because it increases gastric acid production, which could prolong bleeding. A liquid diet is the first diet offered after bleeding and shock are controlled.

The nurse is performing an abdominal assessment and inspects the skin of the abdomen. The nurse performs which assessment technique next? a. Palpates the abdomen for size b. Palpates the liver at the right rib margin c. Listens to bowel sounds in all for quadrants d. Percusses the right lower abdominal quadrant

Answer C. The appropriate sequence for abdominal examination is inspection, auscultation, percussion, and palpation. Auscultation is performed after inspection to ensure that the motility of the bowel and bowel sounds are not altered by percussion or palpation. Therefore, after inspecting the skin on the abdomen, the nurse should listen for bowel sounds.

Nurse Berlinda is assigned to a 41-year-old client who has a diagnosis of chronic pancreatitis. The nurse reviews the laboratory result, anticipating a laboratory report that indicates a serum amylase level of: a. 45 units/L b. 100 units/L c. 300 units/L d. 500 units/L

Answer C. The normal serum amylase level is 25 to 151 units/L. With chronic cases of pancreatitis, the rise in serum amylase levels usually does not exceed three times the normal value. In acute pancreatitis, the value may exceed five times the normal value. Options A and B are within normal limits. Option D is an extremely elevated level seen in acute pancreatitis.

A female client with viral hepatitis A is being treated in an acute care facility. Because the client requires enteric precautions, the nurse should: a. place the client in a private room. b. wear a mask when handling the client's bedpan. c. wash the hands after touching the client. d. wear a gown when providing personal care for the client.

Answer C. To maintain enteric precautions, the nurse must wash the hands after touching the client or potentially contaminated articles and before caring for another client. A private room is warranted only if the client has poor hygiene — for instance, if the client is unlikely to wash the hands after touching infective material or is likely to share contaminated articles with other clients. For enteric precautions, the nurse need not wear a mask and must wear a gown only if soiling from fecal matter is likely.

A nurse is preparing to remove a nasogartric tube from a female client. The nurse should instruct the client to do which of the following just before the nurse removes the tube? a. Exhale b. Inhale and exhale quickly c. Take and hold a deep breath d. Perform a Valsalva maneuver

Answer C. When the nurse removes a nasogastric tube, the client is instructed to take and hold a deep breath. This will close the epiglottis. This allows for easy withdrawal through the esophagus into the nose. The nurse removes the tube with one smooth, continuous pull.

A nurse is inserting a nasogastric tube in an adult male client. During the procedure, the client begins to cough and has difficulty breathing. Which of the following is the appropriate nursing action? a. Quickly insert the tube b. Notify the physician immediately c. Remove the tube and reinsert when the respiratory distress subsides d. Pull back on the tube and wait until the respiratory distress subsides

Answer D. During the insertion of a nasogastric tube, if the client experiences difficulty breathing or any respiratory distress, withdraw the tube slightly, stop the tube advancement, and wait until the distress subsides. Options B and C are unnecessary. Quickly inserting the tube is not an appropriate action because, in this situation, it may be likely that the tube has entered the bronchus.

The nurse is assessing a male client 24 hours following a cholecystectomy. The nurse noted that the T tube has drained 750 mL of green-brown drainage since the surgery. Which nursing intervention is appropriate? a. Clamp the T tube b. Irrigate the T tube c. Notify the physician d. Document the findings

Answer D. Following cholecystectomy, drainage from the T tube is initially bloody and then turns to a greenish-brown color. The drainage is measured as output. The amount of expected drainage will range from 500 to 1000 mL/day. The nurse would document the output.

The nurse caring for a client with small-bowel obstruction would plan to implement which nursing intervention first? a. Administering pain medication b. Obtaining a blood sample for laboratory studies c. Preparing to insert a nasogastric (NG) tube d. Administering I.V. fluids

Answer D. I.V. infusions containing normal saline solution and potassium should be given first to maintain fluid and electrolyte balance. For the client's comfort and to assist in bowel decompression, the nurse should prepare to insert an NG tube next. A blood sample is then obtained for laboratory studies to aid in the diagnosis of bowel obstruction and guide treatment. Blood studies usually include a complete blood count, serum electrolyte levels, and blood urea nitrogen level. Pain medication often is withheld until obstruction is diagnosed because analgesics can decrease intestinal motility.

The nurse is monitoring a female client with a diagnosis of peptic ulcer. Which assessment findings would most likely indicate perforation of the ulcer? a. Bradycardia b. Numbness in the legs c. Nausea and vomiting d. A rigid, board-like abdomen

Answer D. Perforation of an ulcer is a surgical emergency and is characterized by sudden, sharp, intolerable severe pain beginning in the midepigastric area and spreading over the abdomen, which becomes rigid and board-like. Nausea and vomiting may occur. Tachycardia may occur as hypovolemic shock develops. Numbness in the legs is not an associated finding.

Nurse Hannah is teaching a group of middle-aged men about peptic ulcers. When discussing risk factors for peptic ulcers, the nurse should mention: a. a sedentary lifestyle and smoking. b. a history of hemorrhoids and smoking. c. alcohol abuse and a history of acute renal failure. d. alcohol abuse and smoking.

Answer D. Risk factors for peptic (gastric and duodenal) ulcers include alcohol abuse, smoking, and stress. A sedentary lifestyle and a history of hemorrhoids aren't risk factors for peptic ulcers. Chronic renal failure, not acute renal failure, is associated with duodenal ulcers.

A female client with hepatitis C develops liver failure and GI hemorrhage. The blood products that would most likely bring about hemostasis in the client are: a. whole blood and albumin. b. platelets and packed red blood cells. c. fresh frozen plasma and whole blood. d. cryoprecipitate and fresh frozen plasma

Answer D. The liver is vital in the synthesis of clotting factors, so when it's diseased or dysfunctional, as in hepatitis C, bleeding occurs. Treatment consists of administering blood products that aid clotting. These include fresh frozen plasma containing fibrinogen and cryoprecipitate, which have most of the clotting factors. Although administering whole blood, albumin, and packed cells will contribute to hemostasis, those products aren't specifically used to treat hemostasis. Platelets are helpful, but the best answer is cryoprecipitate and fresh frozen plasma.

Polyethylene glycol-electrlyte solution (GoLYTELY) is prescribed for the female client scheduled for a colonoscopy. The client begins to experience diarrhea following administration of the solution. What action by the nurse is appropriate? a. Start an IV infusion b. Administer an enema c. Cancel the diagnostic test d. Explain that diarrhea is expected

Answer D. The solution GoLYTELY is a bowel evacuant used to prepare a client for a colonoscopy by cleansing the bowel. The solution is expected to cause a mild diarrhea and will clear the bowel in 4 to 5 hours. Options A, B, and C are inappropriate actions.

A nurse is preparing to care for a female client with esophageal varices who has just has a Sengstaken-Blakemore tube inserted. The nurse gathers supplies, knowing that which of the following items must be kept at the bedside at all times? a. An obturator b. Kelly clamp c. An irrigation set d. A pair of scissors

Answer D. When the client has a Sengstaken-Blakemore tube, a pair of scissors must be kept at the client's bedside at all times. The client needs to be observed for sudden respiratory distress, which occurs if the gastric balloon ruptures and the entire tube moves upward. If this occurs, the nurse immediately cuts all balloon lumens and removes the tube. An obturator and a Kelly clamp are kept at the bedside of a client with a tracheostomy. An irrigation set may be kept at the bedside, but it is not the priority item.

A nurse is caring for a child who had a laproscopic appendectomy. What interventions should the nurse document on the child's clinical record? Select all that apply. 1) Intake and Output 2) Measurement of Pain 3) Tolerance to low-residue diet 4) Frequency of dressing changes 5) Auscultation of bowel sounds

Answer: 1, 2, 5 1) Assessment and documentation of fluid balance are critical aspects of all postoperative care. 2) Laparoscopic surgery involves insufflating the abdominal cavity with air, which is painful until it is absorbed. The amount of pain should be measured and documented with either a 1-10 scale or the Wong's FACES for younger children. 3) A special diet is not indicated after this surgery. 4) After a laparoscopic appendectomy there is little drainage and no dressings. 5) Auscultating for bowel sounds and documenting their presennce or absence evaluate the child's adaptation to the intestinal trauma caused by the surgery.

The nurse is assessing an adolescent who is admitted to the hospital with appendicitis. The nurse should report which of the following to the HCP? "1) change in pain rating of 7 to 8 on a 10 point scale. 2) sudden relief of sharp pain, shifting to diffuse pain. 3)shallow breathing with normal vital signs. 4) decrease of pain rating from 8 to 6 when parents visit.

Answer: 2 Rationale: The nurse notifies the HCP if the client has sudden relief of sharp pain and on presence of more diffuse pain. this change in the pain indicates the appendix has ruprured. The diffuse pain is typically accompanied by rigid guarding of the abdomen, progressive abdominal distension, tachycardia, pallor, chills, and irritability. The slight increase pain can be expected; the decrease in pain when parents visit may be attributed to being distracted from the pain. shallow breathing is likely due to the pain and is insignificant when other vital signs are normal

A school-aged child has an emergency appendectomy. The nurse should report which of the following to the HCP if notes in the immediate postoperative period. 1. abdominal pain, 2. tugging at the incision line, 3. thirst, 4 a rigid abdomen

Answer: 4 Rationale: A tense, rigid abdomen is an early symptom of peritonitis. The other findings are expected in the immediate postoperative period.

Which of the following complications is thought to be the most common cause of appendicitis? a. A fecalith b. Internal bowel occlusion c. Bowel kinking d. Abdominal wall swelling"

Answer: A. A fecalith Rationale: A fecalith is a hard piece of stool which is stone like that commonly obstructs the lumen. Due to obstruction, inflammation and bacterial invasion can occur. Tumors or foreign bodies may also cause obstruction."

A client with complaints of right lower quadrant pain is admitted to the emergency department. Blood specimens are drawn and sent to the laboratory. Which laboratory finding should be reported to the physician immediately? "a) Hematocrit 42% b) Serum potassium 4.2 mEq/L c) Serum sodium 135 mEq/L d) White blood cell (WBC) count 22.8/mm3.

Answer: D "D) White blood cell (WBC) count 22.8/mm3 The nurse should report the elevated WBC count. This finding, which is a sign of infection, indicates that the client's appendix might have ruptured. Hematocrit of 42%, serum potassium of 4.2 mEq/L, and serum sodium of 135 mEq/L are within normal limits. Alterations in these levels don't indicate appendicitis."

Bobby, a 13 year old is being seen in the emergency room for possible appendicitis. An important nursing action to perform when preparing Bobby for an appendectomy is to:""a) administer saline enemas to cleanse the bowels b) apply heat to reduce pain c) measure abdominal girth d) continuously monitor pain

Answer: D Rationale: Pain is closely monitored in appendicitis. In most cases, pain medication is not given until prior to surgery or until the diagnosis is confirmed to be able to closely monitor the progression of the disease. A sudden change in the character of pain may indicate rupture or bowel perforation. Administering an enema or applying heat may cause perforation and abdominal girth may not change with appendicitis.

The nurse is reviewing the medication record of a female client with acute gastritis. Which medication, if noted on the client's record, would the nurse question? " a. Digoxin (Lanoxin) b. Furosemide (Lasix) c. Indomethacin (Indocin) d. Propranolol hydrochloride (Inderal)

C, Indomethacin (Indocin) is a nonsteroidal anti-inflammatory drug and can cause ulceration of the esophagus, stomach, or small intestine. Indomethacin is contraindicated in a client with gastrointestinal disorders. Furosemide (Lasix) is a loop diuretic. Digoxin is a cardiac medication. Propranolol (Inderal) is a β-adrenergic blocker. Furosemide, digoxin, and propranolol are not contraindicated in clients with gastric disorders

A client has a diagnosis of diabetes. His physician has orderedshort and long acting insulin. When administering two type of insulin, the nurse would :A. withdraw the long acting insulin into the syringe before the shortacting insulin B. withdraw the short acting insulin into the syringe before thelong acting insulin C. draw up in two separate syringes, then combine in one syringe D. withdraw long acting insulin, inject air into regular insulin, andwithdraw insulin

B

A patient diagnosed with inflammatory bowel disease experiences an obstruction in the small bowel. When assessing the patient, which of the following will the healthcare provider anticipate? Please choose from one of the following options. A. Scaphoid abdomen B. Hypovolemia C. Increased flatus D. Passage of melena

B

A patient diagnosed with ulcerative colitis is prescribed the aminosalicylate sulfasalazine. When teaching the patient about this medication, which of the following statements is a priority for the healthcare provider include? Please choose from one of the following options. A. "Be sure to limit your intake of fluids during therapy." B. "Avoid exposure to sunlight while taking this medication." C. "Call our office immediately if your urine turns an orangish color." D. "You may crush the enteric-coated tablet and mix it with applesauce."

B

A person with a diagnosis of adult Diabetes, type 2, shouldunderstand the symptoms of a hyperglycemic reaction. The nurse willknow this client understands if she says these symptoms are: A. thirst, polyuria and decreased appetite B. flushed cheeks, acetone breath, and increased thirst C. nausea, vomiting and diarrhea D. weight gain, normal breath and thirst

B

Digoxin preparations and absorbents should not be given simultaneously. As a nurse, you are aware that if these agents are given simultaneously, which of the following will occur? A. Increased absorption of digoxin B. Decreased absorption of digoxin C. Increased absorption of the absorbent D. Decreased absorption of the absorbent

B

For a client with the diagnosis of acute pancreatitis, the nursewould plan for which critical component of his care? A. testing for Homan's sign B. measuring the abdominal girth C. performing a glucometer test D. straining the urine

B

In teaching patients at risk for upper GI bleeding to prevent bleeding episodes, the nurse stresses that: a. all stools and vomit must be tested for blood b. the use of over the counter meds of any kind should be avoided c. antacids should be taken with all prescribed meds d. Cytotec should be used to protect gastric mucosa

B

Most effective means of suppressing pancreatic secreation during an episode of pancreatitis is the use of: a. antibiotics b. NPO status c. antispasmotics d. H2R blockers

B

Patient with an ulcer of the posterior portion of duodenum experiences: a. pain that occurs after not eating all day b. back pain that occurs 2-4 hrs after eating c. midepigastric pain unrelieved with antacids d. high epigastric burning relieved with food intake

B

The nurse explains to a patient with an episode of acute pancreatitis that the most effective means of relieving pain by suppressing pancreatic secretions is the use of: A. antibiotics B. NPO Status C. Antispasmodics D. H2R blockers or proton pump inhibitors

B

The nurse knows that the recommended diet for a client withAddison's disease includes: A. 1 mg. Na C. low fat, low cholesterol B. 3 gms. Na D. high potassium, high cholesterol

B

Upon examining a patient 8 hrs after formation of a colostomy the nurse would expect to find a. hypoactive, high pitched bowel sounds b. brickred, puffy stoma that oozes blood c. purplish stoma, shiny and moist d. small amt of liquid fecal drainage from stomA

B

When a client is in liver failure, which of the following behavioral changes is the most important assessment to report? A. shortness of breath B. lethargy C. fatigue D. nausea

B

Which of the findings would the nurse most likely note during anAddisonian crisis? A. serum potassium of 3 mEq/L, BP=158/72 mmHg B. serum potassium of 5.8 mEq/L, BP=62/48 mmHg C. serum sodium of 150 mEq/L, BP= 158/72 D. serum sodium of 135 mEq/L, BP=62/48

B

When evaluating a male client for complications of acute pancreatitis, the nurse would observe for: a. increased intracranial pressure. b. decreased urine output. c. bradycardia. d. hypertension.

B Acute pancreatitis can cause decreased urine output, which results from the renal failure that sometimes accompanies this condition. Intracranial pressure neither increases nor decreases in a client with pancreatitis. Tachycardia, not bradycardia, usually is associated with pulmonary or hypovolemic complications of pancreatitis. Hypotension can be caused by a hypovolemic complication, but hypertension usually isn't related to acute pancreatitis.

The most frequently used diagnostic test for persons with GERD is: a) Barium enema b) upper endoscopy c) barium swallow d) acid perfusion test

C, Persons with GERD should be referred to a primary care provider for a thorough cardiac evaluation to rule out cardiac disease. The most frequently used diagnostic test is barium swallow. Upper endoscopy is the best method to assess mucosal injury. Acid perfusion tests usually are not necessary, and require the placement of an esophageal probe above the esophageal sphincter to collect esophageal contents

What is the most serious complication that can result from frequent, untreated gastric reflux? A. Dental enamel erosion B. Barrett's esophagus C. Leukoplakia D. Aspiration pneumonia

B Rationale A complication of frequent untreated gastric reflux is Barrett's esophagus or esophageal metaplasia, which can lead to esophageal adenocarcinoma. Enamel erosion results from frequent induced vomiting (e.g., bulimia) with gastric acid reflux, although the reflux does not ordinarily reach the mouth. Leukoplakia is the white patch in the oral cavity from constant irritation, such as from a pipe. Aspiration of the reflux is not a common problem in healthy individuals; it can be a problem with patients who are on mechanical ventilators. Reference: 972

What finding is common when assessing a patient with a hiatal hernia? A. Pulsating abdominal mass B. Heartburn when lying supine C. Pernicious anemia D. Hypoactive bowel sounds

B Rationale A hiatal hernia is herniation of a portion of the stomach into the esophagus through an opening or hiatus in the diaphragm. Many of the symptoms and treatments are similar to those of gastroesophageal reflux disease (GERD). A pulsating abdominal mass is a classic finding for abdominal aortic aneurysm. Pernicious anemia is related to a lack of the intrinsic factor secreted in the stomach and is a complication of gastric surgery, not a hiatal hernia. Bowel sounds reflect peristalsis of the large intestine, and an alteration is not a common finding in a hiatal hernia. Reference: 975

Results of a patient's recent endoscopy indicate the presence of peptic ulcer disease (PUD). What teaching should you provide to the patient based on this diagnosis? A. "You'll need to drink at least two to three glasses of milk daily." B. "It would be beneficial for you to eliminate drinking alcohol." C. "Many people find that a minced or pureed diet eases their symptoms of PUD." D. "Your medications should allow you to maintain your present diet while minimizing symptoms."

B Rationale Although there is no specific recommended dietary modification for PUD, most patients find it necessary to make some sort of dietary modifications to minimize symptoms. Milk may exacerbate PUD, and alcohol is best avoided because it can delay healing. Reference: 998

What is a risk factor for a bleeding peptic ulcer? A. Daily vitamin C tablets B. Chronic use of nonsteroidal antiinflammatory drugs (NSAIDs) C. Heavy physical exercise when not properly conditioned D. Cholecystitis

B Rationale Approximately 25% of persons on chronic nonsteroidal antiinflammatory drugs (NSAIDs) will develop ulcer disease, and of these, 2% to 4 % will bleed because the drugs irritate and disrupt the gastric mucosal barrier. The other options do not cause gastric bleeding. Reference: 981

Five hours after a family reunion picnic, three members are admitted to an emergency department with nausea, vomiting, and abdominal cramping. Which question is a priority to ask on the admission assessment? A. "Where was the reunion?" B. "What food was served?" C. "Is there a history of PUD?" D. "Was anyone sick beforehand?"

B Rationale Because multiple individuals from the same event have the same symptoms at the same time, the most likely explanation is foodborne illness. This likely occurs with uncooked food that is left out for more than 2 hours at room temperature, such as at a picnic. It would take priority in determining the cause and treatment over the other options. Reference: 1001

6. A daughter of an 85-year-old elderly woman calls you to tell you that her mother has been nauseated all day and has vomited twice. Before you hang up and telephone the health care provider to communicate your assessment data, how should you instruct the daughter? A. Administer antispasmodic drugs, and observe skin turgor. B. Give her mother sips of water and elevate the head of her bed to prevent aspiration. C. Offer her mother a high-protein liquid supplement to drink to maintain her nutritional needs. D. Offer her mother large quantities of Gatorade to drink because elderly people are at risk for sodium depletion.

B Rationale Excessive replacement of fluid and electrolytes may result in adverse consequences for the older person who has heart failure or renal disease. The older adult with a decreased level of consciousness may be at high risk for aspiration of vomitus. The elderly are particularly susceptible to the central nervous system (CNS) side effects of antiemetic drugs; these drugs may produce confusion. Dosages should be reduced and efficacy closely evaluated. Older patients are more likely to have cardiac or renal insufficiency that places them at greater risk for life-threatening fluid and electrolyte imbalances. High-protein drinks or high-sodium liquids may be contraindicated. Reference: 965

A 19-year-old male is on chemotherapy and has open sores in the mouth. What intervention should you encourage? A. Cold foods B. Providing oral care every 2 to 4 hours C. Mixing water with the alcohol-based mouthwash D. Having him gargle with viscous lidocaine

B Rationale Frequent oral care rids mucosal surfaces of debris, which is an excellent medium for bacterial and fungal growth. Tepid foods are recommended instead of those with temperature extremes. Alcohol-based mouthwash increases pain and tissue trauma because it is drying. Viscous lidocaine can depress the gag reflex and increase the risk of aspiration. Reference: 968

Which action will help prevent injury to the suture line after a surgical repair of a hiatal hernia? A. Maintain the patient in the Sims position. B. Ensure proper anchoring of the nasogastric tube. C. Avoid deep breathing and coughing. D. Keep suction at less than 80 mm Hg.

B Rationale Manipulating the nasogastric tube can cause direct irritation to the suture line. Sims' position is a semiprone position with the patient on the left side, with the right knee and thigh drawn up; it is used for enema administration. The patient should be maintained in the semi-Fowler or Fowler position to prevent reflux. Deep breathing and coughing is required to prevent atelectasis and should be done every 2 hours. Low suction (less than 80 mm Hg) does not adequately remove gastric juices, which can result in overdistention. Reference: 976, 998

Gastroesophageal reflux disease (GERD) weakens the lower esophageal spinchter, predisposing older persons to risk for impaired swallowing. In managing the symptoms associated with GERD, the nurse should assign the highest priority to which of the following interventions? A. Decrease daily intake of vegetables and water and ambulate frequently, B. drink coffee diluted with milk at each meal and remain in an upright position for thirty minutes, C. Eat small, frequent meals and remain in an upright position for thirty minutes D. Avoid OTC drugs that have antacids in them

C, Rationale: Small, frequent feedings requires less release of hydrochloric acid. Remaining in an upright position for 30 minutes after meals prevents reflux into the esophagus

The patient who is admitted with a diagnosis of diverticulitis and a history of irritable bowel disease and GERD has received a dose of Mylanta (30 mL PO). You evaluate the effectiveness of the drug by questioning the patient about whether which symptom has been resolved? A. Diarrhea B. Heartburn C. Constipation D. Lower abdominal pain

B Rationale Mylanta is an antacid that contains aluminum and magnesium. It is indicated for the relief of gastrointestinal discomfort such as heartburn, which is associated with GERD. Reference: 973

Several patients are seen at an urgent care center with symptoms of nausea, vomiting, and diarrhea that began 2 hours ago while attending a potluck dinner at a large family reunion. You question the patients specifically about foods they ingested containing A. beef. B. meat and milk. C. poultry and eggs. D. home-preserved vegetables.

B Rationale Staphylococcus aureus toxins produce onset of symptoms (vomiting, nausea, abdominal cramping, and diarrhea) within 30 minutes and for up to 7 hours. Meat, bakery products, cream fillings, salad dressings, and milk are the usual sources of contamination from the skin and respiratory tract of food handlers. Reference: 1002

What should you teach a patient taking sucralfate (Carafate)? A. Watch for deficiency of fat-soluble vitamins. B. Will interfere with the absorption of many medications. C. Administer 1 hour after meals. D. Take with antacids for maximum effectiveness.

B Rationale Sucralfate is used for the short-term treatment of peptic ulcers because it forms an ulcer-adherent complex that covers the ulcer. It is known for binding with many drugs, including digoxin and warfarin. It does not affect fat-soluble vitamin absorption. It is most effective at a low pH and should be given at least 30 minutes before or after an antacid and meals. Reference: 992

The patient is being treated for active upper gastrointestinal bleeding. What is the best indicator that the fluid replacement is adequate? A. Intake matches output. B. Urine output is at least 0.5 mL/kg per hour. C. Urine has a light amber color. D. Patient denies dizziness or thirst.

B Rationale The best indication of adequate renal perfusion is a urinary output of at least 0.5 mL/kg per hour. Reference: 984

Which of the following tests should be administered to a client suspected of having diverticulosis? A. Abdominal ultrasound B. Barium enema C. Barium swallow D. Gastroscopy

B A barium enema will cause diverticula to fill with barium and be easily seen on x-ray. An abdominal US can tell more about structures, such as the gallbladder, liver, and spleen, than the intestine. A barium swallow and gastroscopy view upper GI structures.

Which of the following nursing diagnoses is appropriate for a patient receiving famotidine (Pepcid)? A. Increased risk for infection due to immunosuppression B. Potential risk for bleeding related to thrombocytopenia C. Alteration in urinary elimination related to retention D. Alteration in tissue perfusion related to hypertension

B A serious side effect of famotidine is thrombocytopenia, which is manifested by a decrease in platelet count and an increased risk of bleeding.

Following bowel resection, a patient has a nasogastric tube to suction, but complains of nausea and abdominal distention. The nurse irrigates the tube prn as ordered, but the irrigating fluid does not return. Which of the following should be the priority action by the nurse? A) Notify the physician. B) Auscultate for bowel sounds. C) Reposition the tube and check for placement. D) Remove the tube and replace it with a new one.

C) Reposition the tube and check for placement The tube may be resting against the stomach wall. The first action by the nurse, since this intestinal surgery (not gastric surgery), is to reposition the tube and check it again for placement.

The nurse instructs the nursing assistant on how to provide oral hygiene for a client who cannot perform this task for himself. Which of the following techniques should the nurse tell the assistant to incorporate into the client's daily care? A. Assess the oral cavity each time mouth care is given and record observations B. Use a soft toothbrush to brush the client's teeth after each meal C. Swab the client's tongue, gums, and lips with a soft foam applicator every 2 hours D. Rinse the client's mouth with mouthwash several times a day

B A soft toothbrush should be used to brush the client's teeth after each meal and more often as needed. Mechanical cleaning is necessary to maintain oral health, simulate gingiva, and remove plaque. Assessing the oral cavity and recording observations is the responsibility of the nurse, not the nursing assistant. Swabbing with a safe foam applicator does not provide enough friction to clean the mouth. Mouthwash can be a drying irritant and is not recommended for frequent use.

The nurse is assessing a client for constipation. Which of the following is the first review that the nurse should conduct in order to identify the cause of constipation? Choose the correct option. a) Review the client's current medications b) Review the client's alcohol consumption c) Review the client's usual pattern of elimination d) Review the client's activity levels

C) Review the client's usual pattern of elimination Constipation has many possible reasons; assessing the client's usual pattern of elimination is the first step in identifying the cause.

Which of the following symptoms would a client in the early stages of peritonitis exhibit? A. Abdominal distention B. Abdominal pain and rigidity C. Hyperactive bowel sounds D. Right upper quadrant pain

B Abdominal pain causing rigidity of the abdominal muscles is characteristic of peritonitis. Abdominal distention may occur as a late sign but not early on. Bowel sounds may be normal or decreased but not increased. Right upper quadrant pain is characteristic of cholecystitis or hepatitis.

During the first few days of recovery from ostomy surgery for ulcerative colitis, which of the following aspects should be the first priority of client care? A. Body image B. Ostomy care C. Sexual concerns D. Skin care

B Although all of these are concerns the nurse should address, being able to safely manage the ostomy is crucial for the client before discharge.

The nurse would monitor for which of the following adverse reactions to aluminum-containing antacids such as aluminum hydroxide (Amphojel)? A. Diarrhea B. Constipation C. GI upset D. Fluid retention

B Aluminum- and calcium-containing antacids cause constipation, magnesium-containing antacids cause diarrhea, and sodium-containing antacids cause sodium and fluid retention.

Which of the following tests is most commonly used to diagnose cholecystitis? A. Abdominal CT scan B. Abdominal ultrasound C. Barium swallow D. Endoscopy

B An abdominal ultrasound can show if the gallbladder is enlarged, if gallstones are present, if the gallbladder wall is thickened, or if distention of the gallbladder lumen is present. An abdominal CT scan can be used to diagnose cholecystitis, but it usually isn't necessary. A barium swallow looks at the stomach and the duodenum. Endoscopy looks at the esophagus, stomach, and duodenum.

If a gastric acid perforates, which of the following actions should not be included in the immediate management of the client? A. Blood replacement B. Antacid administration C. Nasogastric tube suction D. Fluid and electrolyte replacement

B Antacids aren't helpful in perforation. The client should be treated with antibiotics as well as fluid, electrolyte, and blood replacement. NG tube suction should also be performed to prevent further spillage of stomach contents into the peritoneal cavity.

For Jayvin who is taking antacids, which instruction would be included in the teaching plan? A. "Take the antacids with 8 oz of water." B. "Avoid taking other medications within 2 hours of this one." C. "Continue taking antacids even when pain subsides." D. "Weigh yourself daily when taking this medication.

B Antacids neutralize gastric acid and decrease the absorption of other medications. The client should be instructed to avoid taking other medications within 2 hours of the antacid. Water, which dilutes the antacid, should not be taken with antacid. A histamine receptor antagonist should be taken even when pain subsides. Daily weights are indicated if the client is taking a diuretic, not an antacid.

A client with a peptic ulcer reports epigastric pain that frequently awakens her at night, a feeling of fullness in the abdomen, and a feeling of anxiety about her health. Based on this information, which nursing diagnosis would be most appropriate? A. Imbalanced Nutrition: Less than Body Requirements related to anorexia B. Disturbed Sleep Pattern related to epigastric pain C. Ineffective Coping related to exacerbation of duodenal ulcer D. Activity Intolerance related to abdominal pain

B Based on the data provided, the most appropriate nursing diagnosis would be Disturbed Sleep pattern. A client with a duodenal ulcer commonly awakens at night with pain. The client's feelings of anxiety do not necessarily indicate that she is coping ineffectively.

A nurse is teaching a client with malabsorption syndrome about the disorder and its treatment. The client asks which part of the GI tract absorbs food. The nurse tells the client that products of digestion are absorbed mainly in the: a) rectum. b) stomach. c) small intestine. d) large intestine.

C) Small intestine The small intestine absorbs products of digestion, completes food digestion, and secretes hormones that help control the secretion of bile, pancreatic juice, and intestinal secretions. The stomach stores, mixes, and liquefies the food bolus into chyme and controls food passage into the duodenum; it doesn't absorb products of digestion. Although the large intestine completes the absorption of water, chloride, and sodium, it plays no part in absorbing food. The rectum is the portion of the large intestine that forms and expels feces from the body; its functions don't include absorption.

A nurse is monitoring a client admitted to the hospital with a diagnosis of appendicitis. The client is scheduled for surgery in 2 hours. The client begins to complain of increased abdominal pain and begins to vomit. On assessment the nurse notes that the abdomen is distended and the bowel sounds are diminished. Which of the following is the most appropriate nursing intervention? A. Administer dilaudid B. Notify the physician C. Call and ask the operating room team to perform the surgery as soon as possible D. Reposition the client and apply a heating pad on a warm setting to the client's abdomen

B Based on the signs and symptoms presented in the question, the nurse should suspect peritonitis and should notify the physician. Administering pain medication is not an appropriate intervention. Heat should never be applied to the abdomen of a client with suspected appendicitis. Scheduling surgical time is not within the scope of nursing practice, although the physician probably would perform the surgery earlier than the prescheduled time.

A client with peptic ulcer disease tells the nurse that he has black stools, which he has not reported to his physician. Based on this information, which nursing diagnosis would be appropriate for this client? A. Ineffective coping related to fear of diagnosis of chronic illness B. Deficient knowledge related to unfamiliarity with significant signs and symptoms C. Constipation related to decreased gastric motility D. Imbalanced nutrition: Less than body requirements due to gastric bleeding

B Black, tarry stools are an important warning sign of bleeding in peptic ulcer disease. Digested blood in the stomach causes it to be black. The odor of the stool is very stinky. Clients with peptic ulcer disease should be instructed to report the incidence of black stools promptly to their physician.

he nurse is caring for a male client with a diagnosis of chronic gastritis. The nurse monitors the client knowing that this client is at risk for which vitamin deficiency? A. Vitamin A B. Vitamin B12 C. Vitamin C D. Vitamin E

B Chronic gastritis causes deterioration and atrophy of the lining of the stomach, leading to the loss of the function of the parietal cells. The source of the intrinsic factor is lost, which results in the inability to absorb vitamin B12. This leads to the development of pernicious anemia. The client is not at risk for vitamin A, C, or E deficiency.

For Rico who has chronic pancreatitis, which nursing intervention would be most helpful? A. Allowing liberalized fluid intake B. Counseling to stop alcohol consumption C. Encouraging daily exercise D. Modifying dietary protein

B Chronic pancreatitis typically results from repeated episodes of acute pancreatitis. More than half of chronic pancreatitis cases are associated with alcoholism. Counseling to stop alcohol consumption would be the most helpful for the client. Dietary protein modification is not necessary for chronic pancreatitis. Daily exercise and liberalizing fluid intake would be helpful but not the most beneficial intervention.

The nurse is reviewing the record of a client with Crohn's disease. Which of the following stool characteristics would the nurse expect to note documented on the client's record? A. Chronic constipation B. Diarrhea C. Constipation alternating with diarrhea D. Stool constantly oozing from the rectum

B Crohn's disease is characterized by nonbloody diarrhea of usually not more than four to five stools daily. Over time, the diarrhea episodes increase in frequency, duration and severity. The other option are not associated with diarrhea.

Which goal of the client's care should take priority during the first days of hospitalization for an exacerbation of ulcerative colitis? A. Promoting self-care and independence B. Managing diarrhea C. Maintaining adequate nutrition D. Promoting rest and comfort

B Diarrhea is the primary symptom in an exacerbation of ulcerative colitis, and decreasing the frequency of stools is the first goal of treatment. The other goals are ongoing and will be best achieved by halting the exacerbation. The client may receive antidiarrheal medications, antispasmodic agents, bulk hydrophilic agents, or anti-inflammatory drugs.

Which of the following definitions best describes diverticulosis? A. An inflamed outpouching of the intestine B. A noninflamed outpouching of the intestine C. The partial impairment of the forward flow of intestinal contents D. An abnormal protrusion of an organ through the structure that usually holds it.

B Diverticulosis involves a noninflamed outpouching of the intestine. Diverticulitis involves an inflamed outpouching. The partial impairment of forward flow of the intestine is an obstruction; abnormal protrusion of an organ is a hernia.

The nurse would assess the client experiencing an acute episode of cholecysitis for pain that is located in the right A. Upper quadrant and radiates to the left scapula and shoulder B. Upper quadrant and radiates to the right scapula and shoulder C. Lower quadrant and radiates to the umbilicus D. Lower quadrant and radiates to the back

B During an acute "gallbladder attack," the client may complain of severe right upper quadrant pain that radiates to the right scapula and shoulder. This is governed by the pattern on dermatomes in the body.

An enema is prescribed for a client with suspected appendicitis. Which of the following actions should the nurse take? A. Prepare 750 ml of irrigating solution warmed to 100*F B. Question the physician about the order C. Provide privacy and explain the procedure to the client D. Assist the client to left lateral Sim's position

B Enemas are contraindicated in an acute abdominal condition of unknown origin as well as after recent colon or rectal surgery or myocardial infarction. The other answers are correct only when enema administration is appropriate.

The nurse is assessing a client 24 hours following a cholecystectomy. The nurse notes that the T-tube has drained 750ml of green-brown drainage. Which nursing intervention is most appropriate? A. Notify the physician B. Document the findings C. Irrigate the T-tube D. Clamp the T-tube

B Following cholecystectomy, drainage from the T-tube is initially bloody and then turns to green-brown. The drainage is measured as output. The amount of expected drainage will range from 500 to 1000 ml per day. The nurse would document the output.

When planning home care for a client with hepatitis A, which preventive measure should be emphasized to protect the client's family? A. Keeping the client in complete isolation B. Using good sanitation with dishes and shared bathrooms C. Avoiding contact with blood-soiled clothing or dressing D. Forbidding the sharing of needles or syringes

B Hepatitis A is transmitted through the fecal oral route or from contaminated water or food. Measures to protect the family include good handwashing, personal hygiene and sanitation, and use of standard precautions. Complete isolation is not required. Avoiding contact with blood-soiled clothing or dressings or avoiding the sharing of needles or syringes are precautions needed to prevent transmission of hepatitis B.

The nurse is performing a colostomy irrigation on a male client. During the irrigation, the client begins to complain of abdominal cramps. What is the appropriate nursing action? A. Notify the physician B. Stop the irrigation temporarily C. Increase the height of the irrigation D. Medicate for pain and resume the irrigation

B If cramping occurs during a colostomy irrigation, the irrigation flow is stopped temporarily and the client is allowed to rest. Cramping may occur from an infusion that is too rapid or is causing too much pressure. The physician does not need to be notified. Increasing the height of the irrigation will cause further discomfort. Medicating the client for pain is not the appropriate action in this situation.

Which of the following symptoms is associated with ulcerative colitis? A. Dumping syndrome B. Rectal bleeding C. Soft stools D. Fistulas

B In ulcerative colitis, rectal bleeding is the predominant symptom. Soft stools are more commonly associated with Crohn's disease, in which malabsorption is more of a problem. Dumping syndrome occurs after gastric surgeries. Fistulas are associated with Crohn's disease.

The nurse is reviewing the medication record of a client with acute gastritis. Which medication, if noted on the client's record, would the nurse question? A. Digoxin (Lanoxin) B. Indomethacin (Indocin) C. Furosemide (Lasix) D. Propranolol hydrochloride (Inderal)

B Indomethacin (Indocin) is a NSAID and can cause ulceration of the esophagus, stomach, duodenum, or small intestine. Indomethacin is contraindicated in a client with GI disorders.

A client is suspected of having hepatitis. Which diagnostic test result will assist in confirming this diagnosis? A. Elevated hemoglobin level B. Elevated serum bilirubin level C. Elevated blood urea nitrogen level D. Decreased erythrocyte sedimentation rate

B Laboratory indicators of hepatitis include elevated liver enzyme levels, elevated serum bilirubin levels, elevated erythrocyte sedimentation rates, and leukopenia. An elevated blood urea nitrogen level may indicate renal dysfunction. A hemoglobin level is unrelated to this diagnosis.

When teaching a client about pancreatic function, the nurse understands that pancreatic lipase performs which function? A. Transports fatty acids into the brush border B. Breaks down fat into fatty acids and glycerol C. Triggers cholecystokinin to contract the gallbladder D. Breaks down protein into dipeptides and amino acids

B Lipase hydrolyses or breaks down fat into fatty acids and glycerol. Lipase is not involved with the transport of fatty acids into the brush border. Fat itself triggers cholecystokinin release. Protein breakdown into dipeptides and amino acids is the function of trypsin, not lipase.

A female client complains of gnawing epigastric pain for a few hours after meals. At times, when the pain is severe, vomiting occurs. Specific tests are indicated to rule out: A. Cancer of the stomach B. Peptic ulcer disease C. Chronic gastritis D. Pylorospasm

B Peptic ulcer disease is characteristically gnawing epigastric pain that may radiate to the back. Vomiting usually reflects pyloric spasm from muscular spasm or obstruction. Cancer (1) would not evidence pain or vomiting unless the pylorus was obstructed.

Which of the following best describes the method of action of medications, such as ranitidine (Zantac), which are used in the treatment of peptic ulcer disease? A. Neutralize acid B. Reduce acid secretions C. Stimulate gastrin release D. Protect the mucosal barrier

B Ranitidine is a histamine-2 receptor antagonist that reduces acid secretion by inhibiting gastrin secretion.

Which of the following factors is believed to cause ulcerative colitis? A. Acidic diet B. Altered immunity C. Chronic constipation D. Emotional stress

B Several theories exist regarding the cause of ulcerative colitis. One suggests altered immunity as the cause based on the extraintestinal characteristics of the disease, such as peripheral arthritis and cholangitis. Diet and constipation have no effect on the development of ulcerative colitis. Emotional stress can exacerbate the attacks but isn't believed to be the primary cause.

After a right hemicolectomy for treatment of colon cancer, a 57-year old client is reluctant to turn while on bed rest. Which action by the nurse would be appropriate? A. Asking a coworker to help turn the client B. Explaining to the client why turning is important C. Allowing the client to turn when he's ready to do so D. Telling the client that the physician's order states he must turn every 2 hours

B The appropriate action is to explain the importance of turning to avoid postoperative complications. Asking a coworker to help turn the client would infringe on his rights. Allowing him to turn when he's ready would increase his risk for postoperative complications. Telling him he must turn because of the physician's orders would put him on the defensive and exclude him from participating in care decision.

Which of the following factors should be the main focus of nursing management for a client hospitalized for cholecystitis? A. Administration of antibiotics B. Assessment for complications C. Preparation for lithotripsy D. Preparation for surgery

B The client with acute cholecystitis should first be monitored for perforation, fever, abscess, fistula, and sepsis. After assessment, antibiotics will be administered to reduce the infection. Lithotripsy is used only for a small percentage of clients. Surgery is usually done after the acute infection has subsided.

For a client with hepatic cirrhosis who has altered clotting mechanisms, which intervention would be most important? A. Allowing complete independence of mobility B. Applying pressure to injection sites C. Administering antibiotics as prescribed D. Increasing nutritional intake

B The client with cirrhosis who has altered clotting is at high risk for hemorrhage. Prolonged application of pressure to injection or bleeding sites is important. Complete independence may increase the client's potential for injury, because an unsupervised client may injure himself and bleed excessively. Antibiotics and good nutrition are important to promote liver regeneration. However, they are not most important for a client at high risk for hemorrhage.

A client with irritable bowel syndrome is being prepared for discharge. Which of the following meal plans should the nurse give the client? A. Low fiber, low-fat B. High fiber, low-fat C. Low fiber, high-fat D. High-fiber, high-fat

B The client with irritable bowel syndrome needs to be on a diet that contains at least 25 grams of fiber per day. Fatty foods are to be avoided because they may precipitate symptoms.

Which of the following symptoms is a client with colon cancer most likely to exhibit? A. A change in appetite B. A change in bowel habit C. An increase in body weight D. An increase in body temperature

B The most common complaint of the client with colon cancer is a change in bowel habits. The client may have anorexia, secondary abdominal distention, or weight loss. Fever isn't associated with colon cancer.

The nurse is providing discharge instructions to a client following gastrectomy. Which measure will the nurse instruct the client to follow to assist in preventing dumping syndrome? A. Eat high-carbohydrate foods B. Limit the fluids taken with meals C. Ambulate following a meal D. Sit in a high-Fowlers position during meals

B The nurse should instruct the client to decrease the amount of fluid taken at meals and to avoid high carbohydrate foods including fluids such as fruit nectars; to assume a low-Fowler's position during meals; to lie down for 30 minutes after eating to delay gastric emptying; and to take antispasmodics as prescribed.

Spironolactone (Aldactone) is prescribed for a client with chronic cirrhosis and ascites. The nurse should monitor the client for which of the following medication-related side effects? A. Jaundice B. Hyperkalemia C. Tachycardia D. Constipation

B This is a potassium-sparing diuretic so clients should be monitored closely for hyperkalemia. Diarrhea, dizziness, and headaches are other more common side effects. Tachycardia, jaundice, and constipation are not expected side effects of spironolactone (Aldactone).

Which of the following will the nurse include in the care plan for a client hospitalized with viral hepatitis? A. Increase fluid intake to 3000 ml per day B. Adequate bed rest C. Bland diet D. Administer antibiotics as ordered

B Treatment of hepatitis consists of bed rest during the acute phase to reduce metabolic demands on the liver, thus increasing blood supply and cell regeneration. Forcing fluids, antibiotics, and bland diets are not part of the treatment plan for viral hepatitis.

The client with a colostomy has an order for irrigation of the colostomy. The nurse used which solution for irrigation? A. Distilled water B. Tap water C. Sterile water D. Lactated Ringer's

B Warm tap water or saline solution is used to irrigate a colostomy. If the tap water is not suitable for drinking, then bottled water should be used.

The nurse evaluates the client's stoma during the initial post-op period. Which of the following observations should be reported immediately to the physician? A. The stoma is slightly edematous B. The stoma is dark red to purple C. The stoma oozes a small amount of blood D. The stoma does not expel stool

B A dark red to purple stoma indicates inadequate blood supply. Mild edema and slight oozing of blood are normal in the early post-op period. The colostomy would typically not begin functioning until 2-4 days after surgery.

The nurse is caring for a client with chronic gastritis. The nurse monitors the client, knowing that this client is at risk for which of the following vitamin deficiencies? A. Vitamin A B. Vitamin B12 C. Vitamin C D. Vitamin E

B Chronic gastritis causes deterioration and atrophy of the lining of the stomach, leading to the loss of the functioning parietal cells. The source of the intrinsic factor is lost, which results in the inability to absorb vitamin B12. This leads to the development of pernicious anemia.

The results of a patient's recent endoscopy indicate the presence of peptic ulcer disease (PUD). Which of the following teaching points should the nurse provide to the patient in light of his new diagnosis? A. You'll need to drink at least two to three glasses of milk daily. B."It would likely be beneficial for you to eliminate drinking alcohol." C. Many people find that a minced or pureed diet eases their symptoms of PUD. D. Your medications should allow you to maintain your present diet while minimizing symptoms

B Although there is no specific recommended dietary modification for PUD, most patients find it necessary to make some sort of dietary modifications to minimize symptoms. Milk may exacerbate PUD and alcohol is best avoided because it can delay healing

A client receives a local anesthetic to suppress the gag reflex for a diagnostic procedure of the upper GI tract. Which of the following nursing interventions is advised for this patient? a. The client should be monitored for any breathing related disorder or discomforts b. The client should not be given any food and fluids until the gag reflex returns, c. The client should be monitored for cramping or abdominal distention, d. The client's fluid output should be measured for at least 24 hours after the procedure

B For a client receiving a local anesthetic that suppresses the gag reflex, the nurse is advised to withhold food and fluids until the reflex returns

Caffeinated beverages and smoking are risk factors to assess for in the development of what condition? A. Duodenal ulcers B. Peptic ulcers C. Helicobacter pylori D. Esophageal reflux

B PUD risk factors include family history, blood group O, smoking tobacco, and beverages containing caffeine

The results of a patient's recent endoscopy indicate the presence of peptic ulcer disease (PUD). Which of the following teaching points should the nurse provide to the patient in light of his new diagnosis? A) "You'll need to drink at least two to three glasses of milk daily." B) "It would likely be beneficial for you to eliminate drinking alcohol." C) "Many people find that a minced or pureed diet eases their symptoms of PUD." D) "Your medications should allow you to maintain your present diet while minimizing symptoms."

B) "It would likely be beneficial for you to eliminate drinking alcohol" Although there is no specific recommended dietary modification for PUD, most patients find it necessary to make some sort of dietary modifications to minimize symptoms. Milk may exacerbate PUD and alcohol is best avoided because it can delay healing.

The results of a patients recent endoscopy indicate the presence of peptic ulcer disease (PUD). Which of the following teaching points should the nurse provide to the pt in light of his new diagnosis? A) "You'll need to drink at least two to three glasses of milk daily." B) "It would likely be beneficial for you to eliminate drinking alcohol." C) "Many people find that a minced or pureed diet eases their sxs of PUD." D) "Your medications should allow you to maintain your present diet while minimizing symptoms."

B) "It would likely be beneficial for you to eliminate drinking alcohol." Although there is no specific recommended dietary modification for PUD, most patients find it necessary to make some sort of dietary modifications to minimize symptoms. Milk may exacerbate PUD and alcohol is best avoided because it can delay healing.

A patient who is administering a bisacodyl (Dulcolax) suppository asks the nurse how long it will take to work. The nurse replies that the patient will probably need to use the bedpan or commode within which of the following time frames after administration? A) 2-5 Minutes B) 15-60 Minutes C) 2-4 Hours D) 6-8 Hours

B) 15-60 minutes Bisacodyl suppositories usually are effective within 15 to 60 minutes of administration, so the nurse should plan accordingly to assist the patient to use the bedpan or commode.

A 61-year-old patient with suspected bowel obstruction has had a nasogastric tube inserted at 4:00 am. The nurse shares in the morning report that the day shift staff should check the tube for patency at which of the following times? A) 7:00 am, 10:00 am, and 1:00 pm B) 8:00 am and 12:00 pm C) 9:00 am and 3:00 pm D) 9:00 am, 12:00 pm, and 3:00 pm

B) 8:00 am and 12:00 pm A nasogastric tube should be checked for patency routinely at 4-hour intervals. Thus if the tube were inserted at 4:00 am, it would be due to be checked at 8:00 am and 12:00 pm.

The nurse is planning care for a 68-year-old patient with an abdominal mass and suspected bowel obstruction. Which factor in the patient's history increases the patient's risk for colorectal cancer? A) Osteoarthritis B) History of colorectal polyps C) History of lactose intolerance D) Use of herbs as dietary supplements

B) A history of colorectal polyps places this patient at risk for colorectal cancer. This tissue can degenerate over time and become malignant. Osteoarthritis, lactose intolerance, and the use of herbs do not pose additional risk to the patient.

A 61-year-old patient with suspected bowel obstruction had a nasogastric tube inserted at 4:00 AM. The nurse shares in the morning report that the day shift staff should check the tube for patency at what times? A) 7:00 AM, 10:00 AM, and 1:00 PM B) 8:00 AM, 12:00 PM, and 4:00 PM C) 9:00 AM and 3:00 PM D) 9:00 AM, 12:00 PM, and 3:00 PM

B) A nasogastric tube should be checked for patency routinely at 4-hour intervals. Thus if the tube were inserted at 4:00 AM, it would be due to be checked at 8:00 AM, 12:00 PM, and 4:00 PM.

A client is evaluated for severe pain in the right upper abdominal quadrant, which is accompanied by nausea and vomiting. The physician diagnoses acute cholecystitis and cholelithiasis. For this client, which nursing diagnosis takes top priority? a) Anxiety related to unknown outcome of hospitalization b) Acute pain related to biliary spasms c) Imbalanced nutrition: Less than body requirements related to biliary inflammation d) Deficient knowledge related to prevention of disease recurrence

B) Acute pain related to biliary spams The chief symptom of cholecystitis is abdominal pain or biliary colic. Typically, the pain is so severe that the client is restless and changes positions frequently to find relief. Therefore, the nursing diagnosis of Acute pain related to biliary spasms takes highest priority. Until the acute pain is relieved, the client can't learn about prevention, may continue to experience anxiety, and can't address nutritional concerns.

A client with a history of alcohol abuse comes to the emergency department and complains of abdominal pain. Laboratory studies help confirm a diagnosis of acute pancreatitis. The client's vital signs are stable, but the client's pain is worsening and radiating to his back. Which intervention takes priority for this client? a) Providing mouth care b) Administering morphine I.V. as ordered c) Placing the client in a semi-Fowler's position d) Maintaining nothing-by-mouth (NPO) status

B) Administering morphine IV as ordered The nurse should address the client's pain issues first by administering morphine I.V. as ordered. Placing the client in a Semi-Fowler's position, maintaining NPO status, and providing mouth care don't take priority over addressing the client's pain issues.

A colectomy is scheduled for a 68-year-old woman with an abdominal mass, possible bowel obstruction, and a history of rectal polyps. The nurse should plan to include which of the following prescribed measures in the preoperative preparation of this patient? A) Instruction on irrigating a colostomy B) Administration of a cleansing enema C) A high-fiber diet the day before surgery D) Administration of IV antibiotics for bowel preparation

B) Administration of a cleansing enema Preoperative preparation for bowel surgery typically includes bowel cleansing with antibiotics, such as oral neomycin and cleansing enemas, including Fleet enemas.

A nonresponsive client has a nasogastric tube to low intermittent suction due to gastrointestinal bleeding. It is most important for the nurse to a) Change the nasal tape every 2 to 3 days. b) Auscultate lung sounds every 4 hours. c) Inspect the nose daily for skin irritation. d) Apply water-based lubricant to the nares daily.

B) Auscultate lung sounds every 4 hours Pulmonary complications may occur as a result of nasogastric intubation. It is a high priority according to Maslow's hierarchy of needs and takes a higher priority over assessing the nose, changing nasal tape, or applying a water-based lubricant.

A client with a peptic ulcer is about to begin a therapeutic regimen that includes a bland diet, antacids, and famotidine (Pepcid). Before the client is discharged, the nurse should provide which instruction? a) "Eat three balanced meals every day." b) "Avoid aspirin and products that contain aspirin." c) "Stop taking the drugs when your symptoms subside." d) "Increase your intake of fluids containing caffeine."

B) Avoid aspirin and products that contain aspirin The nurse should instruct the client to avoid aspirin because it's a gastric irritant and should not be taken by clients with peptic ulcer to prevent further erosion of the stomach lining. The client should eat small, frequent meals rather than three large ones. Antacids and ranitidine prevent acid accumulation in the stomach; they should be taken even after symptoms subside. Caffeine should be avoided because it increases acid production in the stomach.

To prevent gastroesophageal reflux in a client with hiatal hernia, the nurse should provide which discharge instruction? a) "Lie down after meals to promote digestion." b) "Avoid coffee and alcoholic beverages." c) "Limit fluid intake with meals." d) "Take antacids with meals."

B) Avoid coffee and alcoholic beverages To prevent reflux of stomach acid into the esophagus, the nurse should advise the client to avoid foods and beverages that increase stomach acid, such as coffee and alcohol. The nurse also should teach the client to avoid lying down after meals, which can aggravate reflux, and to take antacids after eating. The client need not limit fluid intake with meals as long as the fluids aren't gastric irritants.

A patient scheduled to undergo an abdominal ultrasonography is advised to do which of the following? a) Do not undertake any strenuous exercise for 24 hours before the test b) Restrict eating of solid food for 6 to 8 hours before the test. c) Avoid exposure to sunlight for at least 6 to 8 hours before the test d) Do not consume anything sweet for 24 hours before the test

B) Avoid eating of solid food for 6 to 8 hours before the test. For a patient who is scheduled to undergo an abdominal ultrasonography, the patient should restrict herself from solid food for 6 to 8 hours to avoid having images of her test obscured with gas and intestinal contents. Ultrasonography records the reflection of sound waves. Strenuous exercises, the consumption of sweets, and exposure to sunlight do not affect the results of the test in any way.

A patient is seeking emergency care after choking on a piece of steak. The nursing assessment reveals a history of alcoholism, cigarette smoking, and hemoptysis. Which diagnostic study is most likely to be performed on this patient? A) Barium swallow B) Endoscopic biopsy C) Capsule endoscopy D) Endoscopic ultrasonography

B) Because of this patient's history of excessive alcohol intake, smoking, hemoptysis, and the current choking episode, cancer may be present. A biopsy is necessary to make a definitive diagnosis of carcinoma, so an endoscope will be used to obtain a biopsy and observe other abnormalities as well. A barium swallow may show narrowing of the esophagus, but it is more diagnostic for achalasia. An endoscopic ultrasonography may be used to stage esophageal cancer. Capsule endoscopy can show alterations in the esophagus but is more often used for small intestine problems. A barium swallow, capsule endoscopy, and endoscopic ultrasonography cannot provide a definitive diagnosis for cancer when it is suspected.

A patient who is given a bisacodyl (Dulcolax) suppository asks the nurse how long it will take to work. The nurse replies that the patient will probably need to use the bedpan or commode within which time frame after administration? A) 2-5 minutes B) 15-60 minutes C) 2-4 hours D) 6-8 hours

B) Bisacodyl suppositories usually are effective within 15 to 60 minutes of administration, so the nurse should plan accordingly to assist the patient to use the bedpan or commode.

After teaching a group of students about the various organs of the upper gastrointestinal tract and possible disorders, the instructor determines that the teaching was successful when the students identify which of the following structures as possibly being affected? a) Large intestine b) Ileum c) Stomach d) Liver

C) Stomach The upper gastrointestinal (GI) tract begins at the mouth and ends at the jejunum. Therefore, the stomach would be a component of the upper GI tract. The lower GI tract begins at the ileum and ends at the anus. The liver is considered an accessory structure.

The nurse is assessing a client with a bleeding gastric ulcer. When examining the client's stool, which of the following characteristics would the nurse be most likely to find? a) Green color and texture b) Black and tarry appearance c) Clay-like quality d) Bright red blood in stool

B) Black and tarry appearance Black and tarry stools (melena) are a sign of bleeding in the upper gastrointestinal (GI) tract. As the blood moves through the GI system, digestive enzymes turn red blood to black. Bright red blood in the stool is a sign of lower GI bleeding. Green color and texture is a distractor for this question. Clay-like stools are a characteristic of biliary disorders

A client has noticed increased incidence of constipation since he broke his ankle and cannot complete his daily three-mile walk. As his home care nurse, you complete your assessment and discuss the potential causes. During your client education session, what do you explain as the mechanical cause of his constipation? a) No known cause b) Ingesting excessive fiber c) Stool remaining in the large intestine too long. d) Drinking excessive water

C) Stool remaining in the large intestine too long Whenever stool remains stationary in the large intestine, moisture continues to be absorbed from the residue. Consequently, retention of stool, for any number of reasons, causes stool to become dry and hard.

A client with inflammatory bowel disease undergoes an ileostomy. On the first day after surgery, the nurse notes that the client's stoma appears dusky. How should the nurse interpret this finding? a) The ostomy bag should be adjusted. b) Blood supply to the stoma has been interrupted. c) An intestinal obstruction has occurred. d) This is a normal finding 1 day after surgery.

B) Blood supply to the stoma has been interrupted. An ileostomy stoma forms as the ileum is brought through the abdominal wall to the surface skin, creating an artificial opening for waste elimination. The stoma should appear cherry red, indicating adequate arterial perfusion. A dusky stoma suggests decreased perfusion. The nurse should interpret this finding as an indication that the stoma's blood supply is interuppted, which may lead to tissue damage or necrosis. A dusky stoma isn't a normal finding 1 day after surgery. Adjusting the ostomy bag wouldn't affect stoma color, which depends on blood supply to the area. An intestinal obstruction also wouldn't change stoma color.

Which of the follow statements provide accurate information regarding cancer of the colon and rectum? a) There is no hereditary component to colon cancer. b) Cancer of the colon and rectum is the second most common type of internal cancer in the United States. c) Rectal cancer affects more than twice as many people as colon cancer. d) The incidence of colon and rectal cancer decreases with age.

B) Cancer of the colon and rectum is the second most common type of internal cancer in the US Cancer of the colon and rectum is the second most common type of internal cancer in the United States. Colon cancer affects more than twice as many people as does rectal cancer (94,700 for colon, 34,700 for rectum). The incidence increases with age (the incidence is highest in people older than 85). Colon cancer occurrence is higher in people with a family history of colon cancer.

A client comes into the emergency department with complaints of abdominal pain. Which of the following should the nurse ask first? a) Family history of ruptured appendix b) Characteristics and duration of pain c) Concerns about impending hospital stay d) Medications taken in the last 8 hours

B) Characteristics and duration of pain A focused abdominal assessment begins with a complete history. The nurse must obtain information about abdominal pain. Pain can be a major symptom of gastrointestinal disease. The character, duration, pattern, frequency, location, distribution, and timing of the pain vary but require investigation immediately.

A client is scheduled for several diagnostic tests to evaluate her gastrointestinal function. After teaching the client about these tests, the nurse determines that the client has understood the teaching when she identifies which test as not requiring the use of a contrast medium? a) Computer tomography b) Colonoscopy c) Small bowel series d) Upper GI series

B) Colonoscopy A colonoscopy is a direct visual examination of the entire large intestine. It does not involve the use of a contrast agent. Contrast medium may be used with a small bowel series, computed tomography, and upper GI series.

A nurse is caring for a client who underwent a subtotal gastrectomy. To manage dumping syndrome, the nurse should advise the client to: a) restrict fluid intake to 1 qt (1,000 ml)/day. b) drink liquids only between meals. c) don't drink liquids 2 hours before meals. d) drink liquids only with meals.

B) Drink liquids only between meals. A client who experiences dumping syndrome after a subtotal gastrectomy should be advised to ingest liquids between meals rather than with meals. Taking fluids between meals allows for adequate hydration, reduces the amount of bulk ingested with meals, and aids in the prevention of rapid gastric emptying. There is no need to restrict the amount of fluids, just the time when the client drinks fluids. Drinking liquids with meals increases the risk of dumping syndrome by increasing the amount of bulk and stimulating rapid gastric emptying. Small amounts of water are allowable before meals.

The nurse is caring for a patient treated with intravenous fluid therapy for severe vomiting. As the patient recovers and begins to tolerate oral intake, the nurse understands that which of the following food choices would be most appropriate? A) Ice tea B) Dry toast C) Warm broth D) Plain hamburger

B) Dry toast Dry toast or crackers may alleviate the feeling of nausea and prevent further vomiting. Extremely hot or cold liquids and fatty foods are generally not well tolerated.

The nurse is caring for a patient treated with intravenous fluid therapy for severe vomiting. As the pt recovers and begins to tolerate oral intake, the N understands that which of the following food choices would be most appropriate? A) Ice tea B) Dry toast C) Warm broth D) Plain hamburger

B) Dry toast (Dry toast or crackers may alleviate the feeling of nausea and prevent further vomiting. Extremely hot or cold liquids and fatty foods are generally not well tolerated.)

When caring for the patient with heart failure, the nurse knows that which gastrointestinal process is most dependent on cardiac output and may affect the patient's nutritional status? A) Ingestion B) Digestion C) Absorption D) Elimination

C) Substances that interface with the absorptive surfaces of the GI tract (primarily in the small intestine) diffuse across the intestinal membranes into intestinal capillaries and are then carried to other parts of the body for use in energy production. The cardiac output provides the blood flow for this absorption of nutrients to occur.

A nurse is preparing a client with Crohn's disease for a barium enema. What should the nurse do the day before the test? a) Serve dairy products. b) Encourage plenty of fluids. c) Serve the client his usual diet. d) Order a high-fiber diet.

B) Encourage plenty of fluids The nurse should encourage plenty of fluids because adequate fluid intake is necessary to avoid dehydration that may be caused by the bowel preparation and to prevent fecal impaction after the procedure. The client may be placed on a low-residue diet 1 to 2 days before the procedure to reduce the contents in the GI tract. Fiber intake is limited in a low-residue diet. Because dairy products leave a residue, they aren't allowed the evening before the test. Clear liquids only are allowed the evening before the test.

Which of the following medications requires the nurse to contact the pharmacist in consultation when the patient receives all oral medications by feeding tube? a) Buccal or sublingual tablets b) Enteric-coated tablets c) Soft gelatin capsules filled with liquid d) Simple compressed tablets

B) Enteric-coated tablets Enteric-coated tablets are meant to be digested in the intestinal tract and may be destroyed by stomach acids. A change in the form of medication is necessary for patients with tube feedings. Simple compressed tablets may be crushed and dissolved in water for patients receiving oral medications by feeding tube. Buccal or sublingual tablets are absorbed by mucous membranes and may be given as intended to the patient undergoing tube feedings. The nurse may make an opening in the capsule and squeeze out contents for administration by feeding tube.

A nurse is applying an ostomy appliance to the ileostomy of a client with ulcerative colitis. Which action is appropriate? a) Maintaining wrinkles in the faceplate so it doesn't irritate the skin b) Gently washing the area surrounding the stoma using a facecloth and mild soap c) Scrubbing fecal material from the skin surrounding the stoma d) Cutting the faceplate opening no more than 2? larger than the stoma

B) Gently washing the area surrounding the stoma using a facecloth and mild soap For a client with an ostomy, maintaining skin integrity is a priority. The nurse should gently wash the area surrounding the stoma using a facecloth and mild soap. Scrubbing the area around the stoma can damage the skin and cause bleeding. The faceplate opening should be no more than 1/8? to 1/6? larger than the stoma. This size protects the skin from exposure to irritating fecal material. The nurse can create an adequate seal and prevent leakage of fecal material from under the faceplate by applying a thin layer of skin barrier and smoothing out wrinkles in the faceplate. Eliminating wrinkles in the faceplate also protects the skin surrounding the stoma from pressure.

The patient who is admitted with a diagnosis of diverticulitis and a history of irritable bowel disease and gastroesophageal reflux disease (GERD) has received a dose of Mylanta 30 ml PO. The nurse would evaluate its effectiveness by questioning the patient as to whether which of the following symptoms has been resolved? A) Diarrhea B) Heartburn C) Constipation D) Lower abdominal pain

B) Heartburn (Mylanta is an antacid that contains both aluminum and magnesium. It is indicated for the relief of GI discomfort, such as with heartburn associated with GERD.)

The pt who is admitted with a diagnosis of diverticulitis and a history of irritable bowel disease and gastroesophageal reflux disease (GERD) has received a dose of Mylanta 30 ml PO. The nurse would evaluate its effectiveness by questioning the patient as to whether which of the following sxs has been resolved? A) Diarrhea B) Heartburn C) Constipation D) Lower abdominal pain

B) Heartburn (Mylanta is an antacid that contains both aluminum and magnesium. It is indicated for the relief of GI discomfort, such as with heartburn associated with GERD.)

The patient who is admitted with a diagnosis of diverticulitis and a history of irritable bowel disease and gastroesophageal reflux disease (GERD) has received a dose of Mylanta 30 ml PO. The nurse would evaluate its effectiveness by questioning the patient as to whether which of the following symptoms has been resolved? A) Diarrhea B) Heartburn C) Constipation D) Lower abdominal pain

B) Heartburn Mylanta is an antacid that contains both aluminum and magnesium. It is indicated for the relief of GI discomfort, such as with heartburn associated with GERD.

The nurse is planning care for a 68-year-old patient with an abdominal mass and suspected bowel obstruction. Which of the following factors in the patient's history increases the patient's risk for colorectal cancer? A) Osteoarthritis B) History of rectal polyps C) History of lactose intolerance D) Use of herbs as dietary supplements

B) History of rectal polyps A history of rectal polyps places this patient at risk for colorectal cancer. This tissue can degenerate over time and become malignant. The other factors identified do not pose additional risk to the patient.

A nurse is caring for a client newly diagnosed with hepatitis A. Which statement by the client indicates the need for further teaching? a) "I'll wash my hands often." b) "How did this happen? I've been faithful my entire marriage." c) "I'll take all my medications as ordered." d) "I'll be very careful when preparing food for my family."

B) How did this happen? I've been faithful my entire marriage The client requires further teaching if he suggests that he acquired the virus through sexual contact. Hepatitis A is transmitted by the oral-fecal route or through ingested food or liquid that's contaminated with the virus. Hepatitis A is rarely transmitted through sexual contact. Clients with hepatitis A need to take every effort to avoid spreading the virus to other members of their family with precautions such as preparing food carefully, washing hands often, and taking medications as ordered.

The nurse is preparing to interview a client with cirrhosis. Based on an understanding of this disorder, which question would be most important to include? a) "What type of over-the-counter pain reliever do you use?" b) "How often do you drink alcohol?" c) "Have you had an infection recently?" d) "Does your work expose you to chemicals?"

B) How often do you drink alcohol? The most common type of cirrhosis results from chronic alcohol intake and is frequently associated with poor nutrition. Although it can follow chronic poisoning with chemicals or ingestion of hepatotoxic drugs such as acetaminophen, asking about alcohol intake would be most important. Asking about an infection or exposure to hepatotoxins or industrial chemicals would be important if the client had postnecrotic cirrhosis.

Which of the following would be the highest priority information to include in preoperative teaching for a 68-year-old patient scheduled for a colectomy? A) How to care for the wound B) How to deep breathe and cough C) The location and care of drains after surgery D) What medications will be used during surgery

B) How to deep breathe and cough Because anesthesia, an abdominal incision, and pain can impair the patient's respiratory status in the postoperative period, it is of high priority to teach the patient to cough and deep breathe. Otherwise, the patient could develop atelectasis and pneumonia, which would delay early recovery from surgery and hospital discharge.

An elderly patient diagnosed with diarrhea is taking digoxin (Lanoxin). Which of the following electrolyte imbalances should the nurse be alert to? a) Hyponatremia b) Hypokalemia c) Hypernatremia d) Hyperkalemia

B) Hypokalemia The older person taking digitalis must be aware of how quickly dehydration and hypokalemia can occur with diarrhea. The nurse teaches the patient to recognize the symptoms of hypokalemia because low levels of potassium intensify the action of digitalis, leading to digitalis toxicity.

Patients diagnosed with esophageal varices are at risk for hemorrhagic shock. Which of the following is a sign of potential hypovolemia? a) Bradycardia b) Hypotension c) Polyuria d) Warm moist skin

B) Hypotension Signs of potential hypovolemia include cool, clammy skin, tachycardia, decreased blood pressure, and decreased urine output.

Gastroesophageal reflux disease (GERD) weakens the lower esophageal sphincter, predisposing older persons to risk for impaired swallowing. In managing the symptoms associated with GERD, the nurse should assign the highest priority to which of the following interventions? A) Decrease daily intake of vegetables and water, and ambulate frequently. B) Drink coffee diluted with milk at each meal, and remain in an upright position for 30 minutes. C) Eat small frequent meals, and remain in an upright position for at least 30 minutes after eating. D) Avoid over-the-counter drugs that have antacids in them

C, Small, frequent feedings requires less release of hydrochloric acid. Remaining in an upright position for 30 minutes after meals prevents reflux into the esophagus

The patient with sudden pain in the left upper quadrant radiating to the back and vomiting was diagnosed with acute pancreatitis. What intervention(s) should the nurse expect to include in the patient's plan of care? A) Immediately start enteral feeding to prevent malnutrition. B) Insert an NG and maintain NPO status to allow pancreas to rest. C) Initiate early prophylactic antibiotic therapy to prevent infection. D) Administer acetaminophen (Tylenol) every 4 hours for pain relief.

B) Initial treatment with acute pancreatitis will include an NG tube if there is vomiting and being NPO to decrease pancreatic enzyme stimulation and allow the pancreas to rest and heal. Fluid will be administered to treat or prevent shock. The pain will be treated with IV morphine because of the NPO status. Enteral feedings will only be used for the patient with severe acute pancreatitis in whom oral intake is not resumed. Antibiotic therapy is only needed with acute necrotizing pancreatitis and signs of infection.

A patient who has undergone colostomy surgery is experiencing constipation. Which of the following interventions should a nurse consider for such a patient? a) Instruct the patient to keep a record of food intake b) Instruct the patient to avoid prune or apple juice c) Suggest fluid intake of at least 2 L per day d) Assist the patient regarding the correct diet or to minimize food intake

C) Suggest fluid intake of at least 2 L per day For constipation the nurse should suggest a fluid intake of at least 2L per day. The nurse should also offer prune or apple juice because they promote elimination. The nurse should encourage the patient to eat regular meals. Dieting or fasting can decrease stool volume and slow elimination. The nurse should instruct the patient to keep a record of food intake in case of diarrhea because this helps identify specific foods that irritate the GI tract.

"The nurse is teaching a client with a gastric ulcer about dietary management for the disease. Teaching is successful when the client states... Source: Lippincott's Review for NCLEX-RN" "A: "I should eat a low fiber diet to delay gastric emptying." B: "I cannot eat fruits and veggies because they cause too much gas." C: "As long as they don't bother my stomach, I can eat most foods. D: "I can eat bland foods to help my stomach heal

C, The correct answer is C. The antiulcer diet is not severely restricted. it is the ideal to have small frequent feedings but the client can eat foods as long as they do not cause upset. Low fiber diets are more so used in Ulcerative Colitis. A bland diet is used for severe inflammation

A client who is recovering from anesthesia following oral surgery for lip cancer is experiencing difficulty breathing deeply and coughing up secretions. Which of the following measures will help ease the client's discomfort? a) Positioning the client flat on the abdomen or side. b) Keeping the head of the bed elevated. c) Turning the client's head to the side. d) Providing a tracheostomy tray near the bed.

B) Keeping the head of the bed elevated It is essential to position the client with the head of the bed elevated because it is easier for the client to breathe deeply and cough up secretions after recovering from the anesthetic. Positioning the client flat either on the abdomen or side with the head turned to the side will facilitate drainage from the mouth. A tracheostomy tray is kept by the bed for respiratory distress or airway obstruction. When mouth irrigation is carried out, the nurse should turn the client's head to the side to allow the solution to run in gently and flow out.

A 74-year-old client is on the hospital unit where you practice nursing. She will be undergoing rhinoplasty and you are completing her admission assessment and paperwork. She reports medications she uses on a daily basis, which you record for her chart. Which of her daily medications will result in constipation? a) Acetaminophen b) Laxative c) NSAIDs d) Multivitamin without iron

B) Laxative Constipation may also result from chronic use of laxatives ("cathartic colon") because such use can cause a loss of normal colonic motility and intestinal tone. Laxatives also dull the gastrocolic reflex.

A nurse is providing dietary instructions to a client with a history of pancreatitis. Which instruction is correct? a) "Maintain a high-sodium, high-calorie diet." b) "Maintain a high-carbohydrate, low-fat diet." c) "Maintain a high-fat, high-carbohydrate diet." d) "Maintain a high-fat diet and drink at least 3 L of fluid a day."

B) Maintain a high carbohydrate, low fat diet A client with a history of pancreatitis should avoid foods and beverages that stimulate the pancreas, such as fatty foods, caffeine, and gas-forming foods; should avoid eating large meals; and should eat plenty of carbohydrates, which are easily metabolized. Therefore, the only correct instruction is to maintain a high-carbohydrate, low-fat diet. An increased sodium or fluid intake isn't necessary because chronic pancreatitis isn't associated with hyponatremia or fluid loss.

When obtaining a nursing history on a client with a suspected gastric ulcer, which signs and symptoms would the nurse expect to see? Select all that apply. A. Epigastric pain at night B. Relief of epigastric pain after eating C. Vomiting D. Weight loss

C,D Vomiting and weight loss are common with gastric ulcers. Clients with a gastric ulcer are most likely to complain of a burning epigastric pain that occurs about one hour after eating. Eating frequently aggravates the pain. Clients with duodenal ulcers are more likely to complain about pain that occurs during the night and is frequently relieved by eating.

If a client has abdominal surgery and a portion of the small intestine is removed, the client is at risk for which of the following? a) Gastric ulcers b) Malabsorption syndrome c) Constipation d) Cirrhosis

B) Malabsorption syndrome Absorption is the primary function of the small intestine. Vitamins and minerals are absorbed essentially unchanged. Nutrients are absorbed at specific locations in the small intestine.

What part of the GI tract begins the digestion of food? a) Stomach b) Mouth c) Duodenum d) Esophagus

B) Mouth Food that contains starch undergoes partial digestion when it mixes with the enzyme salivary amylase, which the salivary glands secrete.

A nurse is caring for a client with active upper GI bleeding. What is the appropriate diet for this client during the first 24 hours after admission? a) Skim milk b) Nothing by mouth c) Regular diet d) Clear liquids

B) NPO Shock and bleeding must be controlled before oral intake, so the client should receive nothing by mouth. When the bleeding is controlled, the diet is gradually increased, starting with ice chips and then clear liquids. Skim milk shouldn't be given because it increases gastric acid production, which could prolong bleeding. A clear liquid diet is the first diet offered after bleeding and shock are controlled.

The nurse should administer a prn dose of magnesium hydroxide (MOM) after noting which of the following while reviewing a patient's medical record? A) Abdominal pain and bloating B) No bowel movement for 3 days C) A decrease in appetite by 50% over 24 hours D) Muscle tremors and other signs of hypomagnesemia

B) No bowel movement for 3 days MOM is an osmotic laxative that produces a soft, semisolid stool usually within 15 minutes to 3 hours. This medication would benefit the patient who has not had a bowel movement for 3 days.

The patient with right upper quadrant abdominal pain has an abdominal ultrasound that reveals cholelithiasis. What should the nurse expect to do for this patient? A) Prevent all oral intake. B) Control abdominal pain. C) Provide enteral feedings. D) Avoid dietary cholesterol.

B) Patients with cholelithiasis can have severe pain, so controlling pain is important until the problem can be treated. NPO status may be needed if the patient will have surgery but will not be used for all patients with cholelithiasis. Enteral feedings should not be needed, and avoiding dietary cholesterol is not used to treat cholelithiasis.

A colectomy is scheduled for a 38-year-old woman with ulcerative colitis. The nurse should plan to include what prescribed measure in the preoperative preparation of this patient? A) Instruction on irrigating a colostomy B) Administration of a cleansing enema C) A high-fiber diet the day before surgery D) Administration of IV antibiotics for bowel preparation

B) Preoperative preparation for bowel surgery typically includes bowel cleansing with antibiotics, such as oral neomycin and cleansing enemas, including Fleet enemas. Instructions to irrigate the colostomy will be done postoperatively. Oral antibiotics are given preoperatively, and an IV antibiotic may be used in the OR. A clear liquid diet will be used the day before surgery with the bowel cleansing.

A client with a disorder of the oral cavity cannot tolerate tooth brushing or flossing. Which of the following strategies can the nurse employ to assist this client? a) Regularly wipe the outside of the client's mouth to prevent germs from entering. b) Provide the client with an irrigating solution of baking soda and warm water. c) Recommend that the client drink a small glass of alcohol at the end of the day to kill germs. d) Urge the client to regularly rinse the mouth with tap water.

B) Provide the client with an irrigating solution of baking soda and warm water If a client cannot tolerate brushing or flossing, an irrigating solution of 1 tsp of baking soda to 8 oz of warm water, half strength hydrogen peroxide, or normal saline solution is recommended.

When a central venous catheter dressing becomes moist or loose, what should a nurse do first? a) Notify the physician. b) Remove the dressing, clean the site, and apply a new dressing. c) Remove the catheter, check for catheter integrity, and send the tip for culture. d) Draw a circle around the moist spot and note the date and time.

B) Remove dressing, clean the site, and apply a new dressing. A nurse maintaining a central venous catheter should change the dressing every 72 hours or when it becomes soiled, moist, or loose. After removing the soiled dressing, the nurse should use sterile technique to clean around the site in accordance with facility policy. After the cleaning solution has dried, the nurse should cover the site with a transparent semipermeable dressing. A nurse who notes drainage on a wound dressing should draw a circle around the moist spot and note the date and time. She should notify the physician if she observes any catheter-related complications. Only a nurse with the appropriate qualifications may remove a central venous catheter, and a moist or loose dressing isn't a reason to remove the catheter.

A nurse is providing postprocedure instructions for a client who had an esophagogastroduodenoscopy. The nurse should perform which action? a) Tell the client to call back in the morning so she can give him instructions over the phone. b) Review the instructions with the person accompanying the client home. c) Tell the client there aren't specific instructions for after the procedure. d) Give instructions to the client immediately before discharge.

B) Review the instructions with the person accompanying the client home A client who undergoes esophagogastroduodenoscopy receives sedation during the procedure, and his memory becomes impaired. Clients tend not to remember instructions provided after the procedure. The nurse's best course of action is to give the instructions to the person who is accompanying the client home. It isn't appropriate for the nurse to tell the client to call back in the morning for instructions. The client needs to be aware at discharge of potential complications and signs and symptoms to report to the physician.

A client is admitted with a diagnosis of acute appendicitis. When assessing the abdomen, the nurse would expect to find rebound tenderness at which location? a) Left upper quadrant b) Right lower quadrant c) Left lower quadrant d) Right upper quadrant

B) Right lower quadrant The pain of acute appendicitis localizes in the right lower quadrant (RLQ) at McBurney's point, an area midway between the umbilicus and the right iliac crest. Often, the pain is worse when manual pressure near the region is suddenly released, a condition called rebound tenderness.

Patients with chronic liver dysfunction have problems with insufficient vitamin intake. Which of the following may occur as a result of vitamin C deficiency? a) Beriberi b) Scurvy c) Night blindness d) Hypoprothrombinemia

B) Scurvy Scurvy may result from a vitamin C deficiency. Night blindness, hypoprothrombinemia, and beriberi do not result from a vitamin C deficiency.

Patients with chronic liver dysfunction have problems with insufficient vitamin intake. Which of the following may occur as a result of vitamin C deficiency? a) Hypoprothrombinemia b) Scurvy c) Beriberi d) Night blindness

B) Scurvy Scurvy may result from a vitamin C deficiency. Night blindness, hypoprothrombinemia, and beriberi do not result from a vitamin C deficiency.

Nursing assessment of a client with peritonitis reveals hypotension, tachycardia, and signs and symptoms of dehydration. The nurse also expects to find: a) tenderness and pain in the right upper abdominal quadrant. b) severe abdominal pain with direct palpation or rebound tenderness. c) jaundice and vomiting. d) rectal bleeding and a change in bowel habits.

B) Severe abdominal pain with direct palpation or rebound tenderness Peritonitis decreases intestinal motility and causes intestinal distention. A classic sign of peritonitis is a sudden, diffuse, severe abdominal pain that intensifies in the area of the underlying causative disorder (such as appendicitis, diverticulitis, ulcerative colitis, or a strangulated obstruction). The client may also have rebound tenderness. Tenderness and pain in the right upper abdominal quadrant suggest cholecystitis. Jaundice and vomiting are signs of cirrhosis of the liver. Rectal bleeding or a change in bowel habits may indicate colorectal cancer.

Which of the following medications would the nurse expect the physician to order for a client with cirrhosis who develops portal hypertension? a) Kanamycin (Kantrex) b) Spironolactone (Aldactone) c) Cyclosporine (Sandimmune) d) Lactulose (Cephulac)

B) Spironlactone (Aldactone) For portal hypertension, a diuretic usually an aldosterone antagonist such as spironolactone (Aldactone) is ordered. Kanamycin (Kantrex) would be used to treat hepatic encephalopathy to destroy intestinal microorganisms and decrease ammonia production. Lactulose would be used to reduce serum ammonia concentration in a client with hepatic encephalopathy. Cyclosporine (Sandimmune) would be used to prevent graft rejection after a transplant.

A client with acute liver failure exhibits confusion, a declining level of consciousness, and slowed respirations. The nurse finds him very difficult to arouse. The diagnostic information which best explains the client's behavior is: a) elevated blood urea nitrogen and creatinine levels and hyperglycemia. b) subnormal serum glucose and elevated serum ammonia levels. c) subnormal clotting factors and platelet count. d) elevated liver enzymes and low serum protein level.

B) Subnormal serum glucose and elevated serum ammonia levels In acute liver failure, serum ammonia levels increase because the liver can't adequately detoxify the ammonia produced in the GI tract. In addition, serum glucose levels decline because the liver isn't capable of releasing stored glucose. Elevated serum ammonia and subnormal serum glucose levels depress the level of a client's consciousness. Elevated liver enzymes, low serum protein level, subnormal clotting factors and platelet count, elevated blood urea nitrogen and creatine levels, and hyperglycemia aren't as directly related to the client's level of consciousness.

The nurse is preparing to examine the abdomen of a client complaining of a change in his bowel pattern. The nurse would place the client in which position? a) Lithotomy b) Supine with knees flexed c) Knee-chest d) Left Sim's lateral

B) Supine with knees flexed When examining the abdomen, the client lies supine with his knees flexed. This position assists in relaxing the abdominal muscles. The lithotomy position commonly is used for a female pelvic examination and to examine the rectum. The knee-chest position can be used for a variety of examinations, most commonly the anus and rectum. The left Sim's lateral position may be used to assess the rectum or vagina and to administer an enema.

A client with appendicitis is experiencing excruciating abdominal pain. An abdominal X-ray film reveals intraperitoneal air. The nurse should prepare the client for: a) colonoscopy. b) surgery. c) nasogastric (NG) tube insertion. d) barium enema.

B) Surgery The client should be prepared for surgery because his signs and symptoms indicate bowel perforation. Appendicitis is the most common cause of bowel perforation in the United States. Because perforation can lead to peritonitis and sepsis, surgery wouldn't be delayed to perform other interventions, such as colonoscopy, NG tube insertion, or a barium enema. These procedures aren't necessary at this point.

A client receives a local anesthetic to suppress the gag reflex for a diagnostic procedure of the upper GI tract. Which of the following nursing interventions is advised for this patient? a) The client should be monitored for any breathing related disorder or discomforts b) The client should not be given any food and fluids until the gag reflex returns c) The client should be monitored for cramping or abdominal distention d) The client's fluid output should be measured for at least 24 hours after the procedure

B) The client should not be given any food and fluids until the gag reflex returns. For a client receiving a local anesthetic that suppresses the gag reflex, the nurse is advised to withhold food and fluids until the reflex returns.

The patient had a car accident and was "scared to death." The patient is now reporting constipation. What affecting the gastrointestinal (GI) tract does the nurse know could be contributing to the constipation? A) The patient is too nervous to eat or drink, so there is no stool. B) The sympathetic nervous system was activated, so the GI tract was slowed. C) The parasympathetic nervous system is now functioning to slow the GI tract. D) The circulation in the GI system has been increased, so less waste is removed.

B) The constipation is most likely related to the sympathetic nervous system activation from the stress related to the accident. SNS activation can decrease peristalsis. Even without oral intake for a short time, stool will be formed. The parasympathetic system stimulates peristalsis. The circulation to the GI system is decreased with stress.

A client with hepatitis who has not responded to medical treatment is scheduled for a liver transplant. Which of the following most likely would be ordered? a) Chenodiol b) Ursodiol c) Tacrolimus d) Interferon alfa-2b, recombinant

C) Tacrolimus In preparation for a liver transplant, a client receives immunosuppressants to reduce the risk for organ rejection. Tacrolimus or cyclosporine are two immunosuppresants that may be used. Chenodiol and ursodiol are agents used to dissolve gall stones. Recombinant interferon alfa-2b is used to treat chronic hepatitis B, C, and D to force the virus into remission.

Which patient teaching component is important for the nurse to communicate regarding pain management prior to or during diagnostic testing for a disorder of the GI system? a) The patient should not expel gas and test fluids from the bowel when he or she experiences the urge during the procedure. b) The patient should inform the test personnel if he or she experiences pressure or cramping during the instillation of test fluids. c) The patient should take a sedative before the procedure to avoid the possibility of experiencing any discomfort. d) The patient should lie down in a supine position for at least 3 hours before the test to reduce any discomfort during the test.

B) The patient should inform the test personnel if he or she experiences pressure or cramping during instillation of test fluids To ensure that a patient who is to undergo a diagnostic test for a disorder of the gastrointestinal system experiences no or minimal discomfort during the test, the patient should be instructed to inform the test personnel if he or she experiences pressure or cramping during the instillation of test fluids. The test personnel can slow the instillation or take other measures to relieve discomfort. The patient should also be advised to expel gas and test fluids from the bowel when he or she experiences the urge. Ignoring the urge to expel the bowel contents increases pain and discomfort. The patient should be advised not to take any sedative or analgesic before the test, unless prescribed. Lying down in a supine position is not known to have any consequence on the level of discomfort experienced by a patient during a diagnostic test for a GI disorder.

An elderly client asks the nurse how to treat chronic constipation. What is the best recommendation the nurse can make? a) Administer a tap-water enema weekly. b) Take a mild laxative such as magnesium citrate when necessary. c) Take a stool softener such as docusate sodium (Colace) daily. d) Administer a phospho-soda (Fleet) enema when necessary.

C) Take a stool softener such as docusate sodium (Colace) daily Stool softeners taken daily promote absorption of liquid into the stool, creating a softer mass. They may be taken on a daily basis without developing a dependence. Dependence is an adverse effect of daily laxative use. Enemas used daily or on a frequent basis can also lead to dependence of the bowel on an external source of stimulation.

A client is admitted to the hospital for diagnostic testing to rule out colorectal cancer. Which intervention should the nurse include on the plan of care? a) Prepare the client for a gastrostomy tube placement. b) Administer morphine (Duramorph PF) routinely, as ordered. c) Test all stools for occult blood. d) Administer topical ointment to the rectal area to decrease bleeding.

C) Test all stools for occult blood Blood in the stools is one of the warning signs of colorectal cancer. The nurse should plan on checking all stools for both frank and occult blood. The blood in the stool is coming from the colon or rectum; administering an ointment wouldn't help decrease the bleeding. Preparing a client for a gastrostomy tube isn't appropriate when diagnosing colorectal cancer. Colorectal cancer is usually painless; administering opioid pain medication isn't needed.

Paul Cavanagh, a 63-year-old retired teacher, had oral cancer and had extensive surgery to excise the malignancy. While is surgery was deemed successful, it was quite disfiguring and incapacitating. What is essential to Mr. Cavanagh and his family? a) Knowing that everything will work out just fine b) Time to mourn, accept, and adjust to the loss c) Not giving in to anger d) Having a courageous attitude

B) Time to mourn, accept, and adjust to the loss The first time family members or clients see the effects of surgery, the experience usually is traumatic. The nurse needs to promote effective coping and therapeutic grieving at this time. Responses may range from crying or extreme sadness and avoiding contact with others to refusing to talk about the surgery or changes in appearance. Allowing the client time to mourn, accept, and adjust to losses is essential.

A client with severe and chronic liver disease is showing manifestations related to inadequate vitamin intake and metabolism. He reports difficulty driving at night because he cannot see well. Which of the following vitamins is most likely deficient for this client? a) Vitamin K b) Vitamin A c) Riboflavin d) Thiamine

B) Vitamin A Problems common to clients with severe chronic liver dysfunction result from inadequate intake of sufficient vitamins. Vitamin A deficiency results in night blindness and eye and skin changes. Thiamine deficiency can lead to beriberi, polyneuritis, and Wernicke-Korsakoff psychosis. Riboflavin deficiency results in characteristic skin and mucous membrane lesions. Vitamin K deficiency can cause hypoprothrombinemia, characterized by spontaneous bleeding and ecchymoses.

Inspection of an older patient's mouth reveals the presence of white, curd-like lesions on the patient's tongue. What is the most likely etiology for this abnormal assessment finding? A) Herpesvirus B) Candida albicans C) Vitamin deficiency D) Irritation from ill-fitting dentures

B) White, curd-like lesions surrounded by erythematous mucosa are associated with oral candidiasis. Herpesvirus causes benign vesicular lesions in the mouth. Vitamin deficiencies may cause a reddened, ulcerated, swollen tongue. Irritation from ill-fitting dentures will cause friable, edematous, painful, bleeding gingivae.

A client presented with gastrointestinal bleeding 2 days ago and continues to have problems. The physician has ordered a visualization of the small intestine via a capsule endoscopy. Which of the following will the nurse include in the client education about this procedure? a) "An x-ray machine will use a capsule ray to follow your intestinal tract." b) "You will need to swallow a capsule." c) "The physician will use a scope called a capsule to view your intestine." d) "A capsule will be inserted into your rectum."

B) You will need to swallow a capsule A capsule endoscopy allows for noninvasive visualization of the small intestinal mucosa. The technique consists of the client swallowing a capsule that is embedded with a wireless miniature camera, which is propelled through the intestine by peristalsis. The capsule passes from the rectum in 1 to 2 days.

A client with inflammatory bowel disease undergoes an ileostomy. On the first day after surgery, the nurse notes that the client's stoma appears dusky. How should the nurse interpret this finding?" a) The ostomy bag should be adjusted. b) Blood supply to the stoma has been interrupted. c) An intestinal obstruction has occurred. d) This is a normal finding 1 day after surgery

B), Blood supply to the stoma has been interrupted An ileostomy stoma forms as the ileum is brought through the abdominal wall to the surface skin, creating an artificial opening for waste elimination. The stoma should appear cherry red, indicating adequate arterial perfusion. A dusky stoma suggests decreased perfusion. The nurse should interpret this finding as an indication that the stoma's blood supply is interuppted, which may lead to tissue damage or necrosis. A dusky stoma isn't a normal finding 1 day after surgery. Adjusting the ostomy bag wouldn't affect stoma color, which depends on blood supply to the area. An intestinal obstruction also wouldn't change stoma color

The patient who is admitted with a diagnosis of diverticulitis and a history of irritable bowel disease and gastroesophageal reflux disease (GERD) has received a dose of Mylanta 30 ml PO. The nurse would evaluate its effectiveness by questioning the patient as to whether which of the following symptoms has been resolved? A-Diarrhea B.Heartburn C.Constipation D. Lower abdominal pain

B, "Mylanta is an antacid that contains both aluminum and magnesium. It is indicated for the relief of GI discomfort, such as with heartburn associated with GERD

What statement made by the client indicates to the nurse the client may be experiencing GERD?n " A. ""My chest hurts when I walk up the stairs in my home"" B. ""I take antacid tablets with me wherever I go"" C. ""My spouse tells me I snore very loudly at night"" D. ""I drink 6 to 7 soft drinks every day

B, (B) Frequent use of antacids indicates an acid reflux problem

The nurse should recognize that the liver performs which functions (select all that apply)? A) Bile storage B) Detoxification C) Protein metabolism D) Steroid metabolism E) Red blood cell (RBC) destruction

B, C, D) The liver performs multiple major functions that aid in the maintenance of homeostasis. These include metabolism of proteins and steroids as well as detoxification of drugs and metabolic waste products. The Kupffer cells of the liver participate in the breakdown of old RBCs. The liver produces bile, but storage occurs in the gall bladder.

When planning care for a client with ulcerative colitis who is experiencing symptoms, which client care activities can the nurse appropriately delegate to a unlicensed assistant? Select all that apply. A. Assessing the client's bowel sounds B. Providing skin care following bowel movements C. Evaluating the client's response to antidiarrheal medications D. Maintaining intake and output records E. Obtaining the client's weight.

B, D, E The nurse can delegate the following basic care activities to the unlicensed assistant: providing skin care following bowel movements, maintaining intake and output records, and obtaining the client's weight. Assessing the client's bowel sounds and evaluating the client's response to medication are registered nurse activities that cannot be delegated.

The male client tells the nurse he has been experiencing "heartburn" at night that awakens him. Which assessment question should the nurse ask? A. How much weight have you gained recently? B. What have you done to alleviate the heartburn? C. Do you consume many milk and dairy products? D Have you been around anyone with a stomach virus

B, Most clients with GERD have been self medicating with over-the counter medications prior to seeking advice from a healthcare provider. It is important to know what the client has been using to treat the problem

The teaching plan for the patient being discharged following an acute episode of upper gastrointestinal bleeding includes information concerning the importance of (select all that apply) A. taking aspirin only with milk or bread products. B. avoiding taking aspirin and drugs containing aspirin. C. taking only drugs prescribed by the health care provider. D. taking all drugs 1 hour before mealtime to prevent further bleeding. E. reading all over-the-counter drug labels to avoid those containing stearic acid and calcium.

B,C Rationale Before discharge the patient with upper gastrointestinal bleeding and the caregiver should be taught how to avoid future bleeding episodes. Ulcer disease, drug or alcohol abuse, and liver and respiratory diseases can result from upper gastrointestinal bleeding. Help the patient and caregiver be aware of the consequences of noncompliance with drug therapy. Emphasize that no drugs (especially aspirin and NSAIDs) other than those prescribed by the health care provider should be taken. Smoking and alcohol should be eliminated because they are sources of irritation and interfere with tissue repair. Reference: 983-984

While caring for a client with peptic ulcer disease, the client reports that he has been nauseated most of the day and is now feeling lightheaded and dizzy. Based upon these findings, which nursing actions would be most appropriate for the nurse to take? Select all that apply. A. Administering an antacid hourly until nausea subsides. B. Monitoring the client's vital signs C. Notifying the physician of the client's symptoms D. Initiating oxygen therapy E. Reassessing the client on an hour

B,C The symptoms of nausea and dizziness in a client with peptic ulcer disease may be indicative of hemorrhage and should not be ignored. The appropriate nursing actions at this time are for the nurse to monitor the client's vital signs and notify the physician of the client's symptoms. To administer an antacid hourly or to wait one hour to reassess the client would be inappropriate; prompt intervention is essential in a client who is potentially experiencing a gastrointestinal hemorrhage. The nurse would notify the physician of assessment findings and then initiate oxygen therapy if ordered by the physician.

A client diagnosed with chronic cirrhosis who has ascites and pitting peripheral edema also has hepatic encephalopathy. Which of the following nursing interventions are appropriate to prevent skin breakdown? (Select all that apply.) A. Range of motion every 4 hours B. Turn and reposition every 2 hours C. Abdominal and foot massages every 2 hours D. Alternating air pressure mattress E. Sit in chair for 30 minutes each shift

B,D Edematous tissue must receive meticulous care to prevent tissue breakdown. Range of motion exercises preserve joint function but do not prevent skin breakdown. Abdominal or foot massage will not prevent skin breakdown but must be cleansed carefully to prevent breaks in skin integrity. The feet should be kept at the level of heart or higher so Fowler's position should be employed. An air pressure mattress, careful repositioning can prevent skin breakdown.

Which option provides proper information regarding the medications for GERD (select all that apply)? A. Sucralfate (Carafate) should be taken 1 hour after meals. B. Proton pump inhibitors (PPIs) such as esomeprazole (Nexium) are most effective for esophagitis. C. Antacids should be taken before eating for maximum benefit. D. Prokinetic drugs such as metoclopramide (Reglan) work by enhancing motility. E. H2-receptor (H2R) blockers such as cimetidine (Tagamet) work by blocking histamine.

B,D,E Rationale PPIs are more effective in healing esophagitis than H2R blockers. Prokinetic agents are motility-enhancing drugs to promote gastric emptying and reduce the risk of gastric reflux. H2R blockers are available in over-the-counter and prescription formulas. They reduce symptoms and promote esophageal healing by blocking the action of histamine on the H2Rs. Carafate works through its cytoprotective properties and should be taken before meals. Antacids neutralize acid already present and should be taken when acid is the highest, which is 1 hour and 3 hours after meals. Reference: 973-974

The nurse is caring for a client who has had a gastroscopy. Which of the following symptoms may indicate that the client is developing a complication related to the procedure? Select all that apply. A. The client complains of a sore throat B. The client has a temperature of 100*F C. The client appears drowsy following the procedure D. The client complains of epigastric pain E. The client experiences hematemesis

B,D,E Following a gastroscopy, the nurse should monitor the client for complications, which include perforation and the potential for aspiration. An elevated temperature, complaints of epigastric pain, or the vomiting of blood (hematemesis) are all indications of a possible perforation and should be reported promptly. A sore throat is a common occurrence following a gastroscopy. Clients are usually sedated to decrease anxiety and the nurse would anticipate that the client will be drowsy following the procedure.

A nurse is caring for a client with cirrhosis. The nurse assesses the client at noon and discovers that the client is difficult to arouse and has an elevated serum ammonia level. The nurse should suspect which situation? a) The client didn't take his morning dose of lactulose (Cephulac). b) The client is relaxed and not in pain. c) The client's hepatic function is decreasing. d) The client is avoiding the nurse.

C) The client's hepatic function is decreasing The decreased level of consciousness caused by an increased serum ammonia level indicates hepatic disfunction. If the client didn't take his morning dose of lactulose, he wouldn't have elevated ammonia levels and decreased level of consciousness this soon. These assessment findings don't indicate that the client is relaxed or avoiding the nurse.

When planning care for a client with ulcerative colitis who is experiencing symptoms, which client care activities can the nurse appropriately delegate to a unlicensed assistant? Select all that apply. A. Assessing the client's bowel sounds B. Providing skin care following bowel movements C. Evaluating the client's response to antidiarrheal medications D. Maintaining intake and output records E. Obtaining the client's weigh

B,D,E The nurse can delegate the following basic care activities to the unlicensed assistant: providing skin care following bowel movements, maintaining intake and output records, and obtaining the client's weight. Assessing the client's bowel sounds and evaluating the client's response to medication are registered nurse activities that cannot be delegated.

"The nurse has instructed the client who is experiencing diarrhea associated with irritable bowel syndrome on dietary changes to prevent diarrhea. The nurse knows the client understands the dietary changes if the client selects which of the following menu choices? a) Yogurt, crackers and sweet tea b) Salad with chicken, whole wheat crackers c) Bacon, tomato, lettuce with mayonnaise and a soft drink d) Tuna on white bread and coconut cake

B,Rationale: Bacon tomato lettuce with mayonnaise and soft drink is high in fat and the soft drink is hyperosmolar both contributing to diarrhea. Salad, whole wheat crackers may decrease diarrhea due to increased fiber. Dairy increases diarrhea. Food high in carbohydrates increase diarrhea. Coconut may increase diarrhea

A patient with advanced cirrhosis has a nursing diagnosis of imbalanced nutrition: less than body requirements r/t anorexia and inadequate food intake. An appropriate midday snack for the patient would be a. peanut butter and salt free crackers b. a fresh tomato sandwich with salt free butter c. popcorn with salt free butter and herbal seasoning d. canned chicken noodle soup with low protein bread

B- The patient with advanced, complicated cirrhosis requires a high calore, high carbohydrate diet with moderate to low fat. Patients with cirrhosis are at risk for edema and ascites and their sodium intake should be limited. The tomato sandwich with salt free butter best meets these requirements. Rough foods, such as popcorn, may irritate the esophagus and stomach and lead to bleeding. Peanut butter is high in sodium and fat, and canned chicken noodle soup is very high in sodium

Which of the following tests should be administered to a client suspected of having diverticulosis? A. Abdominal ultrasound B. Barium enema C. Barium swallow D. Gastroscopy

B. A barium enema will cause diverticula to fill with barium and be easily seen on x-ray. An abdominal US can tell more about structures, such as the gallbladder, liver, and spleen, than the intestine. A barium swallow and gastroscopy view upper GI structures.

The nurse evaluates the client's stoma during the initial post-op period. Which of the following observations should be reported immediately to the physician? A. The stoma is slightly edematous B. The stoma is dark red to purple C. The stoma oozes a small amount of blood D. The stoma does not expel stool

B. A dark red to purple stoma indicates inadequate blood supply. Mild edema and slight oozing of blood are normal in the early post-op period. The colostomy would typically not begin functioning until 2-4 days after surgery.

The client has just had surgery to create an ileostomy. The nurse assesses the client in the immediate post-op period for which of the following most frequent complications of this type of surgery? A. Intestinal obstruction B. Fluid and electrolyte imbalance C. Malabsorption of fat D. Folate deficiency

B. A major complication that occurs most frequent following an ileostomy is fluid and electrolyte imbalance. The client requires constant monitoring of intake and output to prevent this from happening. Losses require replacement by intravenous infusion until the client can tolerate a diet orally. Intestinal obstruction is a less frequent complication. Fat malabsorption and folate deficiency are complications that could occur later in the postoperative period.

Which of the following symptoms would a client in the early stages of peritonitis exhibit? A. Abdominal distention B. Abdominal pain and rigidity C. Hyperactive bowel sounds D. Right upper quadrant pain

B. Abdominal pain causing rigidity of the abdominal muscles is characteristic of peritonitis. Abdominal distention may occur as a late sign but not early on. Bowel sounds may be normal or decreased but not increased. Right upper quadrant pain is chatacteristic of cholecystitis or hepatitis.

Care for the postoperative client after gastric resection should focus on which of the following problems? A. Body image B. Nutritional needs C. Skin care D. Spiritual needs

B. After gastric resection, a client may require total parenteral nutrition or jejunostomy tube feedings to maintain adequate nutritional status.

During the first few days of recovery from ostomy surgery for ulcerative colitis, which of the following aspects should be the first priority of client care? A. Body image B. Ostomy care C. Sexual concerns D. Skin care

B. Although all of these are concerns the nurse should address, being able to safely manage the ostomy is crucial for the client before discharge.

A nurse is monitoring a client admitted to the hospital with a diagnosis of appendicitis. The client is scheduled for surgery in 2 hours. The client begins to complain of increased abdominal pain and begins to vomit. On assessment the nurse notes that the abdomen is distended and the bowel sounds are diminished. Which of the following is the most appropriate nursing intervention? A. Administer dilaudid B. Notify the physician C. Call and ask the operating room team to perform the surgery as soon as possible D. Reposition the client and apply a heating pad on a warm setting to the client's abdomen.

B. Based on the signs and symptoms presented in the question, the nurse should suspect peritonitis and should notify the physician. Administering pain medication is not an appropriate intervention. Heat should never be applied to the abdomen of a client with suspected appendicitis. Scheduling surgical time is not within the scope of nursing practice, although the physician probably would perform the surgery earlier than the prescheduled time.

The nurse is caring for a male client with a diagnosis of chronic gastritis. The nurse monitors the client knowing that this client is at risk for which vitamin deficiency? a. Vitamin A b. Vitamin B12 c. Vitamin C d. Vitamin E

B. Chronic gastritis causes deterioration and atrophy of the lining of the stomach, leading to the loss of the function of the parietal cells. The source of the intrinsic factor is lost, which results in the inability to absorb vitamin B12. This leads to the development of pernicious anemia. The client is not at risk for vitamin A, C, or E deficiency

A client with gastric cancer can expect to have surgery for resection. Which of the following should be the nursing management priority for the preoperative client with gastric cancer? A. Discharge planning B. Correction of nutritional deficits C. Prevention of DVT D. Instruction regarding radiation treatment

B. Client's with gastric cancer commonly have nutritional deficits and may be cachectic. Discharge planning before surgery is important, but correcting the nutrition deficit is a higher priority. At present, radiation therapy hasn't been proven effective for gastric cancer, and teaching about it preoperatively wouldn't be appropriate. Prevention of DVT also isn't a high priority to surgery, though it assumes greater importance after surgery.

The nurse is reviewing the record of a client with Crohn's disease. Which of the following stool characteristics would the nurse expect to note documented on the client's record? A. Chronic constipation B. Diarrhea C. Constipation alternating with diarrhea D. Stool constantly oozing from the rectum

B. Crohn's disease is characterized by nonbloody diarrhea of usually not more than four to five stools daily. Over time, the diarrhea episodes increase in frequency, duration and severity. The other option are not associated with diarrhea.

Which goal of the client's care should take priority during the first days of hospitalization for an exacerbation of ulcerative colitis? A. Promoting self-care and independence B. Managing diarrhea C. Maintaining adequate nutrition D. Promoting rest and comfort

B. Diarrhea is the primary symptom in an exacerbation of ulcerative colitis, and decreasing the frequency of stools is the first goal of treatment. The other goals are ongoing and will be best achieved by halting the exacerbation. The client may receive antidiarrheal medications, antispasmodic agents, bulk hydrophilic agents, or anti-inflammatory drugs.

Which of the following definitions best describes diverticulosis? A. An inflamed outpouching of the intestine B. A noninflamed outpouching of the intestine C. The partial impairment of the forward flow of intestinal contents D. An abnormal protrusion of an organ through the structure that usually holds it.

B. Diverticulosis involves a noninflamed outpouching of the intestine. Diverticulitis involves an inflamed outpouching. The partial impairment of forward flow of the intestine is an obstruction; abnormal protrusion of an organ is a hernia.

Which of the following complications of gastric resection should the nurse teach the client to watch for? A. Constipation B. Dumping syndrome C. Gastric spasm D. Intestinal spasms

B. Dumping syndrome is a problem that occurs postprandially after gastric resection because ingested food rapidly enters the jejunum without proper mixing and without the normal duodenal digestive processing. Diarrhea, not constipation, may also be a symptom. Gastric or intestinal spasms don't occur, but antispasmidics may be given to slow gastric emptying.

What laboratory finding is the primary diagnostic indicator for pancreatitis? a. Elevated blood urea nitrogen (BUN) b. Elevated serum lipase c. Elevated aspartate aminotransferase (AST) d. Increased lactate dehydrogenase (LD)

B. Elevation of serum lipase is the most reliable indicator of pancreatitis because this enzyme is produced solely by the pancreas. A client's BUN is typically elevated in relation to renal dysfunction; the AST, in relation to liver dysfunction; and LD, in relation to damaged cardiac muscle.

An enema is prescribed for a client with suspected appendicitis. Which of the following actions should the nurse take? A. Prepare 750 ml of irrigating solution warmed to 100*F B. Question the physician about the order C. Provide privacy and explain the procedure to the client D. Assist the client to left lateral Sim's position

B. Enemas are contraindicated in an acute abdominal condition of unknown origin as well as after recent colon or rectal surgery or myocardial infarction. The other answers are correct only when enema administration is appropriate.

A male client with pancreatitis complains of pain. The nurse expects the physician to prescribe meperidine (Demerol) instead of morphine to relieve pain because: a. meperidine provides a better, more prolonged analgesic effect. b. morphine may cause spasms of Oddi's sphincter. c. meperidine is less addictive than morphine. d. morphine may cause hepatic dysfunction.

B. For a client with pancreatitis, the physician will probably avoid prescribing morphine because this drug may trigger spasms of the sphincter of Oddi (a sphincter at the end of the pancreatic duct), causing irritation of the pancreas. Meperidine has a somewhat shorter duration of action than morphine. The two drugs are equally addictive. Morphine isn't associated with hepatic dysfunction.

Which of the following symptoms is associated with ulcerative colitis? A. Dumping syndrome B. Rectal bleeding C. Soft stools D. Fistulas

B. In ulcerative colitis, rectal bleeding is the predominant symptom. Soft stools are more commonly associated with Crohn's disease, in which malabsorption is more of a problem. Dumping syndrome occurs after gastric surgeries. Fistulas are associated with Crohn's disease.

15. Which of the following factors is believed to cause ulcerative colitis? A. Acidic diet B. Altered immunity C. Chronic constipation D. Emotional stress

B. Several theories exist regarding the cause of ulcerative colitis. One suggests altered immunity as the cause based on the extraintestinal characteristics of the disease, such as peripheral arthritis and cholangitis. Diet and constipation have no effect on the development of ulcerative colitis. Emotional stress can exacerbate the attacks but isn't believed to be the primary cause.

After a right hemicolectomy for treatment of colon cancer, a 57-year old client is reluctant to turn while on bed rest. Which action by the nurse would be appropriate? A. Asking a co-worker to help turn the client B. Explaining to the client why turning is important. C. Allowing the client to turn when he's ready to do so D. Telling the client that the physician's order states he must turn every 2 hours

B. The appropriate action is to explain the importance of turning to avoid postoperative complications. Asking a coworker to help turn the client would infringe on his rights. Allowing him to turn when he's ready would increase his risk for postoperative complications. Telling him he must turn because of the physician's orders would put him on the defensive and exclude him from participating in care decision.

A client with irritable bowel syndrome is being prepared for discharge. Which of the following meal plans should the nurse give the client? A. Low fiber, low-fat B. High fiber, low-fat C. Low fiber, high-fat D. High-fiber, high-fat

B. The client with irritable bowel syndrome needs to be on a diet that contains at least 25 grams of fiber per day. Fatty foods are to be avoided because they may precipitate symptoms.

Which of the following symptoms is a client with colon cancer most likely to exhibit? A. A change in appetite B. A change in bowel habits C. An increase in body weight D. An increase in body temperature

B. The most common complaint of the client with colon cancer is a change in bowel habits. The client may have anorexia, secondary abdominal distention, or weight loss. Fever isn't associated with colon cancer.

The client with a colostomy has an order for irrigation of the colostomy. The nurse used which solution for irrigation? A. Distilled water B. Tap water C. Sterile water D. Lactated Ringer's

B. Warm tap water or saline solution is used to irrigate a colostomy. If the tap water is not suitable for drinking, then bottled water should be used.

35. Which of the following vitamins are stored by the normal liver? A. vit. A, vit. B and vit. C B. vit. A, vit. B, vit. C, and vit. D C. vit A and vit B D. vit. A and vit. C

C

A client being treated for esophageal varices has a Sengstaken-Blakemore tube inserted to control the bleeding. The most importantassessment is for the nurse to: A. check that a hemostat is at the bedside B. monitor IV fluids for the shift C. regularly assess respiratory status D. check that the balloon is deflated on a regular basis

C

A client is scheduled for thyroidectomy. The nurse explains thatPTU or an iodine preparation is given prior to surgery in order to: A. increase the size of the thyroid gland B. render the parathyroid glands visible C. induce a euthyroid state in the body D. Separate the thyroid from the laryngeal nerve

C

A client with NIDDM is admitted to the hospital. The client isconfused and has dry mucus membranes and poor skin turgor. Theserum sodium is 149; the blood pressure 90/60 mmHg; the pulse is118; and the serum glucose 465 mg/dl. The nurse anticipates thatinsulin and the following will be needed: A. a potassium drip C. intravenous fluids B. sodium bicarbonate D. calcium gluconate

C

A nurse is caring for a client with cirrhosis. The nurse assesses the client at noon and discovers that the client is difficult to arouse and has an elevated serum ammonia level. The nurse should suspect which situation? a) The client didn't take his morning dose of lactulose (Cephulac). b) The client is relaxed and not in pain. c) The client's hepatic function is decreasing. d) The client is avoiding the nurse.

C

A patient is returned to the surgical unit following a laparoscopic fundoplication for repair of hiatal hernia with an IV, NG tube t osuction, and several small abdominal incisions. to prevent disruption of the surgical site, it is most important for the nurse to: a. monitor for return of perstalsis b. position the patient on the right side c. maintain the patency of the NG tube d. assess abdominal wounds

C

A physician orders the deflation of the esophageal balloon of aSengstaken-Balkemore tube in a client. The nurse prepares for the procedure knowing that the deflation of the esophageal balloon places. The client is at risk for: A. increased ascites B. esophageal necrosis C. recurrent hemorrhage from the esophageal varices D. gastritis

C

Combined with clinical manifestations, the lab finding that is most commonly used to diagnose acute prancreatitis is: a. increased serum lipase c c. increased urinary amylase d. decreased renal amylase creatine clearance

C

Nurse teaches a patient with newly diagnosed PUD to a. maintain bland diet b. use alcohol and caffeine in moderation c. eat as normally as possible, eliminating foods that cause pain d. avoid milk and milk products

C

Nurse teaches patient with diverticulosis to a. use anticholinergic drugs routinely to prevent bowel spasm b. have a nannual colonoscopy to detect malignant changes c. maintain a high fiber diet and use bulk laxatives to increase fecal volume d. exclude whole grains

C

Patient asks nurse if his risks for colon cancer are increased due to a polyp. the best response is: a. it is very rare for polyps ot become malignant b. individuals with polyps have a 100% lifetime risk of developing colorectal cancer c. all polyps are abnormal and should be removed, but the risk for cancer depends on the type and if malignant changes are present d. all polyps are premalignant and source of most colon cancer. get colonoscpy q 6 months.

C

Patient with a gunshot wound to the abdomen develops a bacterial peritonitis after surgery to repair the bowel. The nurse explains to the patient htat this problem is caused prirmarily by: a. immobility and loss of perstaliss of the bowel as reuslt of surgery b. penetration of unsteril foreighn bodies into the abdominal cavity c. spillage of bowel contents int othe normally sterile abdominal cavity d. accumulation of blood and fluid in the abdominal cavity as a result of the trauma

C

Patient with history of PUD is hospitalized with symptoms of a perforation. During initial assessment nurse would expect to find: a. vomit of bright red blood b. projectile vomiting c. sudden, severe upper abdominal pain and shoulder pain d. hyperactive stomach sounds

C

Patient with pancreatitis has nursing dx of pain r/t distention of pancreas and peritoneal irritation. In addition to effective use of analgesics the nurse should: a. provide diversional activiies to distract patient b. provide small frequent meals c. position the patient on the side with the head of the bed elevated 45º d. ambulate the patient q 3-4 hours

C

Teaching is effective when atient with PUD states: a. " I should stop all meds if i develop side effects" b. "i should cintinue treatemnt as long as i have pain" c. "i have learned some relaxation strategies that decrease my stress" d. "i can buy whatever antacids are on sale"

C

The healthcare provider is assessing a patient diagnosed with ulcerative colitis. The patient has an altered level of consciousness, fever, and lower abdominal distension. Which of these additional findings would confirm a diagnosis of toxic megacolon? Please choose from one of the following options. A. Bradycardia B. Splenomegaly C. Leukocytosis D. Constipation

C

When assessing a patient with pancreatitits, nurse would expect to find: a. hyperactive bowel sounds b. hypertension and tachycardia c. severe midepigastric of LUQ pain d. temp greater than 102º

C

Which of the following nursing diagnosis would be mostappropriate for the client with decreased thyroid function: A. alteration in growth and development related to increased growthhormone production B. alteration in thought processes related to decreased neurologicfunction C. fluid volume deficit related to polyuria D. hypothermia related to decreased metabolic rate

C

Which of the following would be the least important assessment in a patient diagnosed with ascites? a) Measurement of abdominal girth b) Palpation of abdomen for a fluid shift c) Foul-smelling breath d) Weight

C

he nurse determines tha further discharge instruction is needed whne the patient with acute pancreatititis states: a. "i should observe for fat in my stools" b. "I must not use alcohol to prevent future attacks" c. "I shouldn't eat salty foods" d. "I will need to continue to monitor my blood glucose levels until my pancreas is healed"

C

the nurse teaches the patient with a hiatal hernia or GERD to control symptoms by: a. drinking 10-12 oz of water with meals b. spacing six small meals a day c. sleeping wit hthe head of the bed elevated 4-6 inches d. perfrming aily exercises of toe touching

C

The hospitalized client with GERD is complaining of chest discomfort that feels like heartburn following a meal. After administering an ordered antacid, the nurse encourages the client to lie in which of the following positions? A. Supine with the head of the bed flat B. On the stomach with the head flat C. On the left side with the head of the bed elevated 30 degrees D. On the right side with the head of the bed elevated 30 degrees

C The discomfort of reflux is aggravated by positions that compress the abdomen and the stomach. These include lying flat on the back or on the stomach after a meal of lying on the right side. The left side-lying position with the head of the bed elevated is most likely to give relief to the client.

The nurse is caring for a male client with cirrhosis. Which assessment findings indicate that the client has deficient vitamin K absorption caused by this hepatic disease? a. Dyspnea and fatigue b. Ascites and orthopnea c. Purpura and petechiae d. Gynecomastia and testicular atrophy

C A hepatic disorder, such as cirrhosis, may disrupt the liver's normal use of vitamin K to produce prothrombin (a clotting factor). Consequently, the nurse should monitor the client for signs of bleeding, including purpura and petechiae. Dyspnea and fatigue suggest anemia. Ascites and orthopnea are unrelated to vitamin K absorption. Gynecomastia and testicular atrophy result from decreased estrogen metabolism by the diseased liver.

Which teaching should you provide to a patient with chronic gastritis? A. Take antacids before meals. B. Avoid pepper. C. Abstain from alcohol. D. Rest after eating.

C Rationale Alcohol can damage gastric mucosa from local injury to destruction of mucosa. It is a problem seen more in alcohol binging and chronic consumption. Antacids should be taken after meals (1 hour and 3 hours) when the acid is highest. Although large quantities of spicy, irritating foods or dietary indiscretions can cause gastritis, sensible use of pepper in and of itself is not considered a cause. Stress can be a contributing factor, but resting after meals is not a key aspect and could contribute to gastroesophageal reflux disease (GERD). Reference: 985

The patient has chronic gastritis. You should monitor the patient for which potential deficiency? A. Iron B. Scurvy C. Pernicious anemia D. Fat-soluble vitamins

C Rationale Chronic gastritis can result in the loss of parietal cells from atrophy and therefore the loss of intrinsic factor. Intrinsic factor is essential for the absorption of cobalamin in the terminal ileus. This causes a deficiency in the nutrient needed for the growth and maturation of red blood cells, causing pernicious anemia. Other deficiencies do not commonly occur with loss of the intrinsic factor. Scurvy is a deficiency of vitamin C. The other deficiencies are not related directly to gastritis. Reference: 986

What clinical manifestations are common in a patient with stomach cancer? A. Shortness of breath and productive cough B. Constipation C. Unexplained weight loss and anemia D. Fat intolerance

C Rationale Clinical manifestations are unexplained weight loss, anorexia, abdominal discomfort, indigestion, and anemia. Anemia is caused by the chronic blood loss as the lesion erodes through the mucosa or from pernicious anemia. The other options are not specific for stomach cancer. Reference: 999

The patient is vomiting large amounts of bright red vomitus. What action should you take? A. Obtain intravenous access with a 22-gauge IV catheter. B. Infuse D5W/.45 normal saline at 60 mL/hr. C. Administer the prescribed pantoprazole (Protonix). D. Insert a Sengstaken-Blakemore tube.

C Rationale During the acute phase of upper gastrointestinal bleeding, empiric use of proton pump inhibitor (PPI) therapy with high-dose bolus and subsequent infusions is often started before endoscopy. It can decrease the amount of bleeding and the need for endoscopy. When active bleeding is present, the IV lines (preferably two) should be 16- or 18-gauge needles for fluid and blood replacement. For replacement, an isotonic crystalloid solution (e.g., lactated Ringer's solution, normal saline) is started, not a solution with dextrose, which becomes hypotonic as the dextrose is metabolized. Replacement rates are 100 to 150 mL/hr; maintenance is 60 to 80 mL/hr. The tube is used only for esophageal varices, and a physician or nurse practitioner inserts it. Reference: 982

The patient reports a relatively sudden onset of bloody diarrhea. What is the most important test to obtain? A. Fecal smear for guaiac B. Blood urea nitrogen (BUN) C. Stool culture D. Hemoglobin

C Rationale E. coli O157:H7 produces a powerful toxin, and all patients with sudden, bloody diarrhea should get a stool culture done to test for E. coli O157:H7. There is no need to test for occult blood because gross blood is evident. Determining the causative organism and treatment is more important that the BUN (which can be used in assessing the degree of dehydration) or hemoglobin level (which is used for assessing the degree of anemia). Reference: 1002

You determine that a patient has experienced the beneficial effects of medication therapy with famotidine (Pepcid) when which symptom is relieved? A. Nausea B. Belching C. Epigastric pain D. Difficulty swallowing

C Rationale Famotidine is an H2-receptor antagonist that inhibits parietal cell output of hydrochloric acid (HCl) and minimizes damage to gastric mucosa related to hyperacidity, relieving epigastric pain. Reference: 991

The pernicious anemia that may accompany gastritis is caused by A. chronic autoimmune destruction of cobalamin stores in the body. B. progressive gastric atrophy from chronic breakage in the mucosal barrier and blood loss. C. a lack of intrinsic factor normally produced by acid-secreting cells of the gastric mucosa. D. hyperchlorhydria resulting from an increase in acid-secreting parietal cells and degradation of red blood cells.

C Rationale Gastritis may cause a loss of parietal cells as a result of atrophy. The source of intrinsic factor is also lost. The loss of intrinsic factor, a substance essential for the absorption of cobalamin in the terminal ileum, ultimately results in cobalamin deficiency. With time the body's storage of cobalamin is depleted, and a deficiency state exists. Because it is essential for the growth and maturation of red blood cells, the lack of cobalamin results in pernicious anemia and neurologic complications. Reference: 986

Which description accurately reflects the problem of achalasia? A. Intolerance to high fat foods related to enzyme secretion B. White patches in the mouth from candidiasis C. Chronic dysphagia because of pain with eating D. Abdominal and stomach cramping 2 hours after meals

C Rationale In achalasia, peristalsis of the lower two thirds (smooth muscle) of the esophagus is absent, and obstruction of the esophagus occurs at or near the diaphragm. Food accumulates, which dilates the lower esophagus. Dysphagia is the most common symptom. Some patients have pain during or immediately after a meal. The other options are not related to this condition. Reference: 980

Which symptom would be likely if the patient sustains an acid-base imbalance from frequent vomiting? A. Kussmaul respirations B. Mental confusion C. Tingling fingertips D. Tachypnea

C Rationale Metabolic alkalosis can result from loss of hydrochloric acid (HCl) in the stomach. Tingling fingertips and lightheadedness are symptoms of alkalosis. Metabolic alkalosis is more common that metabolic acidosis, which can occur if the contents of the small intestine are vomited. Kussmaul respirations are a classic manifestation of metabolic acidosis, particularly diabetic ketoacidosis. Mental confusion and tachypnea are common in respiratory acidosis. Reference: 964

7. After administration of a dose of metoclopramide (Reglan) to the patient, you determine that the medication has been effective when what is noticed? A. Decreased blood pressure B. Absence of muscle tremors C. Relief of nausea and vomiting D. No further episodes of diarrhea

C Rationale Metoclopramide is classified as a prokinetic and antiemetic medication. If it is effective, the patient's nausea and vomiting should resolve. Reference: 965

The patient receiving chemotherapy rings the call bell and reports an onset of nausea. You should prepare a PRN dose of which medication? A. Morphine sulfate B. Zolpidem (Ambien) C. Ondansetron (Zofran) D. Dexamethasone (Decadron)

C Rationale Ondansetron is a 5-HT3 receptor antagonist antiemetic that is especially effective in reducing cancer chemotherapy-induced nausea and vomiting. Reference: 965

A patient who has a peptic ulcer reports sudden, severe upper abdominal pain throughout the abdomen and radiating to the back. You notice a rigid abdomen with positive rebound tenderness. What action should you take? A. Continue to monitor, and repeat vital signs in 1 hour. B. Administer the prescribed prn antacid. C. Contact the primary health care provider. D. Position the patient on the left side and encourage slow breaths.

C Rationale Perforation is the most lethal complication of peptic ulcer disease. The classic manifestation of perforation is a sudden, dramatic onset of pain that then spreads. Signs of peritoneal irritation are rigid abdomen and positive rebound tenderness. These findings require a medical evaluation to determine whether surgery or other treatment is needed. The other measures do not appropriately respond to this potential emergency. Reference: 989

What is a priority action after a patient has had an endoscopy to diagnose the cause of upper gastrointestinal bleeding? A. Assess the dressing. B. Auscultate bowel sounds. C. Check for gag reflex. D. Administer IV fluid replacement.

C Rationale Protecting the airway is a priority before the patient eats because numbing solutions are often administered. The procedure is an insertion of a tube down the throat, and no dressing is used. It is performed under procedural sedation (e.g., conscious sedation) and peristalsis is usually minimally affected. Dehydration is not a key outcome of the procedure. Reference: 982

Which manifestation would most likely indicate an obstruction in the small intestine? A. Hypoactive bowel sounds B. Coffee-ground emesis C. Copious feculent-odor vomitus D. Abdominal cramping

C Rationale Small intestine obstructions tend to result in feculent vomitus; large bowel obstruction is more likely to cause inability to pass flatus and hypoactive bowel sounds. Coffee-ground emesis indicates gastric bleeding. The coffee ground appearance is caused by blood interacting with the gastric acid. Abdominal cramping is a nonspecific symptom and is more likely to be associated with an obstruction in the large intestine. Reference: 964

Which nursing action is appropriate for the patient the first postoperative day after a hemiglossectomy with radical neck dissection for oral cancer? A. Maintain the patient in supine position for 24 hours. B. Start the patient on a clear liquid diet. C. Note the amount of drainage. D. Frequently suction mouth with tonsil-tip suction.

C Rationale The patient usually has tubes inserted in the surgical area that are connected to suction, for which you should monitor the color and amount of drainage. The patient is placed in a semi-Fowler position to aid breathing. Patients usually cannot eat anything for at least 48 hours due to the localized edema and may need nutritional support beyond that. Patients usually have a tracheostomy along with a radical neck dissection and need tracheal suctioning. Oral suctioning is more of an issue after the patient is able to eat. Reference: 970

The patient is diagnosed with E. coli O157:H7 after reporting sudden, bloody diarrhea. What is most important for you to do? A. Administer IV antibiotics. B. Administer loperamide (Imodium). C. Provide IV fluid replacement. D. Obtain a stool culture.

C Rationale Treatment is mainly supportive and includes maintaining intravascular volume. The use of antibiotics remains controversial. Most persons recover without antibiotics or other specific treatment. There is no evidence that antibiotics improve the course of the disease, and they may precipitate kidney complications. Antidiarrheal agents should be avoided. There is no need for a stool culture because the diagnosis and organism is already identified. Reference: 1002

Which of the following tests can be performed to diagnose a hiatal hernia? A. Colonoscopy B. Lower GI series C. Barium swallow D. Abdominal x-rays

C A barium swallow with fluoroscopy shows the position of the stomach in relation to the diaphragm. A colonoscopy and a lower GI series show disorders of the intestine.

In discussing long term management with the patient with alcoholic cirrhosis, the nurse advises the patient that a. a daily exercise regimen is important to increase the blood flow through the liver b. cirrhosis can be reversed if the patient follows a regimen of proper rest and nutrition c. abstinence from alcohol is the most important factor in improvement of the patient's condition d. the only over the counter analgesic that should be used for minor aches and pains is acetaminophen

C- Abstinence from alcohol is very important in alcoholic cirrhosis and may result in improvement if started when liver damage is reduced by rest and nutrition, most changes in the liver cannot be reversed. Exercise does not promote portal circulation, and very moderate exercise is recommended. Acetaminophen should not be used by the patient with liver disease because it is potentially hepatotoxic.

In a client with diarrhea, which outcome indicates that fluid resuscitation is successful? A. The client passes formed stools at regular intervals B. The client reports a decrease in stool frequency and liquidity C. The client exhibits firm skin turgor D. The client no longer experiences perianal burning

C A client with diarrhea has a nursing diagnosis of Deficient fluid volume related to excessive fluid loss in the stool. Expected outcomes include firm skin turgor, moist mucous membranes, and urine output of at least 30 ml/hr. The client also has a nursing diagnosis of diarrhea, with expected outcomes of passage of formed stools at regular intervals and a decrease in stool frequency and liquidity. The client is at risk for impaired skin integrity related to irritation from diarrhea; expected outcomes for this diagnosis include absence of erythema in perianal skin and mucous membranes and absence of perianal tenderness or burning.

In a client with diarrhea, which outcome indicates that fluid resuscitation is successful? A. The client passes formed stools at regular intervals B. The client reports a decrease in stool frequency and liquidity C. The client exhibits firm skin turgor D. The client no longer experiences perianal burning.

C. A client with diarrhea has a nursing diagnosis of Deficient fluid volume related to excessive fluid loss in the stool. Expected outcomes include firm skin turgor, moist mucous membranes, and urine output of at least 30 ml/hr. The client also has a nursing diagnosis of diarrhea, with expected outcomes of passage of formed stools at regular intervals and a decrease in stool frequency and liquidity. The client is at risk for impaired skin integrity related to irritation from diarrhea; expected outcomes for this diagnosis include absence of erythema in perianal skin and mucous membranes and absence of perianal tenderness or burning.

Which of the following treatments is used for rectal cancer but not for colon cancer? A. Chemotherapy B. Colonoscopy C. Radiation D. Surgical resection

C. A client with rectal cancer can expect to have radiation therapy in addition to chemotherapy and surgical resection of the tumor. A colonoscopy is performed to diagnose the disease. Radiation therapy isn't usually indicated in colon cancer.

In a client with Crohn's disease, which of the following symptoms should not be a direct result from antibiotic therapy? A. Decrease in bleeding B. Decrease in temperature C. Decrease in body weight D. Decrease in the number of stools

C. A decrease in body weight may occur during therapy due to inadequate dietary intake, but isn't related to antibiotic therapy. Effective antibiotic therapy will be noted by a decrease in temperature, number of stools, and bleeding.

Which of the following diagnostic tests may be performed to determine if a client has gastric cancer? A. Barium enema B. Colonoscopy C. Gastroscopy D. Serum chemistry levels

C. A gastroscopy will allow direct visualization of the tumor. A colonoscopy or a barium enema would help diagnose colon cancer. Serum chemistry levels don't contribute data useful to the assessment of gastric cancer.

Which of the following factors is believed to be linked to Crohn's disease? A. Constipation B. Diet C. Hereditary D. Lack of exercise

C. Although the definite cause of Crohn's disease is unknown, it's thought to be associated with infectious, immune, or psychological factors. Because it has a higher incidence in siblings, it may have a genetic cause.

Medical management of the client with diverticulitis should include which of the following treatments? A. Reduced fluid intake B. Increased fiber in diet C. Administration of antibiotics D. Exercises to increase intra-abdominal pressur

C. Antibiotics are used to reduce the inflammation. The client isn't typically isn't allowed anything orally until the acute episode subsides. Parenteral fluids are given until the client feels better; then it's recommended that the client drink eight 8-ounce glasses of water per day and gradually increase fiber in the diet to improve intestinal motility. During the acute phase, activities that increase intra-abdominal pressure should be avoided to decrease pain and the chance of intestinal obstruction.

Which of the following associated disorders may the client with Crohn's disease exhibit? A. Ankylosing spondylitis B. Colon cancer C. Malabsorption D. Lactase deficiency

C. Because of the transmural nature of Crohn's disease lesions, malaborption may occur with Crohn's disease. Ankylosing spondylitis and colon cancer are more commonly associated with ulcerative colitis. Lactase deficiency is caused by a congenital defect in which an enzyme isn't present.

The nurse is preparing a discharge teaching plan for the client who had an umbilical hernia repair. Which of the following would the nurse include in the plan? A. Restricting pain medication B. Maintaining bedrest C. Avoiding coughing D. Irrigating the drain

C. Bedrest is not required following this surgical procedure. The client should take analgesics as needed and as prescribed to control pain. A drain is not used in this surgical procedure, although the client may be instructed in simple dressing changes. Coughing is avoided to prevent disruption of the tissue integrity, which can occur because of the location of this surgical procedure.

The nurse is monitoring a client for the early signs of dumping syndrome. Which symptom indicates this occurrence? A. Abdominal cramping and pain B. Bradycardia and indigestion C. Sweating and pallor D. Double vision and chest pain

C. Early manifestations of dumping syndrome occur 5 to 30 minutes after eating. Symptoms include vertigo, tachycardia, syncope, sweating, pallor, palpitations, and the desire to lie down.

A client's ulcerative colitis symptoms have been present for longer than 1 week. The nurse recognizes that the client should be assessed carefully for signs of which of the following complications? A. Heart failure B. DVT C. Hypokalemia D. Hypocalcemia

C. Excessive diarrhea causes significant depletion of the body's stores of sodium and potassium as well as fluid. The client should be closely monitored for hypokalemia and hyponatremia. Ulcerative colitis does not place the client at risk for heart failure, DVT, or hypocalcemia.

Five days after undergoing surgery, a client develops a small-bowel obstruction. A Miller-Abbott tube is inserted for bowel decompression. Which nursing diagnosis takes priority? A. Imbalanced nutrition: Less than body requirements B. Acute pain C. Deficient fluid volume D. Excess fluid volume

C. Fluid shifts to the site of the bowel obstruction, causing a fluid deficit in the intravascular spaces. If the obstruction isn't resolved immediately, the client may experience an imbalanced nutritional status (less than body requirements); however, deficient fluid volume takes priority. The client may also experience pain, but that nursing diagnosis is also of lower priority than deficient fluid volume.

A client who has ulcerative colitis has persistent diarrhea. He is thin and has lost 12 pounds since the exacerbation of his ulcerative colitis. The nurse should anticipate that the physician will order which of the following treatment approaches to help the client meet his nutritional needs? A. Initiate continuous enteral feedings B. Encourage a high protein, high-calorie diet C. Implement total parenteral nutrition D. Provide six small meals a day.

C. Food will be withheld from the client with severe symptoms of ulcerative colitis to rest the bowel. To maintain the client's nutritional status, the client will be started on TPN. Enteral feedings or dividing the diet into 6 small meals does not allow the bowel to rest. A high-calorie, high-protein diet will worsen the client's symptoms.

The nurse has given instructions to the client with an ileostomy about foods to eat to thicken the stool. The nurse determines that the client needs further instructions if the client stated to eat which of the following foods to make the stools less watery? A. Pasta B. Boiled rice C. Bran D. Low-fat cheese

C. Foods that help thicken the stool of the client with an ileostomy include pasta, boiled rice, and low-fat cheese. Bran is high in dietary fiber and thus will increase output of watery stool by increasing propulsion through the bowel. Ileostomy output is liquid. Addition or elimination of various foods can help thicken or loosen this liquid drainage.

Which of the following definitions best describes gastritis? A. Erosion of the gastric mucosa B. Inflammation of a diverticulum C. Inflammation of the gastric mucosa D. Reflux of stomach acid into the esophagus

C. Gastritis is an inflammation of the gastric mucosa that may be acute (often resulting from exposure to local irritants) or chronic (associated with autoimmune infections or atrophic disorders of the stomach). Erosion of the mucosa results in ulceration. Inflammation of a diverticulum is called diverticulitis; reflux of stomach acid is known as gastroesophageal disease.

The nurse is performing a colostomy irrigation on a client. During the irrigation, a client begins to complain of abdominal cramps. Which of the following is the most appropriate nursing action? A. Notify the physician B. Increase the height of the irrigation C. Stop the irrigation temporarily. D. Medicate with dilaudid and resume the irrigation

C. If cramping occurs during a colostomy irrigation, the irrigation flow is stopped temporarily and the client is allowed to rest. Cramping may occur from an infusion that is too rapid or is causing too much pressure. Increasing the height of the irrigation will cause further discomfort. The physician does not need to be notified. Medicating the client for pain is not the most appropriate action (damn).

Which of the following interventions should be included in the medical management of Crohn's disease? A. Increasing oral intake of fiber B. Administering laxatives C. Using long-term steroid therapy D. Increasing physical activity

C. Management of Crohn's disease may include long-term steroid therapy to reduce the inflammation associated with the deeper layers of the bowel wall. Other management focuses on bowel rest (not increasing oral intake) and reducing diarrhea with medications (not giving laxatives). The pain associated with Crohn's disease may require bed rest, not an increase in physical activity.

A client with rectal cancer may exhibit which of the following symptoms? A. Abdominal fullness B. Gastric fullness C. Rectal bleeding D. Right upper quadrant pain

C. Rectal bleeding is a common symptom of rectal cancer. Rectal cancer may be missed because other conditions such as hemorrhoids can cause rectal bleeding. Abdominal fullness may occur with colon cancer, gastric fullness may occur with gastric cancer, and right upper quadrant pain may occur with liver cancer.

A client with ulcerative colitis has an order to begin salicylate medication to reduce inflammation. The nurse instructs the client to take the medication: A. 30 minutes before meals B. On an empty stomach C. After meals D. On arising

C. Salicylate compounds act by inhibiting prostaglandin synthesis and reducing inflammation. The nurse teaches the client to take the medication with a full glass of water and to increase fluid intake throughout the day. This medication needs to be taken after meals to reduce GI irritation.

A client has surgery for a perforated appendix with localized peritonis. In which position should the nurse place the client? A) Sims position B) trendelenburg C) semi-fowlers D)dorsal recumbant

C. Semi-fowlers aids in drainage and prevents spread of infection throughout the abodominal cavity.

The nurse is caring for a female client with active upper GI bleeding. What is the appropriate diet for this client during the first 24 hours after admission? a. regular diet b. skim milk c. nothing by mouth d. clear liquids

C. Shock and bleeding must be controlled before oral intake, so the client should receive nothing by mouth. A regular diet is incorrect. When the bleeding is controlled, the diet is gradually increased, starting with ice chips and then clear liquids. Skim milk shouldn't be given because it increases gastric acid production, which could prolong bleeding. A liquid diet is the first diet offered after bleeding and shock are controlled

A client with gastric cancer may exhibit which of the following symptoms? A. Abdominal cramping B. Constant hunger C. Feeling of fullness D. Weight gain

C. The client with gastric cancer may report a feeling of fullness in the stomach, but not enough to cause him to seek medical attention. Abdominal cramping isn't associated with gastric cancer. Anorexia and weight loss (not increased hunger or weight gain) are common symptoms of gastric cancer.

During the assessment of a client's mouth, the nurse notes the absence of saliva. The client is also complaining of pain near the area of the ear. The client has been NPO for several days because of the insertion of a NG tube. Based on these findings, the nurse suspects that the client is developing which of the following mouth conditions? A. Stomatitis B. Oral candidiasis C. Parotitis D. Gingivitis

C. The lack of saliva, pain near the area of the ear, and the prolonged NPO status of the client should lead the nurse to suspect the development of parotitis, or inflammation of the parotid gland. Parotitis usually develops in cases of dehydration combined with poor oral hygiene or when clients have been NPO for an extended period. Preventative measures include the use of sugarless hard candy or gum to stimulate saliva production, adequate hydration, and frequent mouth care. Stomatitis (inflammation of the mouth) produces excessive salivation and a sore mouth.

Which of the following conditions is most likely to directly cause peritonitis? A. Cholelithiasis B. Gastritis C. Perforated ulcer D. Incarcerated hernia

C. The most common cause of peritonitis is a perforated ulcer, which can pour contaminates into the peritoneal cavity, causing inflammation and infection within the cavity. The other conditions don't by themselves cause peritonitis. However, if cholelithiasis leads to rupture of the gallbladder, gastritis leads to erosion of the stomach wall, or an incarcerated hernia leads to rupture of the intestines, peritonitis may develop.

Which of the following medications is most effective for treating the pain associated with irritable bowel disease? A. Acetaminophen B. Opiates C. Steroids D. Stool softeners

C. The pain with irritable bowel disease is caused by inflammation, which steroids can reduce. Stool softeners aren't necessary. Acetaminophen has little effect on the pain, and opiate narcotics won't treat its underlying cause (I feel this is untrue—dilaudid will help anything!)

A 30-year old client experiences weight loss, abdominal distention, crampy abdominal pain, and intermittent diarrhea after birth of her 2nd child. Diagnostic tests reveal gluten-induced enteropathy. Which foods must she eliminate from her diet permanently? A. Milk and dairy products B. Protein-containing foods C. Cereal grains (except rice and corn) D. Carbohydrates

C. To manage gluten-induced enteropathy, the client must eliminate gluten, which means avoiding all cereal grains except for rice and corn. In initial disease management, clients eat a high calorie, high-protein diet with mineral and vitamin supplements to help normalize nutritional status.

The nurse is monitoring a client diagnosed with appendicitis who is scheduled for surgery in 2 hours. The client begins to complain of increased abdominal pain and begns to vomit. On assessment, the nurse notes that the abdomen is distended and bowel sounds are diminished. Which is the appropriate nursing intervention? "1. Notify the Physician 2. Administer the prescribed pain medication 3. Call and ask the operating room team to perform the surgery as soon as possible 4. Reposition the client and apply a heating pad on warm setting to the client's abdomen

CORRECT ANSWER: 1" "1. Based on the assessment information the nurse should suspect peritonitis, a complication that is associated with appendicitis, and notify the physician. 2. Administering pain medication is not an appropriate intervention 3. Scheduling surgical time is not within the scope of practice of an RN. 4. Heat should never be applied to the abdomen of a patient suspected of having peritonitis because of the risk of rupture."

The nurse is planning to teach a client with GERD about substances to avoid. Which items should the nurse include on this list? Select all that apply. Coffee Chocolate Peppermint Nonfat milk Fried chicken Scrambled eggs

Coffee, chocolate, peppermint, fried chicken

A patient admitted with acute diverticulitis has experienced a sudden increase in temperature and complains of a sudden onset of exquisite abdominal tenderness. The nurse's rapid assessment reveals that the patient's abdomen is uncharacteristically rigid on palpation. What is the nurse's best response? Administer a Fleet enema as ordered and remain with the patient. Contact the primary care provider promptly and report these signs of perforation. Position the patient supine and insert an NG tube. Page the primary care provider and report that the patient may be obstructed

Contact the primary care provider promptly and report these signs of perforation. The patient's change in status is suggestive of perforation, which is a surgical emergency. Obstruction does not have this presentation involving fever and abdominal rigidity. An enema would be strongly contraindicated. An order is needed for NG insertion and repositioning is not a priority.

The nurse is monitoring a client admitted to the hospital with a diagnosis of appendicitis who is scheduled for surgery in 2 hours. The client begins to complain of increased abdominal pain and begins to vomit. On assessment, the nurse notes that the abdomen is distended and bowel sounds are diminished. Which is the appropriate nursing intervention? Saunders Comprehensive Review for the NCLEX-RN Examination 5th ed. 1. Notify the physician 2. Administer the prescribed pain medication 3. Call and ask the operating room team to perform the surgery as soon as possible 4. Reposition the client and apply a heating pad on warm setting to the clien't abdomen

Correct 1 Based on the signs and symptoms presented in the question, the nurse shoudl suspect peritonitis and notify the physician. Administering pain medication is not an appropriate intervention. Heat should never be applied to the abdomen of a client wiht suspected appendicitis because of the risk of rupture. Scheduling surgical time is not within the scope of nursing practice, although the physician probably would perform the surgery earlier than the prescheduled time.

"A client is admitted with right lower quadrant pain, anorexia, nausea, low-grade fever, and an elevated white blood cell count. Which complication is most likely the cause? "1. A fecalith 2. Bowel kinking 3. Internal bowel occlusion 4. Abdominal wall swelling"

Correct 1 The client is experiencing appendicitis. A fecalith is a fecal calculus, or stone, that occludes the lumen of the appendix and is the most common cause of appendicitis. Bowel wall swelling, kinking of the appendix, and external occlusion, not internal occlusion, of the bowel by adhesions can also be causes of appendicitis.

The client with sever abdominal pain is being evaluated for appendicitis. What is the most common cause of appendicistis? http://nursing.slcc.edu/nclexrn3500/ 1. Rupture of the appendix 2.Obstruction of the appendix 3 A high-fat diet 4. A duodenal ulcer

Correct 2 Appendicitis most commonly results from obstruction of the appendix, which may lead to rupture. A high-fat diet or duodenal ulcer doesn't cause appendicitis; however, a client may require dietary restrictions after an appendectomy

which statement made by the client who is postoperative abdominal surgery indicates the discharge teaching has been effective? 1. "i will take my temp each week and report any elevation." 2. "i will not need any pain meds when i go home." 3. i will take all of my antibiotics until they are gone." 4. i will not take a shower until my three month check up.

Correct 3 1. the client should check the temp twice a day. 2. it is not realistic to expect the client to experience no pain after surgery. 3 (CORRECT): this statement about taking all the antibiotics ordered indicates the teaching is effective. 4. clients may shower after surgery, but not taking a tub bath for three months after surgery is too long a time.

A client complains of severe pain in the right lower quadrant of the abdomen. To assist with pain relief, the nurse should take which of the following actions? "1. Encourage the client to change positions frequently in bed 2. Massage the right lower quadrant fo the abdomen 3. Apply warmth to the abdomen with a heating pad 4. Use comfort measures and pillows to position the client"

Correct 4 "1. ""Encourage the client..."" - unnecesary movement will increase pain and should be avoided 2. ""Massage the lower..."" - if appendicitis is suspected, massorge or palpation should never be performed as thes actions may cause the appendix to rupture 3. ""Apply warmth..."" - if pain is casused by appendicitis, increased circulation from the heat may cause appendix to rupture 4. ""Use comfort measures..."" - CORRECT: non-pharmacological methods of pain relief"

"A client with acute appendicitis develops a fever, tachycardia, and hypotension. Based on these assessment findings, the nurse should further assess the client for which of the following complications?... "1. Deficient fluid volume. 2. Intestinal obstruction. 3. Bowel ischemia. 4. Peritonitis

Correct 4 "Complications of acute appendicitis are perforation, peritonitis, and abscess development. Signs of the development of peritonitis include abdominal pain and distention, tachycardia, tachypnea, nausea, vomiting, and fever. Because peritonitis can cause hypovolemic shock, hypotension can develop. Deficient fluid volume would not cause a fever. Intestinal obstruction would cause abdominal distention, diminished or absent bowel sounds, and abdominal pain. Bowel ischemia has signs and symptoms similar to those found with intestinal obstruction."

"A nurse is providing wound care to a client 1 day after the client underwent an appendectomy. A drain was inserted into the incisional site during surgery. Which action should the nurse perform when providing wound care? 1. Remove the dressing and leave the incision open to air. 2. Remove the drain if wound drainage is minimal. 3. Gently irrigate the drain to remove exudate. 4. Clean the area around the drain moving away from the drain.

Correct 4 The nurse should gently clean the area around the drain by moving in a circular motion away from the drain. Doing so prevents the introduction of microorganisms to the wound and drain site. The incision cannot be left open to air as long as the drain is intact. The nurse should note the amount and character of wound drainage, but the surgeon will determine when the drain should be removed. Surgical wound drains are not irrigated.

Which of the following would confirm a diagnosis of appendicitis? "a. The pain is localized at a position halfway between the umbilicus and the right iliac crest. b. Mr. Liu describes the pain as occurring 2 hours after eating c. The pain subsides after eating d. The pain is in the left lower quadrant"

Correct A "Pain over McBurney's point, the point halfway between the umbilicus and the iliac crest, is diagnosis for appendicitis. Options b and c are common with ulcers; option d may suggest ulcerative"

"When evaluating a male client for complications of acute pancreatitis, the nurse would observe for: "a. increased intracranial pressure. b. decreased urine output. c. bradycardia. d. hypertension."

Correct Answer: B Rationale: Acute pancreatitis can cause decreased urine output, which results from the renal failure that sometimes accompanies this condition. Intracranial pressure neither increases nor decreases in a client with pancreatitis. Tachycardia, not bradycardia, usually is associated with pulmonary or hypovolemic complications of pancreatitis. Hypotension can be caused by a hypovolemic complication, but hypertension usually isn't related to acute pancreatitis."

Which of the following position should the client with appendicitis assume to relieve pain ? A. Prone B. Sitting C. Supine D. Lying with legs drawn up

Correct Answer: D Lying still with legs drawn up towards chest helps relive tension on the abdominal muscle, which helps to reduce the amount of discomfort felt. Lying flat or sitting may increase the amount of pain experienced

"When preparing a male client, age 51, for surgery to treat appendicitis, the nurse formulates a nursing diagnosis of Risk for infection related to inflammation, perforation, and surgery. What is the rationale for choosing this nursing diagnosis? "a. Obstruction of the appendix may increase venous drainage and cause the appendix to rupture. b. Obstruction of the appendix reduces arterial flow, leading to ischemia, inflammation, and rupture of the appendix. c. The appendix may develop gangrene and rupture, especially in a middle-aged client. d. Infection of the appendix diminishes necrotic arterial blood flow and increases venous drainage."

Correct B A client with appendicitis is at risk for infection related to inflammation, perforation, and surgery because obstruction of the appendix causes mucus fluid to build up, increasing pressure in the appendix and compressing venous outflow drainage. The pressure continues to rise with venous obstruction; arterial blood flow then decreases, leading to ischemia from lack of perfusion. Inflammation and bacterial growth follow, and swelling continues to raise pressure within the appendix, resulting in gangrene and rupture. Geriatric, not middle-aged, clients are especially susceptible to appendix rupture."

When preparing a male client, age 51, for surgery to treat appendicitis, the nurse formulates a nursing diagnosis of Risk for infection related to inflammation, perforation, and surgery. What is the rationale for choosing this nursing diagnosis? "a. Obstruction of the appendix may increase venous drainage and cause the appendix to rupture. b. Obstruction of the appendix reduces arterial flow, leading to ischemia, inflammation, and rupture of the appendix. c. The appendix may develop gangrene and rupture, especially in a middle-aged client. d. Infection of the appendix diminishes necrotic arterial blood flow and increases venous drainage."

Correct B A client with appendicitis is at risk for infection related to inflammation, perforation, and surgery because obstruction of the appendix causes mucus fluid to build up, increasing pressure in the appendix and compressing venous outflow drainage. The pressure continues to rise with venous obstruction; arterial blood flow then decreases, leading to ischemia from lack of perfusion. Inflammation and bacterial growth follow, and swelling continues to raise pressure within the appendix, resulting in gangrene and rupture. Geriatric, not middle-aged, clients are especially susceptible to appendix rupture.

A nurse is making a home health visit and finds the client experiencing right lower quadrant abdominal pain, which has decreased in intensity over the last day. The client also has a rigid abdomen and a temperature of 103.6 F. The nurse should intervene by: a) administer Tylenol (acetaminophen) for the elevated temperature b) advising the client to increase oral fluids c) asking the client when she last had a bowel movement d) notifying the physician

Correct D D. The client symptoms indicate appendicitis which requires immediate attention

"During the assessment of a patient with acute abdominal pain, the nurse should: a. perform deep palpation before auscultation b. obtain blood pressure and pulse rate to determine hypervolemic changes c. auscultate bowel sounds because hyperactive bowel sounds suggest paralytic ileus d. measure body temperature because an elevated temperature may indicate an inflammatory or infectious process.

Correct D Rationale: for the patient complaining of acute abdominal pain, nurse should take vital signs immediately. Increased pulse and decreasing blood pressure are indicative of hypovolemia. An elevated temperature suggests an inflammatory infectious process. Intake and output measurements provide essential information about the adequate of vascular volume. Inspect abdomen first and then auscultate bowel sounds. Palpation is performed next and should be gentle.

Which of the following would indicate that Bobby's appendix has ruptured? " a) diaphoresis b) anorexia c) pain at Mc Burney's point d) relief from pain

Correct D all are normal signs of having appendicits and once you have relief from pain means you could have a rupture.

The nurse would increase the comfort of the patient with appendicitis by: a. Having the patient lie prone b. Flexing the patient's right knee c. Sitting the patient upright in a chair d. Turning the patient onto his or her left side

Correct answer: B" The patient with appendicitis usually prefers to lie still, often with the right leg flexed to decrease pain.

The nurse instructs an obese 22-year-old man with a sedentary job about the health benefits of an exercise program. The nurse evaluates that teaching is effective when the patient makes which statement? a. "The goal is to walk at least 10,000 steps every day of the week." b. "Weekend aerobics for 2 hours is better than exercising every day." c. "Aerobic exercise will increase my appetite and result in weight gain." d. "Exercise causes weight loss by decreasing my resting metabolic rate."

Correct answer: a Rationale: A realistic activity goal is to walk 10,000 steps a day. Increased activity does not promote an increase in appetite or lead to weight gain. Exercise should be done daily, preferably 30 minutes to an hour a day. Exercise increases metabolic rate.

Priority Decision: The nurse is caring for a patient receiving 1000 mL of parenteral nutrition solution over 24 hours. When it is time to change the solution, 150 mL remain in the bottle. What is the most appropriate action by the nurse? a. Hang the new solution and discard the unused solution. b. Open the IV line and rapidly infuse the remaining solution. c. Notify the health care provider for instructions regarding the infusion rate. d. Wait to change the solution until the remaining solution infuses at the proscribed rate.

Correct answer: a Rationale: Bacterial growth occurs at room temperature in nutritional solutions. Therefore solutions must not be infused for longer than 24 hours. Remaining solution should be discard. Speeding up the solution may cause hyperglycemia and should not be done. The health care provider does not need to be notified.

What information should be included in the dietary teaching for the patient following a Roux-en-Y gastric bypass? a. Avoid sugary foods and limit fluids to prevent dumping syndrome. b. Gradually increase the amount of food ingested to preoperative levels. c. Maintain a long-term liquid diet to prevent damage to the surgical site. d. Consume foods high in complex carbohydrates, protein, and fiber to add bulk to contents.

Correct answer: a Rationale: Fluids and foods high in carbohydrates tend to promote diarrhea and symptoms of dumping syndrome in patients with gastric bypass surgery. The diet generally should be high in protein and low in carbohydrates, fat, and roughage and consists of six small feedings a day because of the small stomach size. Liquid diets are likely to be used longer for the patient with a gastroplasty.

The nurse is caring for a patient who is 5'6" tall and weighs 186 lb. The nurse has discussed reasonable weight loss goals and a low-calorie diet with the patient. Which statement made by the patient indicates a need for further teaching? a. "I will limit intake to 500 calories a day." b. "I will try to eat very slowly during mealtimes." c. "I'll try to pick foods from all of the basic food groups." d. "It's important for me to begin a regular exercise program."

Correct answer: a Rationale: Limiting intake to 500 calories per day is not indicated for this patient, and the severe calorie energy restriction would place this patient at risk for multiple nutrient deficiencies. Decreasing caloric intake at least 500 to 1000 calories a day is recommended for weight loss of one to two pounds per week. The other options show understanding of the teaching.

Priority Decision: Before selecting a weight reduction plan with an obese patient, what is most important for the nurse to first assess? a. The patient's motivation to lose weight b. The length of time that the patient has been obese c. Whether financial considerations will affect the patient's choices d. The patient's anthropometric measures of height, weight, BMI, waist-to-hip ratio, and skinfold thickness

Correct answer: a Rationale: Motivation is essential. Focus on the reasons for wanting to lose weight. The rest of the options will asset in planning the weight loss if the patient is motivated.

A 50-year-old African American woman has a BMI of 35 kg/m2, type 2 diabetes mellitus, hypercholesterolemia, and irritable bowel syndrome (IBS). She is seeking assistance in losing weight, because, "I have trouble stopping eating when I should, but I do not want to have bariatric surgery." Which drug therapy should the nurse question if it is prescribed for this patient? a. Orlistat (Xenical) b. Locaserin (Belviq) c. Phentermine (Adipex-P) d. Phentermine and topiramate (Qsymia)

Correct answer: a Rationale: Orlistat (Xenical), which blocks fat breakdown and absorption in the intestine, produces some unpleasant GI side effects. This drug would not be appropriate for someone with IBS. Locaserin (Belviq) suppresses the appetite and creates a sense of satiety that may be helpful for this patient. Phentermine (Adipex-P) needs to be used for a limited period of time (3 months or less). Qsymia is a combination of two drugs, phentermine and topiramate. Phentermine is a sympathomimetic agent that suppresses appetite and topiramate induces a sense of satiety.

A patient has been on a 1000-calorie diet with a daily exercise routine. In 2 months, the patient has lost 20 lb (9kg) toward a goal of 50 lb (23 kg) but is now discouraged that no weight has been lost in the last 2 weeks. What should the nurse tell the patient about this? a. Plateaus where no weight is lost normally occur during a weight-loss program. b. A weight considered by the body to most efficient for functioning has been reached. c. A return to former eating habits is the most common cause of not continuing to lose weight. d. A steady weight may be due to water gain from eating foods high in sodium.

Correct answer: a Rationale: Plateau periods during which no weight is lost are normal occurrences during weight reduction and may last for several days to several weeks but weight loss will resume if the prescribed weight reduction plan is continued. Weight loss may stop if former eating habits are resumed but this not the most common cause of plateaus.

A patient asks the nurse about taking phentermine and topiramate (Qsymia) for weight loss. To avoid side effects, it is important for the nurse to determine whether the patient has a history of a. glaucoma. b. hypertension. c. valvular heart disease. d. irritable bowel disease.

Correct answer: a Rationale: Qsymia is a combination of phentermine and topiramate. It must not be used in patients with glaucoma or hyperthyroidism.

When planning nutritional interventions for a healthy 83-year-old man, the nurse recognizes what factor is most likely to affect his nutritional status? a. Living alone on a fixed income b. Changes in cardiovascular function c. An increase in GI motility and absorption d. Snacking between meal, resulting in obesity

Correct answer: a Rationale: Socioeconomic conditions frequently have the greatest effect on the nutritional status of the healthy older adult. Limited income and social isolation can result in the "tea and toast" meals of the older adult. The other options do not interfere with nutritional status.

When considering tube feedings for a patient with severe protein-calorie malnutrition, what is an advantage of a gastrostomy tube versus a nasogastric (NG) tube? a. There is less irritation to the nasal and esophageal mucosa. b. The patient experiences the sights and smells associated with eating. c. Aspiration resulting from reflux of formulas into the esophagus is less common. d. Routine checking for placement is not required because gastrostomy tubes do not become displaced

Correct answer: a Rationale: Standard nasogastric (NG) tubes are used for tube feedings for short-term feeding problems because prolonged therapy can result in irritation and erosion of the mucosa of the upper GI tract. Gastric reflux and the potential for aspiration can occur with both tubes that deliver fluids into the stomach. Both NG and gastrostomy tubes can become displaced and deprive the patient of the sensations associated with eating.

During the initial postoperative period following bariatric surgery, the nurse recognizes the importance of monitoring obese patients for respiratory insufficiency based on what knowledge? a. The body stores anesthetics in adipose tissue. b. Postoperative pain may cause a decreased respiratory rate. c. Intubation may be difficult because of extra chin skinfolds. d. The patient's head must remain flat for a minimum of 2 hours postprocedure.

Correct answer: a Rationale: The body stores anesthetics in adipose tissue, placing patients with excess adipose tissue at risk for re-sedation. As adipose cells release anesthetics back into the bloodstream, the patient may become sedated after surgery, increasing the risk of hypoventilation and resultant respiratory insufficiency. Difficult intubation does not cause respiratory insufficiency. Pain usually increases respiratory rate. The patient's head should be elevated after bariatric surgery to decrease abdominal pressure and facilitate respirations.

The nurse evaluates that patient teaching about a high-calorie, high-protien diet has been effective when the patient selects which breakfast option from the hospital menu? a. Two poached eggs, hash brown potatoes, and whole milk b. Two slices of toast with butter and jelly, orange juice, and skim milk c. Three pancakes with butter and syrup, two slices of bacon, and apple juice d. Cream of wheat with 2 tbsp of skim milk powder, one half grapefruit, and a high-protein milkshake

Correct answer: a Rationale: The breakfast with the eggs provides 24 g or protein, compared with 14 g for the protein-fortified cream of wheat and milkshake breakfast. Whole milk instead of skim milk helps to meet the calorie requirements. The toast has 10 g of protein and the pancakes have about 6 g. Bacon is considered a fat rather than a meat serving.

The nurse cares for a 34-year-old woman after bariatric surgery. The nurse determines that discharge teaching related to diet is successful if the patient makes which statement? a. "A high protein diet that is low in carbohydrates and fat will prevent diarrhea." b. "Food should be high in fiber to prevent constipation from the pain medication." c. "Three meals a day with no snacks between meals will provide optimal nutrition." d. "Fluid intake should be at least 2000 mL per day with meals to avoid dehydration."

Correct answer: a Rationale: The diet generally prescribed is high in protein and low in carbohydrates, fat, and roughage and consists of six small feedings daily. Fluids should not be ingested with the meal, and in some cases, fluids should be restricted to less than 1000 mL per day. Fluids and foods high in carbohydrate tend to promote diarrhea and symptoms of the dumping syndrome. Generally, calorically dense foods (foods high in fat) should be avoided to permit more nutritionally sound food to be consumed.

The nurse confirms initial placement of a blindly inserted small-bore NG feeding tube by a. x-ray. b. air insufflation. c. observing patient for coughing. d. pH measurement of gastric aspirate.

Correct answer: a Rationale: The nurse should obtain x-ray confirmation to determine whether a blindly placed nasogastric or orogastric tube (small bore or large bore) is properly positioned in the gastrointestinal tract before administering feedings or medications.

The nurse teaches a 50-year-old woman who has a body mass index (BMI) of 39 kg/m2 about weight loss. Which dietary change would be appropriate for the nurse to recommend to this patient? a. Decrease fat intake and control portion size b. Increase vegetables and decrease fluid intake c. Increase protein intake and avoid carbohydrates d. Decrease complex carbohydrates and limit fiber

Correct answer: a Rationale: The safest dietary guideline for weight loss is to decrease caloric intake by maintaining a balance of nutrients and adequate hydration while controlling portion size and decreasing fat intake.

What contributes to increased protein-calorie needs? a. Surgery b. Vegan diet c. Lowered temperature d. Cultural or religious beliefs

Correct answer: a Rationale: With surgery a patient will recover more rapidly with a balanced nutritional status before the surgery and increased protein is needed for healing after the surgery. Following a vegan diet does not put the patient at risk of low protein intake. A lowered temperature will not cause increased protein need. Following religious and cultural beliefs would not be expected to affect an increased need for protein.

During assessment of the patient with protein-calorie malnutrition, what should the nurse expect to find (select all that apply)? a. Frequent cold symptoms b. Decreased bowel sounds c. Cool, rough, dry, scaly skin d. A flat or concave abdomen e. Prominent bony structures f. Decreased reflexes and lack of attention

Correct answer: a, b, c, e, f Rationale: In malnutrition, metabolic processes are slowed, leading to increased sensitivity to cold, decreased heart rate (HR) and cardiac output (CO), and decreased neurologic function. Because of slowed GI motility and absorption, the abdomen becomes distended and protruding and bowel sounds are decreased. Skin is rough, dry, and scaly whereas bone structures protrude because of muscle loss. Because the immune system is weakened, susceptibility to respiratory infections is increased.

The stable patient has a gastrostomy tube for enteral feeding. Which care could the RN delegate to the LPN (select all that apply)? a. Administer bolus or continuous feedings. b. Evaluate the nutritional status of the patient. c. Administer medications through the gastrostomy tube. d. Monitor for complications related to the tube and enteral feeding. e. Teach the caregiver about feeding via the gastrostomy tube at home.

Correct answer: a, c Rationale: For the stable patient, the LPN can administer bolus or continuous feedings and administer medications through the gastrostomy. The RN must evaluate the nutritional status of the patient, monitor for complications related to the tube and the enteral feeding, and teach the caregiver about feeding via the gastrostomy tube at home.

The nurse is caring for a 45-year-old woman with a herniated lumbar disc. The patient realizes that weight loss is necessary to lessen back strain. The patient is 5'6" tall and weighs 186 lb (84.5 kg) with a body mass index (BMI) of 28 kg/m2. The nurse explains to the patient that this measurement places her in which of the following weight categories? a. Normal weight b. Overweight c. Obese d. Severely obese

Correct answer: b Rationale: A normal BMI is 18.5 to 24.9 kg/m2, whereas a BMI of 25 to 29.9 kg/m2 is considered overweight. A BMI of 30.0-39.9 is considered obese, and a BMI of 40 or greater is severely obese.

A woman is 5 ft, 6 in (166 cm) tall and weighs 200 lb (90.9 kg) with a waist-to-hip ratio of 0.7. The nurse counsels the patient with the knowledge that the patient is at greatest risk for a. heart disease. b. osteoporosis. c. diabetes mellitus. d. endometrial cancer.

Correct answer: b Rationale: A patient who is obese (BMI of 32.2) but has a waist-to-hip ratio of less than 0.8, indicating gynoid obesity, has an increased risk for osteoporosis. The other conditions are risks associated with android obesity.

A 22-year-old female is admitted with anorexia nervosa and a serum potassium level of 2.4 mEq/L. What complication is most important for the nurse to observe for in this patient? a. Muscle weakness b. Cardiac dysrhythmias c. Increased urine output d. Anemia and leukopenia

Correct answer: b Rationale: A serum potassium level less than 2.5 mEq/L indicates severe hypokalemia, which can lead to life-threatening cardiac dysrhythmias (e.g., bradycardia, tachycardia, ventricular dysrhythmias). Other manifestations of potassium deficiency include muscle weakness and renal failure. Patients with anorexia nervosa commonly have iron-deficiency anemia and an elevated blood urea nitrogen level related to intravascular volume depletion and abnormal renal function.

When teaching the older adult about nutritional needs during aging, what does the nurse emphasize? a. Need for all nutrients decreases as one ages. b. Fewer calories, but the same or slightly increased amount of protein, are required as one ages. c. Fats, carbohydrates, and protein should be decreased, but vitamin and mineral intake should be increased. d. High-calorie oral supplements should be taken between meals to ensure that recommended nutrient needs are met.

Correct answer: b Rationale: Although calorie intake should be decreased in the older adult because of decreased activity and basal metabolic rate, the need for specific nutrients, such as proteins and vitamins, does not change.

A patient who has suffered severe burns in a motor vehicle accident will soon be started on parenteral nutrition (PN). Which principle should guide the nurse's administration of the patient's nutrition? a. Administration of PN requires clean technique. b. Central PN requires rapid dilution in a large volume of blood. c. Peripheral PN delivery is preferred over the use of a central line. d. Only water-soluble medications may be added to the PN by the nurse.

Correct answer: b Rationale: Central PN is hypertonic and requires rapid dilution in a large volume of blood. Because PN is an excellent medium for microbial growth, aseptic technique is necessary during administration. Administration through a central line is preferred over the use of peripheral PN, and the nurse may not add any medications to PN.

The nurse is teaching a female patient with type 1 diabetes mellitus about nutrition before discharge. She had surgery to revise a lower leg stump with a skin graft. What food should the nurse teach the patient to eat to best facilitate healing? a. Non-fat milk b. Chicken breast c. Fortified oatmeal d. Olive oil and nuts

Correct answer: b Rationale: High quality protein such as chicken breast is important for tissue repair. Although the non-fat milk, nuts, and fortified oatmeal have some protein, they do not have as much as the chicken breast.

A patient is receiving peripheral parenteral nutrition. The parenteral nutrition solution is completed before the new solution arrives on the unit. The nurse administers a. 20% intralipids. b. 5% dextrose solution. c. 0.45% normal saline solution. d. 5% lactated Ringer's solution

Correct answer: b Rationale: If a peripheral parenteral nutrition (PPN) formula bag empties before the next solution is available, a 5% dextrose solution (based on the amount of dextrose in the peripheral PN solution) may be administered to prevent hypoglycemia.

During starvation, the order in which the body obtains substrate for energy is a. visceral protein, skeletal protein, fat, glycogen b. glycogen, skeletal protein, fat stores, visceral protein c. visceral protein, fat stores, glycogen, skeletal protein d. fat stores, skeletal protein, visceral protein, glycogen

Correct answer: b Rationale: Initially, the body selectively uses carbohydrates (e.g., glycogen) rather than fat and protein to meet metabolic needs. These carbohydrate stores, found in the liver and muscles, are minimal and may be totally depleted within 18 hours. After carbohydrate stores are depleted, skeletal protein begins to be converted to glucose for energy. Within 5 to 9 days, body fat is fully mobilized to supply much of the needed energy. In prolonged starvation, up to 97% of calories are provided by fat, and protein is conserved. Depletion of fat stores depends on the amount available, but fat stores typically are used up in 4 to 6 weeks. After fat stores are used, body or visceral proteins, including those in internal organs and plasma, can no longer be spared and rapidly decrease because they are the only remaining body source of energy available.

Priority Decision: When medications are used in the treatment of obesity, what is most important for the nurse to teach the patient? a. Over-the-counter (OTC) diet aids are safer than other agents and con be useful in controlling appetite. b. Drugs should be used only as adjuncts to a diet and exercise program as treatment for a chronic condition. c. All drugs used for weight control are capable of altering central nervous system (CNS) function and should be used with caution. d. The primary effect of the medications is psychologic, controlling the urge to eat in response to stress or feelings of rejection.

Correct answer: b Rationale: Medications are used only as adjuncts to diet and exercise programs in the treatment of obesity. Drugs do not cure obesity; without changes in food intake and physical activity, weight gain will occur when the medications are discontinued. The medications used work in a variety of ways to control appetite but over-the-counter drugs are probably the least effective and most abused of these drugs.

A frail 74-year-old man with recent severe weight loss is instructed to eat a high-protein, high-calorie diet at home. If the man likes all of the items below, which would be the most appropriate for the nurse to suggest? a. Orange juice and dry toast b. Oatmeal, butter, and cream c. Steamed carrots and chicken broth d. Banana and unsweetened applesauce

Correct answer: b Rationale: Oatmeal, butter, and cream are examples of food items that would be appropriate to include for a patient on a high-protein, high-calorie diet.

Priority Decision: The nurse admitting a patient for bariatric surgery obtains the following information from the patient. Which finding should be brought to the surgeon's attention before proceeding with further patient preparation? a. History of hypertension b. History of untreated depression c. History of multiple attempts at weight loss d. History of sleep apnea treated with continuous positive airway pressure (CPAP)

Correct answer: b Rationale: Patients with histories of untreated depression or psychosis are not good candidates for surgery. All other historical information includes medical complications of severe obesity that would help to qualify the patient for the surgery.

An older patient was admitted with a fractured hip after being found on the floor of her home. She was extremely malnourished and started on parenteral nutrition (PN) 3 days ago. Which assessment finding would be of most concern to the nurse? a. Blood glucose level of 125 mg/dL b. Serum phosphate level of 1.9 mg/dL c. White blood cell count of 10,500/µL d. Serum potassium level of 4.6 mEq/L

Correct answer: b Rationale: Refeeding syndrome can occur if a malnourished patient is started on aggressive nutritional support. Hypophosphatemia (serum phosphate level less than 2.4 mg/dL) is the hallmark of refeeding syndrome and could result in cardiac dysrhythmias, respiratory arrest, and neurologic problems. An increase in the blood glucose level is expected during the first few days after PN is started. The goal is to maintain a glucose range of 110 to 150 mg/dL. An elevated white blood cell count (greater than 11,000/µL) could indicate an infection. Normal serum potassium levels are between 3.5 and 5.0 mEq/L.

Which patient is at highest risk for complications of obesity? a. A 30-year-old woman who is 5 ft (151 cm) tall, weighs 140 lb (63 kg), and carries weight in her thighs. b. A 56-year-old woman with a BMI of 38 kg/m2, a waist measurement of 38 in (96 cm), and a hip measurement of 36 in (91 cm) c. A 42-year-old man with a waist measurement of 36 in (91 cm) and a hip measurement of 36 in (91 cm) who is 5 ft, 6 in (166 cm) tall and weighs 150 lb (68.2 kg) d. A 68-year-old man with a waist measurement of 38 in (96 cm) and a hip measurement of 42 in (76 cm) who is 5 ft, 11 in (179 cm) tall and weighs 200 lb (90.9 kg)

Correct answer: b Rationale: The 56-year-old woman has a body mass index (BMI) of 38 kg/m2 (obese, Class II) with a waist-to-hip ratio of 1.1 with android obesity and is more at risk (very high) than the other patients. The 30-year-old woman has the least risk with a BMI of 27.3 kg/m2 (overweight) and gynoid shape. The 42-year-old man has a BMI of 24.2 kg/m2 (normal weight) with one risk factor in the waist-to-hip ratio of 1.0 and the 68-year-old man has a BMI of 27.9 kg/m2 (overweight) with a waist-to-hip ratio of 0.9.

A 40-year-old severely obese female patient with type 2 diabetes wants to lose weight. After learning about the surgical procedures, she thinks a combination of restrictive and malabsorptive surgery would be best. Which procedure should the nurse teach her about? a. Lipectomy b. Roux-en-Y gastric bypass c. Adjustable gastric banding d. Vertical sleeve gastrectomy

Correct answer: b Rationale: The Roux-en-Y gastric bypass is a common combination of restrictive (limiting the size of the stomach) and malabsorptive (less food is absorbed) surgery. Lipectomy is used to remove unsightly flabby folds of adipose tissue. Adjustable gastric banding is the most common restrictive procedure. Vertical sleeve gastrectomy is a restrictive procedure that preserves stomach function.

A patient who has dysphagia as a consequence of a stroke is receiving enteral feedings through a percutaneous endoscopic gastrostomy (PEG). What intervention should the nurse integrate into this patient's care? a. Flush the tube with 30 mL of normal saline every 4 hours. b. Flush the tube before and after feedings if the patient's feedings are intermittent. c. Flush the PEG with 100 mL of sterile water before and after medication administration. d. To prevent fluid overload, avoid flushing when the patient is receiving continuous feeding.

Correct answer: b Rationale: The nurse should flush feeding tubes with 30 mL of water (not normal saline) every 4 hours and before and after medication administration during continuous feeding or before and after intermittent feeding. Flushes of 100 mL are excessive and may cause fluid overload in the patient.

The nurse has completed initial instruction with a patient regarding a weight loss program. The nurse determines that the teaching has been effective when the patient makes which statement? a. "I plan to lose 4 lb a week until I have lost the 60-pound goal." b. "I will keep a diary of weekly weights to illustrate my weight loss." c. "I will restrict my carbohydrate intake to less than 30 g/day to maximize weight loss." d."I should not exercise more than my program requires since increased activity increases the appetite."

Correct answer: b Rationale: The patient should monitor and record weight once per week. This prevents frustration at the normal variations in daily weights and may help the patient to maintain motivation to stay on the prescribed diet. Weight loss should occur at a rate of 1 to 2 lb/week. The diet should be well balanced rather than lacking in specific components that may cause an initial weight loss but is not usually sustainable. Exercise is a necessary component of any successful weight loss program.

Which assessment should the nurse prioritize in the care of a patient who has recently begun receiving parenteral nutrition (PN)? a. Skin integrity and bowel sounds b. Electrolyte levels and daily weights c. Auscultation of the chest and tests of blood coagulability d. Peripheral vascular assessment and level of consciousness (LOC)

Correct answer: b Rationale: The use of PN necessitates frequent and thorough assessments. Key focuses of these assessments include daily weights and close monitoring of electrolyte levels. Assessments of bowel sounds, integument, peripheral vascular system, LOC, chest sounds, and blood coagulation may be variously performed, but close monitoring of fluid and electrolyte balance supersedes these in importance.

The patient cannot afford to buy the food she needs for her family, so she makes sure her children eat first, and then she eats. When she comes to the clinic, she reports bleeding gums, loose teeth, and dry, itchy skin. The nurse should know that this patient is most likely lacking which vitamin? a. Folic acid b. Vitamin C c. Vitamin D d. Vitamin K

Correct answer: b Rationale: This patient is lacking Vitamin C as evidenced by the bleeding gums, loose teeth, and dry, itchy skin. Clinical manifestations of folic acid deficiency include megaloblastic anemia, anorexia, fatigue, sore tongue, diarrhea, or forgetfulness. Clinical manifestations of Vitamin D deficiency include muscular weakness, excess sweating, diarrhea, bone pain, rickets, or osteomalacia. Clinical manifestations of Vitamin K deficiency include defective blood coagulation.

Which female patient is most likely to have metabolic syndrome? a. BP 128/78 mm Hg, triglycerides 160 mg/dL, fasting blood glucose 102 mg/dL b. BP 142/90 mm Hg, high-density lipoproteins 45 mg/dL, fasting blood glucose 130 mg/dL c. Waist circumference 36 in, triglycerides 162 mg/dL, high-density lipoproteins 55 mg/dL d. Waist circumference 32 in, high-density lipoproteins 38 mg/dL, fasting blood glucose 122 mg/dL

Correct answer: b Rationale: Three of the following five measures are needed for a woman to be diagnosed with metabolic syndrome: waist circumference >35 in, triglycerides >150 mg/dL, high-density lipoproteins <50 mg/dL, BP >130 mm Hg systolic or >85 mm Hg diastolic, fasting blood glucose >110 mg/dL. Although the other options have some abnormal measures, none has all three measures in the diagnostic ranges. The criteria for metabolic syndrome for both women and men are listed in Table 41-10.

Which statement about obesity is explained by genetics? a. Older obese patients have exacerbated changes of aging. b. Android body shape and weight gain are influenced by genetics. c. White Americans have a higher incidence of obesity than African Americans. d. Men have a harder time losing weight, as they have more muscle mass than women.

Correct answer: b Rationale: Twin studies and studies with adopted children have shown that body shape and weight gain are influenced by genetics but more research is needed. Older obese people do have exacerbated aging problems related to declines in physical function. African Americans and Hispanics have a higher incidence of obesity than whites. Women have a higher incidence of obesity and more difficulty losing weight than men because women have a higher percentage of metabolically less-active fat.

Priority Decision: Before administering a bolus of intermittent tube feeding to a patient with a percutaneous endoscopic gastrostomy (PEG), the nurse aspirates 220 mL of gastric contents. How should the nurse respond? a. Return the aspirate to the stomach and recheck the volume of aspirate in an hour. b. Return the aspirate to the stomach and continue with tube feeding as planned. c. Discard the aspirate to prevent over distending the stomach when the new feeding is given. d. Notify the health care provider that the feedings have been scheduled too frequently to allow for stomach emptying.

Correct answer: b Rationale: With intermittent feedings, less than 250 mL residual does not require further action. With continuous feedings and a residual of 250 mL or more after the second residual check, a pro motility agent should be considered.

The patient has parenteral nutrition infusing with amino acids and dextrose. In report, the oncoming nurse is told that the tubing, the bag, and the dressing were changed 22 hours ago. What care should the nurse coming on be prepared to do (select all that apply)? a. Give the patient insulin. b. Check amount of feeding left in the bag. c. Check that the next bag has been ordered. d. Check the insertion site and change the tubing. e. Check the label to ensure ingredients and solution are as ordered.

Correct answer: b,c,e Rationale: The nurse should check the amount of feeding left in the bag, and that the next bag has been ordered to be sure the solution will not run out before the next bag is available. Parenteral nutrition solutions are only good for 24 hours and usually take some time for the pharmacy to mix for each patient. The label on the bag should be checked to ensure that the ingredients and solution are what was ordered. The patient would only receive insulin if the patient is experiencing hyperglycemia and was receiving sliding scale insulin or had diabetes mellitus. The insertion site should be checked, but the tubing is only changed every 72 hours unless lipids are being used.

In developing a weight reduction program with a 45-year-old female patient who weighs 197 lb, the nurse encourages the patient to set a weight loss goal of how many pounds in 4 weeks? a. 1-2 b. 3-5 c. 4-8 d. 5-10

Correct answer: c Rationale: A realistic weight loss goal for patients is 1 to 2 lb/wk, which prevents the patient from becoming frustrated at not meeting weight loss goals.

The nurse recognizes that the majority of patients' caloric needs should come from which source? a. Fats b. Proteins c. Polysaccharides d. Monosaccharides

Correct answer: c Rationale: Carbohydrates should constitute between 45% and 65% of caloric needs, compared with 20% to 35% from fats and 10% to 35% from proteins. Polysaccharides are the complex carbohydrates that are contained in breads and grains. Monosaccharides are simple sugars.

In developing an effective weight reduction plan for an overweight patient who states a willingness to try to lose weight, it is most important for the nurse to first assess which factor? a. The length of time the patient has been obese b. The patient's current level of physical activity c. The patient's social, emotional, and behavioral influences on obesity d. Anthropometric measurements, such as body mass index and skinfold thickness

Correct answer: c Rationale: Eating patterns are established early in life, and eating has many meanings for people. To establish a weight reduction plan that will be successful for the patient, the nurse should first explore the social, emotional, and behavioral influences on the patient's eating patterns. The duration of obesity, current physical activity level, and current anthropometric measurements are not as important for the weight reduction plan.

What is an indication for parenteral nutrition that is not an appropriate indication for enteral tube feedings? a. Head and neck cancer b. Hypermetabolic states c. Malabsorption syndrome d. Protein-calorie malnutrition

Correct answer: c Rationale: In malabsorption syndrome, foods that are ingested into the intestinal tract cannot be digested or absorbed and tube feedings infused into the intestinal tract would not be absorbed. All of the other conditions can be treated with enteral or parenteral nutrition, depending on the patient's needs.

The nurse is evaluating the nutritional status of a 55-year-old man who is undergoing radiation treatment for oropharyngeal cancer. Which laboratory test would be the best indicator to determine if the patient has protein-calorie malnutrition? a. Serum transferrin b. C-reactive protein c. Serum prealbumin d. Alanine transaminase (ALT)

Correct answer: c Rationale: In the absence of an inflammatory condition, the best indicator of protein-calorie malnutrition (PCM) is prealbumin; prealbumin is a protein synthesized by the liver and indicates recent or current nutritional status. Decreased albumin and transferrin levels are other indicators that protein is deficient. C-reactive protein (CRP) is elevated during inflammation and is used to determine if prealbumin, albumin, and transferrin are decreased related to protein deficiency or an inflammatory process. Other indicators of protein deficiency include elevated serum potassium levels, low red blood cell counts and hemoglobin levels, decreased total lymphocyte count, elevated liver enzyme levels (ALT), and decreased levels of both fat-soluble and water-soluble vitamins.

What is the main underlying risk factor for metabolic syndrome? a. Age b. Heart disease c. Insulin resistance d. High cholesterol levels

Correct answer: c Rationale: Insulin resistance is the main underlying risk factor for metabolic syndrome. Aging is associated with metabolic syndrome. High cholesterol, hypertension, and increased clotting risk are characteristics of metabolic syndrome.

Which statement accurately describes vitamin deficiencies? a. The two nutrients most often lacking in the diet of a vegan are vitamin B6 and folic acid. b. Vitamin imbalances occur frequently in the United States because of excessive fat intake. c. Surgery on the GI tract may contribute to vitamin deficiencies because of impaired absorption. d. Vitamin deficiencies in adults most commonly are clinically manifested by disorders of the skin.

Correct answer: c Rationale: Patients who have surgery on the GI tract may be at risk for vitamin deficiencies because of inability to absorb or metabolize them. The strict vegan diet most often lacks cobalamin (vitamin B12) and iron. Although the high intake of fat is a major nutritional problem in the United States, vitamin deficiencies are rare in developed countries except in those with eating disorders or chronic alcohol abusers. Some vitamin deficiencies in adults have neurologic manifestations.

The nurse monitors the laboratory results of the patient with protein-calorie malnutrition during treatment. Which result is an indication of improvement in the patient's condition? a. Decreased lymphocytes b. Increased serum potassium c. Increased serum transferrin d. Decreased serum prealbumin

Correct answer: c Rationale: Serum transferrin is a protein that is synthesized by the liver and used for iron transport and decreases when there is protein deficiency. An increase in the protein would indicate a more positive nitrogen balance with amino acids available for synthesis. Decreased lymphocytes and serum prealbumin are indicators of protein depletion and increased serum potassium shows continuing failure of the sodium-potassium pump.

Priority Decision: During care of the severely obese patient, what is most important for the nurse to do? a. Avoid reference to the patient's weight to avoid embarrassing the patient. b. Emphasize to the patient how important it is to lose weight to maintain health. c. Plan for necessary modifications in equipment and nursing techniques before initiating care. d. Recognize that a full assessment of each body system might not be possible because of numerous layers of skinfolds.

Correct answer: c Rationale: Special considerations are needed for the care of the severely obese patient because most hospital units are not prepared with beds, chairs, BP cuffs, and other equipment that will need to be used with the very obese patient. Consideration of all aspects of care should be made before implementing care for the patient, including extra time and perhaps assistance for positioning, physical assessment, and transferring the patient.

Which statement best describes the etiology of obesity? a. Obesity primarily results from a genetic predisposition. b. Psychosocial factors can override the effects of genetics in the etiology of obesity. c. Obesity is the result of complex interactions between genetic and environmental factors. d. Genetic factors are more important than environmental factors in the etiology of obesity.

Correct answer: c Rationale: The cause of obesity involves significant genetic and biologic susceptibility factors that are highly influenced by environmental and psychosocial factors.

The nurse is caring for a patient admitted to the hospital for asthma who weighs 186 lb (84.5 kg). During dietary counseling, the patient asks the nurse how much protein he should ingest each day. How many grams of protein does the nurse recommend should be included in the diet based on the patient's current weight? a. 24 b. 41 c. 68 d. 93

Correct answer: c Rationale: The daily intake of protein should be between 0.8 and 1 g/kg of body weight. Thus this patient should take in between 68 and 84 g of protein per day in the diet.

What may occur with failure of the sodium-potassium pump during severe protein depletion? a. Ascites b. Anemia c. Hyperkalemia d. Hypoalbuminemia

Correct answer: c Rationale: The sodium-potassium pump uses 20% to 50% of all calories ingested. When energy sources are decreased, the pump fails to function, sodium and water are left in the cell, and potassium remains in extracellular fluids. Hyperkalemia, as well as hyponatremia, can occur.

The nurse is providing care for a 23-year-old woman who is a strict vegetarian. To prevent the consequences of iron deficiency, what should the nurse recommend? a. Brown rice and kidney beans b. Cauliflower and egg substitutes c. Soybeans and hot breakfast cereal d. Whole-grain bread and citrus fruits

Correct answer: c Rationale: Vegetarians are at a particular risk for iron deficiency, a problem that can be prevented by regularly consuming high-iron foods such as hot cereals and soybeans. The other foods listed are not classified as high sources of iron.

Which explanation about weight reduction should be included when teaching the obese patient and her obese husband? a. Weight gain is caused by psychologic factors. b. Daily weighing is recommended to monitor weight loss. c. Fat is not burned until the glycogen-water pool is depleted. d. Men lose weight less quickly than women because they have a higher percentage of metabolically less-active fat.

Correct answer: c Rationale: With reducing diets that severely restrict carbohydrates, the body's glycogen stores become depleted within a few days. The glycogen normally binds to water in fat cells and it is this water loss that causes weight loss in the first few days. Fat is not burned until the glycogen-water pool is depleted. Although psychosocial components (i.e., using food for comfort or reward and inability to buy high-nutritional quality food) may have an influence on weight gain, these factors along with lack of physical exercise, underestimation of portion size, and genetics contribute to weight gain. Weekly weighing is recommended as a more reliable indicator of weight loss because daily weighing shows frequent fluctuation from retained water (including urine) and elimination of feces. Men are able to lose weight more quickly than women because women have a higher percentage of metabolically less-active fat.

When teaching a patient about weight reduction diets, the nurse teaches the patient that an appropriate single serving of a food is a. a 6-inch bagel. b. 1 cup of chopped vegetables. c. a piece of cheese the size of three dice. d. a chicken breast the size of a deck of cards.

Correct answer: d Rationale: A chicken breast the size of a deck of cards is about 3 oz, a recommended portion size of meat. Other normal portions include a 3-inch bagel, 1/2 cup of chopped vegetables, and a piece of cheese the size of six dice.

When the nurse identifies an individual at risk for malnutrition with nutritional screening, what is the next step for the nurse to take? a. Supply supplements between meals. b. Encourage eating meals with others. c. Have family bring in food from home. d. Complete a full nutritional assessment.

Correct answer: d Rationale: A full nutritional assessment includes history and physical examination and laboratory data. The nutritional assessment will need to be done to provide the basis for nutrition intervention. The interventions may include supplements if ordered, family bringing food from home, and socializing with meals.

A complete nutritional assessment including anthropometric measurements is important for the patient who a. has a BMI of 25.5 kg/m2. b. complains of frequent nocturia. c. reports a 5-year history of constipation. d. reports an unintentional weight loss of 10 lb in 2 months.

Correct answer: d Rationale: A loss of more than 5% of usual body weight over 6 months, whether intentional or unintentional, is a critical indicator for further assessment.

Which patient is at highest risk for developing metabolic syndrome? a. A 62-year-old white man who has coronary artery disease with chronic stable angina b. A 54-year-old Hispanic woman who is sedentary and has nephrogenic diabetes insipidus c. A 27-year-old Asian American woman who has preeclampsia and gestational diabetes mellitus d. A 38-year-old Native American man who has diabetes mellitus and elevated hemoglobin A1C

Correct answer: d Rationale: African Americans, Hispanics, Native Americans, and Asians are at an increased risk for development of metabolic syndrome. Other risk factors include individuals who have diabetes that cannot maintain a normal glucose level, have hypertension, and secrete a large amount of insulin, or who have survived a heart attack and have hyperinsulinemia.

In the immediate postoperative period a nurse cares for a severely obese 72-year-old man who had surgery for repair of a lower leg fracture. Which assessment would be most important for the nurse to make? a. Cardiac rhythm b. Surgical dressing c. Postoperative pain d. Oxygen saturation

Correct answer: d Rationale: After surgery an older and/or severely obese patient should be closely monitored for oxygen desaturation. The body stores anesthetics in adipose tissue, placing patients with excess adipose tissue (e.g., obesity, older) at risk for resedation. As adipose cells release anesthetic back into the bloodstream, the patient may become sedated after surgery. This may depress the respiratory rate and result in a drop in oxygen saturation.

To evaluate the effect of nutritional interventions for a patient with protein-calorie malnutrition, what is the best indicator for the nurse to use? a. Height and weight b. Body mass index (BMI) c. Weight in relation to ideal body weight d. Mid-upper arm circumference and triceps skinfold

Correct answer: d Rationale: Anthropometric measurements, including mid-upper arm circumference and triceps skinfold measurements, are good indicators of lean body mass and skeletal protein reserves and are valuable in evaluating persons who may have been or are being treated for acute protein malnutrition. The other measurements do not specifically address muscle mass.

At the first visit to the clinic, the female patient with a BMI of 29 kg/m2 tells the nurse that she does not want to become obese. Which question used for assessing weight issues is the most important question for the nurse to ask? a. "What factors contributed to your current body weight?" b. "How is your overall health affected by your body weight?" c. "What is your history of gaining weight and losing weight?" d. "In what ways are you interested in managing your weight differently?"

Correct answer: d Rationale: Asking the patient about her desire to manage her weight in a different manner helps the nurse determine the patient's readiness for learning, degree of motivation, and willingness to change lifestyle habits. The nurse can help the patient set realistic goals. This question will also lead to discussing the patient's history of gaining and losing weight and factors that have contributed to the patient's current weight. The patient may be unaware of the overall health effects of her body weight, so this question is not helpful at this time.

A severely obese patient has undergone Roux-en-Y gastric bypass surgery. In planning postoperative care, the nurse anticipates that the patient a. may have severe diarrhea early in the postoperative period. b. will not be allowed to ambulate for 1 to 2 days postoperatively. c. will require nasogastric suction until the incision heals. d. may have only liquids orally, and in very limited amounts, during the postoperative period.

Correct answer: d Rationale: During the immediate postoperative period, water and sugar-free clear liquids are given (30 mL every 2 hours while the patient is awake).

What is the most common cause of secondary protein-calorie malnutrition in the United States? a. The unavailability of foods high in protein b. A lack of knowledge about nutritional needs c. A lack of money to purchase high-protien foods d. An alteration in ingestion, digestion, absorption, or metabolism

Correct answer: d Rationale: In the United States, where rote in intake is high and of good quality, protein-calorie malnutrition most commonly results from problems of the GI system. In developing countries, adequate food sources might not exist, the inhabitants may not be well educated about nutritional needs, and economic conditions can prevent purchase of balanced diets.

The obesity classification that is most often associated with cardiovascular health problems is a. primary obesity. b. secondary obesity. c. gynoid fat distribution. d. android fat distribution.

Correct answer: d Rationale: Individuals with fat located primarily in the abdominal area (i.e., whose body is apple-shaped) are at greater risk for obesity-related complications (e.g., heart disease) than are those whose fat is primarily located in the upper legs (i.e., whose body is pear-shaped). Individuals whose fat is distributed over the abdomen and upper body (i.e., neck, arms, and shoulders) are classified as having android obesity.

A patient received a small-bore nasogastric (NG) tube after a laryngectomy. What should be the nurse's priority intervention before starting the enteral feeding? a. Aspiration b. Auscultation of air c. Set head of bed at 40 degrees. d. Verify NG tube placement on x-ray.

Correct answer: d Rationale: It is imperative to ensure that an NG tube is situated in the GI tract rather than the patient's lungs. When an NG tube has been recently inserted, it is important to confirm this placement with an x-ray that will identify the tube's radiopaque tip. Aspiration and air auscultation may not differentiate between gastric and respiratory placement of the tube. The head of bed elevated at least 30 degrees is to prevent aspiration. To determine the maintenance of the feeding tube's proper position, the exit site of the tube is marked at the time of the x-ray and the external portion measured to allow for assessment of a change position with a change in the length of the tube.

Which patient has the highest risk for poor nutritional balance related to decreased ingestion? a. Tuberculosis infection b. Malabsorption syndrome c. Draining decubitus ulcers d. Severe anorexia resulting from radiation therapy

Correct answer: d Rationale: Malnutrition that results form a deceased intake of food is most common in individuals with severe anorexia where there is a decreased desire to eat. Infections created a hypermetabolic state that increases nutritional demand, malabsorption causes loss of nutrients that are ingested, and draining decubitus ulcers are examples of disorders that cause both loss of protein and hypermetabolic states.

The nurse has completed initial instruction with a patient regarding a weight-loss program. Which patient comment indicates to the nurse that the teaching has been effective? a. "I will keep a diary of daily weight to illustrate my weight loss." b. "I plan to lose 4 lb a week until I have lost the 60 lb I want to lose." c. "I should not exercise more than what is required so I don't increase my appetite." d. "I plan to join a behavior modification group to help establish long-term behavior changes."

Correct answer: d Rationale: People who have undergone behavior therapy are more successful in maintaining weight losses over time because most programs deemphasize the diet, focus on how and when the person eats and education, and provide support from others. Weighing daily is not recommended and plateaus may not allow for consistent weight loss. A goal for weight loss must be set and 1 to 2 pounds a week is realistic. A more rapid loss often causes skin and underlying tissue to lose elasticity and become flabby folds of tissue. Exercising more often depresses appetite and exercise need not be limited.

The nurse is reviewing the laboratory test results for a 71-year-old patient with metastatic lung cancer. The patient was admitted with a diagnosis of malnutrition. The serum albumin level is 4.0 g/dL, and prealbumin is 10 mg/dL. What should this indicate to the nurse? a. The albumin level is normal, and therefore the patient does not have protein malnutrition. b. The albumin level is increased, which is a common finding in patients with cancer who have malnutrition. c. Both the serum albumin and prealbumin levels are reduced, consistent with the admitting diagnosis of malnutrition. d. Although the serum albumin level is normal, the prealbumin level more accurately reflects the patient's nutritional status.

Correct answer: d Rationale: Prealbumin has a half-life of 2 days and is a better indicator of recent or current nutritional status. Serum albumin has a half-life of approximately 20 to 22 days. The serum level may lag behind actual protein changes by more than 2 weeks and is therefore not a good indicator of acute changes in nutritional status.

A patient with anorexia nervosa shows signs of malnutrition. During initial referring, the nurse carefully assesses the patient for a. hyperkalemia. b. hypoglycemia. c. hypercalcemia. d. hypophosphatemia.

Correct answer: d Rationale: Refeeding syndrome is characterized by fluid retention, electrolyte imbalances (e.g., hypophosphatemia, hypokalemia, hypomagnesemia), and hyperglycemia. Conditions that predispose patients to refeeding syndrome include long-standing malnutrition states such as those induced by chronic alcoholism, vomiting and diarrhea, chemotherapy, and major surgery. Refeeding syndrome can occur any time a malnourished patient is started on aggressive nutritional support. Hypophosphatemia is the hallmark of refeeding syndrome, and it is associated with serious outcomes, including cardiac dysrhythmias, respiratory arrest, and neurologic disturbances (e.g., paresthesias).

The best nutritional therapy plan for a person who is obese is a. the Zone diet. b. the Atkins diet. c. Sugar Busters. d. foods from the basic food groups.

Correct answer: d Rationale: Restricted food intake is a cornerstone for any weight loss or maintenance program. A good weight loss plan should include foods from the basic food groups.

The severely obese patient has elected to have the Roux-en-Y gastric bypass (RYGB) procedure. The nurse will know the patient understands the preoperative teaching when the patient makes which statement? a. "This surgery will preserve the function of my stomach." b. "This surgery will remove the fat cells from my abdomen." c. "This surgery can be modified whenever I need it to be changed." d. "This surgery decreases how much I can eat and how many calories I can absorb."

Correct answer: d Rationale: The RYGB decreases the size of the stomach to a gastric pouch and attaches it directly to the small intestine so food bypasses 90% of the stomach, the duodenum, and a small segment of the jejunum. The vertical sleeve gastrectomy removes 85% of the stomach, but preserves the function of the stomach. Lipectomy and liposuction remove fat tissue from the abdomen or other areas. Adjustable gastric banding can be modified or reversed at a later date.

Which patient has the highest morbidity risk? a. Male 6 ft. 1 in. tall, BMI 29 kg/m2 b. Female 5 ft. 6 in. tall, weight 150 lb. c. Male with waist circumference 46 in. d. Female 5 ft. 10 in. tall, obesity Class III

Correct answer: d Rationale: The patient in Class III obesity has the highest risk for disease because Class III denotes severe obesity or a BMI greater than 40 kg/m2. The patient with waist circumference 46 in. has a high risk for disease, but without the BMI or obesity class, a more precise determination cannot be made. The female who is 5 ft. 6 in. tall has a normal weight for her height. The male patient who is over 6 ft. tall is overweight, which increases his risk of disease, but a more precise determination cannot be made without the waist circumference.

The patient being admitted has been diagnosed with anorexia nervosa. What clinical manifestations should the nurse expect to see on admission assessment? a. Tan skin, blonde hair, and diarrhea b. Sensitivity to heat, fatigue, and polycythemia c. Dysmenorrhea, gastric ulcer pain, and hunger d. Hair loss; dry, yellowish skin; and constipation

Correct answer: d Rationale: The patient with anorexia nervosa, along with abnormal weight loss, is likely to have hair loss; dry, yellow skin; constipation; sensitivity to cold, and absent or irregular menstruation. Other signs of malnutrition are also noted during physical examination.

Priority Decision: An 18-year-old female patient with anorexia nervosa is admitted to the hospital for treatment. On admission she weighs 82 lb (37 kg) and is 5 ft. 3 in (134.6 cm). Her laboratory test results include the following: K+ 2.8 mEq/L (2.8 mmol/L), Hgb 8.9 g/dL (89 g/L), and BUN 64 mg/dL (22.8 mmol/L). In planning care for the patient, the nurse gives the highest priority to which of the following nursing diagnoses? a. Risk for injury related to dizziness and weakness resulting from anemia b. Imbalanced nutrition: less than body requirements related to inadequate food intake c. Risk for impaired urinary elimination related to elevated BUN resulting from renal failure d. Risk for decreased cardiac output (CO) related to dysrhythmias resulting from hypokalemia

Correct answer: d Rationale: The potential life-threatening cardiac complications related to the hypokalemia are the most important immediate considerations in the patient's care. The other nursing diagnoses are important for the patient's care but do not pose immediate risk that the hypokalemia does.

What is a postoperative nursing intervention for the obese patient who has undergone bariatric surgery? a. Irrigating and repositioning the nasogastric (NG) tube as needed b. Delaying ambulation until the patient has enough strength to support self c. Keeping the patient positioned on the side to facilitate respiratory function d. Providing adequate support to the incision during coughing, deep breathing, and turning

Correct answer: d Rationale: Turning, coughing, and deep breathing are essential to prevent postoperative complications. Protecting the incision from strain is important since wound dehiscence is a problem for obese patients. If a nasogastric (NG) tube that is present following gastric surgery for severe obesity becomes blocked or needs repositioning, the health care provider should be notified. Ambulation is usually started on the evening of surgery and addition help will be needed to support the patient. Respiratory function is promoted by keeping the head of the bed elevated at an angle of 35 to 40 degrees.

A patient who is unable to swallow because of progressive amyotrophic lateral sclerosis is prescribed enteral nutrition through a newly placed gastrostomy tube. Which task is appropriate for the nurse to delegate to unlicensed assistive personnel (UAP)? a. Irrigate the tube between feedings. b. Provide wound care at the gastrostomy site. c. Administer prescribed liquid medications through the tube. d.Position the patient with a 45-degree head of bed elevation.

Correct answer: d Rationale: Unlicensed assistive personnel (UAP) may position the patient receiving enteral feedings with the head of bed elevated. A licensed practical nurse/licensed vocational nurse (LPN/LVN) or an RN could perform the other activities.

This bariatric surgical procedure involves creating a stoma and gastric pouch that is reversible, and no malabsorption occurs. What surgical procedure is this? a. Vertical gastric banding b. Biliopancreatic diversion c. Roux-en-Y gastric bypass d. Adjustable gastric banding

Correct answer: d Rationale: With adjustable gastric banding (AGB), the stomach size is limited by an inflatable band placed around the fundus of the stomach. The band is connected to a subcutaneous port and can be inflated or deflated to change the stoma size to meet the patient's needs as weight is lost. The procedure is performed laparoscopically and, if necessary, can be modified or reversed after the initial procedure.

"A client is admitted with a diagnosis of acute appendicitis. When assessing the abdomen, the nurse would expect to find rebound tenderness at which location? a) Left lower quadrant b) Left upper quadrant c) Right upper quadrant d) Right lower quadrant

Correct answer: d) Right lower quadrant" Rationale: The pain of acute appendicitis localizes in the right lower quadrant (RLQ) at McBurney's point, an area midway between the umbilicus and the right iliac crest. Often, the pain is worse when manual pressure near the region is suddenly released, a condition called rebound tenderness.

Wich of the following criteria must be met for a diagnosis of metabolic syndrome (select all that apply)? a. Hypertension b. Elevated triglycerides c. Elevated plasma glucose d. Increased waist circumference e. Decreased low-density lipoproteins

Correct answers: a, b, c, d Rationale: Three of the following five criteria must be met for a diagnosis of metabolic syndrome: • Waist circumference of 40 inches or more in men and 35 inches or more in women • Triglyceride levels higher than 150 mg/dL, or need for drug treatment for elevated triglyceride levels • High-density lipoprotein (HDL) cholesterol levels lower than 40 mg/dL in men and lower than 50 mg/dL in women, or need for drug treatment for reduced HDL cholesterol levels • Blood pressure: 130 mm Hg or higher systolic or 85 mm Hg or higher diastolic, or need for drug treatment for hypertension • Fasting blood glucose level of 110 mg/dL or higher, or need for drug treatment for elevated glucose levels

Normally, which hormones and peptides affect appetite (select all that apply)? a. Leptin b. Insulin c. Ghrelin d. Peptide YY e. Neuropeptide Y f. Cholecystokinin

Correct answers: a, b, c, d, e, f Rationale: Normally ghrelin and neuropeptide Y stimulate appetite. Leptin suppresses appetite and hunger. Insulin decreases appetite. Peptide YY and cholecystokinin inhibit appetite by slowing gastric emptying and sending satiety signals to the hypothalamus.

Priority Decision: The nurse is teaching a moderately obese woman interventions for the management of obesity. Initially, which strategies will support restricting dietary intake to below energy requirements (select all that apply)? a. Limit alcohol b. Rest when fatigued c. Determine portion sizes d. 1800- to 2200-calorie diet e. Attend Overeaters Anonymous

Correct answers: a, c Rationale: To restrict dietary intake so that it is below energy requirements, the moderately obese woman should limit or avoid alcohol intake because it increases caloric intake and has low nutritional value. Portion sizes have increased over the years and are larger than they should be. Teach the patient to determine portion sizes by weight or learn equivalencies such as that a serving of fruit is the size of a baseball. A progressive exercise program will increase energy requirements and a diet with an initial 800- to 1200-calorie limit would decrease calorie intake. Overeaters Anonymous would not restrict dietary intake below energy requirements, although it may offer support for the patient.

Health risks associated with obesity include (select all that apply) a. colorectal cancer. b. rheumatoid arthritis. c. polycystic ovary syndrome. d. nonalcoholic steatohepatitis. e. systemic lupus erythematosus.

Correct answers: a, c, d Rationale: Health risks associated with obesity include cardiovascular disease (related to increased low-density lipoprotein levels, increased triglyceride levels, and decreased high-density lipoprotein levels), hypertension, sleep apnea, obesity hypoventilation syndrome, reduced chest wall compliance, increased work of breathing, decreased total lung capacity and functional residual capacity, type 2 diabetes mellitus (i.e., hyperinsulinemia and insulin resistance), osteoarthritis, hyperuricemia, gout, gastroesophageal reflux disease, gallstones, nonalcoholic steatohepatitis, fatty liver and cirrhosis, cancer (mainly breast, endometrial, kidney, colorectal, pancreas, esophagus, and gallbladder), psychosocial problems (employment, education, and health care), low self-esteem, withdrawal from social interactions, and major depression.

Which teaching points are important when providing information to a patient with metabolic syndrome (select all that apply)? a. Stop smoking. b. Monitor weight daily. c. Increase level of activity. d. Decrease saturated fat intake. e. Reduce weight and maintain lower weight. f. Check blood glucose each morning prior to eating.

Correct answers: a, c, d, e Rationale: Patients with metabolic syndrome need to lower their risk factors by reducing and maintaining weight, increasing physical activity, establishing healthy diet habits, and smoking cessation. Some patients with metabolic syndrome are diabetic and would need to monitor glucose levels frequently. When monitoring weight reduction, it is recommended to check weight weekly, not daily.

A community health nurse is conducting an initial assessment of a new patient. Which assessments should the nurse include when screening the patient for metabolic syndrome (select all that apply)? a. Blood pressure b. Resting heart rate c. Physical endurance d. Waist circumference e. Fasting blood glucose

Correct answers: a, d, e Rationale: The diagnostic criteria for metabolic syndrome include elevated blood pressure, fasting blood glucose, waist circumference, triglycerides, and HDL cholesterol. Resting heart rate and physical endurance are not part of the diagnostic criteria.

What characteristics describe adjustable gastric banding (select all that apply)? a. 85% of the stomach is removed. b. Stomach restriction can be reversed. c. Eliminates hormones that stimulate hunger. d. Malabsorption of fat-soluble vitamins occurs. e. Inflatable band allows for modification of gastric stoma size. f. Stomach with a gastric pouch surgically anastomosed to the jejunum.

Correct answers: b, e Rationale: The adjustable gastric banding procedure is reversible and allows a change in gastric stoma size by inflation or deflation of the band around the funds of the stomach. The vertical sleeve gastrectomy removes 85% of the stomach and eliminates the hormones produced in the stomach that stimulate hunger. The biliopancreatic diversion is a maladaptive surgery that prevents absorption of nutrients, including fat-soluble vitamins. The Roux-en-Y gastric bypass reduces the stomach size with a gastric pouch anastomosed to the small intestine, so it is both restrictive and malabsorptive.

In preparing to care for the obese patient with cancer, what physiologic problems is this patient at a greater risk for having (select all that apply)? a. Tinnitus b. Fractures c. Sleep apnea d. Trousseau's sign e. Type 2 diabetes mellitus f. Gastroesophageal reflux disease (GERD)

Correct answers: c, e, f Rationale: Obese patients are at a higher risk for cancer, sleep apnea and sleep deprivation, type 2 diabetes mellitus, gastroesophageal reflux disease (GERD), nonalcoholic steatohepatits, osteoarthritis, and cardiovascular problems. The other options are not related to obesity.

The nurse is caring for the following clients on a surgical unit. Which client would the nurse assess first? 1.The client who had an inguinal hernia repair and has not voided in four (4) hours. 2.The client who was admitted with abdominal pain who suddenly has no pain. 3.The client four (4) hours postoperative abdominal surgery with no bowel sounds. 4.The client who is one (1) day postoperative appendectomy who is being discharged"

Correct: 2 "1. A client who has not voided within four (4)hours after any surgery would not be priority. This is an acceptable occurrence, but if the client hasn't voided for eight (8) hours, then the nurse would assess further. 2.This could indicate a ruptured appendix, which could lead to peritonitis, a life-threatening complication; therefore, thenurse should assess this client first. 3.Bowel sounds should return within 24 hoursafter abdominal surgery. Absent bowel soundsat four (4) hours postoperative would not beof great concern to the nurse 4.The client being discharged would be stableand not a priority for the nurse"

The nurse is admitting a client with acute appendicitis to the emergency department. The client has abdominal pain of 10 on a pain scale of 1 to 10. The client will be going to surgery as soon as possible. The nurse should: "1. Contact the surgeon to request an order for a narcotic for the pain. 2. Maintain the client in a recumbent position. 3. Place the client on nothing-by-mouth (NPO) status. 4. Apply heat to the abdomen in the area of the pain."

Correct: 3 The nurse should place the client on NPO status in anticipation of surgery. The nurse can initiate pain relief strategies, such as relaxation techniques, but the surgeon will likely not order narcotic medication prior to surgery. The nurse can place the client in a position that is most comfortable for the client. Heat is contraindicated because it may lead to perforation of the appendix

"The nurse is admitting a client with acute appendicitis to the emergency department. The client has abdominal pain of 10 on a pain scale of 1 to 10. The client will be going to surgery as soon as possible. The nurse should: "1. Contact the surgeon to request an order for a narcotic for the pain. 2. Maintain the client in a recumbent position. 3. Place the client on nothing-by-mouth (NPO) status. 4. Apply heat to the abdomen in the area of the pain."

Correct: 3 - no rationale

"A client has an appendectomy and develops peritonitis. The nurse should asses the client for an elevated temperature and which additional clinical indication commonly associated with peritonitis? "1. hyperactivity 2. extreme hunger 3. urinary retention 4. local muscular rigidity

Correct: 4 muscular rigidity over the affected area is a classic sign of peritonitis

"A client is admitted with a diagnosis of acute appendicitis. When assessing the abdomen, the nurse would expect to find rebound tenderness at which location? "A) Left lower quadrant B) Left upper quadrant C) Right upper quadrant D) Right lower quadrant"

Correct: 4 - no rationale

"When preparing a male client, age 51, for surgery to treat appendicitis, the nurse formulates a nursing diagnosis of Risk for infection related to inflammation, perforation, and surgery. What is the rationale for choosing this nursing diagnosis? "a. Obstruction of the appendix may increase venous drainage and cause the appendix to rupture. b. Obstruction of the appendix reduces arterial flow, leading to ischemia, inflammation, and rupture of the appendix. c. The appendix may develop gangrene and rupture, especially in a middle-aged client. d. Infection of the appendix diminishes necrotic arterial blood flow and increases venous drainage."

Correct: B - no rationale

A client has an appendectomy. This is an example of what kind of surgery? a. Diagnostic b. palliative c. ablative d. constructive

Correct: C Appendectomy is an example of ablative surgery. Diagnostic confirms or establishes a diagnosis, palliative relieves or reduces pain, and constructive restores function or appearance.

"The health care team is assessing a patient for acute pancreatitis after he presented to the emergency department with severe abdominal pain. Which laboratory value is the best diagnostic indicator of acute pancreatitis? A. Gastric pH B. Blood glucose C. Serum amylase D. Serum potassium

Correct: C Serum amylase levels indicate pancreatic function, and they are used to diagnose acute pancreatitis. Blood glucose, gastric pH, and potassium levels are not direct indicators of acute pancreatic dysfunction.

Which client requires immediate nursing intervention? "The client who: a) complains of epigastric pain after eating. b) complains of anorexia and periumbilical pain. c) presents with ribbonlike stools. d) presents with a rigid, boardlike abdomen.

Correct: D A rigid, boardlike abdomen is a sign of peritonitis, a possibly life-threatening condition. Epigastric pain occurring 90 minutes to 3 hours after eating indicates a duodenal ulcer. Anorexia and periumbilical pain are characteristic of appendicitis. Risk of rupture is minimal within the first 24 hours, but increases significantly after 48 hours. A client with a large-bowel obstruction may have ribbonlike stools.

A client admitted to the hospital with a diagnosis of cirrhosis hasmassive ascites and has difficulty breathing. A nurse performs whichintervention as a priority measure to assist the client with breathing? A. auscultates the lung fields every 4 hours B. repositions side to side every 2 hours C. encourages deep breathing exercises every 2 hours D. elevates the head of the bed 60 degrees

D

A client with acute liver failure exhibits confusion, a declining level of consciousness, and slowed respirations. The nurse finds him very difficult to arouse. The diagnostic information which best explains the client's behavior is: a) elevated liver enzymes and low serum protein level. b) subnormal clotting factors and platelet count. c) elevated blood urea nitrogen and creatinine levels and hyperglycemia. d) subnormal serum glucose and elevated serum ammonia levels.

D

A patient has been admitted to the medical unit after several days of watery diarrhea related to Crohn's disease. The healthcare provider recognizes which of the following symptoms as most concerning? Please choose from one of the following options. A. Right upper quadrant pain B. Elevated hematocrit C. Elevated leukocytes D. Palpitations

D

A patient who has been vomiting for several dasy from an unknown cause is admitted to hospital. the nurse anticipates collaborative care to indlude: a. oral admin of broth and tea b. admin of paretneral antiemetics c. insertion of NG tube to suction d. IV replacement of fluid and electrolytes

D

A pt. with advanced cirrhosis with ascites is short of breath and has an increased resp. rate. The nurse should: A. notify HCP so a paracentesis can be performed. B. initiate O2 therapy at 2 L/min to increase gas exchange C. ask pt. to cough and deep breath to clear resp. secretions D. place pt in fowler's position to relieve pressure on diaphragm

D

During an acute exacerbation of inflammatory bowel disease, a patient is to receive total parenteral nutrition (TPN) and lipids. Which of these interventions is the priority when caring for this patient? Please choose from one of the following options. A. Monitor urine specific gravity every shift B. Change the administration set every 72 hours C. Infuse the solution in a large peripheral vein D. Monitor the patient's blood glucose per protocol

D

Early screening for detection of cancers of the right side of colon in individuals over 50 yrs old should be done q year to include: a. serum CEA levels b. flexible sigmoidoscopy c. digital rectal exam d. fecal testing for occult blood

D

Following a Billroth 2 procedure, the patient develops dumping syndrome. The nurse explains that the symptoms associated wti h this problem are caused by: a. distention of smaller stomach by too much food intake b. hyperglycemia caused by uncontrolled gastric emptying into small intestine c. irritation of stomach lining by reflux of bile salts d. movement of fluid into the bowel because concentrated food and fluids move rapidly into the intesting

D

Foods usually omitted from diet of Kim with cirrhosis of liver are: A. whole grain cereals B. milk products C. cereal products D. rich gravies and sauces

D

One of your patients is receiving digitalis orally and is also to receive an antacid at the same time. Your most appropriate action, based on the pharmacokinetics of antacids, is to: A. Delay the digitalis for 1 to 2 hours until the antacid is absorbed B. Give the antacid at least 2 to 4 hours before administering the digitalis C. Administer both medications as ordered and document in nurse's notes D. Contact the physician regarding the drug interaction and request a change in the time of dosing of the drugs

D

Patient with cancer of stomach undergoes total gastrecotmy with esophagojejunostomy. Postop the nurse teaches the patient to expect: a. rapid healing b. ability to return to normal dietary habits c. close follow up for development of ulcers d. lifelong intramuscular or intranasal admin of cobalamin

D

Propanolol (Inderal) is commonly prescribed for clients withhyperthyroidism to: A. block formation of the thyroid hormone B. decrease the vascularity of the thyroid gland C. inhibit peripheral conversion of T4 and T3 D. decrease CNS stimulation

D

The client with ascites is schedule for a paracentesis. The nurse isassisting the physician in performing the procedure. Which of thefollowing positions will the nurse assist the client to assume for this procedure? A. supine C. right side lying B. left side lying D. upright

D

Which of the following is not a contributor to a condition of ascites? A. Elevated levels of aldosterone B. Hypertension C. Low levels of albumin D. Elevated levels of angiotension I

D

A patient reports having dry mouth and asks for some liquid to drink. You reason that this symptom can most likely be attributed to a common adverse effect of which medication? A. Digoxin (Lanoxin) B. Cefotetan (Cefotan) C. Famotidine (Pepcid) D. Promethazine (Phenergan)

D Rationale A common adverse effect of promethazine, an antihistamine antiemetic agent, is dry mouth; another is blurred vision. Reference: 965

The patient has aspirin prescribed. You should notify the primary health care provider if the patient has which medical diagnosis? A. Cerebrovascular accident (stroke) B. Osteoarthritis C. Atrial fibrillation D. Peptic ulcer disease (PUD)

D Rationale Aspirin can have a direct irritating effect on the gastric mucosa. Aspirin it used as part of the treatment in the other options. Reference: 981

What is the description of esophageal diverticula? A. Tearing of the intimal layer from the medial layer of the tissue B. Erosion of the mucosa caused by gastric reflux C. Sliding of gastric tissue through the diaphragm D. Saclike outpouchings of one or more layers of the esophagus

D Rationale Esophageal diverticula are saclike outpouchings in one or more layers of the esophagus that most commonly occur in older adults. Torn layers describe dissection, erosion describes Barrett's esophagitis, and sliding tissue describes a hiatal hernia. Reference: 979

.You explain to the patient with gastroesophageal reflux disease that this disorder A. results in acid erosion and ulceration of the esophagus caused by the frequent vomiting. B. will require surgical wrapping or repair of the pyloric sphincter to control the symptoms. C. is the protrusion of a portion of the stomach into the esophagus through an opening in the diaphragm. D. often involves relaxation of the lower esophageal sphincter, allowing stomach contents to back up into the esophagus.

D Rationale GERD results when the defenses of the esophagus are overwhelmed by the reflux of acidic gastric contents into the lower esophagus. An incompetent LES is a common cause of gastric reflux. Reference: 971

You are reviewing the medication records of a patient with acute gastritis. Which medication on the record would you question? A. Digoxin (Lanoxin) B. Furosemide (Lasix) C. Lisinopril (Zestril) D. Indomethacin (Indocin)

D Rationale NSAIDs have a direct irritating effect on gastric mucosa. NSAIDs also inhibit the synthesis of prostaglandins, which protect the gastric mucosa, leaving the mucosa more susceptible to injury. The other drugs do not have this effect. Reference: 985

You are teaching the patient and family about possible causative factors for peptic ulcers. You explain that ulcer formation is A. caused by a stressful lifestyle and other acid-producing factors such as Helicobacter pylori. B. inherited within families and reinforced by bacterial spread of Staphylococcus aureus in childhood. C. promoted by factors that tend to cause oversecretion of acid, such as excess dietary fats, smoking, and H. pylori. D. promoted by a combination of possible factors that may result in erosion of the gastric mucosa, including certain drugs and alcohol.

D Rationale Peptic ulcers develop only in the presence of an acid environment. However, an excess of HCl may not be necessary for ulcer development. The back-diffusion of HCl into the gastric mucosa results in cellular destruction and inflammation. Histamine is released from the damaged mucosa, resulting in vasodilation and increased capillary permeability and further secretion of acid and pepsin. A variety of agents (certain infections, medications, and lifestyle factors) are known to damage the mucosal barrier. Helicobacter pylori can alter gastric secretion and produce tissue damage leading to peptic ulcer disease. The response to H. pylori is likely influenced by a variety of factors, including genetics, environment, and diet. Ulcerogenic drugs, such as aspirin and nonsteroidal antiinflammatory drugs (NSAIDs), inhibit synthesis of prostaglandins, increase gastric acid secretion, and reduce the integrity of the mucosal barrier. Patients on corticosteroids, anticoagulants, and selective serotonin reuptake inhibitors (fluoxetine [Prozac]) are also at increased risk of ulcer development. High alcohol intake is associated with acute mucosal lesions. Alcohol stimulates acid secretion. Coffee (caffeinated and decaffeinated) is a strong stimulant of gastric acid secretion. Psychologic distress, including stress and depression, can negatively influence the healing of ulcers after they have developed. Smoking also delays the ulcer healing. Infection with herpesvirus and cytomegalovirus in immunocompromised patients may also lead to gastric ulcers. Reference: 987-989

An optimal teaching plan for an outpatient with stomach cancer receiving radiation therapy should include information about A. cancer support groups, alopecia, and stomatitis. B. avitaminosis, ostomy care, and community resources. C. prosthetic devices, skin conductance, and grief counseling. D. wound and skin care, nutrition, drugs, and community resources.

D Rationale Radiation therapy is used as an adjuvant to surgery or as palliation for stomach cancer. Your role is to provide detailed instructions about the therapy and possible side effects, reassure the patient, and ensure completion of the designated number of treatments. You should start by assessing the patient's knowledge of radiation therapy. You should teach the patient about the skin care, the need for nutrition and fluid intake during therapy, and the appropriate use of antiemetic drugs. Reference: 1000-1001

What is important to teach a patient with gastroesophageal reflux disease (GERD)? A. Drink whole milk before going to bed. B. Lie down and rest 30 minutes after meals. C. Avoid spicy foods. D. Elevate head of the bed on 6-inch blocks.

D Rationale The head of the bed should be elevated to approximately 30 degrees (4- to 6-inch blocks) to prevent reflux through a weakened lower esophageal sphincter during the night. Milk (especially whole milk with higher fat) should be avoided at bedtime because it increases gastric acid secretion. To prevent reflux, patients should not be supine for 2-3 hours after eating. Spicy foods are not particularly related to GERD. The patient should avoid foods known to cause reflux. These include foods that decrease lower esophageal sphincter (LES) pressure such as fatty foods, chocolate, peppermint, tomatoes, coffee, tea, and foods that irritate the esophagus, such as tomato-based products, orange juice, cola, and red wine. Reference: 974

The patient reports having a white patch in the mouth that easily bleeds and will not heal. What teaching is most important for you to provide? A. Stop smoking. B. Gargle with salt water. C. Avoid alcohol. D. See a health care provider.

D Rationale The patient has common manifestations of oral cancer, which needs to be evaluated and potentially treated. Although smoking and excessive alcohol are risk factors for oral cancer, diagnosis and treatment are more important at this point. Salt-water gargles are a common intervention for local throat infection but do not help cancer. Reference: 969

You explain to the patient with Vincent's infection that treatment will include A. smallpox vaccinations. B. viscous lidocaine rinses. C. Amphotericin B suspension. D. topical application of antibiotics.

D Rationale Vincent's infection is treated with topical applications of antibiotics. Other treatments include rest (physical and mental); avoidance of smoking and alcoholic beverages; a soft, nutritious diet; correct oral hygiene habits; and mouth irrigations with hydrogen peroxide and saline solutions. Reference: 968

If a client had irritable bowel syndrome, which of the following diagnostic tests would determine if the diagnosis is Crohn's disease or ulcerative colitis? A. Abdominal computed tomography (CT) scan B. Abdominal x-ray C. Barium swallow D. Colonoscopy with biopsy

D A colonoscopy with biopsy can be performed to determine the state of the colon's mucosal layers, presence of ulcerations, and level of cytologic development. An abdominal x-ray or CT scan wouldn't provide the cytologic information necessary to diagnose which disease it is. A barium swallow doesn't involve the intestine.

Which of the following conditions can cause a hiatal hernia? A. Increased intrathoracic pressure B. Weakness of the esophageal muscle C. Increased esophageal muscle pressure D. Weakness of the diaphragmic muscle

D A hiatal hernia is caused by weakness of the diaphragmic muscle and increased intra-abdominal—not intrathoracic—pressure. This weakness allows the stomach to slide into the esophagus. The esophageal supports weaken, but esophageal muscle weakness or increased esophageal muscle pressure isn't a factor in hiatal hernia.

A client with a peptic ulcer is scheduled for a vagotomy. The client asks the nurse about the purpose of this procedure. The nurse tells the client that the procedure: A. Decreases food absorption in the stomach B. Heals the gastric mucosa C. Halts stress reactions D. Reduces the stimulus to acid secretions

D A vagotomy, or cutting the vagus nerve, is done to eliminate parasympathetic stimulation of gastric secretion.

Bismuth subsalicylate (Pepto-Bismol), as an absorbent, has which of the following mechanisms of action? A. Decreased GI motility B. Decreased gastric secretions C. Increased fluid absorption D. Binding to diarrhea-causing bacteria for excretion

D Absorbent antidiarrheal medications bind to diarrhea-causing bacteria to form a nonabsorbable complex, which is then excreted in the stool.

Which clinical manifestation would the nurse expect a client diagnosed with acute cholecystitis to exhibit? A. Jaundice, dark urine, and steatorrhea B. Acute right lower quadrant (RLQ) pain, diarrhea, and dehydration C. Ecchymosis petechiae, and coffee-ground emesis D. Nausea, vomiting, and anorexia

D Acute cholecystitis is an acute inflammation of the gallbladder commonly manifested by the following: anorexia, nausea, and vomiting; biliary colic; tenderness and rigidity the right upper quadrant (RUQ) elicited on palpation (e.g., Murphy's sign); fever; fat intolerance; and signs and symptoms of jaundice. Ecchymosis, petechiae, and coffee-ground emesis are clinical manifestations of esophageal bleeding. The coffee-ground appearance indicates old bleeding. Jaundice, dark urine, and steatorrhea are clinical manifestations of the icteric phase of hepatitis.

Which rationale supports explaining the placement of an esophageal tamponade tube in a client who is hemorrhaging? A. Allowing the client to help insert the tube B. Beginning teaching for home care C. Maintaining the client's level of anxiety and alertness D. Obtaining cooperation and reducing fear

D An esophageal tamponade tube would be inserted in critical situations. Typically, the client is fearful and highly anxious. The nurse therefore explains about the placement to help obtain the client's cooperation and reduce his fear. This type of tube is used only short term and is not indicated for home use. The tube is large and uncomfortable. The client would not be helping to insert the tube. A client's anxiety should be decreased, not maintained, and depending on the degree of hemorrhage, the client may not be alert.

A client is taking an antacid for treatment of a peptic ulcer. Which of the following statements best indicates that the client understands how to correctly take the antacid? A. "I should take my antacid before I take my other medications." B. "I need to decrease my intake of fluids so that I don't dilute the effects of my antacid." C. "My antacid will be most effective if I take it whenever I experience stomach pains." D. "It is best for me to take my antacid 1 to 3 hours after meals."

D Antacids are most effective if taken 1 to 3 hours after meals and at bedtime. When an antacid is taken on an empty stomach, the duration of the drug's action is greatly decreased. Taking antacids 1 to 3 hours after a meal lengthens the duration of action, thus increasing the therapeutic action of the drug. Antacids should be administered about 2 hours after other medications to decrease the chance of drug interactions. It is not necessary to decrease fluid intake when taking antacids.

Which of the following laboratory results would be expected in a client with peritonitis? A. Partial thromboplastin time above 100 seconds B. Hemoglobin level below 10 mg/dL C. Potassium level above 5.5 mEq/L D. White blood cell count above 15,000

D Because of infection, the client's WBC count will be elevated. A hemoglobin level below 10 mg/dl may occur from hemorrhage. A PT time longer than 100 seconds may suggest disseminated intravascular coagulation, a serious complication of septic shock. A potassium level above 5.5 mEq/L may indicate renal failure.

Colon cancer is most closely associated with which of the following conditions? A. Appendicitis B. Hemorrhoids C. Hiatal hernia D. Ulcerative colitis

D Chronic ulcerative colitis, granulomas, and familial polposis seem to increase a person's chance of developing colon cancer. The other conditions listed have no known effect on colon cancer risk.

The client with GERD complains of a chronic cough. The nurse understands that in a client with GERD this symptom may be indicative of which of the following conditions? A. Development of laryngeal cancer B. Irritation of the esophagus C. Esophageal scar tissue formation D. Aspiration of gastric contents

D Clients with GERD can develop pulmonary symptoms such as coughing, wheezing, and dyspnea that are caused by the aspiration of gastric contents. GERD does not predispose the client to the development of laryngeal cancer. Irritation of the esophagus and esophageal scar tissue formation can develop as a result of GERD. However, GERD is more likely to cause painful and difficult swallowing.

A client being treated for chronic cholecystitis should be given which of the following instructions? A. Increase rest B. Avoid antacids C. Increase protein in diet D. Use anticholinergics as prescribed

D Conservative therapy for chronic cholecystitis includes weight reduction by increasing physical activity, a low-fat diet, antacid use to treat dyspepsia, and anticholinergic use to relax smooth muscles and reduce ductal tone and spasm, thereby reducing pain.

Crohn's disease can be described as a chronic relapsing disease. Which of the following areas in the GI system may be involved with this disease? A. The entire length of the large colon B. Only the sigmoid area C. The entire large colon through the layers of mucosa and submucosa D. The small intestine and colon; affecting the entire thickness of the bowel

D Crohn's disease can involve any segment of the small intestine, the colon, or both, affecting the entire thickness of the bowel. Answers 1 and 3 describe ulcerative colitis, answer 2 is too specific and therefore, not likely.

A nurse is inserting a nasogastric tube in an adult male client. During the procedure, the client begins to cough and has difficulty breathing. Which of the following is the appropriate nursing action? A. Quickly insert the tube B. Notify the physician immediately C. Remove the tube and reinsert when the respiratory distress subsides D. Pull back on the tube and wait until the respiratory distress subsides

D During the insertion of a nasogastric tube, if the client experiences difficulty breathing or any respiratory distress, withdraw the tube slightly, stop the tube advancement, and wait until the distress subsides. Options B and C are unnecessary. Quickly inserting the tube is not an appropriate action because, in this situation, it may be likely that the tube has entered the bronchus.

When teaching an elderly client how to prevent constipation, which of the following instructions should the nurse include? A. "Drink 6 glasses of fluid each day." B. "Avoid grain products and nuts." C. "Add at least 4 grams of bran to your cereal each morning." D. "Be sure to get regular exercise."

D Exercise helps prevent constipation. Fluids and dietary fiber promote normal bowel function. The client should drink eight to ten glasses of fluid each day. Although adding bran to cereal helps prevent constipation by increasing dietary fiber, the client should start with a small amount and gradually increase the amount as tolerated to a maximum of 2 grams a day.

A client presents to the emergency room, reporting that he has been vomiting every 30 to 40 minutes for the past 8 hours. Frequent vomiting puts him at risk for which of the following? A. Metabolic acidosis with hyperkalemia B. Metabolic acidosis with hypokalemia C. Metabolic alkalosis with hyperkalemia D. Metabolic alkalosis with hypokalemia

D Gastric acid contains large amounts of potassium, chloride, and hydrogen ions. Excessive loss of these substances, such as from vomiting, can lead to metabolic alkalosis and hypokalemia.

The nurse would question an order for which type of antacid in patients with chronic renal failure? A. Aluminum-containing antacids B. Calcium-containing antacids C. Magnesium-containing antacids D. All of the above

D Magnesium-containing antacids can cause hypermagnesemia in patients with chronic renal failure. Aluminum-containing antacids may be used as a phosphate binder in patients with chronic renal failure. Calcium-containing antacids are also appropriate because these patients may be hypocalcemic.

The nurse is reviewing the physician's orders written for a client admitted with acute pancreatitis. Which physician order would the nurse question if noted on the client's chart? A. NPO status B. Insert a nasogastric tube C. An anticholinergic medication D. Morphine for pain

D Meperidine (Demerol) rather than morphine is the medication of choice because morphine can cause spasm in the sphincter of Oddi.

When counseling a client in ways to prevent cholecystitis, which of the following guidelines is most important? A. Eat a low-protein diet B. Eat a low-fat, low-cholesterol diet C. Limit exercise to 10 minutes/day D. Keep weight proportionate to height

D Obesity is a known cause of gallstones, and maintaining a recommended weight will help protect against gallstones. Excessive dietary intake of cholesterol is associated with the development of gallstones in many people. Dietary protein isn't implicated in cholecystitis. Liquid protein and low-calorie diets (with rapid weight loss of more than 5 lb [2.3kg] per week) are implicated as the cause of some cases of cholecystitis. Regular exercise (30 minutes/three times a week) may help reduce weight and improve fat metabolism. Reducing stress may reduce bile production, which may also indirectly decrease the chances of developing cholecystitis.

The client with peptic ulcer disease is scheduled for a pyloroplasty. The client asks the nurse about the procedure. The nurse plans to respond knowing that a pyloroplasty involves: A. Cutting the vagus nerve B. Removing the distal portion of the stomach C. Removal of the ulcer and a large portion of the cells that produce hydrochloric acid D. An incision and resuturing of the pylorus to relax the muscle and enlarge the opening from the stomach to the duodenum

D Option 4 describes the procedure for a pyloroplasty. A vagotomy involves cutting the vagus nerve. A subtotal gastrectomy involves removing the distal portion of the stomach. A Billroth II procedure involves removal of the ulcer and a large portion of the tissue that produces hydrochloric acid.

The pain of a duodenal ulcer can be distinguished from that of a gastric ulcer by which of the following characteristics? A. Early satiety B. Pain on eating C. Dull upper epigastric pain D. Pain on empty stomach

D Pain on empty stomach is relieved by taking foods or antacids. The other symptoms are those of a gastric ulcer.

The nurse is monitoring a female client with a diagnosis of peptic ulcer. Which assessment findings would most likely indicate perforation of the ulcer? A. Bradycardia B. Numbness in the legs C. Nausea and vomiting D. A rigid, board-like abdomen

D Perforation of an ulcer is a surgical emergency and is characterized by sudden, sharp, intolerable severe pain beginning in the midepigastric area and spreading over the abdomen, which becomes rigid and board-like. Nausea and vomiting may occur. Tachycardia may occur as hypovolemic shock develops. Numbness in the legs is not an associated finding.

Surgical management of ulcerative colitis may be performed to treat which of the following complications? A. Gastritis B. Bowel herniation C. Bowel outpouching D. Bowel perforation

D Perforation, obstruction, hemorrhage, and toxic megacolon are common complications of ulcerative colitis that may require surgery. Herniation and gastritis aren't associated with irritable bowel diseases, and outpouching of the bowel is diverticulosis.

The nurse is caring for a hospitalized client with a diagnosis of ulcerative colitis. Which finding, if noted on assessment of the client, would the nurse report to the physician? A. Bloody diarrhea B. Hypotension C. A hemoglobin of 12 mg/dL D. Rebound tenderness

D Rebound tenderness may indicate peritonitis. Blood diarrhea is expected to occur in ulcerative colitis. Because of the blood loss, the client may be hypotensive and the hemoglobin level may be lower than normal. Signs of peritonitis must be reported to the physician.

Which of the following symptoms may be exhibited by a client with Crohn's disease? A. Bloody diarrhea B. Narrow stools C. N/V D. Steatorrhea

D Steatorrhea from malaborption can occur with Crohn's disease. N/V, and bloody diarrhea are symptoms of ulcerative colitis. Narrow stools are associated with diverticular disease.

Which area of the alimentary canal is the most common location for Crohn's disease? A. Ascending colon B. Descending colon C. Sigmoid colon D. Terminal ileum

D Studies have shown that the terminal ileum is the most common site for recurrence in clients with Crohn's disease. The other areas may be involved but aren't as common.

Which of the following diagnostic tests should be performed annually over age 50 to screen for colon cancer? A. Abdominal CT scan B. Abdominal x-ray C. Colonoscopy D. Fecal occult blood test

D Surface blood vessels of polyps and cancers are fragile and often bleed with the passage of stools. Abdominal x-ray and CT scan can help establish tumor size and metastasis. A colonoscopy can help locate a tumor as well as polyps, which can be removed before they become malignant.

Which of the following tests can be used to diagnose ulcers? A. Abdominal x-ray B. Barium swallow C. Computed tomography (CT) scan D. Esophagogastroduodenoscopy (EGD)

D The EGD can visualize the entire upper GI tract as well as allow for tissue specimens and electrocautery if needed. The barium swallow could locate a gastric ulcer. A CT scan and an abdominal x-ray aren't useful in the diagnosis of an ulcer.

A client is admitted to the hospital after vomiting bright red blood and is diagnosed with a bleeding duodenal ulcer. The client develops a sudden, sharp pain in the mid epigastric area along with a rigid, board-like abdomen. These clinical manifestations most likely indicate which of the following? A. An intestinal obstruction has developed B. Additional ulcers have developed C. The esophagus has become inflamed D. The ulcer has perforated

D The body reacts to perforation of an ulcer by immobilizing the area as much as possible. This results in boardlike muscle rigidity, usually with extreme pain. Perforation is a medical emergency requiring immediate surgical intervention because peritonitis develops quickly after perforation. An intestinal obstruction would not cause midepigastric pain. Esophageal inflammation or the development of additional ulcers would not cause a rigid, boardlike abdomen.

The client who has undergone creation of a colostomy has a nursing diagnosis of Disturbed body image. The nurse would evaluate that the client is making the most significant progress toward identified goals if the client: A. Watches the nurse empty the colostomy bag B. Looks at the ostomy site C. Reads the ostomy product literature D. Practices cutting the ostomy appliance

D The client is expected to have a body image disturbance after colostomy. The client progresses through normal grieving stages to adjust to this change. The client demonstrates the greatest deal of acceptance when the client participates in the actual colostomy care. Each of the incorrect options represents an interest in colostomy care but is a passive activity. The correct option shows the client is participating in self-care.

Which of the following tasks should be included in the immediate postoperative management of a client who has undergone gastric resection? A. Monitoring gastric pH to detect complications B. Assessing for bowel sounds C. Providing nutritional support D. Monitoring for symptoms of hemorrhage

D The client should be monitored closely for signs and symptoms of hemorrhage, such as bright red blood in the nasogastric tube suction, tachycardia, or a drop in blood pressure. Gastric pH may be monitored to evaluate the need for histamine-2 receptor antagonists. Bowel sounds may not return for up to 72 hours postoperatively. Nutritional needs should be addressed soon after surgery.

Which of the following therapies is not included in the medical management of a client with peritonitis? A. Broad-spectrum antibiotics B. Electrolyte replacement C. I.V. fluids D. Regular diet

D The client with peritonitis usually isn't allowed anything orally until the source of peritonitis is confirmed and treated. The client also requires broad-spectrum antibiotics to combat the infection. I.V. fluids are given to maintain hydration and hemodynamic stability and to replace electrolytes.

Jordin is a client with jaundice who is experiencing pruritus. Which nursing intervention would be included in the care plan for the client? A. Administering vitamin K subcutaneously B. Applying pressure when giving I.M. injections C. Decreasing the client's dietary protein intake D. Keeping the client's fingernails short and smooth

D The client with pruritus experiences itching, which may lead to skin breakdown and possibly infection from scratching. Keeping his fingernails short and smooth helps prevent skin breakdown and infection from scratching. Applying pressure when giving I.M. injections and administering vitamin K subcutaneously are important if the client develops bleeding problems. Decreasing the client's dietary intake is appropriate if the client's ammonia levels are increased.

Which of the following nursing interventions should be implemented to manage a client with appendicitis? A. Assessing for pain B. Encouraging oral intake of clear fluids C. Providing discharge teaching D. Assessing for symptoms of peritonitis

D The focus of care is to assess for peritonitis, or inflammation of the peritoneal cavity. Peritonitis is most commonly caused by appendix rupture and invasion of bacteria, which could be lethal. The client with appendicitis will have pain that should be controlled with analgesia. The nurse should discourage oral intake in preparation of surgery. Discharge teaching is important; however, in the acute phase, management should focus on minimizing preoperative complications and recognizing when such may be occurring.

Which of the following terms best describes the pain associated with appendicitis A. Aching B. Fleeting C. Intermittent D. Steady

D The pain begins in the epigastrium or periumbilical region, then shifts to the right lower quadrant and becomes steady. The pain may be moderate to severe.

Polyethylene glycol-electrolyte solution (GoLYTELY) is prescribed for the female client scheduled for a colonoscopy. The client begins to experience diarrhea following administration of the solution. What action by the nurse is appropriate? A. Start an IV infusion B. Administer an enema C. Cancel the diagnostic test D. Explain that diarrhea is expected

D The solution GoLYTELY is a bowel evacuant used to prepare a client for a colonoscopy by cleansing the bowel. The solution is expected to cause a mild diarrhea and will clear the bowel in 4 to 5 hours. Options A, B, and C are inappropriate actions

Which of the following associated disorders may a client with ulcerative colitis exhibit? A. Gallstones B. Hydronephrosis C. Nephrolithiasis D. Toxic megacolon

D Toxic megacolon is extreme dilation of a segment of the diseased colon caused by paralysis of the colon, resulting in complete obstruction. This disorder is associated with both Crohn's disease and ulcerative colitis. The other disorders are more commonly associated with Crohn's disease.

Which of the following mechanisms can facilitate the development of diverticulosis into diverticulitis? A. Treating constipation with chronic laxative use, leading to dependence on laxatives B. Chronic constipation causing an obstruction, reducing forward flow of intestinal contents C. Herniation of the intestinal mucosa, rupturing the wall of the intestine D. Undigested food blocking the diverticulum, predisposing the area to bacterial invasion

D Undigested food can block the diverticulum, decreasing blood supply to the area and predisposing the area to invasion of bacteria. Chronic laxative use is a common problem in elderly clients, but it doesn't cause diverticulitis. Chronic constipation can cause an obstruction—not diverticulitis. Herniation of the intestinal mucosa causes an intestinal perforation.

A nurse is preparing to care for a female client with esophageal varices who has just has a Sengstaken-Blakemore tube inserted. The nurse gathers supplies, knowing that which of the following items must be kept at the bedside at all times? A. An obturator B. .Kelly clamp C. An irrigation set D. A pair of scissors

D When the client has a Sengstaken-Blakemore tube, a pair of scissors must be kept at the client's bedside at all times. The client needs to be observed for sudden respiratory distress, which occurs if the gastric balloon ruptures and the entire tube moves upward. If this occurs, the nurse immediately cuts all balloon lumens and removes the tube. An obturator and a Kelly clamp are kept at the bedside of a client with a tracheostomy. An irrigation set may be kept at the bedside, but it is not the priority item.

Which assessment data support to the the nurse the client's diagnosis of gastric ulcer? A. Presence of blood in the client's stool for the past month? B. Reports of a burning sensation moving like a wave. C. Sharp pain in the upper abdomen after eating a heavy meal. D. Complaints of epigastric pain 30-60 minutes after ingesting food

D In a client diagnosed with a gastric ulcer, pain usually occurs 30-60 minutes after eating, but not at night. In contrast, a client with a duodenal ulcer has pain during the night often relieved by eating foods. Other answers: the presence of blood does not specifically indicate diagnose of an ulcer. The client could have hemorrhoids or cancer. A waveline burning sensation is a symptom of GERD. Sharp pain in the upper abdomen after eating a heavy meal is a symptom of gallbladder disease

When teaching the patient about the diet for diverticular disease, which foods should the nurse recommend? A) White bread, cheese, and green beans B) Fresh tomatoes, pears, and corn flakes C) Oranges, baked potatoes, and raw carrots D) Dried beans, All Bran (100%) cereal, and raspberries

D) A high fiber diet is recommended for diverticular disease. Dried beans, All Bran (100%) cereal, and raspberries all have higher amounts of fiber than white bread, cheese, green beans, fresh tomatoes, pears, corn flakes, oranges, baked potatoes, and raw carrots.

Which of the following is an outcome of histamine 2 (H2)-receptor antagonists blocking the action of histamine in the stomach? a) Blood phosphate levels are elevated. b) Symptoms of gastroesophageal reflux are relieved. c) Acid indigestion is relieved. d) Acid secretion is reduced.

D) Acid secretion is reduced H2-receptor antagonists decrease the amount of hydrochloric acid that the stomach produces by blocking the action of histamine on histamine receptors of potential cells in the stomach.

A client is evaluated for severe pain in the right upper abdominal quadrant, which is accompanied by nausea and vomiting. The physician diagnoses acute cholecystitis and cholelithiasis. For this client, which nursing diagnosis takes top priority? a) Imbalanced nutrition: Less than body requirements related to biliary inflammation b) Anxiety related to unknown outcome of hospitalization c) Deficient knowledge related to prevention of disease recurrence d) Acute pain related to biliary spasms

D) Acute pain related to biliary spasms The chief symptom of cholecystitis is abdominal pain or biliary colic. Typically, the pain is so severe that the client is restless and changes positions frequently to find relief. Therefore, the nursing diagnosis of Acute pain related to biliary spasms takes highest priority. Until the acute pain is relieved, the client can't learn about prevention, may continue to experience anxiety, and can't address nutritional concerns.

A nurse is teaching a group of middle-aged men about peptic ulcers. When discussing risk factors for peptic ulcers, the nurse should mention: a) alcohol abuse and a history of acute renal failure. b) a history of hemorrhoids and smoking. c) a sedentary lifestyle and smoking. d) alcohol abuse and smoking.

D) Alcochol abuse and smoking The nurse should mention that risk factors for peptic (gastric and duodenal) ulcers include alcohol abuse, smoking, and stress. A sedentary lifestyle and a history of hemorrhoids aren't risk factors for peptic ulcers. Chronic renal failure, not acute renal failure, is associated with duodenal ulcers.

The nurse who inserted a nasogastric tube for a 68-year-old patient with suspected bowel obstruction should write which of the following priority nursing diagnoses on the patient's problem list? A) Anxiety related to nasogastric tube placement B) Abdominal pain related to nasogastric tube placement C) Risk for deficient knowledge related to nasogastric tube placement D) Altered oral mucous membrane related to nasogastric tube placement

D) Altered oral mucous membrane related to nasogastric tube placement With nasogastric tube placement, the patient is likely to breathe through the mouth and may experience irritation in the affected nares. For this reason, the nurse should plan preventive measures based on this nursing diagnosis.

When planning care for a patient with cirrhosis, the nurse will give highest priority to which of the following nursing diagnoses? A. Imbalanced nutrition: less than body requirements B. Impaired skin integrity related to edema, ascites, and pruritus C. Excess fluid volume related to portal hypertension and hyperaldosteronism D. Ineffective breathing pattern related to pressure on diaphragm and reduced lung volume

D) Although all of these nursing diagnoses are appropriate and important in the care of a patient with cirrhosis, airway and breathing are always the highest priorities.

The nurse is performing a focused abdominal assessment of a patient who has been recently admitted. In order to palpate the patient's liver, where should the nurse palpate the patient's abdomen? A) Left lower quadrant B) Left upper quadrant C) Right lower quadrant D) Right upper quadrant

D) Although the left lobe of the liver is located in the left upper quadrant of the abdomen, the bulk of the liver is located in the right upper quadrant.

The family of a patient newly diagnosed with hepatitis A asks the nurse what they can do to prevent becoming ill themselves. Which of the following responses by the nurse is most appropriate? A) "The hepatitis vaccine will provide immunity from this exposure and future exposures." B) "I am afraid there is nothing you can do since the patient was infectious before admission." C) "You will need to be tested first to make sure you don't have the virus before we can treat you." D) "An injection of immunoglobulin will need to be given to prevent or minimize the effects from this exposure."

D) An injection of immunoglobulin will need to be given to prevent or minimize the effects from this exposure Immunoglobulin provides temporary (1-2 months) passive immunity and is effective for preventing hepatitis A if given within 2 weeks after exposure. It may not prevent infection in all persons, but it will at least modify the illness to a subclinical infection. The hepatitis vaccine is only used for preexposure prophylaxis.

Which of the following would a nurse expect to assess in a client with peritonitis? a) Decreased pulse rate b) Deep slow respirations c) Hyperactive bowel sounds d) Board-like abdomen

D) Board-like abdomen The client with peritonitis would typically exhibit a rigid, board-like abdomen, with absent bowel sounds, elevated pulse rate, and rapid, shallow respirations.

The nurse is preparing to administer a scheduled dose of docusate sodium (Colace) when the patient complains of an episode of loose stool and does not want to take the medication. Which of the following is the appropriate action by the nurse? A) Write an incident report about this untoward event. B) Attempt to have the family convince the patient to take the ordered dose. C) Withhold the medication at this time and try to administer it later in the day. D) Chart the dose as not given on the medical record and explain in the nursing progress notes.

D) Chart the dose as not given on the medical record and explain in the nursing progress notes. Whenever a patient refuses medication, the dose should be charted as not given. An explanation of the reason should then be documented in the nursing progress notes. In this instance, the refusal indicates good judgment by the patient.

What kind of feeding should be administered to a client who is at the risk of diarrhea due to hypertonic feeding solutions? a) Bolus feeding b) Intermittent feeding c) Cyclic feeding d) Continuous feedings

D) Continuous feedings. Continuous feedings should be administered to a client who is at the risk of diarrhea due to hypertonic feeding solutions.

The patient with a history of lung cancer and hepatitis C has developed liver failure and is considering liver transplantation. After the comprehensive evaluation, the nurse knows that which factor discovered may be a contraindication for liver transplantation? A) Has completed a college education B) Has been able to stop smoking cigarettes C) Has well-controlled type 1 diabetes mellitus D) The chest x-ray showed another lung cancer lesion.

D) Contraindications for liver transplant include severe extrahepatic disease, advanced hepatocellular carcinoma or other cancer, ongoing drug and/or alcohol abuse, and the inability to comprehend or comply with the rigorous post-transplant course.

To ensure patency of central venous line ports, diluted heparin flushes are used in which of the following situations? a) Before drawing blood b) With continuous infusions c) When the line is discontinued d) Daily when not in use

D) Daily when not in use Daily instillation of dilute heparin flush when a port is not in use will maintain the port. Continuous infusion maintains the patency of each port. Heparin flushes are used after each intermittent infusion. Heparin flushes are used after blood drawing in order to prevent clotting of blood within the port. Heparin flush of ports is not necessary if a line is to be discontinued.

The health care provider orders lactulose for a patient with hepatic encephalopathy. The nurse will monitor for effectiveness of this medication for this patient by assessing which of the following? A) Relief of constipation B) Relief of abdominal pain C) Decreased liver enzymes D) Decreased ammonia levels

D) Decreased ammonia levels Hepatic encephalopathy is a complication of liver disease and is associated with elevated serum ammonia levels. Lactulose traps ammonia in the intestinal tract. Its laxative effect then expels the ammonia from the colon, resulting in decreased serum ammonia levels and correction of hepatic encephalopathy.

The nurse asks a 68-year-old patient scheduled for colectomy to sign the operative permit as directed in the physician's preoperative orders. The patient states that the physician has not really explained well what is involved in the surgical procedure. Which of the following is the most appropriate action by the nurse? A) Ask family members whether they have discussed the surgical procedure with the physician. B) Have the patient sign the form and state the physician will visit to explain the procedure before surgery. C) Explain the planned surgical procedure as well as possible, and have the patient sign the consent form. D) Delay the patient's signature on the consent and notify the physician about the conversation with the patient.

D) Delay the patient's signature on the consent and notify the physician about the conversation with the patient. The patient should not be asked to sign a consent form unless the procedure has been explained to the satisfaction of the patient. The nurse should notify the physician, who has the responsibility for obtaining consent.

What should the nurse instruct the patient to do to best enhance the effectiveness of a daily dose of docusate sodium (Colace)? A) Take a dose of mineral oil at the same time. B) Add extra salt to food on at least one meal tray. C) Ensure dietary intake of 10 g of fiber each day. D) Take each dose with a full glass of water or other liquid.

D) Docusate lowers the surface tension of stool, permitting water and fats to penetrate and soften the stool for easier passage. The patient should take the dose with a full glass of water and should increase overall fluid intake, if able, to enhance effectiveness of the medication. Dietary fiber intake should be a minimum of 20 g daily to prevent constipation. Mineral oil and extra salt are not recommended.

The most common symptom of esophageal disease is a) nausea. b) odynophagia. c) vomiting. d) dysphagia.

D) Dysphagia This symptom may vary from an uncomfortable feeling that a bolus of food is caught in the upper esophagus to acute pain on swallowing. Nausea is the most common symptom of gastrointestinal problems in general. Vomiting is a nonspecific symptom that may have a variety of causes. Odynophagia refers specifically to acute pain on swallowing.

The nurse is caring for a 68-year-old patient admitted with abdominal pain, nausea, and vomiting. The patient has an abdominal mass, and a bowel obstruction is suspected. The nurse auscultating the abdomen listens for which type of bowel sounds that are consistent with the patient's clinical picture? A) Low-pitched and rumbling above the area of obstruction B) High-pitched and hypoactive below the area of obstruction C) Low-pitched and hyperactive below the area of obstruction D) High-pitched and hyperactive above the area of obstruction

D) Early in intestinal obstruction, the patient's bowel sounds are hyperactive and high-pitched, sometimes referred to as "tinkling" above the level of the obstruction. This occurs because peristaltic action increases to "push past" the area of obstruction. As the obstruction becomes complete, bowel sounds decrease and finally become absent.

The client is experiencing swallowing difficulties and is now scheduled to receive a gastric feeding. She has the following oral medications prescribed: furosemide (Lasix), digoxin, enteric coated aspirin (Ecotrin), and vitamin E. The nurse withholds a) furosemide b) digoxin c) vitamin E d) enteric coated aspirin

D) Enteric coated aspirin Simple compressed tablets (furosemide, digoxin) may be crushed and dissolved in water. Soft gelatin capsules filled with liquid (vitamin E) may be opened, and the contents squeezed out. Enteric coated tablets (enteric coated aspirin) are not to be crushed and a change in the form of the medications is required.

The nurse is caring for a postoperative patient with a colostomy. The nurse is preparing to administer a dose of famotidine (Pepcid) when the patient asks why the medication was ordered since the patient does not have a history of heartburn or gastroesophageal reflux disease (GERD). What response by the nurse would be the most appropriate? A) "This will prevent air from accumulating in the stomach, causing gas pains." B) "This will prevent the heartburn that occurs as a side effect of general anesthesia." C) "The stress of surgery is likely to cause stomach bleeding if you do not receive it." D) "This will reduce the amount of HCl in the stomach until the nasogastric tube is removed and you can eat a regular diet again."

D) Famotidine is an H2-receptor antagonist that inhibits gastric HCl secretion and thus minimizes damage to gastric mucosa while the patient is not eating a regular diet after surgery. Famotidine does not prevent air from accumulating in the stomach or stop the stomach from bleeding. Heartburn is not a side effect of general anesthesia.

What are medium-length nasoenteric tubes are used for? a) Aspiration b) Emptying c) Decompression d) Feeding

D) Feeding Placement of the tube must be verified prior to any feeding. A gastric sump and nasoenteric tube are used for gastrointestinal decompression. Nasoenteric tubes are used for feeding. Gastric sump tubes are used to decompress the stomach and keep it empty.

A client has a gastrointestinal tube that enters the stomach through a surgically created opening in the abdominal wall. The nurse documents this as which of the following? a) Jejunostomy tube b) Nasogastric tube c) Orogastric tube d) Gastrostomy tube

D) Gastrostomy tube A gastrostomy tube enters the stomach through a surgically created opening into the abdominal wall. A jejunostomy tube enters jejunum or small intestine through a surgically created opening into the abdominal wall. A nasogastric tube passes through the nose into the stomach via the esophagus. An orogastric tube passes through the mouth into the stomach.

A nurse is applying an ostomy appliance to the ileostomy of a client with ulcerative colitis. Which action is appropriate? a) Maintaining wrinkles in the faceplate so it doesn't irritate the skin b) Scrubbing fecal material from the skin surrounding the stoma c) Cutting the faceplate opening no more than 2? larger than the stoma d) Gently washing the area surrounding the stoma using a facecloth and mild soap

D) Gently washing the area surrounding the stoma using a facecloth and mild soap For a client with an ostomy, maintaining skin integrity is a priority. The nurse should gently wash the area surrounding the stoma using a facecloth and mild soap. Scrubbing the area around the stoma can damage the skin and cause bleeding. The faceplate opening should be no more than 1/8? to 1/6? larger than the stoma. This size protects the skin from exposure to irritating fecal material. The nurse can create an adequate seal and prevent leakage of fecal material from under the faceplate by applying a thin layer of skin barrier and smoothing out wrinkles in the faceplate. Eliminating wrinkles in the faceplate also protects the skin surrounding the stoma from pressure.

Which of the following is the major carbohydrate that tissue cells use as fuel? a) Proteins b) Fats c) Chyme d) Glucose

D) Glucose Glucose is the major carbohydrate that tissue cells use as fuel. Proteins are a source of energy after they are broken down into amino acids and peptides. Chyme stays in the small intestine for 3 to 6 hours, allowing for continued breakdown and absorption of nutrients. Ingested fats become monoglycerides and fatty acids by the process of emulsification.

Which of these agents is a major contributing factor in the promotion of peptic ulcer disorder? A) Candida albicans. B) staphyloccus infection. C) streptococcus infection D) Helibacter pylori infection

D) Helibacter pylori infection Helobacter pylori infection. Rationale: Recurrence of peptic ulcers is related to Helicobacter pylori, use of NSAIDs, smoking, and continued acid hypersecretion

Which type of jaundice seen in adults is the result of increased destruction of red blood cells? a) Obstructive b) Nonobstructive c) Hepatocellular d) Hemolytic

D) Hemolytic Hemolytic jaundice results because, although the liver is functioning normally, it cannot excrete the bilirubin as quickly as it is formed. Obstructive jaundice is the result of liver disease. Nonobstructive jaundice occurs with hepatitis. Hepatocellular jaundice is the result of liver disease.

The health care provider orders lactulose for a patient with hepatic encephalopathy. The nurse will monitor for effectiveness of this medication for this patient by assessing which of the following? A. Relief of constipation B. Relief of abdominal pain C. Decreased liver enzymes D. Decreased ammonia levels

D) Hepatic encephalopathy is a complication of liver disease and is associated with elevated serum ammonia levels. Lactulose traps ammonia in the intestinal tract. Its laxative effect then expels the ammonia from the colon, resulting in decreased serum ammonia levels and correction of hepatic encephalopathy.

When reviewing the history of a client with pancreatic cancer, the nurse would identify which of the following as a possible risk factor? a) Ingestion of a low-fat diet b) One-time exposure to petrochemicals c) Ingestion of caffeinated coffee d) History of pancreatitis

D) History of pancreatitis Pancreatitis is associated with the development of pancreatic cancer. Other factors that correlate with pancreatic cancer include diabetes mellitus, a high-fat diet, and chronic exposure to carcinogenic substances (i.e., petrochemicals). Although data are inconclusive, a relationship may exist between cigarette smoking and high coffee consumption (especially decaffeinated coffee) and the development of pancreatic carcinoma.

A home care nurse is caring for a client with complaints of epigastric discomfort who is scheduled for a barium swallow. Which statement by the client indicates an understanding of the test? a) "I'll drink full liquids the day before the test." b) "There is no need for special preparation before the test." c) "I'll take a laxative to clear my bowels before the test." d) "I'll avoid eating or drinking anything 6 to 8 hours before the test."

D) I'll avoid eating or drinking anything 6 to 8 hours before the test The client demonstrates understanding of a barium swallow when he states that he must refrain from eating or drinking for 6 to 8 hours before the test. No other preparation is needed. Before a lower GI series, the client should eat a low-residue or clear liquid diet for 2 days and take a potent laxative and an oral liquid preparation.

The family of a patient newly diagnosed with hepatitis A asks the nurse what they can do to prevent becoming ill themselves. Which of the following responses by the nurse is most appropriate? A. "The hepatitis vaccine will provide immunity from this exposure and future exposures." B. "I am afraid there is nothing you can do since the patient was infectious before admission." C. "You will need to be tested first to make sure you don't have the virus before we can treat you." D. "An injection of immunoglobulin will need to be given to prevent or minimize the effects from this exposure."

D) Immunoglobulin provides temporary (1-2 months) passive immunity and is effective for preventing hepatitis A if given within 2 weeks after exposure. It may not prevent infection in all persons, but it will at least modify the illness to a subclinical infection. The hepatitis vaccine is only used for preexposure prophylaxis

Which of the following is a parasympathetic response in the GI tract? a) Blood vessel constriction b) Decreased gastric secretion c) Decreased motility d) Increased peristalsis

D) Increased peristalsis Increased peristalsis is a parasympathetic response in the GI tract. Decreased gastric secretion, blood vessel constriction, and decreased motility are sympathetic responses in the GI tract.

The nurse is preparing to administer a dose of bisacodyl (Dulcolax). In explaining the medication to the patient, the nurse would state that it acts in which of the following ways? A) Increases bulk in the stool B) Lubricates the intestinal tract to soften feces C) Increases fluid retention in the intestinal tract D) Increases peristalsis by stimulating nerves in the colon wall

D) Increases peristalsis by stimulating nerves in the colon wall Bisacodyl is a stimulant laxative that aids in producing a bowel movement by irritating the colon wall and stimulating enteric nerves. It is available in oral and suppository forms.

When planning care for a patient with cirrhosis, the nurse will give highest priority to which of the following nursing diagnoses? A) Imbalanced nutrition: less than body requirements B) Impaired skin integrity related to edema, ascites, and pruritus C) Excess fluid volume related to portal hypertension and hyperaldosteronism D) Ineffective breathing pattern related to pressure on diaphragm and reduced lung volume

D) Ineffective breathing pattern related to pressure on diaphragm and reduced lung volume Although all of these nursing diagnoses are appropriate and important in the care of a patient with cirrhosis, airway and breathing are always the highest priorities.

Crohn's disease is a condition of malabsorption caused by which of the following pathophysiological processes? a) Infectious disease b) Gastric resection c) Disaccharidase deficiency d) Inflammation of all layers of intestinal mucosa

D) Inflammation of all layers of intestinal mucosa Crohn's disease is also known as regional enteritis and can occur anywhere along the GI tract, but most commonly at the distal ileum and in the colon. Infectious disease causes problems such as small bowel bacterial overgrowth leading to malabsorption. Disaccharidase deficiency leads to lactose intolerance. Postoperative malabsorption occurs after gastric or intestinal resection.

Which of the following terms describes a gastric secretion that combines with vitamin B12 so that it can be absorbed? a) Amylase b) Trypsin c) Pepsin d) Intrinsic factor

D) Intrinsic factor Intrinsic factor, secreted by the gastric mucosa, combines with dietary vitamin B12 so that the vitamin can be absorbed in the ileum. In the absence of intrinsic factor, vitamin B12 cannot be absorbed and pernicious anemia results. Amylase is an enzyme that aids in the digestion of starch. Pepsin, an important enzyme for protein digestion, is the end product of the conversion of pepsinogen from the chief cells. Digestive enzymes secreted by the pancreas include trypsin, which aids in digesting protein.

A client is diagnosed with megaloblastic anemia caused by vitamin B12 deficiency. The physician begins the client on cyanocobalamin (Betalin-12), 100 mcg I.M. daily. Which substance influences vitamin B12 absorption? a) Hydrochloric acid b) Histamine c) Liver enzyme d) Intrinsic factor

D) Intrinsic factor Vitamin B12 absorption depends on intrinsic factor, which is secreted by parietal cells in the stomach. The vitamin binds with intrinsic factor and is absorbed in the ileum. Hydrochloric acid, histamine, and liver enzymes don't influence vitamin B12 absorption.

A nurse is preparing a client for a protcosigmoidoscopy. Identify the quadrant on which this diagnostic test will focus. A) RUQ B) RLQ C) LUQ D) LLQ

D) LLQ The sigmoid colon is in the left lower quadrant. Proctosigmoidoscopy is examination of the rectum and sigmoid colon using a rigid endoscope inserted anally about 10 inches.

A physician has ordered a liver biopsy for a client whose condition is deteriorating. Which of the following places the client at high risk due to her altered liver function during the biopsy? a) Low hemoglobin b) Decreased prothrombin time c) Low sodium level d) Low platelet count

D) Low platelet count Certain blood tests provide information about liver function. Prolonged prothrombin time (PT) and low platelet count place the client at high risk for hemorrhage. The client may receive intravenous (IV) administration of vitamin K or infusions of platelets before liver biopsy to reduce the risk of bleeding.

Regarding oral cancer, the nurse provides health teaching to inform the patient that a) most oral cancers are painful at the outset. b) blood testing is used to diagnose oral cancer. c) a typical lesion is soft and craterlike. d) many oral cancers produce no symptoms in the early stages.

D) Many oral cancers produce no symptoms in the early stages The most frequent symptom of oral cancer is a painless sore that does not heal. The patient may complain of tenderness, and difficulty with chewing, swallowing, or speaking as the cancer progresses. Biopsy is used to diagnose oral cancer. A typical lesion in oral cancer is a painless hardened ulcer with raised edges.

The nurse would question the use of which of the following cathartic agents in a patient with renal insufficiency? A) Bisacodyl B) Lubiprostone C) Cascara sagrada D) Milk of magnesia

D) Milk of Magnesia Milk of magnesia may cause hypermagnesemia in patients with renal insufficiency. The nurse should question this order with the health care provider before administration.

The nurse would question the use of which cathartic agent in a patient with renal insufficiency? A) Bisacodyl (Dulcolax) B) Lubiprostone (Amitiza) C) Cascara sagrada (Senekot) D) Magnesium hydroxide (Milk of Magnesia)

D) Milk of Magnesia may cause hypermagnesemia in patients with renal insufficiency. The nurse should question this order with the health care provider. Bisacodyl, lubiprostone, and cascara sagrada are safe to use in patients with renal insufficiency as long as the patient is not currently dehydrated.

A nurse is caring for a client who is undergoing a diagnostic workup for a suspected GI problem. The client reports gnawing epigastric pain following meals and heartburn. The nurse suspects the client has: a) diverticulitis. b) peptic ulcer disease. c) appendicitis. d) ulcerative colitis.

D) Peptic Ulcer Disease Peptic ulcer disease is characterized by dull, gnawing pain in the midepigastrium or the back that worsens with eating. Ulcerative colitis is characterized by exacerbations and remissions of severe bloody diarrhea. Appendicitis is characterized by epigastric or umbilical pain along with nausea, vomiting, and low-grade fever. Pain caused by diverticulitis is in the left lower quadrant and has a moderate onset. It's accompanied by nausea, vomiting, fever, and chills.

Which client requires immediate nursing intervention? The client who: a) complains of epigastric pain after eating. b) complains of anorexia and periumbilical pain. c) presents with ribbonlike stools. d) presents with a rigid, boardlike abdomen.

D) Presents with a rigid, boardlike abdomen A rigid, boardlike abdomen is a sign of peritonitis, a possibly life-threatening condition. Epigastric pain occurring 90 minutes to 3 hours after eating indicates a duodenal ulcer. Anorexia and periumbilical pain are characteristic of appendicitis. Risk of rupture is minimal within the first 24 hours, but increases significantly after 48 hours. A client with a large-bowel obstruction may have ribbonlike stools.

A patient reports having dry mouth and asks for some liquid to drink. The nurse reasons that this symptom can most likely be attributed to a common adverse effect of which of the following medications? A) Digoxin (Lanoxin) B) Cefotetan (Cefotan) C) Famotidine (Pepcid) D) Promethazine (Phenergan)

D) Promethazine (Phenergan) A common adverse effect of promethazine, an antihistamine antiemetic agent, is dry mouth; another is blurred vision.

A pt reports having dry mouth and asks for some liquid to drink. The nurse reasons that this symptom can most likely be attributed to a common adverse effect of which of the following medications? A) Digoxin (Lanoxin) B) Cefotetan (Cefotan) C) Famotidine (Pepcid) D) Promethazine (Phenergan)

D) Promethazine (Phenergan) A common adverse effect of promethazine, an antihistamine antiemetic agent, is dry mouth; another is blurred vision.

A client being treated for pancreatitis faces the risk of atelectasis. Which of the following interventions would be important to implement to minimize this risk? a) Withhold oral feedings for the client. b) Instruct the client to avoid coughing. c) Monitor pulse oximetry every hour. d) Reposition the client every 2 hours.

D) Reposition the client every 2 hours Repositioning the client every 2 hours minimizes the risk of atelectasis in a client who is being treated for pancreatitis. The client should be instructed to cough every 2 hours to reduce atelectasis. Monitoring the pulse oximetry helps show changes in respiratory status and promote early intervention, but it would do little to minimize the risk of atelectasis. Withholding oral feedings limits the reflux of bile and duodenal contents into the pancreatic duct.

A client being treated for pancreatitis faces the risk of atelectasis. Which of the following interventions would be important to implement to minimize this risk? a) Withhold oral feedings for the client. b) Instruct the client to avoid coughing. c) Monitor pulse oximetry every hour. d) Reposition the client every 2 hours.

D) Reposition the client every 2 hours Repositioning the client every 2 hours minimizes the risk of atelectasis in a client who is being treated for pancreatitis. The client should be instructed to cough every 2 hours to reduce atelectasis. Monitoring the pulse oximetry helps show changes in respiratory status and promote early intervention, but it would do little to minimize the risk of atelectasis. Withholding oral feedings limits the reflux of bile and duodenal contents into the pancreatic duct.

A client is admitted with a diagnosis of acute appendicitis. When assessing the abdomen, the nurse would expect to find rebound tenderness at which location? a) Left lower quadrant b) Left upper quadrant c) Right upper quadrant d) Right lower quadrant

D) Right lower quadrant The pain of acute appendicitis localizes in the right lower quadrant (RLQ) at McBurney's point, an area midway between the umbilicus and the right iliac crest. Often, the pain is worse when manual pressure near the region is suddenly released, a condition called rebound tenderness.

The patient is on a continuous tube feeding. The tube placement should be checked every a) 24 hours. b) 12 hours. c) hour. d) shift.

D) Shift Each nurse caring for the patient is responsible for verifying that the tube is located in the proper area for continuous feeding. Checking for placement each hour is unnecessary unless the patient is extremely restless or there is basis for rechecking the tube based on other patient activities. Checking for placement every 12 or 24 hours does not meet the standard of care due the patient receiving continuous tube feedings.

The nurse is providing care to a client who has had a percutaneous liver biopsy. The nurse would monitor the client for which of the following? a) Intake and output b) Passage of stool c) Return of the gag reflex d) Signs and symptoms of bleeding

D) Signs and symptoms of bleeding A major complication after a liver biopsy is bleeding so it would be important for the nurse to monitor the client for signs and symptoms of bleeding. Return of the gag reflex would be important for the client who had an esophagogastroduodenoscopy to prevent aspiration. Monitoring the passage of stool would be important for a client who had a barium enema or colonoscopy. Monitoring intake and output is a general measure indicated for any client. It is not specific to a liver biopsy.

After teaching a group of students about the various organs of the upper gastrointestinal tract and possible disorders, the instructor determines that the teaching was successful when the students identify which of the following structures as possibly being affected? a) Large intestine b) Ileum c) Liver d) Stomach

D) Stomach The upper gastrointestinal (GI) tract begins at the mouth and ends at the jejunum. Therefore, the stomach would be a component of the upper GI tract. The lower GI tract begins at the ileum and ends at the anus. The liver is considered an accessory structure.

After teaching a group of students about the various organs of the upper gastrointestinal tract and possible disorders, the instructor determines that the teaching was successful when the students identify which of the following structures as possibly being affected? a) Ileum b) Liver c) Large intestine d) Stomach

D) Stomach The upper gastrointestinal (GI) tract begins at the mouth and ends at the jejunum. Therefore, the stomach would be a component of the upper GI tract. The lower GI tract begins at the ileum and ends at the anus. The liver is considered an accessory structure.

A client with acute liver failure exhibits confusion, a declining level of consciousness, and slowed respirations. The nurse finds him very difficult to arouse. The diagnostic information which best explains the client's behavior is: a) elevated liver enzymes and low serum protein level. b) subnormal clotting factors and platelet count. c) elevated blood urea nitrogen and creatinine levels and hyperglycemia. d) subnormal serum glucose and elevated serum ammonia levels.

D) Subnormal serum glucose and elevated serum ammonia levels. In acute liver failure, serum ammonia levels increase because the liver can't adequately detoxify the ammonia produced in the GI tract. In addition, serum glucose levels decline because the liver isn't capable of releasing stored glucose. Elevated serum ammonia and subnormal serum glucose levels depress the level of a client's consciousness. Elevated liver enzymes, low serum protein level, subnormal clotting factors and platelet count, elevated blood urea nitrogen and creatine levels, and hyperglycemia aren't as directly related to the client's level of consciousness.

Why should total parental nutrition (TPN) be used cautiously in clients with pancreatitis? a) Such clients can digest high-fat foods. b) Such clients are at risk for hepatic encephalopathy. c) Such clients are at risk for gallbladder contraction. d) Such clients cannot tolerate high-glucose concentration.

D) Such clients cannot tolerate high glucose concentration Total parental nutrition (TPN) is used carefully in clients with pancreatitis because some clients cannot tolerate a high-glucose concentration even with insulin coverage. Intake of coffee increases the risk for gallbladder contraction, whereas intake of high protein increases risk for hepatic encephalopathy in clients with cirrhosis. Patients with pancreatitis should not be given high-fat foods because they are difficult to digest.

An elderly client asks the nurse how to treat chronic constipation. What is the best recommendation the nurse can make? a) Take a mild laxative such as magnesium citrate when necessary. b) Administer a tap-water enema weekly. c) Administer a phospho-soda (Fleet) enema when necessary. d) Take a stool softener such as docusate sodium (Colace) daily.

D) Take a stool softener such as docuaste sodium (Colace) daily. Stool softeners taken daily promote absorption of liquid into the stool, creating a softer mass. They may be taken on a daily basis without developing a dependence. Dependence is an adverse effect of daily laxative use. Enemas used daily or on a frequent basis can also lead to dependence of the bowel on an external source of stimulation.

The nurse would instruct the patient to do which of the following to best enhance the effectiveness of a daily dose of docusate sodium (Colace)? A) Take a dose of mineral oil at the same time. B) Add extra salt to food on at least one meal tray. C) Ensure dietary intake of 10 g of fiber each day. D) Take each dose with a full glass of water or other liquid.

D) Take each dose with a full glass of water or other liquid Docusate lowers the surface tension of stool, permitting water and fats to penetrate and soften the stool for easier passage. The patient should take the dose with a full glass of water and should increase overall fluid intake, if able, to enhance effectiveness of the medication. Dietary fiber intake should be a minimum of 20 g daily to prevent constipation.

A client with an esophageal stricture is about to undergo esophageal dilatation. As the bougies are passed down the esophagus, the nurse should instruct the client to do which action to minimize the vomiting urge? a) Hold his breath b) Bear down as if having a bowel movement c) Pant like a dog d) Take long, slow breaths

D) Take long, slow breaths During passage of the bougies used to dilate the esophagus, the client should take long, slow breaths to minimize the vomiting urge. Having the client hold the breath, bear down as if having a bowel movement, or pant like a dog is neither required nor helpful.

Blood shed in sufficient quantities into the upper GI tract, produces which color of stool? a) Bright red b) Milky white c) Green d) Tarry-black

D) Tarry-black Blood shed in sufficient quantities into the upper GI tract produces a tarry-black stool. Blood entering the lower portion of the GI tract or passing rapidly through it will appear bright or dark red. A milky white stool is indicative "of" a patient who received barium. A green stool is indicative of a patient who has eaten spinach.

A patient who has hepatitis B surface antigen (HBsAg) in the serum is being discharged with pain medication after knee surgery. Which medication order should the nurse question because it is most likely to cause hepatic complications? A) Tramadol (Ultram) B) Hydromorphone (Dilaudid) C) Oxycodone with aspirin (Percodan) D) Hydrocodone with acetaminophen (Vicodin)

D) The analgesic with acetaminophen should be questioned because this patient is a chronic carrier of hepatitis B and is likely to have impaired liver function. Acetaminophen is not suitable for this patient because it is converted to a toxic metabolite in the liver after absorption, increasing the risk of hepatocellular damage.

The patient with suspected pancreatic cancer is having many diagnostic studies done. Which one can be used to establish the diagnosis of pancreatic adenocarcinoma and for monitoring the response to treatment? A) Spiral CT scan B) A PET/CT scan C) Abdominal ultrasound D) Cancer-associated antigen 19-9

D) The cancer-associated antigen 19-9 (CA 19-9) is the tumor marker used for the diagnosis of pancreatic adenocarcinoma and for monitoring the response to treatment. Although a spiral CT scan may be the initial study done and provides information on metastasis and vascular involvement, this test and the PET/CT scan or abdominal ultrasound do not provide additional information.

When teaching the patient with acute hepatitis C (HCV), the patient demonstrates understanding when the patient makes which statement? A) "I will use care when kissing my wife to prevent giving it to her." B) "I will need to take adofevir (Hepsera) to prevent chronic HCV." C) "Now that I have had HCV, I will have immunity and not get it again." D) "I will need to be checked for chronic HCV and other liver problems."

D) The majority of patients who acquire HCV usually develop chronic infection, which may lead to cirrhosis or liver cancer. HCV is not transmitted via saliva, but percutaneously and via high-risk sexual activity exposure. The treatment for acute viral hepatitis focuses on resting the body and adequate nutrition for liver regeneration. Adofevir (Hepsera) is taken for severe hepatitis B (HBV) with liver failure. Chronic HCV is treated with pegylated interferon with ribavirin. Immunity with HCV does not occur as it does with HAV and HBV, so the patient may be reinfected with another type of HCV.

A client with diabetes begins to have digestive problems and is told by the physician that they are a complication of the diabetes. Which of the following explanations from the nurse is most accurate? a) Insulin has an adverse effect of constipation. b) The nerve fibers of the intestinal lining are experiencing neuropathy. c) Elevated glucose levels cause bacteria overgrowth in the large intestine. d) The pancreas secretes digestive enzymes.

D) The pancreas secretes digestive enzymes While the pancreas has the well-known function of secreting insulin, it also secretes digestive enzymes. These enzymes include trypsin, amylase, and lipase. If the secretion of these enzymes are affected by a diseased pancreas as foundi with diabetes, the digestive functioning may be impaired.

The nurse asks a 68-year-old patient scheduled for colectomy to sign the operative permit as directed in the physician's preoperative orders. The patient states that the physician has not really explained very well what is involved in the surgical procedure. What is the most appropriate action by the nurse? A) Ask family members whether they have discussed the surgical procedure with the physician. B) Have the patient sign the form and state the physician will visit to explain the procedure before surgery. C) Explain the planned surgical procedure as well as possible and have the patient sign the consent form. D) Delay the patient's signature on the consent and notify the physician about the conversation with the patient.

D) The patient should not be asked to sign a consent form unless the procedure has been explained to the satisfaction of the patient. The nurse should notify the physician, who has the responsibility for obtaining consent.

The most significant complication related to continuous tube feedings is a) an interruption in fat metabolism and lipoprotein synthesis. b) a disturbance in the sequence of intestinal and hepatic metabolism. c) the interruption of GI integrity, d) the potential for aspiration,

D) The potential for aspiration Because the normal swallowing mechanism is bypassed, consideration of the danger of aspiration must be foremost in the mind of the nurse caring for the patient receiving continuous tube feedings. Tube feedings preserve GI integrity by intraluminal delivery of nutrients. Tube feedings preserve the normal sequence of intestinal and hepatic metabolism. Tube feedings maintain fat metabolism and lipoprotein synthesis.

The nurse is teaching a group of high school students about the prevention of food poisoning. Which comment by the student shows understanding of foodborne illness protection? A) "We like to mix up the ingredients so the flavors will melt before we cook our beef stew." B) "For a snack, I like to eat raw cookie dough from the package instead of baking the cookies." C) "We only have one cutting board, so we cut up our chicken and salad vegetables at the same time." D) "When they gave me a pink hamburger I sent it back and asked for a new bun and clean plate."

D) The student who did not accept the pink hamburger and asked for a new bun and clean plate understood that the pink meat may not have reached 160° and could be contaminated with bacteria. Mixing ingredients and leaving them long enough for the flavors to melt, eating raw cookie dough from a refrigerated package, and only using one cutting board without washing it with hot soapy water between the chicken and salad vegetables could all lead to food poisoning from contamination.

The nurse is caring for a postoperative patient with a colostomy. The nurse is preparing to administer a dose of famotidine (Pepcid) when the patient asks why the medication was ordered since the patient does not have a history of heartburn or gastroesophageal reflux disease (GERD). Which of the following would be the most appropriate response by the nurse? A) "This will prevent air from accumulating in the stomach, causing gas pains." B) "This will prevent the heartburn that occurs as a side effect of general anesthesia." C) "The stress of surgery is likely to cause stomach bleeding if you do not receive it." D) "This will reduce the amount of HCl in the stomach until the nasogastric tube is removed, and you can eat a regular diet again."

D) This will reduce the amount of HCl in the stomach until the nasogastric tube is removed, and you can eat a regular diet again Famotidine is an H2-receptor antagonist that inhibits gastric HCl secretion and thus minimizes damage to gastric mucosa while the patient is not eating a regular diet after surgery.

Why are antacids administered regularly, rather than as needed, in peptic ulcer disease? a) To increase pepsin activity b) To maintain a regular bowel pattern c) To promote client compliance d) To keep gastric pH at 3.0 to 3.5

D) To keep gastric pH at 3.0 to 3.5 To maintain a gastric pH of 3.0 to 3.5 throughout each 24-hour period, regular (not as needed) doses of an antacid are needed to treat peptic ulcer disease. Frequent administration of an antacid tends to decrease client compliance rather than promote it. Antacids don't regulate bowel patterns, and they decrease pepsin activity.

Which of the following symptoms characterizes regional enteritis? a) Severe diarrhea b) Diffuse involvement c) Exacerbations and remissions d) Transmural thickening

D) Transmural thickening Transmural thickening is an early pathologic change of Crohn's disease. Later pathology results in deep, penetrating granulomas. Regional enteritis is characterized by regional discontinuous lesions. Severe diarrhea is characteristic of ulcerative colitis while diarrhea in regional enteritis is less severe. Regional enteritis is characterized by a prolonged and variable course while ulcerative colitis is characterized by exacerbations and remissions.

A nurse is providing care for a client recovering from gastric bypass surgery. During assessment, the client exhibits pallor, perspiration, palpitations, headache, and feelings of warmth, dizziness, and drowsiness. The client reports eating 90 minutes ago. The nurse suspects: a) Peritonitis b) A normal reaction to surgery c) Dehiscence of the surgical wound d) Vasomotor symptoms associated with dumping syndrome

D) Vasomotor symptoms associated with dumping syndrome Early manifestations of dumping syndrome occur 15 to 30 minutes after eating. Signs and symptoms include vertigo, tachycardia, syncope, sweating, pallor, palpitations, diarrhea, nausea, and the desire to lie down. Dehiscence of the surgical wound is characterized by pain and a pulling or popping feeling at the surgical site. Peritonitis presents with a rigid, boardlike abdomen, tenderness, and fever. The client's signs and symptoms aren't a normal reaction to surgery.

The ED nurse has inspected, auscultated, and palpated the abdomen with no obvious abnormalities, except pain. When the nurse palpates the abdomen for rebound tenderness, there is severe pain. The nurse should know that this could indicate what problem? A) Hepatic cirrhosis B) Hypersplenomegaly C) Gall bladder distention D) Peritoneal inflammation

D) When palpating for rebound tenderness, the problem area of the abdomen will produce pain and severe muscle spasm when there is peritoneal inflammation. Hepatic cirrhosis, hypersplenomegaly, and gall bladder distention do not manifest with rebound tenderness.

The nurse is preparing to administer a scheduled dose of docusate sodium (Colace) when the patient reports an episode of loose stool and does not want to take the medication. What is the appropriate action by the nurse? A) Write an incident report about this untoward event. B) Attempt to have the family convince the patient to take the ordered dose. C) Withhold the medication at this time and try to administer it later in the day. D) Chart the dose as not given on the medical record and explain in the nursing progress notes.

D) Whenever a patient refuses medication, the dose should be charted as not given with an explanation of the reason documented in the nursing progress notes. In this instance, the refusal indicates good judgment by the patient, and the patient should not be encouraged to take it today.

A client with complaints of right lower quadrant pain is admitted to the emergency department. Blood specimens are drawn and sent to the laboratory. Which laboratory finding should be reported to the physician immediately? a) Hematocrit 42% b) Serum potassium 4.2 mEq/L c) Serum sodium 135 mEq/L d) White blood cell (WBC) count 22.8/mm3

D) White blood cell (WBC) count 22.8/mm3 The nurse should report the elevated WBC count. This finding, which is a sign of infection, indicates that the client's appendix might have ruptured. Hematocrit of 42%, serum potassium of 4.2 mEq/L, and serum sodium of 135 mEq/L are within normal limits. Alterations in these levels don't indicate appendicitis.

A client with cholelithiasis has a gallstone lodged in the common bile duct. When assessing this client, the nurse expects to note: a) black, tarry stools. b) circumoral pallor. c) light amber urine. d) yellow sclerae.

D) Yellow sclerae Yellow sclerae are an early sign of jaundice, which occurs when the common bile duct is obstructed. Urine normally is light amber. Circumoral pallor and black, tarry stools don't occur in common bile duct obstruction; they are signs of hypoxia and GI bleeding, respectively.

"The nurse is teaching the patient and family about possible causative factors for peptic ulcers. The nurse explains that ulcer formation is a. caused by a stressful lifestyle and other acid-producing factors such as H. pylori. b. inherited within families and reinforced by bacterial spread of Staphylococcus aureus in childhood. c. promoted by factors that tend to cause oversecretion of acid, such as excess dietary fats, smoking, and H. pylori. d. promoted by a combination of possible factors that may result in erosion of the gastric mucosa, including certain drugs and alcohol

D, Rationale: Peptic ulcers develop only in the presence of an acidic environment. However, an excess of hydrochloric acid (HCl) may not be necessary for ulcer development. The back-diffusion of HCl into the gastric mucosa results in cellular destruction and inflammation. Histamine is released from the damaged mucosa, resulting in vasodilation and increased capillary permeability and further secretion of acid and pepsin. A variety of agents (i.e., certain infections, medications, and lifestyle factors) can damage the mucosal barrier. Helicobacter pylori can alter gastric secretion and produce tissue damage leading to peptic ulcer disease. The response to H. pylori is likely influenced by a variety of factors, including genetics, environment, and diet. Ulcerogenic drugs, such as aspirin and NSAIDs, inhibit synthesis of prostaglandins, increase gastric acid secretion, and reduce the integrity of the mucosal barrier. Patients on corticosteroids, anticoagulants, and selective serotonin reuptake inhibitors (e.g., fluoxetine [Prozac]) are also at increased risk for ulcers. High-alcohol intake is associated with acute mucosal lesions. Alcohol stimulates acid secretion. Coffee (caffeinated and uncaffeinated) is a strong stimulant of gastric acid secretion. Psychologic distress, including stress and depression, can negatively influence the healing of ulcers after they have developed. Smoking also delays ulcer healing. Infection with herpes and cytomegalovirus (CMV) in immunocompromised patients may also lead to gastric ulcers

A patient with cirrhosis that is refractory to other treatments for esophageal varices undergoes a peritoneovenous shunt. As a result of this procedure, the nurse would expect the patient to experience a. an improved survival rate b. decreased serum ammonia levels c. improved metabolism of nutrients d. improved hemodynamic function and renal perfusion

D- By shunting fluid sequestered in the peritoneum into the venous system, pressur eon esophageal veins is decreased, and more volume is returned to the circulation, improving CO and renal perfusion. However, because ammonia is diverted past the liver, hepatic encephalopathy continues. These procedures do not prolong life or promote liver function.

The nurse recognizes early signs of hepatic encephalopathy in the patient who a. manifests asterixis b. becomes unconscious c. has increasing oliguria d. is irritable and lethargic

D- Early signs of this neurologic condition include euphoria, depression, apathy, irritability, confusion, agitation, drowsiness, and lethargy. Loss of consciousness is usually preceded by asterixis, disorientation, hyperventilation, hypothermia, and alterations in reflexes. Increasing oliguria is a sign of hepatorenal syndrome.

The family members of a patient with hepatitis A ask if there is anything that will prevent them from developing the disease. The best response by the nurse is a. "no immunization is available for hepatitis A, nor are you likely to get the disease" b. "only individuals who have had sexual contact with the patient should receive immunization" c. "all family members should receive the hepatitis A vaccine to prevent or modify the infection" d. "those who have had household or close contact with the patient should receive immune globulin"

D- Individuals who have been exposed to hepatitis A through household contact or foodborne outbreaks should be given immune globulin within 1 to 2 weeks of exposure to prevent or modify the illness. Hep A vaccine is used to provide pre-exposure immunity to the virus and is indicated for individuals at high risk for hep A exposure. Although hep A can be spread by sexual contact, the risk is higher for transmission with the oral-fecal route.

If a client had irritable bowel syndrome, which of the following diagnostic tests would determine if the diagnosis is Crohn's disease or ulcerative colitis? A. Abdominal computed tomography (CT) scan B. Abdominal x-ray C. Barium swallow D. Colonoscopy with biopsy

D. A colonoscopy with biopsy can be performed to determine the state of the colon's mucosal layers, presence of ulcerations, and level of cytologic development. An abdominal x-ray or CT scan wouldn't provide the cytologic information necessary to diagnose which disease it is. A barium swallow doesn't involve the intestine.

The nurse is administering morning medications at 0730. Which medication should have priority? A. a proton pump inhibitor B. A nonnarcotic analgesic C. A histamine receptor antagonist D. A mucosal barrier agent

D. A mucosal barrier agent must be adminstered on an empty stomach for the medication to coat the stomach lining

The nurse is assessing for stoma prolapse in a client with a colostomy. The nurse would observe which of the following if stoma prolapse occurred? A. Sunken and hidden stoma B. Dark- and bluish-colored stoma C. Narrowed and flattened stoma D. Protruding stoma

D. A prolapsed stoma is one which the bowel protruded through the stoma. A stoma retraction is characterized by sinking of the stoma. Ischemia of the stoma would be associated with dusky or bluish color. A stoma with a narrowed opening at the level of the skin or fascia is said to be stenosed.

Which of the following laboratory results would be expected in a client with peritonitis? A. Partial thromboplastin time above 100 seconds B. Hemoglobin level below 10 mg/dL C. Potassium level above 5.5 mEq/L D. White blood cell count above 15,000

D. Because of infection, the client's WBC count will be elevated. A hemoglobin level below 10 mg/dl may occur from hemorrhage. A PT time longer than 100 seconds may suggest disseminated intravascular coagulation, a serious complication of septic shock. A potassium level above 5.5 mEq/L may indicate renal failure.

Colon cancer is most closely associated with which of the following conditions? A. Appendicitis B. Hemorrhoids C. Hiatal hernia D. Ulcerative colitis

D. Chronic ulcerative colitis, granulomas, and familial polposis seem to increase a person's chance of developing colon cancer. The other conditions listed have no known effect on colon cancer risk.

Crohn's disease can be described as a chronic relapsing disease. Which of the following areas in the GI system may be involved with this disease? A. The entire length of the large colon B. Only the sigmoid area C. The entire large colon through the layers of mucosa and submucosa D. The small intestine and colon; affecting the entire thickness of the bowel

D. Crohn's disease can involve any segment of the small intestine, the colon, or both, affecting the entire thickness of the bowel. Answers 1 and 3 describe ulcerative colitis, answer 2 is too specific and therefore, not likely.

When teaching an elderly client how to prevent constipation, which of the following instructions should the nurse include? A. "Drink 6 glasses of fluid each day." B. "Avoid grain products and nuts." C. "Add at least 4 grams of brain to your cereal each morning." D. "Be sure to get regular exercise."

D. Exercise helps prevent constipation. Fluids and dietary fiber promote normal bowel function. The client should drink eight to ten glasses of fluid each day. Although adding bran to cereal helps prevent constipation by increasing dietary fiber, the client should start with a small amount and gradually increase the amount as tolerated to a maximum of 2 grams a day.

A client presents to the emergency room, reporting that he has been vomiting every 30 to 40 minutes for the past 8 hours. Frequent vomiting puts him at risk for which of the following? A. Metabolic acidosis with hyperkalemia B. Metabolic acidosis with hypokalemia C. Metabolic alkalosis with hyperkalemia D. Metabolic alkalosis with hypokalemia

D. Gastric acid contains large amounts of potassium, chloride, and hydrogen ions. Excessive loss of these substances, such as from vomiting, can lead to metabolic alkalosis and hypokalemia.

The nurse is reviewing the physician's orders written for a client admitted with acute pancreatitis. Which physician order would the nurse question if noted on the client's chart? A. NPO status B. Insert a nasogastric tube C. An anticholinergic medication D. Morphine for pain

D. Meperidine (Demerol) rather than morphine is the medication of choice because morphine can cause spasm in the sphincter of Oddi.

Which of the following substances is most likely to cause gastritis? A. Milk B. Bicarbonate of soda, or baking soda C. Enteric coated aspirin D. Nonsteriodal anti-imflammatory drugs

D. NSAIDS are a common cause of gastritis because they inhibit prostaglandin synthesis. Milk, once thought to help gastritis, has little effect on the stomach mucosa. Bicarbonate of soda, or baking soda, may be used to neutralize stomach acid, but it should be used cautiously because it may lead to metabolic acidosis. ASA with enteric coating shouldn't contribute significantly to gastritis because the coating limits the aspirin's effect on the gastric mucosa.

"The nurse explains to the patient with gastroesophageal reflux disease that this disorder: "A. results in acid erosion of the esophagus caused by frequent vomiting B. Will require surgical wrapping of the pyloric sphincter to control the symptoms C. Is the protrusion of a portion of the stomach into the esophagus through the opening in the diaphragm D. Often involves relaxation of the lower esophageal sphincter, allowing the stomach contents to back up into the esophagus

D. Often involves relaxation of the lower esophageal sphincter, allowing the stomach contents to back up into the esophagus Rationale: Gastroesophageal reflux disease (GERD) results when the defenses of the esophagus are overwhelmed by the reflux of acidic gastric contents into the lower esophagus. An incompetent lower esophageal sphincter (LES) is a common cause of gastric reflux

Surgical management of ulcerative colitis may be performed to treat which of the following complications? A. Gastritis B. Bowel herniation C. Bowel outpouching D. Bowel perforation

D. Perforation, obstruction, hemorrhage, and toxic megacolon are common complications of ulcerative colitis that may require surgery. Herniation and gastritis aren't associated with irritable bowel diseases, and outpouching of the bowel is diverticulosis.

The nurse is caring for a hospitalized client with a diagnosis of ulcerative colitis. Which finding, if noted on assessment of the client, would the nurse report to the physician? A. Bloody diarrhea B. Hypotension C. A hemoglobin of 12 mg/dL D. Rebound tenderness

D. Rebound tenderness may indicate peritonitis. Blood diarrhea is expected to occur in ulcerative colitis. Because of the blood loss, the client may be hypotensive and the hemoglobin level may be lower than normal. Signs of peritonitis must be reported to the physician.

22. Nurse Hannah is teaching a group of middle-aged men about peptic ulcers. When discussing risk factors for peptic ulcers, the nurse should mention: A. a sedentary lifestyle and smoking. B. a history of hemorrhoids and smoking, C. alcohol abuse and a history of acute renal failure. D. alcohol abuse and smoking

D. Risk factors for peptic (gastric and duodenal) ulcers include alcohol abuse, smoking, and stress. A sedentary lifestyle and a history of hemorrhoids aren't risk factors for peptic ulcers. Chronic renal failure, not acute renal failure, is associated with duodenal ulcers

Which of the following symptoms may be exhibited by a client with Crohn's disease? A. Bloody diarrhea B. Narrow stools C. N/V D. Steatorrhea

D. Steatorrhea from malaborption can occur with Crohn's disease. N/V, and bloody diarrhea are symptoms of ulcerative colitis. Narrow stools are associated with diverticular disease.

Which area of the alimentary canal is the most common location for Crohn's disease? A. Ascending colon B. Descending colon C. Sigmoid colon D. Terminal ileum

D. Studies have shown that the terminal ileum is the most common site for recurrence in clients with Crohn's disease. The other areas may be involved but aren't as common.

Which of the following diagnostic tests should be performed annually over age 50 to screen for colon cancer? A. Abdominal CT scan B. Abdominal x-ray C. Colonoscopy D. Fecal occult blood test

D. Surface blood vessels of polyps and cancers are fragile and often bleed with the passage of stools. Abdominal x-ray and CT scan can help establish tumor size and metastasis. A colonoscopy can help locate a tumor as well as polyps, which can be removed before they become malignant.

"The nurse explains to the patient with gastroesophageal reflux disease that this disorder: "A. results in acid erosion of the esophagus caused by frequent vomiting B. Will require surgical wrapping of the pyloric sphincter to control the symptoms C. Is the protrusion of a portion of the stomach into the esophagus through the opening in the diaphragm D. Often involves relaxation of the lower esophageal sphincter, allowing the stomach contents to back up into the esophagus

D. The acidic contents of the stomach touching the inside of the esophagus are responsible for the physical sensation known as "heart-burn" that is a cardinal symptom of GERD

The nurse explains to the patient with gastroesophageal reflux disease that this disorder: A. results in acid erosion and ulceration of the esophagus caused by frequent vomiting, B. will require surgical wrapping or repair of the pyloric sphincter to control the symptoms, C. is the protrusion of a portion of the stomach into to esophagus through an opening in the diaphragm, D. often involves relaxation of the lower esophageal sphincter, allowing stomach contents to back up into the espophagus

D. The acidic contents of the stomach touching the inside of the esophagus are responsible for the physical sensation known as "heart-burn" that is a cardinal symptom of GERD

The client who has undergone creation of a colostomy has a nursing diagnosis of Disturbed body image. The nurse would evaluate that the client is making the most significant progress toward identified goals if the client: A. Watches the nurse empty the colostomy bag B. Looks at the ostomy site C. Reads the ostomy product literature D. Practices cutting the ostomy appliance

D. The client is expected to have a body image disturbance after colostomy. The client progresses through normal grieving stages to adjust to this change. The client demonstrates the greatest deal of acceptance when the client participates in the actual colostomy care. Each of the incorrect options represents an interest in colostomy care but is a passive activity. The correct option shows the client is participating in self-care.

Which of the following therapies is not included in the medical management of a client with peritonitis? A. Broad-spectrum antibiotics B. Electrolyte replacement C. I.V. fluids D. Regular diet

D. The client with peritonitis usually isn't allowed anything orally until the source of peritonitis is confirmed and treated. The client also requires broad-spectrum antibiotics to combat the infection. I.V. fluids are given to maintain hydration and hemodynamic stability and to replace electrolytes.

Which of the following nursing interventions should be implemented to manage a client with appendicitis? A. Assessing for pain B. Encouraging oral intake of clear fluids C. Providing discharge teaching D. Assessing for symptoms of peritonitis

D. The focus of care is to assess for peritonitis, or inflammation of the peritoneal cavity. Peritonitis is most commonly caused by appendix rupture and invasion of bacteria, which could be lethal. The client with appendicitis will have pain that should be controlled with analgesia. The nurse should discourage oral intake in preparation of surgery. Discharge teaching is important; however, in the acute phase, management should focus on minimizing preoperative complications and recognizing when such may be occurring.

Which of the following terms best describes the pain associated with appendicitis? A. Aching B. Fleeting C. Intermittent D. Steady

D. The pain begins in the epigastrium or periumbilical region, then shifts to the right lower quadrant and becomes steady. The pain may be moderate to severe.

Which of the following associated disorders may a client with ulcerative colitis exhibit? A. Gallstones B. Hydronephrosis C. Nephrolithiasis D. Toxic megacolon

D. Toxic megacolon is extreme dilation of a segment of the diseased colon caused by paralysis of the colon, resulting in complete obstruction. This disorder is associated with both Crohn's disease and ulcerative colitis. The other disorders are more commonly associated with Crohn's disease.

Which of the following mechanisms can facilitate the development of diverticulosis into diverticulitis? A. Treating constipation with chronic laxative use, leading to dependence on laxatives B. Chronic constipation causing an obstruction, reducing forward flow of intestinal contents C. Herniation of the intestinal mucosa, rupturing the wall of the intestine D. Undigested food blocking the diverticulum, predisposing the area to bacteria invasion.

D. Undigested food can block the diverticulum, decreasing blood supply to the area and predisposing the area to invasion of bacteria. Chronic laxative use is a common problem in elderly clients, but it doesn't cause diverticulitis. Chronic constipation can cause an obstruction—not diverticulitis. Herniation of the intestinal mucosa causes an intestinal perforation.

"The nurse is caring for a 68 year old patient admitted with abdominal pain, nausea, and vomiting. The patient has an abdominal mass and a bowel obstruction is suspected. The nurse auscultating the abdomen listens for which of the following types of bowel sounds that is consistent with the patient's clinical picture? "A. low pitched and rumbling above the area of obstruction B. High pitched and hypoactive below the area of obstruction C. low pitched and hyperactive below the area of obstruction (D). high pitched and hyperactive above the area of obstruction

Early in intestinal obstruction, the patient's bowel sounds are hyperactive adn high pitched. This occurs because peristaltic action increases to "push past" the area of obstruction. As the obstruction becomes complete, bowel sounds decrease and finally become absent

A nurse caring for a patient who has had bariatric surgery is developing a teaching plan in anticipation of the patient's discharge. Which of the following is essential to include? Drink a minimum of 12 oz of fluid with each meal. Eat several small meals daily spaced at equal intervals. Choose foods that are high in simple carbs. Sit upright when eating and for 30 minutes afterward.

Eat several small meals daily spaced at equal intervals. Due to decreased stomach capacity, the patient must consume small meals at intervals to meet nutritional requirements while avoiding a feeling of fullness and complications such as dumping syndrome. The patient should not consume fluids with meals and low-Fowler's positioning is recommended during and after meals. Carbohydrates should be limited.

A nurse is assisting with a percutaneous liver biopsy. Place the steps involved in care in the correct sequence from first to last. Ensure that the biopsy equipment is assembled and in order. Help the client assume a supine position. Make sure that the specimen container is labeled and delivered to the laboratory. While the physician inserts the needle, instruct the client to take a deep breath and hold it to keep the liver as near to the abdominal wall as possible. Place a rolled towel beneath the client's right lower ribs.

Ensure that the biopsy equipment is assembled and in order. Help the client assume a supine position. Place a rolled towel beneath the client's right lower ribs. While the physician inserts the needle, instruct the client to take a deep breath and hold it to keep the liver as near to the abdominal wall as possible. Make sure that the specimen container is labeled and delivered to the laboratory. When assisting with a percutaneous liver biopsy, the nurse ensures that the biopsy equipment is assembled and in order. He or she helps the client assume a supine position with a rolled towel beneath the right lower ribs. Before the physician inserts the needle, the nurse instructs the client to take a deep breath and hold it to keep the liver as near to the abdominal wall as possible. After specimen cells are obtained, they are placed in a preservative. The nurse makes sure that the specimen container is labeled and delivered to the laboratory.

True or False? A drug that clocks the release of secretions from the stomach's chief cells will decrease gastric acidity.

False A drug that clocks the release of secretions from the stomach's parietal cells will decrease gastric acidity.

True or False? Obsruction of the biliary tract is indicated by increased unconjugated (indirect) bilirubin levels in the blood.

False Obsruction of the biliary tract is indicated by increased conjugated (direct) bilirubin levels in the blood.

True or False? The structure that prevents reflux of stomach contents into the esophagus is the upper esophageal sphincter.

False The structure that prevents reflux of stomach contents into the esophagus is the lower esophageal sphincter.

The nurse is caring for a patient who is undergoing diagnostic testing for suspected malabsorption. When taking this patient's health history and performing the physical assessment, the nurse should recognize what finding as most consistent with this diagnosis? Recurrent constipation coupled with weight loss Foul-smelling diarrhea that contains fat Fever accompanied by a rigid, tender abdomen Bloody bowel movements accompanied by fecal incontinence

Foul-smelling diarrhea that contains fat The hallmarks of malabsorption syndrome from any cause are diarrhea or frequent, loose, bulky, foul-smelling stools that have increased fat content and are often grayish (steatorrhea). Constipation and bloody bowel movements are not suggestive of malabsorption syndromes. Fever and a tender, rigid abdomen are associated with peritonitis.

A client with chronic pancreatitis is being discharged from the hospital. Which of the following statements by the client demonstrates that the client understands how to take prescribed pancreatic enzyme replacements? "I must take capsules three times daily spaced about 8 hours apart." "I must take the medication when I wake up and before bed." "I must take the medication with meals and snacks." "I must take the capsules every 4 hours while awake."

I must take the med with meals and snacks

A nurse is caring for a patient with liver failure and is performing an assessment in the knowledge of the patient's increased risk of bleeding. The nurse recognizes that this risk is related to the patient's inability to synthesize prothrombin in the liver. What factor most likely contributes to this loss of function? Alterations in glucose metabolism Retention of bile salts Inadequate production of albumin by hepatocytes Inability of the liver to use vitamin K

Inability of the liver to use vitamin K Decreased production of several clotting factors may be partially due to deficient absorption of vitamin K from the GI tract. This probably is caused by the inability of liver cells to use vitamin K to make prothrombin. This bleeding risk is unrelated to the roles of glucose, bile salts, or albumin.

A 35-year-old male patient presents at the emergency department with symptoms of a small bowel obstruction. In collaboration with the primary care provider, what intervention should the nurse prioritize? Insertion of a nasogastric tube Insertion of a central venous catheter Administration of a mineral oil enema Administration of a glycerin suppository and an oral laxative

Insertion of a nasogastric tube Decompression of the bowel through a nasogastric tube is necessary for all patients with small bowel obstruction. Peripheral IV access is normally sufficient. Enemas, suppositories, and laxatives are not indicated if an obstruction is present.

A client is admitted with an alteration in neurological status and is in the process of being diagnosed with hepatic encephalopathy. Which of the following is known about this diagnosis? Reduced cardiac output is the leading cause of death in these clients. It is caused by a fat metabolism dysfunction. It is caused by a protein metabolism dysfunction. It is caused by a buildup of ammonia. It is caused by a carbohydrate metabolism dysfunction. It is caused by a buildup of urea.

It is caused by a buildup of ammonia. It is caused by a protein metabolism dysfunction.

A nurse is reviewing laboratory test results from a client. The report indicates that the client has jaundice. What serum bilirubin level must the client's finding exceed? Enter the correct number only.

Jaundice becomes clinically evident when the serum bilirubin level exceeds 2.5 mg/dL (43 fmol/L).

what to include when caring for client with cirrhosis

Lasix measure abd girth avoid salt but avoid warfarin (possible destruction of plt caused by splenomegaly resulting in spontaneous bleeding)

A nurse is caring for a patient admitted with a suspected malabsorption disorder. The nurse knows that one of the accessory organs of the digestive system is the pancreas. What digestive enzymes does the pancreas secrete? Select all that apply. Pepsin Lipase Amylase Trypsin Ptyalin

Lipase Amylase Trypsin

A nurse is caring for a patient who has a diagnosis of GI bleed. During shift assessment, the nurse finds the patient to betachycardic and hypotensive, and the patient has an episode of hematemesis while the nurse is in the room. In addition to monitoring the patient's vital signs and level of conscious, what would be a priority nursing action for this patient? place the patient in a prone position. provide the patient with ice water to slow any GI bleeding. Prepare for the insertion of an NG tube. Notify the physician.

Notify the physician The nurse must always be alert for any indicators of hemorrhagic gastritis, which include hematemesis (vomiting of blood), tachycardia, and hypotension. If these occur, the physician is notified and the patient's vital signs are monitored as the patient's condition warrants. Putting the patient in a prone position could lead to aspiration. Giving ice water is contraindicated as it would stimulate more vomiting.

A 74 y.o. female pt w/ GERD takes over-the-counter meds. For which med, if taken long-term, should the nurse teach about increased risk of fractures? Sucralfate (Carafate) Cimetidine (Tagamet) Omeprazole (Prilosec) Metoclopramide (Reglan)

Omeprazole (Prilosec) There is a potential link between proton pump inhibitor use and bone metabolism. Long-term use or high doese of these may increase the risk of fractures of the hip, wrist, and spine. Lower doses or shorter duration of therapy should be considered.

"The client diagnosed with appendicitis has undergone an appendectomy. At two hours postoperative, the nurse takes the vital signs and notes T 102.6 F, P 132, R 26, and BP 92/46. Which interventions should the nurse implement? List in order of priority. 1. Increase the IV rate. 2. Notify the health care provider. 3. Elevate the foot of the bed. 4. Check the abdominal dressing. 5. Determine if the IV antibiotics have been administered.

Order of priority: 1, 3, 4, 5, 2." "1. The nurse should increase the IV rate to maintain the circulatory system function until further orders can be obtained. 3. The foot of the bed should be elevated to help treat shock, the symptoms of which include elevated pulse and decreased BP. Those signs and an elevated temperature indicate an infection may be present and the client could be developing septicemia. 4. The dressing should be assessed to determine if bleeding is occurring. 5. The nurse should administer any IV antibiotics ordered after addressing hypovolemia. The nurse will need this information when reporting to the HCP. 2. The HCP should be notified when the nurse has the needed information."

The nurse is teaching the patient and family about possible causative factors for peptic ulcers. The nurse explains that ulcer formation is: a) caused by stressful lifestyle and other acid producing factors such as H. pylori. b) inherited within families and reinforced by bacterial spread of Staphylococcus aureus in childhood. c) promoted by factors that tend to cause oversecretion of acid, such as excess dietary fats, smoking, and H. pylor. d) promoted by a combination of possible factors that may result in erosion of the gastric mucosa, including certain drugs and alcohol

Peptic ulcers develop only in the presence of an acidic environment. However, an excess of hydrochloric acid (HCl) may not be necessary for ulcer development. The back-diffusion of HCl into the gastric mucosa results in cellular destruction and inflammation. Histamine is released from the damaged mucosa, resulting in vasodilation and increased capillary permeability and further secretion of acid and pepsin. A variety of agents (i.e., certain infections, medications, and lifestyle factors) can damage the mucosal barrier

A nurse is performing an admission assessment of a patient with a diagnosis of cirrhosis. What technique should the nurse use to palpate the patient's liver? Place hand under the right lower abdominal quadrant and press down lightly with the other hand. Place the left hand over the abdomen and behind the left side at the 11th rib. Place hand under right lower rib cage and press down lightly with the other hand. Hold hand 90 degrees to right side of the abdomen and push down firmly.

Place hand under right lower rib cage and press down lightly with the other hand. To palpate the liver, the examiner places one hand under the right lower rib cage and presses downward with light pressure with the other hand. The liver is not on the left side or in the right lower abdominal quadrant.

The nurse is caring for a client acute pancreatitis. Which of the following is an appropriate intervention while waiting for orders to be written? Provide sips of water. Provide ice chips. Keep NPO. Prepare for TPN.

Prepare for TPN.

A client with acute hepatitis is prescribed lactulose. The nurse knows this medication will: Mobilize iron stores from the liver. Remove bilirubin from the blood. Prevent hypoglycemia. Prevent the absorption of ammonia from the bowel.

Prevent the absorption of ammonia from the bowel.

A client diagnosed with acute pancreatitis is being transferred to another facility. The nurse caring for the client completes the transfer summary, which includes information about the client's drinking history and other assessment findings. Which assessment findings confirm his diagnosis? a) Recent weight loss and temperature elevation b) Presence of easy bruising and bradycardia c) Adventitious breath sounds and hypertension d) Presence of blood in the client's stool and recent hypertension

Recent weight loss and temperature elevation Assessment findings associated with pancreatitis include recent weight loss and temperature elevation. Inflammation of the pancreas causes a response that elevates temperature and leads to abdominal pain that typically occurs with eating. Nausea and vomiting may occur as a result of pancreatic tissue damage that's caused by the activation of pancreatic enzymes. The client may experience weight loss because of the lost desire to eat. Blood in stools and recent hypertension aren't associated with pancreatitis; fatty diarrhea and hypotension are usually present. Presence of easy bruising and bradycardia aren't found with pancreatitis; the client typically experiences tachycardia, not bradycardia. Adventitious breath sounds and hypertension aren't associated with pancreatitis.

The nurse educator is reviewing the blood supply of the GI tract with a group of medical nurses. The nurse is explaining the fact that the veins that return blood from the digestive organs and the spleen form the portal venous system. What large veins will the nurse list when describing this system? Select all that apply. Splenic vein Inferior mesenteric vein Gastric vein inferior vena cava Saphenous vein

Splenic vein Inferior mesenteric vein Gastric vein

Locate the abdominal quadrant where the nurse would expect to palpate the liver.

The liver is located in the right upper abdominal quadrant.

True or False? The nurse encourages that patient with chronic constipation to attempt defecation after the first meal of the day because gastrocolic and duodenocolic reflexes increase colon peristalsis at that time.

True

True or False? The secretion of hydrochloric acid and pepsinogen is stimulated by the sight, smell, and taste of food.

True

a nurse is caring for a newly admitted patient with a suspected GI bleed. The nurse assesses the patient's stool anfter a bowel movement and notes it to be a tarry-black color. This finding is suggestive of bleeding from what location? *sigmoid colon *upper GI tract *large intestine *anus or rectum

Upper GI tract Blood shed in sufficient quantities in the upper GI tract will produce a tarry-black color (melena). Blood entering the lower portion of the GI tract or passing rapidly through it will appear bright or dark red. Lower rectal or anal bleeding is suspected if there is streaking of blood on the surface of the stool or if blood is noted on toilet tissue.

When evaluating the patient's understanding about the care of the ileostomy, what statement by the patient indicates the patient needs more teaching? a. "I will be able to regulate when I have stools." b. "I will be able to wear the pouch until it leaks." c. "Dried fruit and popcorn must be chewed very well." d. "The drainage from my stoma can damage my skin."

a. "I will be able to regulate when I have stools." The ileostomy is in the ileum and drains liquid stool frequently, unlike the colostomy which has more formed stool the further distal the ostomy is in the colon. The ileostomy pouch is usually worn 4-7 days or until it leaks. It must be changed immediately if it leaks because the drainage is very irritating to the skin. To avoid obstruction, popcorn, dried fruit, coconut, mushrooms, olives, stringy vegetables, food with skin, and meats with casings must be chewed extremely well before swallowing because of the narrow diameter of the ileostomy lumen.

The nurse is preparing to insert a nasogastric (NG) tube into a 68-year-old female patient who is nauseated and vomiting. She has an abdominal mass and suspected small intestinal obstruction. The patient asks the nurse why this procedure is necessary. What response by the nurse is most appropriate? a. "The tube will help to drain the stomach contents and prevent further vomiting." b. "The tube will push past the area that is blocked and thus help to stop the vomiting." c. "The tube is just a standard procedure before many types of surgery to the abdomen." d. "The tube will let us measure your stomach contents so that we can plan what type of IV fluid replacement would be best."

a. "The tube will help to drain the stomach contents and prevent further vomiting." The NG tube is used to decompress the stomach by draining stomach contents and thereby prevent further vomiting. The NG tube will not push past the blocked area. Potential surgery is not currently indicated. The location of the obstruction will determine the type of fluid to use, not measure the amount of stomach contents.

What problem should the nurse assess the patient for if the patient was on prolonged antibiotic therapy? a. Coagulation problems b. Elevated serum ammonia levels c. Impaired absorption of amino acids d. Increased mucus and bicarbonate secretion

a. Coagulation problems Rationale: Bacteria int he colon (1) synthesize vitamin K, which is needed for the production of prothrombin by the liver and (2) deaminate undigested or non absorbed proteins, producing ammonia, which is converted to urea by the liver. A reduction in normal flora bacteria by antibiotic therapy can lead to decreased vitamin K, resulting in decreased prothrombin and coagulation problems. Bowel bacteria do not influence protein absorption or the secretion of mucus.

The nurse is caring for a patient diagnosed with IBS (irritable bowel syndrome). What symptoms are the patient most likely to exhibit? Select all that apply: a. Constipation b. Diarrhea c. Bloating d. Generalized abdominal pain e. High pitched bowel sounds.

a. Constipation b. Diarrhea c. Bloating d. Generalized abdominal pain

Checking for the return of the gag reflex and monitoring for LUQ pain, nausea and vomiting are necessary nursing actions after which diagnostic procedure? a. ERCP b. Colonoscopy c. Barium swallow d. Esophagogastroduodenoscopy (EGD)

a. ERCP Rationale: The left upper quadrant (LUQ) pain and nausea and vomiting could occur from perforation. The return of gag reflex is essential to prevent aspiration after an ERCP. The gag reflex is also assessed with an EGD. These are not relevant assessments for the colonoscopy and barium swallow.

When preparing a patient for a capsule endoscopy study, what should the nurse do? a. Ensure the patient understands the required bowel preparation. b. Have the patient return to the procedure room for removal of the capsule. c. Teach the patient to maintain a clear liquid diet throughout the procedure. d. Explain to the patient that conscious sedation will be used during placement of the capsule.

a. Ensure the patient understands the required bowel preparation. Rationale: A capsule endoscopy study involves the patient performing a bowel prep to cleanse the bowel before swallowing the capsule. The patient will be on a clear liquid diet for 1 to 2 days before the procedure and will remain NPO for 4 to 6 hours after swallowing the capsule. The capsule is disposable and will pass naturally with the bowel movement, although the monitoring device will need to be removed.

A 35-year-old man with a family history of adenomatous polyposis had a colonoscopy with removal of multiple polyps. Which signs and symptoms should the nurse teach the patient to report immediately? a. Fever and abdominal pain b. Flatulence and liquid stool c. Loudly audible bowel sounds d. Sleepiness and abdominal cramps

a. Fever and abdominal pain Rationale: The patient should be taught to observe for signs of rectal bleeding and peritonitis. Fever, malaise, and abdominal pain and distention could indicate a perforated bowel with peritonitis.

A patient had a stomach resection for stomach cancer. The nurse should teach the patient about the loss of the hormone that stimulates gastric acid secretion and motility and maintains lower esophageal sphincter tone. Which hormone will be decreased with a gastric resection? a. Gastrin b. Secretin c. Cholecystokinin d. Gastric inhibitory peptide

a. Gastrin Rationale: Gastrin is the hormone activated in the stomach (and duodenal mucosa) by stomach distention that stimulates gastric acid secretion and motility and maintains lower esophageal sphincter tone. Secretin inhibits gastric motility and acid secretion and stimulates pancreatic bicarbonate secretion. Cholecystokinin allows increased flow of bile into the duodenum and release of pancreatic digestive enzymes. Gastric inhibitory peptide inhibits gastric acid secretion and motility.

The nurse is caring for a patient with a transverse colostomy. Which of the following should the nurse perform during ostomy care. Select all that apply: a. Inspect the Stoma b. Test the stool for Guiac c. Clean the surrounding skin with alcohol d. Trim the wafer to the approximate size of the stoma e. Apply skin prep to the skin before applying the ostomy appliance.

a. Inspect the Stoma d. Trim the wafer to the approximate size of the stoma e. Apply skin prep to the skin before applying the ostomy appliance.

Priority Decision: Following auscultation of the abdomen, what should the nurse's next action be? a. Lightly percuss over all four quadrants b. Have the patient empty his or her bladder c. Inspect perianal and anal areas for color, masses, rashes, and scars d. Perform deep palpation to delineate abdominal organs and masses

a. Lightly percuss over all four quadrants Rationale: The abdomen should be assessed in the following sequence: inspection, auscultation, percussion, palpation. The patient should empty his or her bladder before assessment begins.

The nurse is conducting discharge teaching for a patient with metastatic lung cancer who was admitted with a bowel impaction. Which instructions would be most helpful to prevent further episodes of constipation? a. Maintain a high intake of fluid and fiber in the diet. b. Reduce intake of medications causing constipation. c. Eat several small meals per day to maintain bowel motility. d. Sit upright during meals to increase bowel motility by gravity.

a. Maintain a high intake of fluid and fiber in the diet. Increased fluid intake and a high-fiber diet reduce the incidence of constipation caused by immobility, medications, and other factors. Fluid and fiber provide bulk that in turn increases peristalsis and bowel motility. Analgesics taken for lung cancer probably cannot be reduced. Other medications may decrease constipation, but it is best to avoid laxatives. Eating several small meals per day and position do not facilitate bowel motility. Defecation is easiest when the person sits on the commode with the knees higher than the hips.

A 58-year-old woman has just returned to the nursing unit after an esophagogastroduodenoscopy (EGD). Which action by unlicensed assistive personnel (UAP) requires that the registered nurse (RN) intervene? a. Offering the patient a drink of water b. Positioning the patient on the right side c. Checking the vital signs every 30 minutes d. Swabbing the patient's mouth with cold water

a. Offering the patient a drink of water

A patient is being scheduled for endoscopic retrograde cholangiopancreatography (ERCP) as soon as possible. Which actions from the agency policy for ERCP should the nurse take first? a. Place the patient on NPO status. b. Administer sedative medications. c. Ensure the consent form is signed. d. Teach the patient about the procedure.

a. Place the patient on NPO status.

A patient with a history of peptic ulcer disease has presented to the emergency department reporting severe abdominal pain and has a rigid, boardlike abdomen that prompts the health care team to suspect a perforated ulcer. What intervention should the nurse anticipate? a. Providing IV fluids and inserting a nasogastric (NG) tube b. Administering oral bicarbonate and testing the patient's gastric pH level c. Performing a fecal occult blood test and administering IV calcium gluconate d. Starting parenteral nutrition and placing the patient in a high-Fowler's position

a. Providing IV fluids and inserting a nasogastric (NG) tube A perforated peptic ulcer requires IV replacement of fluid losses and continued gastric aspiration by NG tube. Nothing is given by mouth, and gastric pH testing is not a priority. Calcium gluconate is not a medication directly relevant to the patient's suspected diagnosis, and parenteral nutrition is not a priority in the short term.

What characterizes auscultation of the abdomen? a. The presence of borborygmi indicates hyper peristalsis. b. The bell of the stethoscope is used to auscultate high-pitched sounds. c. High-pitched, rushing, and tinkling bowel sounds are heard after eating. d. Absence of bowel sounds for 1 minute in each quadrant is reported as abnormal.

a. The presence of borborygmi indicates hyper peristalsis. Rationale: Borborygmi are loud gurgles (stomach growling) that indicate hyper peristalsis. Normal bowel sounds are relatively high-pitched and are heard best with the diaphragm of the stethoscope. High-pitched, tinkling bowel sounds occur when the intestines are under tension, as in bowel obstructions. Absent bowel sounds may be reported when no sounds are heard for 2 to 3 minutes in each quadrant.

A patient complains of nausea. When administering a dose of metoclopramide (Reglan), the nurse should teach the patient to report which potential adverse effect? a. Tremors b. Constipation c. Double vision d. Numbness in fingers and toes

a. Tremors Extrapyramidal side effects, including tremors and tardive dyskinesias, may occur as a result of metoclopramide (Reglan) administration. Constipation, double vision, and numbness in fingers and toes are not adverse effects of metoclopramide.

A 90-year-old healthy man is suffering from dysphagia. The nurse explains what age-related change of the GI tract is the most likely cause of his difficulty? a. Xerostomia b. Esophageal cancer c. Decreased taste buds d. Thinner abdominal wall

a. Xerostomia Rationale: Xerostomia, decreased volume of saliva, leads to dry oral mucosa and dysphagia. Esophageal cancer is not an age-related change. Decreased taste buds and a thinner abdominal wall do not contribute to difficulty swallowing.

The nurse is instructing a patient with chronic pancreatitis on measures to prevent further attacks. What information should be provided (select all that apply) a. avoid nicotine b. eat bland foods c. observe stools for steatorrhea d. eat high fat, low protein, high carbohydrate meals e. take prescribed pancreatic enzymes immediately following meals

a. avoid nicotine b. eat bland foods c. observe stools for steatorrhea Measures to prevent attacks of pancreatitis are those that decrease stimulation of the pancreas. lower fat intake, and foods that are irritating (eat bland), higher carbs are less stimulating, avoid alcohol and nicotine, monitor for steatorrhea (fat in feces). Pancreatic enzymes should be taken with meals, not after.

A patient is jaundiced and her stools are clay colored (gray). This is most likely related to a. decreased bile flow into the intestine. b. increase production of urobilinogen. c. increased production of cholecystokinin. d. increased bile and bilirubin in the blood.

a. decreased bile flow into the intestine. Rationale: Bile is produced by the hepatocytes and is stored and concentrated in the gallbladder. When bile is released from the common bile duct, it enters the duodenum. In the intestines, bilirubin is reduced to stercobilinogen and urobilinogen by bacterial action. Stercobilinogen accounts for the brown color of stool. Stools may be clay-colored if bile is not released from the common bile duct into the duodenum. Jaundice may result if the bilirubin level in the blood is elevated.

Two days following a colectomy for an abdominal mass, a patient reports gas pains and abdominal distention. The nurse plans care for the patient based on the knowledge that the symptoms are occurring as a result of a. impaired peristalsis. b. irritation of the bowel. c. nasogastric suctioning. d. inflammation of the incision site.

a. impaired peristalsis. Until peristalsis returns to normal following anesthesia, the patient may experience slowed gastrointestinal motility leading to gas pains and abdominal distention. Irritation of the bowel, nasogastric suctioning, and inflammation of the surgical site do not cause gas pains or abdominal distention.

The patient with suspected gallbladder disease is scheduled for an ultrasound of the gallbladder. The nurse explains to the patient that this test a. is noninvasive and is a very reliable method of detecting gallstones b. is used only when other tests cannot be used because of allergy to contrast media c. is an adjunct to liver function tests to determine whether the gallbladder is inflamed d. will outline the gallbladder and the ductal system to enable visualization of stones

a. is noninvasive and is a very reliable method of detecting gallstones Ultrasonography is 90-95% accurate in detecting gallstones, and is noninvasive. Liver function tests will be elevated if there is damage to the liver, not with gallbladder.

A normal physical assessment finding of the GI system is/are (select all that apply) a. nonpalpable liver and spleen. b. borborygmi in upper right quadrant. c. tympany on percussion of the abdomen. d. liver edge 2 to 4 cm below the costal margin. e. finding of a firm, nodular edge on the rectal examination.

a. nonpalpable liver and spleen. c. tympany on percussion of the abdomen. Rationale: Normal assessment findings for the gastrointestinal system include a nonpalpable liver and spleen and generalized tympany on percussion. Normally, bowel sounds are high pitched and gurgling; loud gurgles indicate hyperperistalsis and are called borborygmi (stomach growling). If the patient has chronic obstructive pulmonary disease, large lungs, or a low-set diaphragm, the liver may be palpated 0.4 to 0.8 inch (1 to 2 cm) below the right costal margin. On palpation, the rectal wall should be soft and smooth and should have no nodules.

To palpate the liver during a head-to-toe physical assessment, the nurse a. places one hand on the patient's back and presses upward and inward with the other hand below the patient's right costal margin. b. places one hand on top of the other and uses the upper fingers to apply pressure and the bottom fingers to feel for the liver edge. c. presses slowly and firmly over the right costal margin with one hand and withdraws the fingers quickly after the liver edge is felt. d. places one hand under the patient's lower ribs and presses the left lower rib cage forward, palpating below the costal margin with the other hand.

a. places one hand on the patient's back and presses upward and inward with the other hand below the patient's right costal margin.

what will be elevated in acute pancreatitis

amylase

The nurse is admitting a client with the diagnosis of appendicitis to the surgical unit. Which question is essential to ask? A."When did you last eat?" B."Have you had surgery before?" C."Have you ever had this type of pain before?" D."What do you usually take to relieve your pain?"

answer A. When a person is admitted with possible appendicitis, the nurse should anticipate surgery. It will be important to know when she last ate when considering the type of anesthesia so that the chance of aspiration can be minimized. The other inoformation is "nice to know", but not essential.

Which of the nursing interventions should be implemented to manage appendicitis? a. Assess pain b. encourage oral intake of clear fluids. c. provide discharge teaching D. assess for symptoms of peritonitis.

answer D. Monitor for peritonitis because if the appendix ruptures, bacteria can enter the peritoneum. Pain will be managed with analgesics, and pt should be NPO for surgery. Discharge is not done at this time

The patient has chronic peptic ulcer disease. Which manifestations indicate potential gastric outlet obstruction? A. Diarrhea B. Projectile vomiting C. Early morning nausea D. Cullen's sign

b Rationale In gastric outlet obstruction from edema, inflammation, and scar tissue, there is retention of food that can be vomited in a projectile fashion. Constipation, not diarrhea, is a classic manifestation. Nausea and a sense of fullness, if present, is more likely as the day progresses and the stomach fills and dilates. Cullen's sign is bluish black discoloration around the umbilicus, indicating intraperitoneal hemorrhage. Reference: 989

The nurse is assessing an alert and independent 78-year-old woman for malnutrition risk. The most appropriate initial question is which of the following? a. "How do you get to the store to buy your food?" b. "Can you tell me the food that you ate yesterday?" c. "Do you have any difficulty in preparing or eating food?" d. "Are you taking any medications that alter your taste for food?"

b. "Can you tell me the food that you ate yesterday?"

The results of a patient's recent endoscopy indicate the presence of peptic ulcer disease (PUD). Which teaching point should the nurse provide to the patient based on this new diagnosis? a. "You'll need to drink at least two to three glasses of milk daily." b. "It would likely be beneficial for you to eliminate drinking alcohol." c. "Many people find that a minced or pureed diet eases their symptoms of PUD." d. "Your medications should allow you to maintain your present diet while minimizing symptoms."

b. "It would likely be beneficial for you to eliminate drinking alcohol." Alcohol increases the amount of stomach acid produced so it should be avoided. Although there is no specific recommended dietary modification for PUD, most patients find it necessary to make some sort of dietary modifications to minimize symptoms. Milk may exacerbate PUD.

A 62-year-old woman patient is scheduled for a percutaneous transhepatic cholangiography to restore biliary drainage. The nurse discusses the patient's health history and is most concerned if the patient makes which statement? a."I am allergic to bee stings." b. "My tongue swells when I eat shrimp." c. "I have had epigastric pain for 2 months." d. "I have a pacemaker because my heart rate was slow."

b. "My tongue swells when I eat shrimp." Rationale: The percutaneous transhepatic cholangiography procedure will include the use of radiopaque contrast medium. Patients allergic to shellfish and iodine are also allergic to contrast medium. Having a pacemaker will not affect the patient during this procedure. It would be expected that the patient would have some epigastric pain given the patient's condition.

The nurse identifies that which patient is at highest risk for developing colon cancer? a. A 28-year-old male who has a body mass index of 27 kg/m2 b. A 32-year-old female with a 12-year history of ulcerative colitis c. A 52-year-old male who has followed a vegetarian diet for 24 years d. A 58-year-old female taking prescribed estrogen replacement therapy

b. A 32-year-old female with a 12-year history of ulcerative colitis Risk for colon cancer includes personal history of inflammatory bowel disease (especially ulcerative colitis for longer than 10 years); obesity (body mass index ≥ 30 kg/m2); family (first-degree relative) or personal history of colorectal cancer, adenomatous polyposis, hereditary nonpolyposis colorectal cancer syndrome; red meat (=7 servings/week); cigarette use; and alcohol (=4 drinks/week).

The patient has an Salem sump placed for gastric decompression. The nurse notes there has been no drainage for the past 4 hours. What should the nurse do first? a. Assess for abdominal distension b. Assess the patency of the tube. c. Assess the output from the last shift. d. Assess the bowel sounds.

b. Assess the patency of the tube.

Inspection of an older patient's mouth reveals the presence of white, curd-like lesions on the patient's tongue. What is the most likely etiology for this abnormal assessment finding? a. Herpesvirus b. Candida albicans c. Vitamin deficiency d. Irritation from ill-fitting dentures

b. Candida albicans Rationale: White, curd-like lesions surrounded by erythematous mucosa are associated with oral candidiasis. Herpesvirus causes benign vesicular lesions in the mouth. Vitamin deficiencies may cause a reddened, ulcerated, swollen tongue. Irritation from ill-fitting dentures will cause friable, edematous, painful, bleeding gingivae.

The nurse should recognize that the liver performs which functions (select all that apply) a. Bile storage b. Detoxification c. Protein metabolism d. Steroid metabolism e. Red blood cell (RBC) destruction

b. Detoxification c. Protein metabolism d. Steroid metabolism Rationale: The liver performs multiple major functions that aid in the maintenance of homeostasis. These include metabolism of proteins and steroids as well as detoxification of drugs and metabolic waste products. The Kupffer cells of the liver participate in the breakdown of old RBCs. The liver produces bile, but storage occurs in the gall bladder.

The patient who is admitted with a diagnosis of diverticulitis and a history of irritable bowel disease and gastroesophageal reflux disease (GERD) has received a dose of Mylanta 30 mL PO. The nurse should evaluate its effectiveness by questioning the patient as to whether which symptom has been resolved? a. Diarrhea b. Heartburn c. Constipation d. Lower abdominal pain

b. Heartburn Mylanta is an antacid that contains both aluminum and magnesium. It is indicated for the relief of GI discomfort, such as heartburn associated with GERD. Mylanta can cause both diarrhea and constipation as a side effect. Mylanta does not affect lower abdominal pain.

The nurse is planning care for a 68-year-old patient with an abdominal mass and suspected bowel obstruction. Which factor in the patient's history increases the patient's risk for colorectal cancer? a. Osteoarthritis b. History of colorectal polyps c. History of lactose intolerance d. Use of herbs as dietary supplements

b. History of colorectal polyps A history of colorectal polyps places this patient at risk for colorectal cancer. This tissue can degenerate over time and become malignant. Osteoarthritis, lactose intolerance, and the use of herbs do not pose additional risk to the patient.

What information would have the highest priority to be included in preoperative teaching for a 68-year-old patient scheduled for a colectomy? a. How to care for the wound b. How to deep breathe and cough c. The location and care of drains after surgery d. Which medications will be used during surgery

b. How to deep breathe and cough Because anesthesia, an abdominal incision, and pain can impair the patient's respiratory status in the postoperative period, it is of high priority to teach the patient to cough and deep breathe. Otherwise, the patient could develop atelectasis and pneumonia, which would delay early recovery from surgery and hospital discharge. Care for the wound and location and care of the drains will be briefly discussed preoperatively, but done again with higher priority after surgery. Knowing which drugs will be used during surgery may not be meaningful to the patient and should be reviewed with the patient by the anesthesiologist.

The nurse is caring for a patient with a stress ulcer. The nurse understands thus ulcer is caused by : a. The body's response to psychosocial stressors. b. Lack of blood supply to the gastric mucosa. c. H. pylori infection. d. Suppression of stomach acid

b. Lack of blood supply to the gastric mucosa.

Which area of the abdomen will the nurse palpate to assess for splenomegaly? a. Right Upper Quadrant b. Left Upper Quadrant c. Right Lower Quadrant d. Left Lower Quadrant

b. Left Upper Quadrant

A 30-year-old man is being admitted to the hospital for elective knee surgery. Which assessment finding is most important to report to the health care provider? a. Tympany on percussion of the abdomen b. Liver edge 3 cm below the costal margin c. Bowel sounds of 20/minute in each quadrant d. Aortic pulsations visible in the epigastric area

b. Liver edge 3 cm below the costal margin

The nurse should administer an as-needed dose of magnesium hydroxide (MOM) after noting what information while reviewing a patient's medical record? a. Abdominal pain and bloating b. No bowel movement for 3 days c. A decrease in appetite by 50% over 24 hours d. Muscle tremors and other signs of hypomagnesemia

b. No bowel movement for 3 days MOM is an osmotic laxative that produces a soft, semisolid stool usually within 15 minutes to 3 hours. This medication would benefit the patient who has not had a bowel movement for 3 days. MOM would not be given for abdominal pain and bloating, decreased appetite, or signs of hypomagnesemia.

What is a normal finding on physical examination of the abdomen? a. Auscultation of bruits b. Observation of visible pulsations c. Percussion of liver dullness in the left midclavicular line d. Palpation of the spleen 1 to 2 cm below the left costal margin

b. Observation of visible pulsations Rationale: The pulsation of the aorta in the epigastric area is a normal finding. Bruits indicate that blood flow is abnormal, the liver is percussed in the right midclavicular line, and a normal spleen cannot be palpated.

A patient is admitted to the hospital with left upper quadrant (LUQ) pain. What may be a possible source of the pain? a. Liver b. Pancreas c. Appendix d. Gallbladder

b. Pancreas Rationale: The pancreas is located in the left upper quadrant, the liver is in the right upper quadrant, the appendix is in the right lower quadrant, and the gallbladder is in the right upper quadrant.

The nurse is caring for several patients. Which patient is most likely to need a guaiac test? a. Patient reports dark amber colored urine b. Patient reports black stool. c. Patient vomits small amounts of bile. d. Patient complains of right upper quadrant pain.

b. Patient reports black stool.

Which digestive substances are active or activated in the stomach (select all that apply)? a. Bile b. Pepsin c. Gastrin d. Maltase e. Secretin f. Amylase

b. Pepsin c. Gastrin Rationale: Pepsinogen is changed to pepsin by acidity of the stomach, where it begins to break down proteins. Gastrin stimulates gastric acid secretion and motility and maintains lower esophageal sphincter tone. The stomach also secretes lipase for fat digestion.Bile is secreted by the liver and stored in the gallbladder for emulsifying fats. Maltase is secreted in the small intestine and converts maltose to glucose. Secretin is secreted y the duodenal mucosa and inhibits gastric motility and acid secretion. Amylase is secured in the small intestine and by the pancreas for carbohydrate digestion.

The patient had a car accident and was "scared to death." The patient is now reporting constipation. What affecting the gastrointestinal (GI) tract does the nurse know could be contributing to the constipation? a. The patient is too nervous to eat or drink, so there is no stool. b. The sympathetic nervous system was activated, so the GI tract was slowed. c. The parasympathetic nervous system is now functioning to slow the GI tract. d. The circulation in the GI system has been increased, so less waste is removed.

b. The sympathetic nervous system was activated, so the GI tract was slowed. Rationale: The constipation is most likely related to the sympathetic nervous system activation from the stress related to the accident. SNS activation can decrease peristalsis. Even without oral intake for a short time, stool will be formed. The parasympathetic system stimulates peristalsis. The circulation to the GI system is decreased with stress.

Which information about an 80-year-old man at the senior center is of most concern to the nurse? a. Decreased appetite b. Unintended weight loss c. Difficulty chewing food d. Complaints of indigestion

b. Unintended weight loss

An 85-year-old woman seen in the primary care provider's office for a well check complains of difficulty swallowing. What common effect of aging should the nurse assess for as a possible cause? a. Anosmia b. Xerostomia c. Hypochlorhydria d. Salivary gland tumor

b. Xerostomia Rationale: Xerostomia (decreased saliva production), or dry mouth, affects many older adults and may be associated with difficulty swallowing (dysphagia). Anosmia is loss of sense of smell. Hypochlorhydria, a decrease in stomach acid, does not affect swallowing. Salivary gland tumors are not common.

A 62- year-old man reports chronic constipation. To promote bowel evacuation, the nurse will suggest that the patient attempt defecation a. in the mid-afternoon. b. after eating breakfast. c. right after getting up in the morning. d. immediately before the first daily meal.

b. after eating breakfast.

A patient has an elevated blood level of indirect (unconjugated) bilirubin. One cause of this finding is that a. the gallbladder is unable to contract to release stored bile. b. bilirubin is not being conjugated and excreted into the bile by the liver. c. the Kupffer cells in the liver are unable to remove bilirubin from the blood. d. there is an obstruction in the biliary tract preventing flow of bile into the small intestine.

b. bilirubin is not being conjugated and excreted into the bile by the liver. Rationale: Bilirubin is a pigment derived from the breakdown of hemoglobin and is insoluble in water. Bilirubin is bound to albumin for transport to the liver and is referred to as unconjugated. An indirect bilirubin determination is a measurement of unconjugated bilirubin, and the level may be elevated in hepatocellular and hemolytic conditions.

A risk factor associated with cancer of the pancreas is a. alcohol intake b. cigarette smoking c. exposure to asbestos d. increased dietary intake of milk and milk products

b. cigarette smoking

An 80-year-old man states that, although be adds a lot of salt to his food, it still does not have much taste. The nurse's response is based on the knowledge that the older adult a. should not experience changes in taste. b. has a loss of taste buds, especially for sweet and salty. c. has some loss of taste but no difficulty chewing food. d. loses the sense of taste because the ability to smell is decreased.

b. has a loss of taste buds, especially for sweet and salty. Rationale: Older adults have decreased numbers of taste buds and a decreased sense of smell. These age-related changes diminish the sense of taste (especially of salty and sweet substances).

Combined with clinical manifestations, the laboratory finding that is most commonly used to diagnose acute pancreatitis is a. increased serum calcium b. increased serum amylase c. increased urinary amylase d. decreased serum glucose

b. increased serum amylase

During an examination of the abdomen the nurse should a. position the patient in the supine position with the head of the bed flat and knees straight. b. listen in the epigastrium and all four quadrants for 2 minutes for bowel sounds. c. use the following order of techniques: inspection, palpation, percussion, auscultation. d. describe bowel sounds as absent if no sound is heard in the lower right quadrant after 2 minutes.

b. listen in the epigastrium and all four quadrants for 2 minutes for bowel sounds. Rationale: The nurse should listen in the epigastrium and all four quadrants for bowel sounds for at least 2 minutes. The patient should be in the supine position and should slightly flex the knees; the head of the bed should be raised slightly. During examination of the abdomen, the nurse auscultates before performing percussion and palpation because the latter procedures may alter the bowel sounds. Bowel sounds cannot be described as absent until no sound is heard for 5 minutes in each quadrant.

As gastric contents move into the small intestine, the bowel is normally protected from the acidity of gastric contents by the a. inhibition of secretin release. b. release of bicarbonate by the pancreas. c. release of pancreatic digestive enzymes. d. release of gastrin by the duodenal mucosa.

b. release of bicarbonate by the pancreas. Rationale: The hormone secretin stimulates the pancreas to secrete fluid with a high concentration of bicarbonate. This alkaline secretion enters the duodenum and neutralizes acid in the chyme.

In preparing a patient for a colonoscopy, the nurse explains that a. a signed permit is not necessary. b. sedation may be used during the procedure. c. only one cleansing enema is necessary for preparation. d. a light meal should be eaten the day before the procedure.

b. sedation may be used during the procedure. Rationale: Sedation is induced during a colonoscopy. A signed consent form is necessary for a colonoscopy. A cathartic or enema is administered the night before the procedure, and more than one enema may be necessary. Patients may need to be kept on clear liquids 1 to 2 days before the procedure.

The nurse will plan to monitor a patient with an obstructed common bile duct for a. melena. b. steatorrhea. c. decreased serum cholesterol levels. d. increased serum indirect bilirubin levels.

b. steatorrhea.

In planning care for the patient with Crohn's disease, the nurse recognizees that a mojor difference between ulcdrative colitis and Crohn's disease is that Crohn's disease "a. frequently results in toxic megacolon b. causes fewer nutritional deficiencies than does ulcerative colitis C. Often recurs after surgery whereas UC is curable with colectomy d. is manifested by rectal bleeding and anemia more frequently than is ulcerative colitis

c Rationale: Because there is a high recurrence rate after surgical treatment of Crohn's disease, medications are the preferred treatment

A client is admitted to the healthcare facility suspected of having acute pancreatitis and undergoes laboratory testing. Which of the following would the nurse expect to find? a) Decreased white blood cell count b) Increased serum calcium levels c) Elevated urine amylase levels d) Decreased liver enzyme levels

c) Elevated urine amylase levels Elevated serum and urine amylase, lipase, and liver enzyme levels accompany significant pancreatitis. If the common bile duct is obstructed, the bilirubin level is above normal. Blood glucose levels and white blood cell counts can be elevated. Serum electrolyte levels (calcium, potassium, and magnesium) are low.

A patient receives atropine, an anticholinergic drug, in preparation for surgery. The nurse expects this drug to affect the GI tract by: a) increasing gastric emptying b) relaxing pyloric and ileocecal sphincters c) decreasing secretions and peristaltic action d) stimulating the nervous system of the GI tract

c) decreasing secretions and peristaltic action The parasympathetic nervous system increasing motility and secretions and relaxing sphincters to promote movement of contents. A drug that blocks this activity decreases secretions and peristalsis, slows gastric emptying, and contracts sphincters. The enteric nervous system of the GI tract is modulated by sympathetic and parasympathetic influence

When caring for a patient who has had most of the stomach surgically removed, the nurse plans to teach the client a) that extra iron will need to be taken to prevent anemia b) to avoid foods with lactose to prevent diarrhea and bloating c) that lifelong supplementation of cobalamin will be needed d) that, because of the absence of digestive enzymes, protein malnurition is likely

c) that lifelong supplementation of cobalamin will be needed The stomach secretes intrinsic factor necessary for cobalamin absorption in the intestine. In removal of part or all of the stomach, cobalamin must be supplemented for life.

When the nurse is assessing the health perception-health maintenance pattern as related to GI function, an appropriate question to ask is a. "What is your usual bowel elimination pattern?" b. "What percentage of your income is spent on food?" c. "Have you traveled to a foreign country in the last year?" d. "Do you have diarrhea when you are under a lot of stress?"

c. "Have you traveled to a foreign country in the last year?" Rationale: When assessing gastrointestinal function in relation to the health perception-health management pattern, the nurse should ask the patient about recent foreign travel with possible exposure to hepatitis, parasitic infestation, or bacterial infection.

Which statement to the nurse from a patient with jaundice indicates a need for teaching? a. "I used cough syrup several times a day last week." b. "I take a baby aspirin every day to prevent strokes." c. "I use acetaminophen (Tylenol) every 4 hours for back pain." d. "I need to take an antacid for indigestion several times a week"

c. "I use acetaminophen (Tylenol) every 4 hours for back pain."

The pernicious anemia that may accompany gastritis is due to which of the following? a. Chronic autoimmune destruction of cobalamin stores in the body b. Progressive gastric atrophy from chronic breakage in the mucosal barrier and blood loss c. A lack of intrinsic factor normally produced by acid-secreting cells of the gastric mucosa d. Hyperchlorhydria resulting from an inrease in acid-secreting parietal cells and degradation of RBC's

c. A lack of intrinsic factor normally produced by acid-secreting cells of the gastric mucosa Rationale: Gastritis may cause a loss of parietal cells as a result of atrophy. The source of intrinsic factor is also lostthe loss of intrinsic factor, a substance essential for the absorption of cobalamin in the terminal ileum, ultimately results in cobalamin deficiency. With time, the body's storage of cobalamin is depleted, and a deficiency state exists. Because it is essential for the growth and maturation of red blood cells, the lack of cobalamin results in pernicious anemia and neurologic complications

A 58-year-old woman is being discharged home today after ostomy surgery for colon cancer. The nurse should assign the patient to which staff member? a. A nursing assistant on the unit who also has hospice experience b. A licensed practical nurse who has worked on the unit for 10 years c. A registered nurse with 6 months of experience on the surgical unit d. A registered nurse who has floated to the surgical unit from pediatrics

c. A registered nurse with 6 months of experience on the surgical unit The patient needs ostomy care directions/reinforcement at discharge and should be assigned to a registered nurse with experience in providing discharge teaching for ostomy care. Teaching should not be delegated to a licensed practical/vocational nurse or unlicensed assistive personnel.

What is a normal finding during physical assessment of the mouth? a. A red, slick appearance of the tongue b. Uvular deviation to the side on saying "Ahh" c. A thin, white coating of the dorsum of the tongue d. Scattered red, smooth areas on the dorsum of the tongue

c. A thin, white coating of the dorsum of the tongue Rationale: A thin white coating of the dorsum (top) of the tongue is normal. A red, slick appearance is characteristic of cobalamin deficiency and scattered red, smooth areas on the tongue are known as geographic tongue. The uvula should remain in the midline while the patient is saying "Ahh"

Which finding by the nurse during abdominal auscultation indicates a need for a focused abdominal assessment? a. Loud gurgles b. High-pitched gurgles c. Absent bowel sounds d. Frequent clicking sounds

c. Absent bowel sounds

When caring for the patient with heart failure, the nurse knows that which gastrointestinal process is most dependent on cardiac output and may affect the patient's nutritional status? a. Ingestion b. Digestion c. Absorption d. Elimination

c. Absorption Rationale: Substances that interface with the absorptive surfaces of the GI tract (primarily in the small intestine) diffuse across the intestinal membranes into intestinal capillaries and are then carried to other parts of the body for use in energy production. The cardiac output provides the blood flow for this absorption of nutrients to occur.

The patient with chronic gastritis is being put on a combination of medications to eradicate H. pylori. Which drugs does the nurse know will probably be used for this patient? a. Antibiotic(s), antacid, and corticosteroid b. Antibiotic(s), aspirin, and antiulcer/protectant c. Antibiotic(s), proton pump inhibitor, and bismuth d.Antibiotic(s) and nonsteroidal antiinflammatory drugs (NSAIDs)

c. Antibiotic(s), proton pump inhibitor, and bismuth To eradicate H. pylori, a combination of antibiotics, a proton pump inhibitor, and possibly bismuth (for quadruple therapy) will be used. Corticosteroids, aspirin, and NSAIDs are drugs that can cause gastritis and do not affect H. pylori.

You are performing an abdominal assessment. The patient begins to vomit. What is your priority action? a. Assess the emesis for blood, bile or fecal material. b. Offer the patient mouth care. c. Assist the patient to a sitting or sidelying position. d. Administer a PRN anti-emetic.

c. Assist the patient to a sitting or sidelying position.

When assessing a patient's abdomen, what would be most appropriate for the nurse to do? a. Palpate the abdomen before auscultation. b. Percuss the abdomen before auscultation. c. Auscultate the abdomen before palpation. d. Perform deep palpation before light palpation.

c. Auscultate the abdomen before palpation. Rationale: During examination of the abdomen, auscultation is done before percussion and palpation because these latter procedures may alter the bowel sounds.

Which clinical manifestations of inflammatory bowel disease are common to both patients with ulcerative colitis (UC) and Crohn's disease (select all that apply)? a. Restricted to rectum b. Strictures are common. c. Bloody, diarrhea stools d. Cramping abdominal pain e. Lesions penetrate intestine.

c. Bloody, diarrhea stools d. Cramping abdominal pain Clinical manifestations of UC and Crohn's disease include bloody diarrhea, cramping abdominal pain, and nutritional disorders. Intestinal lesions associated with UC are usually restricted to the rectum before moving into the colon. Lesions that penetrate the intestine or cause strictures are characteristic of Crohn's disease.

How will an obstruction at the ampulla of Vater affect the digestion of all nutrients? a. Bile is responsible for emulsification of all nutrients and vitamins. b. Intestinal digestive enzymes are released through the ampulla of Vater. c. Both bile and pancreatic enzymes enter the duodenum at the ampulla of Vater. d. Gastric contents can ply pass to the duodenum when the ampulla of Vater is open.

c. Both bile and pancreatic enzymes enter the duodenum at the ampulla of Vater. Rationale: The ampulla of Vater is the site where the pancreatic duct and common bile duct enter the duodenum and the opening and closing of the ampulla is controlled by the sphincter of Oddi. Because bile from the common bile duct is needed for emulsification of fat to promote digestion and pancreatic enzymes from the pancreas are needed for digestion of all nutrients, a blockage at this point would affect the digestion of all nutrients. Gastric contents pass into the duodenum through the pylorus or pyloric valve.

The nurse is caring for a patient with Diverticulitis. The patient asks what foods should be included in the diet. What is the best response by the nurse? Select all that apply: a. Raspberries b. Corn c. Cooked pears d. Broiled chicken e. Steamed green beans

c. Cooked pears d. Broiled chicken e. Steamed green beans

A patient receives atropine, an anticholinergic drug, in preparation for surgery. The nurse expects this drug to affect the GI tract by doing what? a. Increasing gastric emptying b. Relaxing pyloric and ileocecal sphincters c. Decreasing secretions and peristaltic action d. Stimulation the nervous system of the GI tract

c. Decreasing secretions and peristaltic action Rationale: The parasympathetic nervous system stimulates activity of the gastrointestinal (GI) tract, increasing motility and secretions and relaxing sphincters to promote movement of contents. A drug that blocks this activity decreases secretions and peristalsis, slows gastric emptying, and contracts sphincters. The enteric nervous system of the GI tract is modulated by sympathetic and parasympathetic influence.

The patient is having a gastroduodenostomy (Billroth I operation) for stomach cancer. What long-term complication is occurring when the patient reports generalized weakness, sweating, palpitations, and dizziness 15 to 30 minutes after eating? a. Malnutrition b. Bile reflux gastritis c. Dumping syndrome d. Postprandial hypoglycemia

c. Dumping syndrome After a Billroth I operation, dumping syndrome may occur 15 to 30 minutes after eating because of the hypertonic fluid going to the intestine and additional fluid being drawn into the bowel. Malnutrition may occur but does not cause these symptoms. Bile reflux gastritis cannot happen when the stomach has been removed. Postprandial hypoglycemia occurs with similar symptoms, but 2 hours after eating.

A patient who is scheduled for surgery with general anesthesia in 1 hour is observed with a moist, but empty water glass in his hand. Which assessment finding may indicate that the patient drank a glass of water? a. Flat abdomen without movement upon inspection b. Tenderness at left upper quadrant upon palpation c. Easily heard, loud gurgling in the right upper quadrant d. High-pitched, hollow sounds in the left upper quadrant

c. Easily heard, loud gurgling in the right upper quadrant Rationale: If the patient drank water on an empty stomach, gurgling can be assessed without a stethoscope or assessed with auscultation. High-pitched, hollow sounds are tympanic and indicate an empty cavity. A flat abdomen and tenderness do not indicate that the patient drank a glass of water.

The nurse determines that a patient has experienced the beneficial effects of therapy with famotidine (Pepcid) when which symptom is relieved? a. Nausea b. Belching c. Epigastric pain d. Difficulty swallowing

c. Epigastric pain Famotidine is an H2-receptor antagonist that inhibits parietal cell output of HCl acid and minimizes damage to gastric mucosa related to hyperacidity, thus relieving epigastric pain. Famotidine is not indicated for nausea, belching, and dysphagia.

A female patient has a sliding hiatal hernia. What nursing interventions will prevent the symptoms of heartburn and dyspepsia that she is experiencing? a. Keep the patient NPO. b. Put the bed in the Trendelenberg position. c. Have the patient eat 4 to 6 smaller meals each day. d. Give various antacids to determine which one works for the patient.

c. Have the patient eat 4 to 6 smaller meals each day. Eating smaller meals during the day will decrease the gastric pressure and the symptoms of hiatal hernia. Keeping the patient NPO or in a Trendelenberg position are not safe or realistic for a long period of time for any patient. Varying antacids will only be done with the care provider's prescription, so this is not a nursing intervention.

The nurse is assessing a 50-year-old woman admitted with a possible bowel obstruction. Which assessment finding would be expected in this patient? a. Tympany to abdominal percussion b. Aortic pulsation visible in epigastric region c. High-pitched sounds on abdominal auscultation d. Liver border palpable 1 cm below the right costal margin

c. High-pitched sounds on abdominal auscultation Rationale: The bowel sounds are more high pitched (rushes and tinkling) when the intestines are under tension, as in intestinal obstruction. Bowel sounds may also be diminished or absent with an intestinal obstruction. Normal findings include aortic pulsations on inspection and tympany with percussion, and the liver may be palpable 1 to 2 cm along the right costal margin.

Priority Decision: When caring for a patient who has had most of the stomach surgically removed, what is important for the nurse to teach the patient? a. Extra iron will need to be taken to prevent anemia. b. Avoid foods with lactose to prevent bloating and diarrhea. c. Lifelong supplementation of cobalamin (vitamin B12) will be needed. d. Because of the absence of digestive enzymes, protein malnutrition is likely.

c. Lifelong supplementation of cobalamin (vitamin B12) will be needed. Rationale: The stomach secretes intrinsic factor, necessary for cobalamin (vitamin B12) absorption in the intestine. When part or all of the stomach is removed, cobalamin must be supplemented for life. The other options will not be a problem.

Which nursing actions are indicated for a liver biopsy (select all that apply)? a. Observe for white stools b. Monitor for rectal bleeding c. Monitor for internal bleeding d. Position to right side after test e. Ensure bowel preparation was done f. Check coagulation status before test

c. Monitor for internal bleeding d. Position to right side after test f. Check coagulation status before test Rationale: Because the liver is a vascular organ, vital signs are monitored to assess for internal bleeding. Prevention of bleeding is the reason for positioning on the right side for at least 2 hours and for splinting the puncture site. Again, because of the vasculature of the liver, coagulation status is checked before the biopsy is done. White stools occur with upper gastrointestinal (UGI) or barium swallow tests. No smoking is to be done after midnight before the study with an UGI. The bowel must be cleared before a lower GI or barium enema, a virtual colonoscopy, or a colonoscopy. Rectal bleeding may occur with a sigmoidoscopy or colonoscopy. A perforation may occur with an esophagogastroduodenoscopy (EGD), ERCP, or peritoneoscopy.

Management of the patient with acute pancreatitis include a. surgery to remove the inflamed pancreas b. pancreatic enzymes administered with meals c. NG suction to prevent gastric contents from entering the duodenum d. endoscopic pancreatic sphncterectomy using ERCP

c. NG suction to prevent gastric contents from entering the duodenum

The nurse is caring for a patient with stomatitis. Which nursing intervention is appropriate? a. Check the gastric residual before the feeding. b. Assess a typical 24 hour eating pattern. c. Offer frequent mouth care. d. Assess BMI

c. Offer frequent mouth care.

A 74-year-old female patient with gastroesophageal reflux disease (GERD) takes over-the-counter medications. For which medication, if taken long-term, should the nurse teach about an increased risk of fractures? a. Sucralfate (Carafate) b. Cimetidine (Tagamet) c. Omeprazole (Prilosec) d. Metoclopramide (Reglan)

c. Omeprazole (Prilosec) There is a potential link between proton pump inhibitors (PPIs) (e.g., omeprazole) use and bone metabolism. Long-term use or high doses of PPIs may increase the risk of fractures of the hip, wrist, and spine. Lower doses or shorter duration of therapy should be considered.

The patient receiving chemotherapy rings the call bell and reports the onset of nausea. The nurse should prepare an as-needed dose of which medication? a. Morphine sulfate b. Zolpidem (Ambien) c. Ondansetron (Zofran) d. Dexamethasone (Decadron)

c. Ondansetron (Zofran)* Ondansetron is a 5-HT3 receptor antagonist antiemetic that is especially effective in reducing cancer chemotherapy-induced nausea and vomiting. Morphine sulfate may cause nausea and vomiting. Zolpidem does not relieve nausea and vomiting. Dexamethasone is usually used in combination with ondansetron for acute and chemotherapy-induced emesis.

Following administration of a dose of metoclopramide (Reglan) to the patient, the nurse determines that the medication has been effective when what is noted? a. Decreased blood pressure b. Absence of muscle tremors c. Relief of nausea and vomiting d. No further episodes of diarrhea

c. Relief of nausea and vomiting Metoclopramide is classified as a prokinetic and antiemetic medication. If it is effective, the patient's nausea and vomiting should resolve. Metoclopramide does not affect blood pressure, muscle tremors, or diarrhea.

The health care team is assessing a male patient for acute pancreatitis after he presented to the emergency department with severe abdominal pain. Which laboratory value is the best diagnostic indicator of acute pancreatitis? a. Gastric pH b. Blood glucose c. Serum amylase d. Serum potassium

c. Serum amylase Rationale: Elevated serum amylase levels indicate early pancreatic dysfunction and are used to diagnose acute pancreatitis. Serum lipase levels stay elevated longer than serum amylase in acute pancreatitis. Blood glucose, gastric pH, and potassium levels are not direct indicators of acute pancreatic dysfunction.

A 54-year-old man has just arrived in the recovery area after an upper endoscopy. Which information collected by the nurse is most important to communicate to the health care provider? a. The patient is very drowsy. b. The patient reports a sore throat. c. The oral temperature is 101.6° F. d. The apical pulse is 104 beats/minute.

c. The oral temperature is 101.6° F.

A patient's serum liver enzyme tests reveal an elevated aspartate aminotransferase (AST). The nurse recognizes what about the elevated AST? a. It eliminates infection as a cause of liver damage. b. It is diagnostic for liver inflammation and damage. c. Tissue damage in organs other than the liver may be identified. d. Nervous system symptoms related to hepatic encephalopathy may be the cause.

c. Tissue damage in organs other than the liver may be identified. Rationale: The aspartate aminotransferase (AST) level is elevated in liver disease but it is important to note that it is also elevated in damage to the heart and lungs and is not a specific test for liver function. Measurements of most of the transaminases involves nonspecific tests unless isoenzyme fractions are determined. Hepatic encephalopathy is related to elevated ammonia levels.

The patient tells the nurse she had a history of abdominal pain, so she had a surgery to make an opening into the common bile duct to remove stones. The nurse knows that this surgery is called a a. colectomy b. cholecystectomy c. choledocholithotomy d. choledochojejunostomy

c. choledocholithotomy Rationale: A choledocholithotomy is an opening into the common bile duct for the removal of stones. A colectomy is the removal of the colon. The cholecystectomy is the removal of the gallbladder. The choledochojejunostomy is an opening between the common bile duct and the jejunum.

When patients undergo diagnostic tests of the GI system The nurse is aware that the elderly patient must be closely monitored for : a. diarrhea b. nausea and vomiting c. electrolyte imbalances and dehydration d. constipation.

c. electrolyte imbalances and dehydration

The patient with chronic pancreatitis is more likely than the patient with acute pancreatitis to a. need to abstain from alcohol b. experience acute abdominal pain c. have malabsorption and diabetes mellitus d. require a high carbohydrate, high protein, low fat diet

c. have malabsorption and diabetes mellitus Chronic damage to the pancreas causes pancreatic exocrine and endocrine insufficiency, resulting in a deficiency of digestive enzymes and insulin. Malabsorption and diabetes often result

The nurse determines that further discharge instruction is needed when the patient with acure pancreatitis states, a. i should observe for fat in my stools b. i must not use alcohol to prevent future attacks of pancreatitis c. i shouldn't eat salty foods or foods with high amounts of sodium d. i will need to continue to monitor my blood glucose levels until my pancreas is healed

c. i shouldn't eat salty foods or foods with high amounts of sodium Sodium restriction isn't indicated for pancreatitis

After assisting with a needle biopsy of the liver at a patient's bedside, the nurse should a. put pressure on the biopsy site using a sandbag. b. elevate the head of the bed to facilitate breathing. c. place the patient on the right side with the bed flat. d. check the patient's postbiopsy coagulation studies.

c. place the patient on the right side with the bed flat.

A patient with acute pancreatitis has a nursing diagnosis of pain related to distention of pancreas and peritoneal irritation. In addition to effective use of analgesics, the nurse should a. provider diversional activities to distract the patient from pain b. provide small frequent meals to increase the patient's tolerance of food c. position the patient on the side with the head of the bed elevated 45 degrees for pain relief d. ambulate the patient every 3 to 4 hours to increase circulation and decrease abdominal congestion

c. position the patient on the side with the head of the bed elevated 45 degrees for pain relief

While interviewing a 30-year-old man, the nurse learns that the patient has a family history of familial adenomatous polyposis (FAP). The nurse will plan to assess the patient's knowledge about a. preventing noninfectious hepatitis. b. treating inflammatory bowel disease. c. risk for developing colorectal cancer. d. using antacids and proton pump inhibitors.

c. risk for developing colorectal cancer.

The nurse explains to the pt with GERD that this disorder: a) results in acid erosion and ulceration of the esophagus caused by frequent vomiting. b) will require surgical wrapping or repair of the pyloric sphincter to control the symptoms. c) is the protrusion of a portion of the stomach into the esophagus through an opening in the diaphragm d) often involves relaxation of the LES, allowing stomach contents to back up into the esophagus.

d

A client is to have an upper GI procedure with barium ingestion and abdominal ultrasonography. While scheduling these diagnostic tests, the nurse must consider which factor? a) The client may eat a light meal before either test. b) Both tests need to be done before breakfast. c) The upper GI should be scheduled before the ultrasonography. d) The ultrasonography should be scheduled before the GI procedure.

d) The ultrasonography should be scheduled before the GI procedure Both an upper GI procedure with barium ingestion and an ultrasonography may be completed on the same day. The ultrasonography test should be completed first, because the barium solution could interfere with the transmission of the sound waves. The ultrasonography test uses sound waves that are passed into internal body structures, and the echoes are recorded as they strike tissues. Fluid in the abdomen prevents transmission of ultrasound.

After eating, a patient with an inflamed gallbladder experiences pain caused by contraction of the gallbladdder. The mechanism responsible for this action is a) production of bile by the liver b) production of secretin by the duodenum c) release of gastrin from the stomach antrum d) production of cholecystokinin by the duodenum

d) production of cholecystokinin by the duodenum Cholecystokinin is secreted by the duodenal mucosa when fats and amino acids enter the duodenum and stimulates the gallbladder to release bile and emulsify the fats for digestion. The bile is produced by the liver but stored in the gallbladder. Secretin is responsible for stimulating pancreatic bicarbonate secretion, and gastrin increases gastric motility and acid secretion.

The nurse is caring for a postoperative patient with a colostomy. The nurse is preparing to administer a dose of famotidine (Pepcid) when the patient asks why the medication was ordered since the patient does not have a history of heartburn or gastroesophageal reflux disease (GERD). What response by the nurse would be the most appropriate? a. "This will prevent air from accumulating in the stomach, causing gas pains." b. "This will prevent the heartburn that occurs as a side effect of general anesthesia." c. "The stress of surgery is likely to cause stomach bleeding if you do not receive it." d. "This will reduce the amount of HCl in the stomach until the nasogastric tube is removed and you can eat a regular diet again."

d. "This will reduce the amount of HCl in the stomach until the nasogastric tube is removed and you can eat a regular diet again." Famotidine is an H2-receptor antagonist that inhibits gastric HCl secretion and thus minimizes damage to gastric mucosa while the patient is not eating a regular diet after surgery. Famotidine does not prevent air from accumulating in the stomach or stop the stomach from bleeding. Heartburn is not a side effect of general anesthesia.

The nurse is reviewing the home medication list for a 44-year-old man admitted with suspected hepatic failure. Which medication could cause hepatotoxicity? a. Nitroglycerin b. Digoxin (Lanoxin) c. Ciprofloxacin (Cipro) d. Acetaminophen (Tylenol)

d. Acetaminophen (Tylenol) Rationale: Many chemicals and drugs are potentially hepatotoxic (see Table 39-6) and result in significant patient harm unless monitored closely. For example, chronic high doses of acetaminophen and nonsteroidal antiinflammatory drugs (NSAIDs) may be hepatotoxic.

A 68-year-old patient is in the office for a physical. She notes that she no longer has regular bowel movements. Which suggestion by the nurse would be most helpful to the patient? a. Take an additional laxative to stimulate defecation. b. Eat less acidic foods to enable the gastrointestinal system to increase peristalsis. c. Eat less food at each meal to prevent feces from backing up related to slowed peristalsis. d. Attempt defecation after breakfast because gastrocolic reflexes increase colon peristalsis at that time.

d. Attempt defecation after breakfast because gastrocolic reflexes increase colon peristalsis at that time. Rationale: When food inters the stomach and duodenum, the gastrocolic and duodenocolic reflexes are initiated and are more active after the first daily meal. Additional laxatives or laxative abuse contribute to constipation in older adults. Decreasing food intake is not recommended, as many older adults have a decreased appetite. Fibre and fluids should be increased.

The nurse asks a 68-year-old patient scheduled for colectomy to sign the operative permit as directed in the physician's preoperative orders. The patient states that the physician has not really explained very well what is involved in the surgical procedure. What is the most appropriate action by the nurse? a. Ask family members whether they have discussed the surgical procedure with the physician. b. Have the patient sign the form and state the physician will visit to explain the procedure before surgery. c. Explain the planned surgical procedure as well as possible and have the patient sign the consent form. d. Delay the patient's signature on the consent and notify the physician about the conversation with the patient.

d. Delay the patient's signature on the consent and notify the physician about the conversation with the patient. The patient should not be asked to sign a consent form unless the procedure has been explained to the satisfaction of the patient. The nurse should notify the physician, who has the responsibility for obtaining consent.

The nurse is admitting a 68-year-old man with severe dehydration and frequent watery diarrhea. He just completed a 10-day outpatient course of antibiotic therapy for bacterial pneumonia. It is most important for the nurse to take which action? a. Wear a mask to prevent transmission of infection. b. Wipe equipment with ammonia-based disinfectant. c. Instruct visitors to use the alcohol-based hand sanitizer. d. Don gloves and gown before entering the patient's room

d. Don gloves and gown before entering the patient's room Clostridium difficile is an antibiotic-associated diarrhea transmitted by contact, and the spores are extremely difficult to kill. Patients with suspected or confirmed infection with C. difficile should be placed in a private room and gloves and gowns should be worn by visitors and health care providers. Alcohol-based hand cleaners and ammonia-based disinfectants are ineffective and do not kill all of the spores. Equipment cannot be shared with other patients, and a disposable stethoscope and individual patient thermometer are kept in the room. Objects should be disinfected with a 10% solution of household bleach.

When teaching the patient about the diet for diverticular disease, which foods should the nurse recommend? a. White bread, cheese, and green beans b. Fresh tomatoes, pears, and corn flakes c. Oranges, baked potatoes, and raw carrots d. Dried beans, All Bran (100%) cereal, and raspberries

d. Dried beans, All Bran (100%) cereal, and raspberries

The nurse is caring for a 68-year-old patient admitted with abdominal pain, nausea, and vomiting. The patient has an abdominal mass, and a bowel obstruction is suspected. The nurse auscultating the abdomen listens for which type of bowel sounds that are consistent with the patient's clinical picture? a. Low-pitched and rumbling above the area of obstruction b. High-pitched and hypoactive below the area of obstruction c. Low-pitched and hyperactive below the area of obstruction d. High-pitched and hyperactive above the area of obstruction

d. High-pitched and hyperactive above the area of obstruction Early in intestinal obstruction, the patient's bowel sounds are hyperactive and high-pitched, sometimes referred to as "tinkling" above the level of the obstruction. This occurs because peristaltic action increases to "push past" the area of obstruction. As the obstruction becomes complete, bowel sounds decrease and finally become absent.

The nurse is preparing to administer a dose of bisacodyl (Dulcolax). In explaining the medication to the patient, the nurse would explain that it acts in what way? a. Increases bulk in the stool b. Lubricates the intestinal tract to soften feces c. Increases fluid retention in the intestinal tract d. Increases peristalsis by stimulating nerves in the colon wall

d. Increases peristalsis by stimulating nerves in the colon wall Bisacodyl is a stimulant laxative that aids in producing a bowel movement by irritating the colon wall and stimulating enteric nerves. It is available in oral and suppository forms. Fiber and bulk forming drugs increase bulk in the stool; water and stool softeners soften feces, and saline and osmotic solutions cause fluid retention in the intestinal tract.

Hyperactive bowel sounds in one quadrant and absent bowel sounds in other quadrants plus nausea and vomiting may indicate: a. Pancreatitis b. Cholesystitis c. Peptic ulcer d. Intestinal obstruction

d. Intestinal obstruction

After an abdominal hysterectomy, a 45-year-old woman complains of severe gas pains. Her abdomen is distended. It is most appropriate for the nurse to administer which prescribed medication? a. Morphine sulfate b. Ondansetron (Zofran) c. Acetaminophen (Tylenol) d. Metoclopramide (Reglan)

d. Metoclopramide (Reglan) Swallowed air and reduced peristalsis after surgery can result in abdominal distention and gas pains. Early ambulation helps restore peristalsis and eliminate flatus and gas pain. Medications used to reduce gas pain include metoclopramide (Reglan) or alvimopan (Entereg) to stimulate peristalsis.

The ED nurse has inspected, auscultated, and palpated the abdomen with no obvious abnormalities, except pain. When the nurse palpates the abdomen for rebound tenderness, there is severe pain. The nurse should know that this could indicate what problem? a. Hepatic cirrhosis b. Hypersplenomegaly c. Gall bladder distention d. Peritoneal inflammation

d. Peritoneal inflammation Rationale: When palpating for rebound tenderness, the problem area of the abdomen will produce pain and severe muscle spasm when there is peritoneal inflammation. Hepatic cirrhosis, hypersplenomegaly, and gall bladder distention do not manifest with rebound tenderness.

After eating, a patient with an inflamed gallbladder experiences pain caused by contraction of the gallbladder. What is the mechanism responsible for this action? a. Production of bile by the liver b. Production of secretin by the duodenum c. Release of gastrin from the stomach antrum d. Production of cholecystokinin by the duodenum

d. Production of cholecystokinin by the duodenum Rationale: Cholecystokinin is secreted by the duodenal mucosa when fats and amino acids enter the duodenum and stimulate the gallbladder to release bile to emulsify the fats for digestion. The bile is produced by the liver but stored in the gallbladder. Secretin is responsible for stimulating pancreatic bicarbonate secretion and gastrin increases gastric motility and acid secretion.

What is a clinical manifestation of age-related changes in the GI system that the nurse may find in an older patient? a. Gastric hyperacidity b. Intolerance to fatty foods c. Yellowish tinge to the skin d. Reflux of gastric contents into the esophagus

d. Reflux of gastric contents into the esophagus Rationale: There is decreased tone of the lower esophageal sphincter with again and regurgitation of gastric contents back into the esophagus occurs, causing heartburn and belching. There is a decrease in hydrochloric acid secretion with aging. Jaundice and intolerance to fatty foods are symptoms of liver or gallbladder disease and are not normal age-related findings.

The nurse is performing a focused abdominal assessment of a patient who has been recently admitted. In order to palpate the patient's liver, where should the nurse palpate the patient's abdomen? a. Left lower quadrant b. Left upper quadrant c. Right lower quadrant d. Right upper quadrant

d. Right upper quadrant Rationale: Although the left lobe of the liver is located in the left upper quadrant of the abdomen, the bulk of the liver is located in the right upper quadrant.

A 72-year-old patient was admitted with epigastric pain due to a gastric ulcer. Which patient assessment warrants an urgent change in the nursing plan of care? a. Chest pain relieved with eating or drinking water b. Back pain 3 or 4 hours after eating a meal c. Burning epigastric pain 90 minutes after breakfast d. Rigid abdomen and vomiting following indigestion

d. Rigid abdomen and vomiting following indigestion A rigid abdomen with vomiting in a patient who has a gastric ulcer indicates a perforation of the ulcer, especially if the manifestations of perforation appear suddenly. Midepigastric pain is relieved by eating, drinking water, or antacids with duodenal ulcers, not gastric ulcers. Back pain 3-4 hours after a meal is more likely to occur with a duodenal ulcer. Burning epigastric pain 1-2 hours after a meal is an expected manifestation of a gastric ulcer related to increased gastric secretions and does not cause an urgent change in the nursing plan of care.

A 42-year-old woman is admitted to the outpatient testing area for an ultrasound of the gallbladder. Which information obtained by the nurse indicates that the ultrasound may need to be rescheduled? a. The patient took a laxative the previous evening. b. The patient had a high-fat meal the previous evening. c. The patient has a permanent gastrostomy tube in place. d. The patient ate a low-fat bagel 4 hours ago for breakfast.

d. The patient ate a low-fat bagel 4 hours ago for breakfast.

The nurse receives the following information about a 51-year-old woman who is scheduled for a colonoscopy. Which information should be communicated to the health care provider before sending the patient for the procedure? a. The patient has a permanent pacemaker to prevent bradycardia. b. The patient is worried about discomfort during the examination. c. The patient has had an allergic reaction to shellfish and iodine in the past. d. The patient refused to drink the ordered polyethylene glycol (GoLYTELY).

d. The patient refused to drink the ordered polyethylene glycol (GoLYTELY).

When caring for a patient with a history of a total gastrectomy, the nurse will monitor for a. constipation. b. dehydration. c. elevated total serum cholesterol. d. cobalamin (vitamin B12) deficiency.

d. cobalamin (vitamin B12) deficiency.

A patient is admitted to the hospital with a diagnosis of diarrhea with dehydration. The nurse recognizes that increased peristalsis resulting in diarrhea can be related to a. sympathetic inhibition. b. mixing and propulsion. c. sympathetic stimulation. d. parasympathetic stimulation.

d. parasympathetic stimulation. Rationale: Peristalsis is increased by parasympathetic stimulation.

adverse effects of Lactulose

diarrhea

A patient has received a diagnosis of gastric cancer and is awaiting a surgical date. During the preoperative period, the patient should adopt what dietary guidelines? eat small, frequent meals with high calorie and vitamin content eat frequent meals with an equal balance of fat, carbs, and protein. Eat frequent, low-fat meals with high protein content. Try to maintain the pre-diagnosis pattern of eating.

eat small, frequent meals with high calorie and vitamin content. The nurse encourages the patient to eat small, frequent portions of nonirritating foods to decrease gastric irritation. Food supplements should be high in calories, as well as vitamins A and C and iron, to enhance tissue repair.

a client with cirrhosis has ascites, bleeding varices, and portal HTN. What lab findings indicates that the GI tract is digesting and absorbing blood?

elevated BUN as the body digest blood, BUN rises

client has acute pancreatitis, what should nurse anticipate in client's health history?

gallstones

A client with advanced cirrhosis has been diagnosed with hepatic encephalopathy. The nurse expects to assess for: hand tremors. weight loss. malaise. stomatitis.

hand tremors

client with bleeding variciese has a sengstaken-blakemore tube. what is an appropriate nursing intervention?

provide oral and nasal care every 2 hrs HOB elevated NPO bedrest

A patient has come to the clinic complaining of blood in his stool. A FOBT test is performed but is negative. Based on the patient's history, the physician suggests a colonoscopy, but the patient refuses, citing a strong aversion to the invasive nature of the test. What other test might the physician order to check for blood in the stool? a laprascopic intestinal mucosa biopsy A quantitative fecal immunochemical test computed tomography MRI

quantitative fecal immunochemical test Quantitative fecal immunochemical tests may be more accurate than guaiac testing and useful for patients who refuse invasive testing. CT or MRI cannot detect blood in stool. Laparoscopic intestinal mucosa biopsy is not performed.

The initial treatment plan for a client with acute pancreatitis included insertion of an NG tube. The nurse explains to the client that the rationale for this procedure is to: ensure adequate nutrition via tube feedings. rest the gi tract. provide adequate water intake. administer medications that cannot be given intravenously.

rest GI tract


संबंधित स्टडी सेट्स

Chapter 11 - The Diversity of Prokaryotic Organisms

View Set

Congenital and genetic disorders

View Set

Chapter 10: Taxation of Life Insurance and Annuities - Premiums and Proceeds

View Set

Effects of Recreational Drugs Exam 4 | Textbook Portion

View Set

PrepU Chapter 45 Nursing Care of the Child With an Alteration in Tissue Integrity/Integumentary Disorder

View Set

Government - Chapter 5, Chapter 6 - Voting and Elections - Government, Chapter 7- Voting and Elections- Government, Chapter 8: The media - government

View Set